You are on page 1of 286

Uploaded by:

Ebooks Chemical Engineering


(https://www.facebook.com/pages/Ebooks-Chemical-Engineering/238197077030)

For More Books, softwares & tutorials Related to Chemical Engineering

Join Us

@google+: http://gplus.to/ChemicalEngineering

@facebook: https://www.facebook.com/AllAboutChemcalEngineering
@facebook: https://www.facebook.com/groups/10436265147/
@facebook: https://www.facebook.com/pages/Ebooks-Chemical-
Engineering/238197077030
P~9a~~--------------------------
PHYSICAL
CHEMISTRY

Dr. J.N. Gurtu Dr. H.C. Khera


M.Sc., Ph.D. M.Sc., Ph.D.
Former Principal Reader & Head, Deptt. of Chemistry,
Meerut College, Meerut loP. College, Bulandshahr.

» PRAGATI PRAKASHAN
© Authors-Physical Chemistry Vol. I

PRAGATI PRAKASHAN
Head Office: Educational Publishers
PRAGATI BHAWAN Edition 2010
240, W. K. Road, Meerut-250 001
SMS/Ph. : (0121) 6544642, 6451644
Tele/Fax: (0121) 2640642, 2643636
ISBN-978-81-8398-496-6
Visit us at : www.pragatiprakashan.in
e-mail: pragatiprakashan@gmail.com
Regd. Office:
New Market, Begum Bridge, Meerut-250 001

Published by : K.K. Mittal for Pragati Prakashan, Laser Typesetting : Pragati Laser Type
Setters Pvt. Ltd., (Phone: 2661657) Meerut. Printed at: Arihant Electric Presss, Meerut.
CONTENTS
IIUlij'
MATHEMATICAL CONCEPTS AND COMPUTER 1-55
1. Logarithmic and Antilogarithmic Relations 1
2. Find out the values of the following 5
3. Differentiation with Examples 7
4-12. Numerical Problems 11-13
13. Integration and give important formulae 13
14-23. Numerical Problems 16
24. Terms Permutation and Combination with Examples 22
24-41. Numerical Problems . 23
42. Probability Definition 29
43-50. Numerical Problems 32
51. Logarithmic, Trigonometric Series 32
52. Maxima and Minima 33
53-57. Numerical Problem 34
58. Functioning, Characteristics, Limitations 38
Computer Programing Flow Charts 49
Fortan, Cobol, Basic, Pascal 50
Operating System 51
Exercise 52
oMultiple Choice Questions 52
oFill in the Blanks 54
oTrue or False 54

111~lij"
GASEOUS STATE 5~102

1. Nature of R and its value in different units 56


2. Short account of kinetic theory of gases and derivation of 57
kinetic equation
3. (a) Short account of kinetic theory of gases. Derivation of 59
PV = RT and show how the various gas laws are
consistent with it?
(b) Expression for kinetic energy of one mole of gas
4. Values of Cv and Cp from kinetic equation and variation of 62
CplCv with molecular complexity of the gas
(viii)

5. Distribution of molecular velocities of Maxwell's law 65


6. (a) Average velocity, root me~n square velocity and most 67
probable velocity and relation among them
(b) Calculation of RMS velocity from kinetic theory of gases 67
7. Kinetic equation of gases 68
8. Ideal gas and its difference with a real gas 70
9. (a) Limitations of PV = RT and improvements suggested by 70
vander Waals. Derivation of vander Waals equation.
(b) Units of vander Waals constants 70
(c) Show that effective volume of gas molecules is four 70
times greater than actual volume of molecules
10. (a) Critical phenomenon, calculation and determination of 75
critical constants, short note on continuity of state
11. Some short questions on vander Waals equation 79
12. Short notes on : 81
(a) Various equation of state
(b) Law of corresponding states
(c) Mean free path
(d) Critical phenomenon and its utility
(e) Collision frequency
(0 Law of equipartition of energy
(g) Specific heat ratio
(h) Boyle temperature
(i) Continuity of state
13. Methods for producing cold and liquefaction of gases, 86
inversion temperature
o Numerical Problems 98
o Multiple Choice Questions 100
o Fill in the Blanks 101
o True or False 102

IIn"j'"
CHEMICAL AND PHASE EQUILmRIUM 103-171
I
Chemical Equilibrium I
1. Law of mass action and eqUilibrium constant 103
2. Short notes on the following : 104
(i) Work function (ii) Free energy
3. Thermodynamic derivation of law of mass action 108
4. Thermodynamic derivation of van't Hoff isotherm 109
5. Thermodynamlc··-derivation of van't Hoff isochore or van't 111
Hoff equation
6. Thermodynamic derivation of Clapeyron equation and 114
Clausius-Clapeyron equation
7. Le-Chatelier-Braun principle and applications to different 118
equilibria
o Numerical Problems 122
IPhase Equilibrium I
1. Explanation and illustration of phase, component and degree 126
of freedom
2. Phase rule and its thermodynamic derivation -129
3. Explain: Can all four phases in a one component system 131
co-exist in equilibrium?
4. Application of phase rule to water system 131
5. Application of phase rule to sulphur system 134
6. Short notes on : 137
(a) Non-variant system in phase rule studies
(b) Triple point
(c) Transition point
7. Two component system, graphical representation, reduced 138
phase rule equation, condensed state
8. Application of phase rule to lead-silver system 140
9. Application of phase rul~ to potassium iodide and water 142
system
10. Determination of number of phases and components of 143
different systems
11. Calculation of degree of freedom 143
12. Determination of number of phases, components and degree 144
of freedom of different systems
13. Ideal solutions, vapour pressure of such solutions 144
14. Non-ideal or real solutions, vapour pressure curves of 147
completely miscible binary solutions
15. Theory of fractional distillation of binary solutions 150
16. (a) Theory of partially miscible liquid pairs, e.g., 154
(i) Phenol-water system
Oi) Triethyl-amine water system
(x)

(iii) Nicotine-water system


(b) Influence of impurities on critical solution temperature 154
IDistribution Law I
1. Nemst's distribution law. limitations. applications 157
2. Nemst's distribution law. modification when the solute 162
undergoes dissociation or association
o Numerical Problems 163
o Multiple Choice Questions 169
o Fill in the Blanks 170
o True or False 170

COLLOIDAL STATE 172-210


1. Explain the terms : colloidal state and colloidal solution, 172
methods for preparation and purification of colloidal solutions
2. (a) Difference between true solution. colloidal solution and 177
suspension
(b) Types of colloidal systems. 177
3. Preparation of colloidal solutions of AS2S3. Fe(OH)3 gold. 178
sulphur. silicic acid. carbon. mastic. iodine
4. Short notes on : 180
(i) Lyophilic and lyophobic colloids
(ii) Peptisation
(iii) Dialysis
(iv) Ultramicroscope
(v) Tyndall effect
(vi) Brownian motion
(vii) Electrophoresis
(viii) Electro-osmosis
(ix) Coagulation
(x) Hardy-Schulze law
(xi) Protection
(xii) Gold number
(xiii) Stability of lyophilic colloids
(xiv) Iso-electric point
(xv) Emulsion
(xvi) Gel
(xvii)Electrical double layer or Zera potential
(xi)

5. (a) Origin and significance of charge on a colloidal particle 198


(b) Classification of the sols: 198
Gold, Fe(OH)3, gelatin, blood, sulphur, AS2S3
6. Explain the following facts: 199
(a) A sulphur sol is coagulated by adding a little electrolyte,
whereas a gelatin sol is apparently unaffected.
(b) What happens when a colloidal solution of gold is
brought under the influence of electric field?
(c) What happens when an electrolyte is added to colloidal
solution of gold?
(d) What happens when a beam of light is passed through a
colloidal solution of gold?
(e) A colloidal solution is stabilised by addition of gelatin.
(f) Presence of H2S is essential in AS2S3 sol though H2S
ionises and should precipitate the sol.
(g) Why ferric chloride or alum is used for stoppage of
bleeding?
7. Applications of colloids in chemistry 200
8. Sol-gel transformation 2C4
9. Note on thixotropy 206
o Multiple Choice Questions 208
o Fill in the Blanks 209
o True or False 209

III~I'M
CHEMICAL KINETICS AND CATALYSIS 211-276
I
Chemical Kinetics I
1. (a) Explain the terms: rate of chemical reaction, velocity 211
coefficient, molecularity and order of reaction
(b) Difference between molecularity and order of reaction
(c) Why reactions of higher orders are rare?
(d) Factors which affect reaction rates?
2. Zero order reaction, rate expression, characteristics. 215
3. Half order reaction, rate expression, characteristic 216
4. First order reaction, rate expression, characteristics, examples 217
5. (a) Pseudo-unimolecular reactions 219
(b) Study of kinetics of hydrolysis of methyl acetate
6. Half life period for a first order reaction 220
7. Second order reaction, rate expression, characteristics, 221
examples and study of kinetics
8. Half life period for a second order reaction 224
9. Third order reaction, rate expressions, characteristics and 225
examples
10. nth order reactions, rate equation and characteristic 227
11. Methods employed in determining the order of reaction 228
12. Energy of activation and temperature coefficient 230
13. Activation energy, potential energy barrier and Arrhenius law 233
14. Collision theory for unimolecular reactions 235
15. Mathematical treatment of transition state theory, comparison 236
with collision theory.
o Numerical Problems 242

ICatalysis I
1. Catalyst, catalysis, types and classification of catalysis. 251
characteristics of catalytic reactions
2. Notes on the following: 258
(a) Catalytic promoters
(b) Catalytic poisons
3. Theories of catalysis, industrial applications of catalysts 261
4. Enzyme catalysis, characteristics and examples of enzyme 265
catalysis, kinetics of enzyme catalysis
5. Note on acid-base catalysis 269
o Multiple Choiae Questions 270
o Fill in the Blanks 274
o True or False 275
o Log and Antilog Tables (i)-(iv)
MATHEMATICAL CONCEPTS AND COMPUTER
MATHEMATICAL CONCEPTS
Problem 1: Expillin the logarithmic and antilogarithmic reilltions with
suitable examples.
[A] Index
Multiplication of equal terms: If the term x is multIplIed Il tImes,
then multiplication will be x n , i.e.,
x x x x x x x x ... n times = x"
Here x is called the base and n is called index.
[8] Laws of Index
m
and ~-----
tn-II
x" X

3. xo= 1 4. (Xlll)" = Xlllll


5. (xyt =x"l 6. (xlyr = x"ly"
1
7.xn =-an d x -n =-
1 8. xlln = n{;
-n n
X X

[C] Logarithms
Definition: If ab = c; then exponent 'b' is called the logarithm of number
'c' to the base 'a' and is written as log.. c = b, e.g., J~ = 81 ~ logarithm of
81 to the base 3 is 4, i.e., log3 81 = 4.
Note: a b = c, is called the exponential form and loga c = b is called the
logarithmic form, i.e.,
(i) T3 = 0.125 (Exponential form)
log2 0.125 =-3 (Logarithmic form)
1
(ii) log64 8 ="2 (Logarithmic form)
(64)112 = 8 (Exponential form)
Laws of Logarithms
[I] First Law (product law) :
The logarithm of a product is equal to the slim of logarithms of its
factors.
logo (m X 11) = log" III + log" /I
2 PHYSICAL CHEMISTRY-I

loga (m X n xp) = loga m + logan + lo~p


Remember:
10& (m + n);/:. logam + logan
[II] Second Law (Quotient law) :
The logarithm of a fractl~n is equal to the difference between the
logarithm of numerator and (he logarithm of denominator.

Remember:
loga m
-1-- ;/:. loga m - loga n
oga Il
[III] Third Law (Power law) :
The logarithm of a power of a number is equal to the logarithm of
the number multiplied by the power.
loga (mt = n loga m
Jlr- l/n
Corollary: Since -vm = m
1Jr- l/n 1
., lo~ -vm =lo&m =-Io~m
n
Note:
(a) Logarithms to the base 10 are known as common logarithms.
(b) If no base is given, the base is always taken as 10.
(c) Logarithm of a number to the same base is always one, i.e.,
loga a = 1; 10glO 10 = 1 and so on.
(d) The logarithm of 1 to any base is zero, i.e.,
loga 1 = 0; logs 1 = 0; 10glO 1 =0 and so on.
(e) 10gJO 1 = 0; 10gJO 10 = 1;
10gIO 100 = 2

[.: 10gIO 100 = 10gJO 102 = 2 log 10 10 = 2 xl = 2]


Similarly, 10g]O 1000 = 3; 10gIO 10000 = 4 and so on.
Example: If log 2 = 0.3010 and log 3 = 0.4771; find the value of:
(i) log 6 (ii) log 5
(iii) log -V24
Solution:
(i) log 6 = log (2 x 3) = log 2 + log 3 = 0.3010 + 0.4771 = 0.7781
10
(ii) log 5 = log 2" = log 10 -log 2 = 1 - 0.3010 = 0.6990

(': log 10= 1)


MATHEMATICAL CONCEPTS AND COMPUTE:,:R-'--_ _ _ _ _ _ _ _ _---=::3

(iii) log ..J24 = log (24)1/2 = ~ log (23 X 3)

="21 [3 log 2 + log 3] ="21 [3 X 0.3010 + 0.4771]


=0.69005
[0] Common Logarithms and Use of Four Figure Log
Tables
[I] Common Logarithms : Logarithms to the base 10 are known as
common Logarithms. If no base is given, the base is always taken as 10.
[II] Characteristics and Mantissa: The logarithm of anum ber con-
sists of two parts :
(i) Characteristic-It is the integral part of the logarithm.
(ii) Mantissa-It is the fractional or decimal part of the logarithm.
For exampLe, in log 273 = 2.4362, the integral part is 2 and the decimal
part is .4362.
Therefore, characteristic = 2 and mantissa = .4362.
[III] How to Find Characteristic?
(i) The characteristic of the logarithm ofa number greater than olle
is positive and is numerically one less than the number of digits before the
decimal point.
In number 475.8~ the number of digits before the decimal point is three.
.. Characteristic of log 475.8 = 2, i.e., (3 - 1 = 2)
Similarly, Characteristic oflog 4758 = 3, i.e., (4 - 1 = 3)
Characteristic of log 47.58 = 1, i.e., (2 - 1 = 1)
Characteristic of log 4.758 =0, i.e., (l - 1 =0)
(il) The characteristic of the logarithm of a number less than one in
negative and is numerically one more than the number of zeros immediately
after decimal point.
The number 0.004758 is less t;lan one and the number of zeros im-
mediately after decimal point in it are two.
~ :. Characteristic of log 0.004758 = - (2 + 1) = -3, which is also written
as 3.
Note: To find the characteristic of the logarithm of a number less than
one, count the number of zeros immediately after the decimal point and add
one to it. The number so obtained with negati~ sign gives the characteristic.
.. Characteristic of log 0.3257 = - 1 = 1
[Since, the number of ~eros after decimal point = 0]
Characteristic of log 0.03257 = - 2 = 2
[Since, the number of zeros aftet; decimal point = 1]
Characteristic of log 0.0003257 = -4 = 4 and so on.
4 PHYSICAL CHEMISTRY-I

[IV] How to Find Mantissa?


The mantissa of the logarithm of a number can be obtained from the
logarithmic table.
A logarithmic table consists of three parts:
(1) A column at the extreme left contains two digit numbers starting
from 10 to 99.
(2) Ten columns headed by the digits O. 1,2, 3,4,5,6, 7, 8,9.
(3) Nine more columns headed by digits 1,2,3,4,5,6.7,8,9.
A part of the logarithmic table is given below: (Difference to be added)

0 1 2 3 4 5 6 7 8 9 1 23 456 789
30 4771 4786 4800 4814 4829 4843 4857 4871 4886 4900 134 678 JO 11 13
31 4914 4928 4942 4955 4969 4983 4997 5011 5024 5038 1 34 678 JO 11 12
32 5051 5065 5079 5092 5105 5119 5132 5145 5159 5172 134 678 9 11 12

(a) To find the mantissa of the logarithm of one digit number.


Let the number be 3.
:. Mantissa of log 3 =Value of the number 30 under O.
= 0.477l.
(b) To find the mantissa of the logarithm of two digit number.
Let the number be 32.
:. Mantissa of log 32 = Value of the number 32 under O.
= 0.5051.
(c) To find the mantissa of the logarithm of three digit number.
Let the number be 325.
:. Mantissa of log 325 = Value of 32 under 5 = 0.5119
(d) To find the mantissa of the logarithm of a four digit number.
Let the number be 3257.
Mantissa of log 3257 = Value of 32 under 5 plus the difference
under 7
=0.5128 [5119+9=5128]
1. How to find the logarithm of a number from the logarithm table?
First. find the characteristic and then mantissa.
Suppose the number is 3257.
Characteristic of log 3257 =3
Mantissa of log 3257 = 0.5128
Therefore, log 3257 =3.5128
[Note: The mantissa of the logarithms of all the numbers having the
same significant digits is the same. While tinding the mantissa, ignore the
decimal point.]
MATHEMATICAL CONCEPTS AND COMPUTER 5

For example: log 3257 = 3.5128


log 325.7 = 2.5128
log 3.257 = 0.5128
log 0.3257 = 1.5128
log 0.003257 = 3.5128. and so on]
[Note ..:.
(1) 3.4682 is equivalent to (-3 + 0.4682) and -3.4682
= -(3 + 0.4682) =-3 - 0.4682 i.e., in 3.4682, the mantissa is pos-
itive, while in -3.4682, the mantissa is negative.
(2) Remember, the mantissa should always be written positive.
(3) To make the mantissa positive, subtract 1 from the integral part
and add 1 to the decimal paJ;t, Thus,
-3.4682 =-3 - 0.4682 =(-3 - 1) + (l - 04682)
=- 4 + 0.5318 =4.5318]
Problem ~ : Find out the values of the following:
(i) ~.8321 + 1.4307 (ii) !.9256 - 4.5044
(iii) 1.7544 x 2 (iv) 2.3206 + 3
Solution:
(i) 3.8321
1.4307 Since, after adding decimal parts, we have 1 to carry.
1.2628 - -
:. 1 + 3 + 1 = 1 - 3 + 1 =-1 == 1.
(ii) 1.9256
-4.5044
-
3.4212 Since, 1 - 4 =-3 =3
(iii) 1.7544
x2
- --
1.5088 Since, lX2+1=2+1=1
(iv) 2.3206 + 3
2.3206
3
3 + 1.3206
To make the integral part 2 divisible by3.
3
3 + 1.3206
Take 2= 3+1
3
- -
= 1 + 0.4402 = 1.4402
6 PHYSICAL CHEMISTRY-I

[E] Antilogarithms
If log 5274 = 3.7221, then 5274 is called the antilogarithm of 3.722l.
We write, antilog 3.7221 = 5274.
To find an antilogarithm, the antilogarithm tables are used.
The antilogarithm tables are used in the same way as the logarithm
tables. The only difference between the two tables is that the column at the
extreme left of the log table contains all two digit numbers starting from 10
to 99; whereas an antilog table contains numbers from .00 to .99 (i.e., all
fractional numbers with only two digits after decimal) in the extreme left
column of it.
x 0 1 2 3 4 5 6 7 8 9 123 456 789
0.35 2239 2244 2249 2254 2259 2265 2270 2275 2280 2286 1 1 2 233 445
0.36 2291 2296 2301 2307 2312 2317 2323 2328 2333 2339 1 12 233 445
037 2344 2350 2355 2360 2366 2371 2377 2382 2388 2393 I 12 234 455
0.38 2399 2404 2410 2415 2421 2427 2432 2438 2443 2449 1 1 2 234 455
[Note :
(i) Antilog tables are used only to find the antilogarithm of decimal
part.
(ii) To find the antilog of 2.368 means to find the number whose log
is 2.368].
Ex. 1 : If log x = 2.368,jind x.
Solution : log x = 2.368
x = antilog 2.368
Antilog 2.368 = the number characteristic of whose log is 2 and mantissa
is 368.
From antilog table, the value of .36 under 8 is 2333.
Since the characteristic of log of the number is 2.
:. The number has 2 + 1 =3 digits in its integral part (i.e., 3 digits before
the decimal point).
Antilog 2.368 = 233.3.
Ex. 2 : Find the antilog of 2.3536
Solution: From the antilog table, find the value of .35 under 3 and add
to it the mean difference under 6. The number thus obtained is 2257.
Now place the decimal point so that the characteristic of its log is 2.
.. Antilog 2.3536 = 0.02257
[Note:
(i) Antilog 0.5362 = 3.438 Antilog 1.5362 = 0.3438
(ii) Antilog 2.5362 = 343.8 Antilog 1.5362 = 0.003438
(iii) Antilog 4.5362 = 34380 Antilog 5.5362 = 0.00003438 and so on].
MATHEMATICAL CONCEPTS AND COMPUTER 7

Problem 3: Explain differentiation with suitable examples.


[I] Differentiation
If y = f(x) , x is the function of x, then there will be a change inf(x) with
respect to every change in x. If the increment in x is denoted by cSx , increment
inf(x) will be
. f(x + ax) - f(x)
The ratio of increment in function f(x) and increment in variable x,
i.e.,
f(x + cSx) - f(x)
Ox
is called the difference quotient.
But, if cSx -+ 0, then this ratio tends to a definite quantity. This definite
quantity is called the differential coefficient of f(x) with respect to x and it is
denoted by
d1;) or ~ or f'(x) or y'

Therefore, differential coefficient of f(x) with respect to x,


!!l = !:lfJE = lim f(x + cSx) - fix)
dx dx x~o cSx
The process of finding differential coefficient of f(x) with respect to x
is called 'differentiation' If Ox = It, then
!!l = lim f(x + It) - f(x)
dx x~o It
[II] Some Standard Derivatives
(1) ! (xn) = nxn - 1, where n is a real number
For example, differential coefficient of ;?= 5x5 - 1 = 5x4
(2) .!£(..Jx) = _1_
dx 2Vx
(3) _d (eX) = eX
dx
d mx mx
(4) .dx (e ) = me
For example, differential coefficient of e3x = 3e 3x
d < X
(5 ) dx (a') = a loge a
For example, differential coefficient of 5x is

.!£ (5") =5x log 5


dx e
8 PHYSICAL CHEMISTRY-I

d 1
(6) - (log x) =-
dx e x
For example, differential coefficient of log (x + 1)
d [log (x + 1)] _1_. 1 = _1_
dx x+1 x+l
d 1
(7) --- (log x) = - log e
dx a X a
For example, differential coefficient of log2 x, i.e.,
d 1
- (log x) =-log~ e
d'( 2 X -

[III] Differentials of Trigonometric Functions

(l) dx
d (SIn
. x) = cos x (2) ~ (cos x) = - sinx
d ~ d 2
(3) dx (tan x) = sec- x (4) dx (cot x) = cosec x

d d
(5) -d (sec x) = sec x tan x (6) dx (cosec x) = - cosec x cotx
x
(7) ~ (sin mx) = m cos mx (8) ~ (cos I1lx) = - m sin mx
d 2
(9) -d (tan IIlx) = 111 sec mx
x
[IV] Differentials of Inverse Functions
(1) dd (sin-I x) =
x
bI-x dx
x = - -1 -
(2) -d (cos-I)
{l=7
(3) dd (tan-I x) = _1_ry (4 ) -d (cot-I)
x = - - -1- ,
x 1+x- dx 1 +x~
d (-I 1 d ( _) 1
(5) -d sec x) = _~ (6) -d cosec x =- _~
x x'lx- - 1 x x 'IX- - 1

[V] Differentiation of a Product


The differential coefficient of product of two functions is the sum of
the product of one function and the differential coefficient of the other.
d du du
-- (ll . u) =u . - + u . -
dx dx dx
For example, to find the differential coefficient of xe-' with respect to
x, we have
d d r x d
dx (xe-) =x . d; (e') + e . d-; (x)
t

=x.e-'+ex.l
l
=e (x+1)
MATHEMATICAL CONCEPTS AND COMPUTER 9

[VI] Differentiation of a Division


If u and v are any two functions of x, then
d d
~(~)== v .~(u)-u.~(v)
dx v (v/
For example, to find the differential coefficient of xm Iloge x with respect
to x, we have

!!.- (~) == loge (x) . d;d(m


X ) - X
m d
• d; (loge x
)

dx loge x (loge x)2


loge x (mxm -I) - x'" . (lIx)
== (loge x)2
m-I m-I
m. 1ogeX. X -x
==
(loge xi
m I
x - • (m loge x - 1)
ADS.
(loge x)2

[VII] Partial Differentiation


We know that the differential coefficient off (x) with respect to x is
. f(x + ox) - f(x)
11m
Iix~O x
a '
provided that limit exceeds and is expressed as,

/' (x) or ~ [f(x)]


If u == fix, y) be a continuous function of two independent variables
x and y, then the differential cofficient w.r.t. x (taking y as constant) is called
the partial derivative or partial differential coefficient of u w.r.t. x and is
represented by different symbols such as
au
ax ' E1
ax ,Ix (x, y) ./x·
Symbolically, if u == f(x, y) then
· f(x + Ox, y) - f(x, y)
11m
Iix ..... O Ox
If it exists then. it is called the partial derivative or partial differential
coefficient of u w.r.t. x. It is denoted by,
au
ax or
af
ax or Ix or Ux
10 PHYSICAL CHEMISTRY-I

Similarly, by keeping x constant and allowing y alone to vary, we can


define the partial derivative or partial differential coefficient of u w.r.t. y. It
is represented by anyone of the following symbols.
au af
oy' Oy,/y (x, y),/y
Symbolically, ou = lim f(x, y + By) - fix, y)
oy dy-+O By
provided this limit exists.
For example, if u = ~ + 2hx2y + bi
ou 2·
then ox = 3ax + 4hxy
ou 2
and -=2hx +2by
oy
[I] Rules of Partial Differentiation
Rule (1) : (a) If u is a function of x, y and we are to differentiate partially
w.r.t. x, then y is treated as constant.
(b) Similarly, if we are to differentiate u partially w.r.t. y, then x is treated
as constant.
(c) If u is a function of x, y, z, and we are to differentiate u w.r.t. x, then
y and z are treated as constants.

Rule (2) : If z = u ± v, where u and v are functions of x and y, then


oz = ou + ov and oz = ou + ov
ax ax - ax ay ay - ay
Rule (3) : If z = uv, where u and v are functions of x and y, then
oz 0 ov ou
-=-(uv)=u-+v-
ox ox ox ox
oz 0 ov ou
and -=-(uv)=u-+v-
oy oy oy oy
Rule (4) : If z =~, where u and v are functions of x and y, then
v
oz =~ (~ ) = v ( ~ ) - u( f)
ox ox V v2

ou.) (OV)
~; =;y (~)= v ay ~ u ay
(
and

Rule (5) : If z = f(u) , where u is a function of x and y, then


OZ = oz OU and OZ = oz ou
ox ou· ox oy ou· oy
MATHEMATICAL CONCEPTS AND COMPUTER 11

Problem 4. Determine thejirst order partial derivatives ofyx.


Solution. Let z = yx
Taking logs. log z =x log y ... (i)

O I·ftierenttatmg
. . w.r.t. x. -1 -;- = Iog y az
Z aX

az = z Iog y= yx Iog y
ax
Now differentiate equation (i), w.r.t y, we get

;~~=x(~)=~
or az (X)
ay=Z y =yx(X)
y =xy x-I

Problem S. Find the differential coefficients ofthejollowing:


(i) x S (U) M
(iii) X-SIl
Solution,
(i) !!:.-. (x5 ) = 5~ - I = 5x4
dx
(ii) !!:.-. U =.!!:.- (x)3/2 =1 x~-l = 1 xl/2-
dx dx 2 2
'1'1') d (-5/2) -5 -7/2 - I -5 -7/2
(1
dx X =T X 2 =T x
Problem 6. Find the differential coefficients of the following:
(i) ax4 (ii) 7 loglo x
Solution.

(i) :.x (ax = a :.x (x4) = 4ax3


4
)

(ii) :.x (7 loglO x) = 7 :.x (log lO x) = 7 ~ loglo (e)


Problem 7. Find the differential coefficients oj the following:
(i) 4x + ~ + 10
2
(ii) tan x 1- sin x
2
(iii) x sin x
Solution .
. d 2 d 2 d d
(I) dx(4x +6x+1O)= dx(4x)+ dx"(6x) + dx(1O)=8x+6
..) dx
(II d (tan x sm
.) d (tan x ) + dx
x = dx d (-sm
.)x = sec2 x + cos x

...) dx
(III d (2. d (.
x sm x ) -::: X .z dx sm x ) + di
d (2).
x . sm x = "'='
~ SIn-X
12 PHYSICAL CHEMISTRY-I

Problem 8. Find the differential coefficients of the following:


(i) sin x (ii) x
xl (x+lf
Solution.
2d(. ) . d(2)
(i) ~ (Sin x) = x ;t;; sm x - sm x.;t;; x
dx x2 (X 2)2

=
i cos x - sin x . 2.x
X4
X cos X - 2 sin x
= 2
x
d
2 d 2
(x+l) ;t;;(x)-x.;t;;(x+l)
••) - d [
(II X
dx (x + 1)2 (x+ 1)4
_ (x+ 1)2. I-x. 2. (x+ 1)
- (x+l)4
_ (x+ 1) [x+ 1-2.x] _ (I-x)
- (x+l)4 -(x+l)3
Problem 9. Differentiate the following with respect to x :
(i) sin 5x (ii) log sin x (iii) (3x 2 + Il (Meerut 2007)
Solution.
(i) Let y = sin 5x => y = sin t, where t =5x

.. ~=cost
and dt =5
dx
dv dv dt
d; = di .dx = cos t . 5 = 5 cos 5x.

(ii) Let y = log sin x => y = log t, where t = sin x


4r_l
dt - t
dt
and dx =cosx

4r = 4r. dt = 1 . cos x
dx dtdx t
or 4r =_._1_ . cos x = cot x
dx smx
(iii) 1 (3x 2 2
+ 1)2 = 2 (3x + 1)2-1 . (6x)
= 2 (3x2 + 1) + (6x)
=12x (3x2 + 1)
MATHEMATICAL CONCEPTS AND COMPUTER 13

Problem 10. Calculate the value of ~ from the following:


(i) x=acost,y=asint
Solution.
(i) x = a cos t, y = a sin t
dx . dv
- = - a sm t ::::.L = a cos t
dt ' dt
!:!l. =d dt dt = a cos t cos t
dx y . dx - a sin t = - si~

or !:!l. = - cott
dx

Problem ll.lfy.= tan- [1


1
~x2} then calculate the value of ~.
Solution. Let x = tan 8, then
.• Y = tan -I [ 2 tan 82
I-tan 8
J= tan- 1 ( tan 28) = 28 = 2 tan - 1 x

dy _ '2
.. dx - 1 +x2"

Problem 12. If y =:
d .
tan-1 (Sin x + C?S x) then calculate the value of f!:1...dx
cos x - SID X

. x~
SoIu tion. Let y = tan -I (sin x + cos
cosx- smx

Dividing ilie :::::~(f;;::n~)~ ~; r:~(: ~Jl~: + ~ :


... !:!l.
dx -- 1.

Problem 13. Explain integration and give important integral formulae.


[I] Integration
The reverse process of differentiation is called the integration and it is

represented by a symbol f.
If differential coefficient of f(x) is <I>(x), i.e.,
d
- [((x)] = <I>
dx

then f <I> dx =f(x)

Therefore, f <I>(x) dx
14 PHYSICAL CHEMISTRY-I

is called the integration of c1>(x) with respect to x.


[II] Tables 01 Integral Formulae

(1) J al(x) dx = a JI(X) dx + C

(2) J If(x) ± c1>(x)} dx = JI(X) dx ± J c1>(x) dx

(3) Jl. dx=x, JOdx=C

n :n;:, (n;t - 1)
(4) Jx dx:%

For example, integration of x5 is given by,


1 6
f
5+1
5
x dx = ; + 1 =6" x

(6) J ~dx=IOgeX
x
(7) f C dx=e

(8)faXdx=~
loge a

(9) ISin x dx = - cos x + C

(10) Jcos x dx = sinx + C

(11) f tan x dx = loge sec x + C = - 10& cos u + C

(12) fcotx dx= 10& sin x + C = -loge cosec x + C

(13) fcosec x dx = loge (cosec x - cot u) + C = loge tan ~+C


2
(14) f sin xdx= t x-t sin x cos x + C

2
(15) J cos x dx = t +t
x sin x cos x + C

2
(16) Jsec x dx = tan x + C
MATHEMATICAL CONCEPTS AND COMPUTER 15

(17) fcosec 2 x dx = - cot x + C

2
(18) ftan x dx = tan x - x + C

2
(19) fcot xdx=-cotx-x+C

(20) f sec x tan x dx = sec x

(21) Icosec xcotx dx= - cosec x

(22) f ~1 ~X2 l
dx=sin- x=-cos- x
l

(23) f~
l+x
dx = tan-
l
x = - coe x
l

(24) ff(y) dx = ff(y) dy :

(25) f~=.!tan-l~+
2 2
C
x +a a a

(26)
J dx
~=-2
x- -a
1
1

a-x
a
2
x-a 2
loge--+C,x >a

2
x+a
2

=-2 log--+C,x <a


a a+x

(27) Ih= a -x
sin-
1
~a + C, a > 0
[III] Definite Integral
When any functionf(x) is integrated between the lower limit and upper
limit of x, then it is called the definite integral.
For f(x), if the lower and upper limits of x are a and b, respectively,
then

f
a
b

f(x) dx = [F(x)]~ =F(b) - F(a)

where ff(X) dx = F(x).

For example,J(x) =x 5 and a = 2, b =3, then


16 PHYSICAL CHEMISTRY-I

1 [(3)6 -
(2)6] = 729 - 64 = 665
=
6
Problem 14. Evaluate the following integrals:

(i) f X7 dx (li) f eX dx

f
7+ 1 8
S~lution. (i)
7
x dx =; + 1 =~
(ii) f eX dx = eX

Problem 15. Find the value of the following integrals:

(i) f 3
(x + 2x + 7) dx(ii) f (x
3
- 2)2 dx

Solution. (i) f 3
(x + 2x + 7) dx = f f f
4
3
x dx +
2
2x dx +
4
7 dx

=£+~+7x=£+i+7x
4 2 4

(ii) f (x
3
- 2)2 dx = f 6
(x + 4 - 4x ) dx
3

= f f f 6
x dx + 4 dx - 4x dx
3

x7 4x3+ 1
=-+4x---
7 3+1
x7 4X4
=-+4x--
7 4
7
= ~+4x-x4
7

Problem 16. Evaluate the integral f 1 d~


-smx
.

·
So Iubon. (.)
I
f~-!-1-
.
SIO X
-
- f (1
-
(l. + sin x) dx
. X )-
sm x). (1 + Sin
MATHEMATICAL CONCEPTS AND COMPUTER 17

= f( 1 + sin x) dx
1 - sin 2 x

=f(l+Si~X)dX
cos x

= f[ cos1 x--2-+~-
sin x ]
cos x
dx

= f sec
2
x dx + f tan x . !>ec x dx

=tanx+secx

Problem 17. Find the value of the integral

Solution. f+-dx=fx4~ 1 + 1 dx=fx:-l dx+f-,_l_dx


x- + 1 x- + I x- + I x- + 1

3
X -1
=--x+tan x
3

'Problem 18. Evaluate f sec x + tan x dx


sec x - tan x
(Meerut 2006)

Solution. Multiplying numerator and denominator by (sec x + tan x).

sec x + tan x dx = f (sec x + tan x) (sec x + tan x) dx


f sec x - tan x (sec x - tan x) (sec x + tan x)

-f
-
(sec x + tanx)2
sec 2 x - tan 2 x
dx
18 PHYSICAL CHEMISTRY-I

= f (sec x + tan X)2 dx (".. sec 2 x - tan 2 x = 1)

= I see 2 x + tan2 x + 2 sec x tan x dx

= I 2
see 2 x + (see x-I) + 2 sec x tan x dx

= I(2 see 2 x-I + 2 sec x tan x) dx

= I 2
2 see x dx - I I I . dx + 2 sec x tan x dx

=2 f 2
f
see x dx - fl. dx + 2 sec x tan x dx

=2tanx-x+2secx
Problem 19. Evaluate the following integrals by substitution method:

(i) f (ax + b)' dx (ii) f cos (ax + b) dx

Solution. (i) In I (ax + bf dx, put ax + b = t, so that


adx=dt

or dx= dt
a

I 7
(ax+b) dx= I7 II7
dt
t -;=~ t dt

=~(~)
1 8
= 8a (ax + b)

(ii) In f cos (ax + b) dx, put ax + b = t

adx=dt or dx=dt
a
MATHEMATICAL CONCEPTS AND COMPUTER 19

f cos (ax + b) dx = f cos t . ~ =~ f cos t dt

=-a1SIn.t 1
=-a .SIn(ax + b)
Problem 20. Evaluate the /oUowing integrals:

(i) f~dx
eX + 1
(ii) f log x dx
x

(iii) f eX 2x dx
l+e

Solution. (i) In f:L


e'+1
dx, put eX + 1 = t

= log (eX + 1)
e

(ii) In f 10! x dx, put log x = t

ldx=dt
x

f IO! =f
x dx t dt = ~
-~
-2

(iii) In f41 +e r
dx, put eX =t

f _e_t-dx=f~=tan-I
l+x2x 1+t2
t

=tan- I e'.
20 PHYSICAL CHEMISTRY-I

Problem 21. Evaluate the following integrals:

(i) (ii) f_1_ dX


X log X

Solution. (i) Put x2 + 3x + 2 = t


(2x + 3) dx =dt

2
2x+3 dx= fdt
-= log t
f x + 3x + 2 t e

= log e (x 2 + 3x + 2)
1
(ii) Put log x = t or - dx =dt
x

f 1
--dx=
x log x
f dt
-=log
t e
t

= loge (log x)
Problem 22. Evaluate the following integrals:

(i) f log X dx (ii) f x


xe dx

(iii) f x log x dx (iv) f xsinxdx


Solution. (i) f log x dx = fl. log x dx

= log x f dx - f { ~ (log x) f dx } dx

= x log x - f ~ .x dx
=xlogx- f dx

= x log x - x = x (log x-I)

(ii) f x
xe dx = x f f{~ f
eX dx - (x) . eX dx } dx
MATHEMATICAL CONCEPTS AND COMPUTER 21

= xe' - fl. eX dx

= eX (x - 1)

(iii) f x log x dx = log x f f1fx


x dx - (log x) . f x dx } dx

x2 1x
2
=--Iogx---
2 2 2
2 1
=~ (logX-
2)
i
= 4 (2 log x-I)

(iv) f x sin x dx = x f sin x dx - f{fx f (x) sin x dx } dx

=x(-cosx)- f 1.(-cosx)dx=-xcosx+ f cosxdx

= - x COSJ + sin x
Problem 23. Evaluate the integral eX sin x dx.

Solution. (i) Let 1= f e" sin x dx

= sin xf f{fx f
eX dx - (sin x) eX dx } dx

\. f
= e' sm x - cos x . e''dx
22 PHYSICAL CHEMISTRY-I

= eX sin x - [ eX cos x - f- sin x . eX dx ]

= eX sin x - [ eX cos x +f sin x eot dx ]

I = eX sin x - eX cos x - f (sin x) eX dx

or I = eX sin x - eX cos x - I
or 21 = eX sin x - e'\ cos x

1= t eX (sin x- cos x)

Problem 24: Explain the terms permutation and combination with suit-
able examples.
[I] Permutation
The number of different arrangements which can b~ made by Taking one
til1le some or all elements of any set of things, thell such every arrangement
is called a permutation.
Example: If in the set {a, b, c}, we take two elements in one time,
then the different arrangements which can be made are
ab,ba,bc,cb,ca,ac
Therefore, it is clear that the number of arrangements which can be
made by taking any two things at one time out of three things will be six.
[II] Principle of Permutation
If one work can be done in m ways and second work in n ways, then
both the works can be done together in m X n ways.
Important Formulae: The number of arrangements of taking r things
out of II different things is
"P r = n (11 - 1) (n - 2) ...... (11 - r + 1)
Factorial n :
II! = n (II - 1) (n - 2) ... 3.2.1
Here n ! is read as factorial Il and this is also represented by 1lL
Clearly, if r < 11
"Pr=n!

Also tip =_II_!_


r (II - r) !
MATHEMATICAL CONCEPTS AND COMPUTER 23

When aU things are not different: If out of n things, p things are of


one type, q things are of second type and r things are of r type, then
Number of arragements = , n '; ,
p.q.r.
[III] Combination
The number of combination of r things taken from a set of II dissimilar
things is given by
nc = n! =_np_r
r r! (n - r) ! r !
[IV] Complementary Combinations

Important Results :
(l)nCn =1

(3) nCr + nCr _ 1= n + IC r (4) 2 =


n
Co + CI + ... + CII
Problem 25. Prove that 0 ! = 1.
Solution. The number of permutations of n different objects taking all of
them at a time, is given by,
nPn = (n - l)(n - 2) ... (n - n + 1)
= n{n - 1) (n - 2) ... 1 = n!
nPn = nl. .... (l)
Also by the formula,
Pp = n! nl ... (2)
n (n-n)!=O!'
From equations (1) and (2), we get

II! = 0;n' or O! = 1.

Problem 26. Find the value of6P3 .

So Iuti· on. 6p
3
6!
= (6 _ 3)1 = 3!
6! 6. 5 .4 . 3! 6 5 4 120
3! =.. = .
Problem 27. Prove that nPr = n x n -lpr _ 1•
. S - n- I _ n (n - 1) !
So Iutlon. R.H .. - n x Pr - 1 - [n _ 1 _ (r - 1)] !
n!
,
II. ="p =LHS
(n - 1 - r + I)! (II - r) ! r ...

Problem 28. Three persons enter into a car offive seats. In how many ways
can they occupy their seats?
24 PHYSICAL CHEMISTRY-I

Solution. First person can sit in five ways as all the five seats are lying
vacant at the time of his entrance. The second person can sit on anyone of
the remaining four seats (one is already filled by first person). So. the second
person can sit in 4 different ways. Similarly, thIrd person can SIt in 3 different
ways on anyone of the three vacant seats (two already filled). Hence. all the
three can sit in 5 X 4 x 3 = 60 different ways.
Problem 29. (i) How many words can be formed with the letters of the
word "DELHI"?
(ii) How many of these will begin with D?
(iii) How many of these will end at D?
(iv) How many of these will begin with D or L?
(v) How many of these will begin with D and end at L?
(vi) How many of these will begin with D or end at L?
(vii) How mallY of the vowels "E, I" occupy the even number of
places?
(viii) How many of these will end at vowel only?
(ix) III how many of these vowels come together?
(x) How many of these will begin and end at vowel?
(xi) How many of these will begin and end with D or L?
Solution. (i) There are five letters "D, E, L, H, I" in the word DELHI.
These five letters can be rcalTanged among themselves in 5 ! ::; 120 ways, (or
say 5P s• i.e., the number of permutations of these five letters taking allthe
five at a time). Hence, we can form 120 different words.
(ii) All the arrangements in which D is 'in the beginning can be obtained
by fixing 'D' at the first place and then rearranging the remaining four letters.
Remaining four letters can be arranged in 4 ! ways. Hence, in 4 ! = 24 ar-
rangements the words will begin with D.
Alternatively. First place can be filled by Din 1 way only, the second
place in 4 ways as any of the letters L. H, E, I can be put there; 3rd place can
be filled in 3 ways, 4th place in 2 ways and 5th place in one way only. Hence,
all the five can be filled in 1 x 4 x 3 x 2 x 1 = 4 ! = 24 ways.
(iii) The problem is similar to problem (ii) except that now we fix the
letter D on the last place. Hence, the remaining four letters can be arranged
in 4 ! ways. Hence, the required numb~r of words, ending at D, is 4 ! = 24.
(iv) As above there will be 4 ! arrangements starting with D, and also
4 ! an'angements will begin with L. Hence, the total number of arrangements
beginning with D or L are 4 ! + 4 ! = 48.
AlternaHvely. First place can be filled in two ways as any of the letters
D or L can be put there. Then second place can be filled in 4 ways as anyone
of the remaining fOur letters can be put there. The third place can be filled in
three ways. fourth place in two ways and fifth place can be filled in one way
only. Hence, all the five places can be filled in 2 x 4 x 3 x 2 x 1 = 48 ways.
MATHEMATICAL CONCEPTS AND COMPUTER 25

(v) In this we fix D at the beginning and L at the end. The remaining
three letters E, H, I can be rearranged in 3 ! ways. Hence, 3 ! = 6 arrangements
will begin with D and end at L.
(vi) The arrangements which begin with Dare 4 ! and which end at L
are also 4 !. Hence total number of arrangements which begin with D or end
at L are 4 ! + 4 ! = 48.
(vii) There are only two even places namely 2nd. 4th. E and I can be
rearranged on these places in 2! ways. Further. remaining three letters
D, L, H can be put on 1st, 3rd and 5th places in 3 ! ways. Hence. the total
number of arrangements in which E, I occur at even places only, is
2 ! x 3! = 12.
(viii) The arrangements ending at E or I give the arrangements ending
at vowels. Such arrangements are 4 ! + 4 ! = 48. Proceed as in (iv).
(ix) We consider the two vowels forming as one letter say (E I). Thus
there are four letters D, L, H, (E I). These can be rearranged in 4 ! ways.
Further, two vowels E, I can be rearranged among themselves in 2 ! ways.
Hence, total number of arrangements in which vowels come together is
2 ! x4! =48.
(x) We want to put E or I in the beginning and at the end. For the 3 !
arrangements will begin with E and end at I. Again 3 ! arrangements will
begin with I and end at E. Hence, the number of required ways are
3 ! + 3 ! = 12.
(xi) As above the required number of ways is 12 (only letters E and I
are replac~d by D and L).
Problem 30. How mallY words call be formed from the letters of the word
'DAUGHTER' so that the vowels always come together? (Meerut 2006)
Solution: We consider three vowels forming as one letter say (AUE).
So, there are six letters DGHTR(AUE). These can be arranged in 6! ways.
Further three vowels, A, U. E. can be rearanged among thermselves in 3!
ways. So, total number of arrangements in which vowels come together is 6!
x 31 = 4320 ways.
Problem 31. How mallY words call beformed with the letters of the word
"MEERUT"? In how many of these words vowels occupy only even places?
Solution: (i) There are 6 letters in the word "MEERUT", and the letter
E is repeated 2 times in this word. Therefore, the total number of words that
can be formed with the letters of the word "MEERUT"
_ 6! _ 6 x 5 x 4 x 3 x (2!)
- 2! - 2!
=6 x 5 x 4 x 3 =360
(ii) There are three even places in the word "MEERUT", namely,
second. fourth and sixth places, respectively. At these places the vowels
E, E and U are to be arranged. So, the number of ways to arrange
E, E and U at these three places
26 PHYSICAL CHEMISTRY-I

= 3! =3
2!
Now at the odd places, the letters M, R, T are to be arranged; so the
number of ways to arrange M, R, T at three odd places
=3! =6
Thus, the total number of words in which the vowels occupy even places
=3 x6= 18.
Problem 32. How many permutations can be made out of the letters of
the word "BUSINESS"? How many ofthese will begin with B and end with
N?
Solution: (i) There are 8 letters in the word "BUSINESS", and the
letter S is repeated 3 times and others are different. Then the total number of
words formed by the letters of the word "BUSINESS" are
8!
- 3!
=8x7x6x5x4=6720
(ii) If each word begins with letter B and ends with N, then except these
two letters, the remaining letters are 6 and out of 6 the letter S is repeated 3
times.
Now in this case, the total words formed by the letters of word
"BUSINESS" which begin with B and end with N are

=~=6
6' x 5 x 4 = 120
3!
Problem 33. Prove that nCr = nCn _ r

Solution. R.H.S. = nCn _ r = ( ) , [n ! ( -)] ,


n-r . n- Il-f .

n!
(Il - r) ! (II - II + r) !
n!
(n - r) ! r!
= nCr = L.H.S.
Problem 34. Find the value of the following 24 C4 .
Solution.
24 24!
C4 = 4 ! (24 - 4) !
[ .,'
nC _
r-
n.,
r! (n - r) !
]
24 !
4! 20!
_ 24 x 23 x 22 x 21 x (20!)
- 4 x 3 x 2 x I x (20!)
24 x 23 x 22 x 21
4x3x2xl
MATHEMATICAL CONCEPTS AND COMPUTER 27

= 6 x 23 x 11 x 7
= 10626
Problem 35. Find the value ofr if20Cr _ 1 = 20Cr + l'
Solution. Since. we know that if nCx = nCy. then either x = y or x + y = n
Here. 2OCr _ I = 20Cr + I and r - 1 "* r + 1

So. r+ 1 + r- 1 = 20 (":x+y=n)
=> 2r=20
=> r=lO
Problem 36. In how many ways can a cricket eleven be chosen out of 15
players? How many of them will always (i) include a particular player? (ii)
exclude a particular player?
Solution. Number of ways of selecting cricket eleven
= Number of ways of selecting 11 players out of 15
= lSC = ~ = 15 . 14 . 13 . 12 . 11! = 1365
11 11! 4! 11! 4. 3. 1 .
(i) Now since a player is to be included always. we are to select
remaining 10 players out of the rest 14 players. This can be done in
14CIO = 1001 ways.
(ii) Again since a player is never to be included. i.e .• always excluded.
we are selecting 11 players out of 14 only. This can be done in IO ClI = 364
ways.
Problem 37. How many triangles can be made by joining 12 points in a
plane, given that 7 are in one line?
Solution. The triangles can be formed by joining any three points. But
7 points are in one line. Hence. with three points out of these 7 points in one
line. we cannot form a triangle. Hence. the required number of triangles is
12C3 - 7 C 3 = 185.
Alternative. Triangles can be formed in the following ways :
(i) Three points are taken from the five non-collinear points. Number
of ways is 5C3 = 10.
(ii) One point is taken from 5 non-collinear and 2 out of 7 collinear
points. This can be done in SCI x 7C2 ways = 105 ways.
(iii) Two points are taken from 5 non-collinear and 1 from 7 collinear
points. This can be done in sC2 x 7C1 ways = 70 ways.
Therefore. the required number of triangles = 10 + 105 + 70 = 185.
(Note that possibilities (i). (ii) are (iii) are mutually exclusive.)
Problem 38. Find the number of diagonals that can be drawn by joining
angular points. of a sixteen sided figure?
Solution. In a sixteen sided figure there will be 16 angular points
(vertices).
28 PHYSICAL CHEMISTRY-I

Total number of lines which can be drawn by joining any two angular
points = 16 C2 = 120.
But out of these lines, 16 will be sides. Hence. the required number of
diagonals = 120 - 16 = 104.
Problem 39. In how many ways can 12 things be divided equally among
4 persons?
Solution: 12 things are divided equally among 4 persons, so each
person gets three things. Therefore. the total number of ways
12!
3! 3! 3! 3!
12xUxlOx9x8x7x6x5x4x3x2xl
=
6x 6 x 6 x 6
= 47900 1600 = 369600
6x6x6x6
Problem 40. There are six points on the circumference of a circle. How
many straight lines can be drawn through these points?
Solution: Here no three tJoints lie on a line and through any two
points, a straight line can be drawn. Therefore. the total number of lines that
can be drawn from 6 points
=6 C2

=~= 6x5 = 15
2! 4! 2 xl
Problem 41. Out of 6 teachers and 4 students, a committee of 5 is to be
formed. How many such committees can be formed including (i) at least
one student (ii) 3 teachers and 2 students?
Solution: (i) In this case we have to select at least one student; this
means that from one to all students are to be selected. So. there can be the
following formations, of a committee of 5 persons :
(a) 1 student and 4 teachers (b) 2 students and 3 teachers
(c) 3 students and 2 teachers (d) 4 students and 1 teacher

Tota I ways
.
III
(4C
case a) = I X
6
C4= I!4!3! x 4!6!2!

= 4 x 15 = 60
Total ways in case (b) = 4C2 x 6C3
4! 6!
::: 2! 2! x 3! 3! = 6 x 20 = 120
Total ways in case (c) = 4C3 X 6 C2
4! 6!
= 3!1! x 2! 4! = 60
~M~AT~H~E~M=A~T~IC=A~L~C~O~N~C~E=P~T~S~A~N~D~C~O~M~P~U~T=E~R___________________ 29

Total ways in case (d) = 4C4 x 6C 1


=lx6=6
Hence, the total ways = 60 + 120 + 60 + 6 = 246.
(ii) In this case the committee has 3 teachers and 2 students. So the
total ways to form this committee
=6C3 x 4C2
6!
=--x--
4!
3! 3! 2! 2!
=20x6= 120.
Problem 42 : What is probability? Give its definition also.
In our daily life, we generally come across with the following state-
ments:
(i) Most probably Amit will stand first in his class.
(ii) It is quite probable that Amit may stand first in his class.
(iii) It is least expected that Amit may stand first in his class.
(iv) It is impossible that Amit will stand first in his class.
In all these above statements we have tried to express the chances of
Amit for standing first in his class qualitatively. This is an event which may
and may not happen. But we are predicting the result of the event with some
uncertainty. This uncertainty associated with the event may be lesser or
greater, i.e., it may vary. In mathematics we measure this uncertainty in terms
of number quantitatively which we call probability or chance. With the help
of probability we can predict the outcome of any random experiment by
associating some probability to that outcome.
Probability: If an event E can happen in m ways and fails (cannot
happen) in n ways, all the ways are equally likely to occur, then the probability
of happening of the event E, denoted by pee), is given by
P(E)=~ ... (1)
m+n
Note that 0 :5 peE) :5 1.
If we denote the event of "not happening of event E" by symbol E or by
E', then according to the above definition
- n
pee) = pcE') = - ... (2)
m+n
From equations (1) and (2), we get
peE) + peE) = 1
or peE) + peE) = 1 - peE)
and peE) = 1 - peE)
In equation (1) note that (m + II) are the total number of ways (outcomes)
in which a trial or an experiment may end. Out of these (m + n) ways in m
30 PHYSICAL CHEMISTRY-I

ways, event E happens or say m ways are favourable to the event E. Therefore,
the probability of event E is also given by
P(E) = Favourable number of ways to event E
Total number of ways of the experiment
Problem 43 : Find the chance of throwing more than 4 in one throw of
cubic dice marked 1 to 6 its six faces,
Solution: Here are 6 equalJy likely cases of which only 2 are favourable
because we want 5 or 6 on the upper face of the cubical dice. Hence, the
required probability of throwing more than 4 in one throw with one dice
2 1
=6=3"
Problem 44. In a single throw with two dices, what is the probability of
throwing 9?
Solution. The number of the first dice may appear in 6 ways. Similarly,
on the second dice also the number may appear in 6 ways. Hence, the two
dices may appear in 6 x 6 ways namely,
(1, 1), (1,2), (1,3), (1,4), (1,5), (1,6),
(2, 1), (2,2), (2, 3), (2, 4), (2,5), (2,6),
(3, 1), (3,2), (3, 3), (3, 4), -(3,5), (3,6),
(4, 1), (4,2), (4,3), (4,4), (4,5), (4,6),
(5, 1), (5, 2), (5, 3), (5, 4), (5,5), (5,6),
(6, 1), (6, 2), (6, 3), (6, 4), (6,5), (6,6).
Out of these 36 ways, those which give desired sum of 9 are (3, 6), (4,
5), (5, 4) and (6, 3), i.e., only 4 favourable ways.
:. The probability of throwing 9 = 4/36 = 119.
Problem 45. A bag contains 5 white, 8 black and 3 red balls. If three balls
are drawn at random from the bag, then find the probability of the event,
(i) that all the balls may be white,
(ii) that one ball may be black and the other two white.
Solution. Total number of balls = 5 + 8 + 3 = 16.
Total number of ways of drawing three balls from the 16 balls in the
bag

C3 = ~ = 16. 15 . 14 = 560
16
3!13! 3.2.1
(i) Total number of ways of drawing three white balls out of 5
= 5 C3 = 10.
. .. The probability of drawing 3 white balls together
10 1
= 560 = 56'
MATHEMATICAL CONCEPTS AND COMPUTER 31

(ij) Total number of ways of drawing two white balls out of 5 and one
black ball out of 8
= 5C x 8C =80 .
2 J
So, number of ways of drawing three balls of which one is black and
the other two white
80
=-=-
560 7"
Problem 46. Three cards are drawn from a pack of 52 cards. Find the
probability that:
(i) aU the three will be kings,
(ii) the cards are a king, a queen and ajack.
Solution. (i) Total number of ways of drawing 3 cards from a pack of
52 cards

C =~52X51X50=22100.
=52
3 3! 49! 3x2x 1
Number of kings in the packet is 4, so the favourable number of ways
of drawing three kings
4 4!
= C3 = 3! I! = 4.
So, the required probability
4 1
= 22100 = 5525·
(ii) The favourable ways are 4CJ x 4C J X 4C1 = 4 x 4 x 4 =.64 .
... Required probability
64 16
:::: 22100 = 5525·
Problem 47. Two cards are drawn from a pack of 52 Find the probability
that one may be queen and the other a king.
Solution. Total number of ways of drawing 2 cards out of pack of 52.
=~= 52 x51 x50! = 1326
=52C
2! 50!
2 2 x 1 x 50!
Now, number of favourable ways of drawing 2 cards together, of which
one is a queen and the other a king
4
=4 C1 X C 1 =4 x4 = 16.

.. . d probab·l·
the reqUIre 16 = 663·
I Ity = 1326 8

Problem 48. Two cards are drawn from a full pack of 52 cards. What is
the chance that (i) both are aces of different colours (li) one is red and other
is black?
32 PHYSICAL CHEMISTRY-I

Solution. Total number of ways of drawing two cards from 52 cards


=52
C = 52! 52x51 x50! =26x51
2 2!(52-2)! 2xlx50! .
(i) Now there are 2 black and 2 red aces. So, two aces can be drawn in
= 2 C , x 2C , ways = 2 x 2 ways = 4 ways.
Required probability
6 2
26 x 51 = 663"
(ii) There are 26 red and 26 black cards. So, one red and one black card
can be drawn in 26C, x 26 C , ways = 26 x 26 ways.
=> Required probability
26 x26 26
26 x 51 51"
Problem 49. A pair of dice is thrown. What is the probability of getting a
totalofn
Solution. Total number of possible outcomes = 6 x 6 = 36
Let E be the event of getting a totai of 7, then
E = [(1, 6), (2, 5), (3, 4), (4, 3), (5, 2), (6, 1)]
n{17'\ 6 1
Pea total of7) = ~ = -- =-.
II (S) 36 6
Problem 50. Find the probability that a leap year selected at random will
contain 53 Sundays.
Solution. A leap year contains 366 days, i.e., 52 weeks and 2 days. The
different possibilities for the remaining two days are:
(1) Monday and Tuesday (2) Tuesday and Wednesday (3) Wednesday
and Thursday (4) Thursday and Friday (5) Friday and Saturday (6) Saturday
and Sunday (7) Sunday and Monday.
So, we see that the last two cases are favourable to the happening of 53
Sundays, out of a total of seven equally likely cases. So, the required proba-
.l. 2
bllty = 7.

Problem 51: Explain with examples the following:


(a) Logarithmic series
(b) Trigonometric series
(a) Logarithmic Series
MATHEMATICAL CONCEPTS AND COMPUTER 33

(b) Trigonometric Series


3 5
' x x
(1) smx=x-'3!+5"!- ...... 00,
(-oo<x<oo)

where x is very small and measured in radians.


2. 4
X X x6
(2) cosx= 1 - -, +-4' - - , + ...... 00, (-oo<x<oo)
2. . 6.
x3 2 5
(3) tanx=x+'3!+sTx + ... ,(-nI2<x<nI2)

Problem 52: Define the terms maxima and minima and also mention the
conditions lor finding them.
[I] Definition
Let y =f(x) be some given function and x = a is some given point.
Suppose on the left hand side of x = a, there exists some nearest point
x = a -/z and on the right hand side of x = a there exists some point
x = a + h, where h is very small.
At x =a f(x) =f(a)
At x =a - h f(x) =f(a - h)
At x = a + h f(x) = f(a + h)
Maximum: At x = a, the function is maximum, if
1(0 - h) </(0) > I(a + h)
Minimum: At x = a, the function is mimmum, if
I(a - h) > I(a) </(a + h)

[II] Conditions for Finding Maximum and Minimum


At any point x = a, the function f(x) has maximum or minimum value
if the following two conditions are satisfied.
(a) Necessary condition: For the existence of both maximum or
minimum, the necessary condition is
=0
f'ex)

or ~=O
dx
34 PHYSICAL CHEMISTRY-I

(b) Sufficient condition:


(a) At x = a, if d2~ is negative, then we get maximum.
dx-
2
(b) At x = a, if d ~ is posItive, then we get minimum.
dx-
2 3
(c) At x = a, if
dx
d
~= °and ~
d
dx
*- 0, then neither we get maximum nor
minimum.
(c) To determine maximum or minimum with the help of differential
coefficient :
(i) For maximum:
If for nearest small value of x = a
~ ..
dx = posItIve va Iue

and for greater value of x which is nearest to x = a,


~ = negative value
Then there exists maximum of y =f(x) at x =a
(ii) For minimum :

If for nearest small value of x = a, 2: == positive value


and for greater value of x which is nearest to x =a
dv .. I
d; = posItIve va ue

then there exists minimum of y =f(x) at x = a.

(iii) For neither maximum nor minimum :

If for small and great nearest values of x = o'!:!x has same sign (either
L
(X
both positive or both negative), then at x = a, y :::: f(x) has neither maximum
nor minimum.
Problem 53 : (a) Find the maximum and minimum values of
x 3 - 3x 2 - 9x.
(b) Show that the function y = X S - 5x 4 + 5x3 - 10 has a maximum at
x = 1, while minimum at x = 3 (Meerut 2007)
2
Solution: (a) f(x) = x 3 - 3x - 9x
or y = x3 - 3.l - 9x
Differentiating with respect to x
!!.l. = 3x2 - 6x - 9
dx
MATHEMATICAL CONCEPTS AND COMPUTER 35

Differentiating again with respect to x


d2
E..l'.2 = 6x - 6
dx
For maximum or minimum value of x
4l. =0
dx
2
=> 3x - 6x-9 =0
or x2-2x-3=0
or (x + 1) (x - 3) = 0
x = - 1,3

Atx =-1 6=6(-1)-6


2
dx
= - 6 - 6 = - 12 (negative)
:. Maximum at x = - 1.
2
Atx= 3 ~=6(3)
2
-6
dx
= 18 - 6 = 12 (positive)
:. Minimum atx= 3.
(b) y=x5 -5x4 +5x3 -10
Differentiating with respect to x,
i= 5x
4
-
3
20x + 15x
2

Differentiating again with respect to x,


6dx 2 = 20x'3 - 2
60x + 30x

For finding out the maximum and minimum value of x, put


4l.=0
dx
or
2
or x - 4x + 3 = 0
or (x - 3) (x - 1) = 0
At x-I =0, i.e., x = 1,
2
d
~ = 20 - 60 + 30 = - 10 (negative)
dx
:. Maximum at x = 1
At x - 3 = 0, i.e., x = 3,
2
~2 = 20 (3/ - 60 (3)2 + 30(3)
dx
= 540 - 540 + 90 = + 90 (positive)
:. Minimum at x = 3
36 PHYSICAL CHEMISTRY-I

Problem 54. Show that in all the rectangles of the. same area, the sum of
the sides of the square is the least.
Solution. Suppose x and y are the length and breadth of a rectangle,
then its,
(a) Area =xy =A (given) ... (i)
(b) Perimeter" P = 2 (x + y) = 2 ( x + ~ ) [Using equation 0)]
.. P is a function of x, ~o for maximum and minimum value of P, we must
have,

Now

Now

2
d P
At x"'-{;\, ->
2
0
dx
Therefore, the perimeter (P) of the rectangle is least when x =-{;\
1
or x- =A =xy.
2
x =xy =H (x - y) == 0
=} x == Y ( ... x 0) *
So, all the rectangles with given area are square if the perimeter of
rectangle is least.
Problem 55. What is the maximum area of a rectangle of perimeter 176
em?
Solution. Suppose x and yare the length and breadth of the rectangle,
respectively. Then,
2(x+y)== 176 or x+y=88
Area, A == xy = x(88 - x)

dA == 88 - 2x and
dx
dA =O=}x=44
dx
x=: 44 is a point of maximum.
Maximum area == 44 (88 - 44) = 44 x 44 = 1936 cm2•
MATHEMATICAL CONCEPTS AND COMPUTER 37

Problem 56. If x > 0 and xy = 1, then find the minimum value of x + y,


Solution. Let z = x + y = x +.1 [. : xy = 1]
x
dz = 1 _ ~ and d2~ = 23
dx x2 dx- x
dz =0
dx
1
means that 1 - -:; = 0 or x = ± 1
x-
Thus, x = 1, because x > 0 (given), so
2
dz
Clx2 )x= 1= 2 > 0

:. x = 1 is a point of minimum.
:. Minimum value = 1 + 1 = 2.
Problem 57: What is a linear graph? How will you calculate the slope
of a linear curve?
Solution: If a graph is plotted on two axes, say X and Y and we get a
straight line, then it is known as a linear graph. The equation of a straIght
line is y = mx + c, where m = slope which the line makes on X-axis and c is
:::.......... '," .... .

Intercept

vt
B~--------------~-- o
-X
-x
m = slope = tar, e = (~)
m = slope = e
c = Intercept =AB =
c Intercept = 0

the intercept (or portion) cut by the straight line on Y-axis. This can be
represented by the curves shown in figures (1) and (2).

Problem 58. Write the equation of a straight line whose slope is


2 an d Y-axIs
- -3 . 6.
. .Intercept IS
(Meerut 2007)
38 PHYSlCAL CHEMISTRY-I

Solution. The general equation of a straight line is y = mx + c


where m = slope and C = intercept on Y-axIs. Here m = - ~ and C = 6, so the
equation of the straight line will be given by,

y=( -~)x+6
or 3y=-2x+ 18
or 2x + 3y= 18

COMPUTER
Problem 58: Explain the terms functioning, characteristics, limitations
and components of a computer. (Meerut 2005. 04)
[I] Introduction to Computer
The word computer comes from the word 'compute' which means to
calculate. So, normally a computer is considered to be a calculating device
that can perform arithmetic operations at enormous speed.
Computer is an invention that has taken more than a hundred years to
take the shape it is in today. The way the nature of computers have manifested
in our lives, is slightly less than the effect that the discovery of fire or the
Illvention of the wheel had on evolution of mankind. Launching of a space-
craft, making an airline reservatIOn, scheduling year, your day or keeping a
child occupied with some games is like a game for this wonder machine. The
essence being, computer can no longer be termed as a mere invention-it is a
revolution.
Computer can be defined as an electronic machille which processes raw
data to give meaningful if!formation. In other words, the term computer is
used to describe a device f//ade up of combination of electronic and electro-
mechanical components.
Data : Data are collection of facts, figures, statistics which can be
processed to produce meanlllgful informations.
Processing: Proce~sing IS the manipulation of the letters, numbers or
graphic symbols that constitute data. It includes calculation, comparison,
decision making, logic.
Information: Information is the term given to meaningful form of
data. In other words, it is processed data.

IInput Data 1--7 IProcess 1--7 ~tput (Informatic;;J


[II] Characteristics of Computer
By itself, a computer has no intelligence. A computer or computer
system does not come to hght until it is connected to the other parts of its
system. A computer is generally used due to the following attributes:
MATHEMATICAL CONCEPTS AND COMPUtER 39

1. Speed: A computer can work very fast. It can perform in min-


utes/seconds the tasks that would take a person long time to complete. Present
day computer can perform 100 million computation in 1 second.
2. Accuracy: A computer always give accurate results.
3. Reliability: Computers consistently provide the same accurate
results under all operating conditions.
4. Storage capacity: A computer can store large amounts of data.
5. Diligence: Computer can keep on working for long hours and never
gets tired.
[III] Limitations of Computer
Unlike human brains, a computer cannot think on its own. It cannot
detect flaws in logic input to it. Computer cannot draw a conclusion without
going through all intermediate steps. It does not learn from experience.
[IV] Components of a Computer
A computer system is a combination of three elements :
(1) Hardware
(2) Software
{1) Person
(1) Hardware: Hardware as the term indicates is anything which is
hard. Those parts of a computer which we can see, touch or feel are called
hard wares. The physical components and equipments which make up a
computer system are called hardware. Hardware consists of the following
components:

Memory
Control Arithmetic
Unit and logical
urut
Now let us study each of the components:
C.P.u.: The central processing unit is also referred as 'brain' of the
computer. It controls the flow of data through the system, directing the data
to enter the system, placing data in memory, reentering them when needed
and directing the output of information. It consists of :
(a) Arithmetic Logic Unit (A.L.U.): It performs all the arithmetic
calculations and takes logical decisions. It can perform addition, subtraction
etc.
40__________________________________~P~H~Y~S~I~C~A~L~C~H~E~M~IS~T~R~Y~-I

(b) Control Unit: This unit controls and coordinates the activities of
all other parts of a computer system. It performs the following functions:
(i) It can get instructions out of the memory unit.
(il) It can determine the storage from which it is to get the net
ll1structions after the previous instruction has been executed.
(iIi) It can decode the instructions.
(c) Memory: All the data, intermediate and final results are stored in
memory. It is used to hold the instructIOns to be carried out and the data to
be processed. It is c1as~ifled mto two types:
(i) Primary memory Oi) Secondary memory.
(i) Primary memory: It is also called the main memory or the central
memory. It is again of two types RAM and ROM.
RAM is essentially a 'ReadlWrite MemO/y'. Information can be written
into and read from a RAM. It is volatile in nature, i.e., it retains the stored
information as long as the power supply is not switched off. ROM is a
permanent type memory. ROM retains the data in it even in the absence of
power and IS thus non-volatile storage. ROM does not allow user to write data
onto it. It stands for 'Read Only Memory'.
(ii) Secondary memory: It is found outside the CPU box and hence
~ometimes called external memory or the external storage. Examples are
floppy disks, t~lpe~. compact disks (C.D's) etc.
(2) Input Devices: These devices are required to get raw data into
the computer. It is considered as 'interface' between the user and the system.
Various types of input devices are as follows:
(i) Keyboard: It is just like a typewriter. But there are additional keys
that control functions. These are alpha-nulIleric keys, special keys and/ullction
keys.
(ii) Mouse: It is one of the most popular type of specialized input
deVIce for computers. It is used as a pointing device.
(iii) OMR: The 'Optical Mark Reader' is a device which is capable
of recognising pre-specified type of mark made by pencil or pen.
(iv) OCR: The 'Optical Character Reader' cannot only detect a mark
but also recognizes its shape and identify characters directly from source
documents.
(v) Light pen: Light pen uses a light sensitive photo-electric cell to
a single screen position to the computer.
(3) Output devices: Output devices are those which output the pro-
cessed information. These devices are as follows:
(i) Visual display unit: It is used for interactive processing. It is
commonly known as monitor.
(ii) Printers: The most common form of computer output is printed
output is also called hard copy. Printers are classified as/to how they print and
how fast they operate. Character printer, page printer and line printer are
different types of printers.
(iii) Storage: It has storage areas, often referred to as memory. The
memory can receive, hold and deliver data when instructed to do so. This
MATHEMATICAL CONCEPTS AND COMPUTER 41

fonn of storage stores the data permanently in the given media, examples of
which are floppy diskettes, magnetic discs, C.D's etc.
Problem 2 : Explain the binary system used in computers. Define the terms,
bit, nibble and byte.
Solution : Binary System : A computer interprets infonnation com-
posed of only zeros and ones. So, instructions and data processed by a
computer must be in the fonn of zeros and ones. In other words. the data are
stored and processed as strings of two symbols or two state devices. A switch
is for example, a two state device, it can be either ON or OFF. As decimal
system uses total ten digits to make any number, similarly the binary system
uses only two digits (0 and 1) to represent any large number. Each character
in computer is represented by a number of decimal system and further con-
verted into 0 and 1 (binary number). Since only two digits 0 and 1 are used
in the system, it is called, binary number system. The each digit of this number
system is called binary digit or bit. This system of numbers can also be said
as base 2 number system. In this system also each digit has its place value as
in decimal number system. The digit written at extreme left is called 'Most
Significant Bit' (MSB) and the digit written at extreme right is called 'Least
Significant Bit' (LSB). Note the place ofLSB and MSB in the example given
below.
1 o o o o 1

1
Most Significant Bit Least Significant Bit
f
(1) Bit. As it has already been stated above that each digit of binary
number system is known as 'Billat:v Digit'. To make its name short, the B has
been taken from Binary and IT is derived from Digit, thus making its name
as BIT. A bit represents the smallest part of memory locations in computer,
which stores either 0 or 1. Refer figure (3) for bit.

Binary
I
I 1 () 0 I 0 Blllary digit or bit.
number I, which represents
Power the smallest part of
24

-
2' .,6 25 23 22 21
of base computer memory
Decimal
equivalent 128 64 32 16 8 4 :2 Binary digits are
sho\',.11 in the form
ON / of switches. If any
switch is ON. It .

~~~~~~~~ represents digit L


otherWise (} in it's
OFF OFl· condItion
I 1 0 0 I 0

Fig. 3. Binary digits


I 0 = 20210
I
42,___________________________________P~H~Y~S~I~C~A~L~C~H~E~M~IS~T~R~Y~-I

(2) Nibble. The combination of 4 adjacent bits is known as a nibble.


(3) Byte. The combination of 8 bits is called byte. The byte represents
one character in computer memory. It is now clear that to store one character
in computer memory we need at least the space of 1 byte or 8 bits. Now the
question arises here that why should there be only 8 bits to represent one
character in computer memory? The answer to this question is that the com-
puter works with the help of only 256 characters generated by its keyboard.
These characters are A to Z, a to z, 0 to 9 and some special characters such
as punctuation marks, arithmetic operator symbols, colon. semicolon, questiQn
mark, = , >, <, etc. Every character is represented by a numeral code. These
numeral codes are converted into binary numbers and stored in computer
memory. The codes starting from 0 to 255 are represented by a group of binary
digits. Now we have to find out the total number of digits required in a group
to represent 256 combinations using only two digits, i.e., 0 and 1. We find
that 8 is the number which can make 256 combinations using two digits.
28 or 2 x 2 x 2 x 2 x 2 x 2 x 2 x 2 = 256
Group of binary numbers Decimal
using 8 bits equivalent
0 0 0 0 0 0 0 0 0
0 0 0 0 0 0 0 1 1
0 0 0 0 0 0 1 0 2
0 0 0 0 0 0 1 1 3
0 0 0 0 0 1 0 0 4
0 0 0 0 0 1 0 1 5
0 0 0 0 0 1 1 0 6
0 0 0 0 0 1 1 1 7
0 0 0 0 1 0 0 0 8
0 0 0 0 1 0 0 1 9
0 0 0 0 1 0 1 0 10
0 o 0 0 1 0 1 1 11
0 0 0 0 1 1 0 0 12

1 1 1 1 1 1 0 255
1 1 1 1 1 1 1 256
Problem 3 : Discuss the conversion of numbers from one system to another
with examples.
Solution: Conversion of Numbers: We know that computer under-
stands only binary numbers, so the methods of converting the numbers from
one system to another, has been explained below :
(1) Decimal to binary conversion. The base of binary number system
is 2 because it uses only two digits to represent the number. The base of
decimal number system is 10 because total 10 digits are used to represent the
MATHEMATICAL CONCEPTS AND COMPUTER 43

decimal numbers. Each decimal number is converted to binary before it is


stored in computer memory. For converting the decimal number to binary, we
divide the decimal number by 2 (because the base of binary system is 2) unless
we get remainder less than 2. This can be understood by the following problem.
2 7 4 2
2 3 7 1 0 (LSB)

2 1 8 5 1
2 9 2 1
2 4 1 0 I Remainders
2 2 0 1
2 1 0 0
2 5 0
2 2 1 (MSB)
1 0
Problem 4. Convert decimal number 742 to binary.
Solution. After dividing the number by 2, the remainder is noted in each
step. The last remainder will make most significant digit and first remainder
as least significant digit.
The binary number will be as follows :
(0 1 0 0 1 0 1 Oh = (742ho
(2) Binary to decimal conversion. As the numerals are converted to
binary system for storing into computer memory, these numbers are converted
back into decimal form for output purpose in the same way. The conversion
of these numbers is done automatic by the electronic parts fitted in computer.
For converting, binary numbers into decimal, following steps are to be fol-
lowed.
(a) Find the least significant digit and most. significant digit in the
number.
(b) Mark the place position of each digit starting from LSB to MSB.
The first position will be zero position.
(c) Find the place value of each digit with the help of the following
formula,
Digit X Base number place pOSItIOn of digit.
It is important to note that the base value in binary system is 2.
(d) Add all the place values to get the equivalent decimal number.
This can be understood by the following problem.
Problem 5. Convert the binary number (1 0 0 1 0 1 Oh to its decimal
equivalent. (Meerut 2006)
44 PHYSICAL CHEMISTRY-I

Solution. Binary Place Place value


digit position
MSB 1 6 1 x 26 64
0 5 Ox25 = 0
0 4 1 X24 0
1 3 1 x23 8
0 2 Ox2 2 0
1 1 lX2' 2
LSB 0 0 Ox2° 0
The equivalent decimal number will be :
64 + 0 + 0 + 8 + 0 + 2 + 0 = 74
(3) Decimal to octal conversion. Like binary number system, octal
system is also used for coding of decimal numbers. As its name clarifies, octal
means 8, so total 8 digits are used in octal system. These digits are 0 to 7.
The highest number that can be derived from the group of octal digits without
any conversion is equivalent to (777)8 of octal number system. For converting
decimal number to binary number, we divide the decimal number by the base
value of binary number system (which is 2). Similarly, in octal system also
we divide the decimal number by base value of octal system which is 8. This
can be understood by the following problem.
Problem 6. Convert a decimal number (3 4 8 9 2)10 to its octal equivalent.
Solution.

834 892 4 (LSB)


84364
8 5 4 5 Remainders
8 6 8 4
8 8 4 o
o (MSB)
Thus, the octal equivalent of (34892)10 decimal number is
(1041 14)8. Just like binary conversion here also we keep dividing the
number by 8 unless we get remainder less than 8.
(4) Octal to decimal conversion. We know that only 8 digits (0 to 7)
are used in octal system. It is clear now that digits used in octal system cannot
be more than 7. For converting an octal number to decimal. same steps are
applicable as from binary system to decimal.
(a) Find the most significant and least significant digits.
(b) Mark the place position of each digit considering LSB at zero
place and MSB at highest place.
(c) Find the place value of each digit by applying the following
formula,
· ·t x B ase num be r Place positIOn of dIgit
D Igl
MATHEMATICAL CONCEPTS AND COMPUTER 45

(d) Add all the place values to get the decimal equivalent number.
This can be understood by the following problem.
Problem 7. Convert (6 5 3 4 2)8 octal number to its equivalent decimal
number.
Solution.
Octal digit Place position Place value
4
(MSB) 6 4 6x8 24576
5 x 83

i
5 2 2560
3 2 3 x 82 ::: 192
4 1 4 x 8' 32
(LSB) 2 o 2 x 8° 2
:. Decimal equivalent number is:
24576 + 2560 + 192 + 32 + 2::: 27362
So, (6 5 3 4 2)g::: (2 7 3 6 2)10
(5) Decimal to hexadecimal conversion. An in binary number system
only two digits (0 and 1) are used and in octal ~stem 8 digits (0 to 7) are
used to represent the numbers, similarly in hexadecimal number system total
16 digits are used to store and represent the numbers. The digits are 0 to 9
and A to F, where A is used for 10, B for 11, C for 12, D for 13, E for 14 and
F for 15. The reason for using these alphabets in place of numbers is that each
digit should be represented by single entity. If we write 10 (i.e .. one zero) to
represent eleventh digit making it a combination of two digits i.e., 1 and 0,
which have already been used for representing the digits 1 (one) and 0 (zero)
independently, it will create confusion to recognise among the numbers 0, 1
and 10. So, t;:> avoid this confusion, alphabets from A to F are used to represent
numbers from 10 to 15.
As shown above, total 16 digits are used in hexadecimal system. While
converting decimal number to hexadecimal number, we divide decimal num-
ber by 16 unless we get remainder less than 16 because the base of hexadec-
imal system is 16. This can be understood by the following problem.
Problem 8. Convert (186275 2ho decimal number to hexadecimal.
Solution.
16 8 6 2 7 5 2
16 1 6 4 2 2 0 (LSB)

16 7 2 7 0 2
16 4 5 4 6
16 2 8 6

l~'
1 (MSB)
46 PHYSICAL CHEMISTRY-I

So, (1 8 6 2 7 5 2)10 =(1 C 6 6 2 0)16


(6) Hexadecimal to decimal conversion. The system of number con-
version from hexadecimal to decimal system is same as from binary to decimal
and octal to decimal. The difference is only of their base value because binary
number system has its base value as 2. octal has 8 and hexadecimal has 16.
This can be understood by the following problem.
Problem 9. Convert (5 7 A C ~16 hexadecimal number to decimal num-
ber.
Solution. Hexadecimal Place position Place value
digit
MSB 5 ,f 4 5 X 164 327680
7 3 7 X 163 = 28672

I
LSB
A
C
2
2
1
0
Ax 162
ex 16
2 x 16°
1
= 2560
192
2
The decimal equivalent number will be :
327680 + 28672 + 2560 + 192 + 2 = 359106
or (5 7 A C 2)16 = (3 5 9 1 o 6)10
(7) Hexadecimal to binary conversion. The conversion of hexadecimal
to binary number is very simple just by converting each digit of hexadecimal
to its binary equivalent. To make it simpler. let us know the binary equivalent
of each decimal number from 0 to 15 as shown in table-I.
Table-1. Relationship between the four number systems
Decimal Binary Octal Hexadecimal
0 000 0 0 0
I 000 I I I
2 0 o I 0 2 2
3 001 I 3 3
4 010 0 4 4
5 0 I 0 I 5 5
6 o I I 0 6 6
7 0 I I I 7 7
8 I o 0 0 10 8
9 100 I II 9
10 I 010 12 A
II I o 1 1 13 B
12 1 1 0 0 14 C
13 1 1 0 1 15 D
14 1 I 1 0 16 E
15 1 I 1 I 17 F

This can be understood by the following problem.


MATHEMATICAL CONCEPTS AND COMPUTER 47

Problem 10. Convert (7 3 0 A D Eh6 hexadecimal number into its bi-


nary equivalent.
Solution. 7 3 o A D E
01ll 0011 0000 1010 1101 lll0
The equivalent binary number will be :
(0111 0011 0000 1010 1101 11lOh = (730ADE)16
(8) Binary to hexadecimal conversion. We know that the hexadecimal
numbers are represented in a group of 4 bits because the largest digit of
hexadecimal system is F, which is equivalent to decimal number 15 and binary
number 1111. As the largest digit in hexadecimal system is represented by
group of four bits, so each hexadecimal digit will be represented by a group
of four bits.
To convert a number from binary to hexadecimal, we have to make the
group of each four digits starting from LSB (Least Significant Bit) to MSB
(Most Significant Bit). Add the zeros to left side of MSB if required to make
the combination of four bits. Now write the hexadecimal equivalent of each
group of four bits starting from MSB to LSB. This can be understood by the
following problem.
Problem 11. Convert (0010 0110 1001 1110 0101h to its equivalent
hexadecimal number.
Solution. Let us make the group of each four bits starting from LSB to
MSB.
10 0110 1001 1110 0101
(These two zeroes have been added
to make the MSB as a combination
of four bits)
While making the group of each four bits we find that only two bits i.e.,
1 and 0 (one and zero) are left in group ofMSB, so we have added two zeroes
at the left most side to make the combination of four bits which in our problem
now shows 00 10.
Now we write the hexadecimal equivalent of each group of four bits.
0010 0110 1001 1110 0101
2 6 9 E 5
So, the hexadecimal equivalent number will be (2 6 9 E 5)16'
(9) Binary arithmetic.
(a) Addition: Now let us consider addition in binary. The following
scales of binary addition are to be remembered :
0+0=0
0+1=1
1+1=0 carry 1 to next column to the left
1 + 1 + 1 =1 carry 1 to next column
48 PHYSICAL CHEMISTRY-I

Example:
1111
11011
+ III
LaOOlO
(b) Subtraction: This method is also known as complementary sub-
traction. We will be doing the following three steps to perform subtraction:
(i) We have to find the complement of the number we are subtracting.
(ii) To the complement of number obtained in step 1, we add the number
we are subtracting from.
(iii) If there is a carry 1, add the carry to the result of the addition, else
recomplement the sum and attach a negative sign.
Example: Number Complement
10001101 01110010
Consider the following example of subtraction.
Example: 1010101 - 100100
Step 1. Find the complement of 1001100
11011
Step 2. Add the number you are subtracting from

Carry 1110111
1010101
+ 0110011
0001000
+1 (Since there is a carry of 1)
0001001 -7 Result
(c) Multiplication: The table to be remembered is :
Oxl=O
lxO=O
1 x 1 == 1
Example: 10101 x 11001
10101
11001
10101
00000
00000
10101
10101
1000001101 -7 Result
(d) Division: The table 'for binary division is
Oil =0
1/1 = 1
The steps for binary division are:
(i) Start from the left of the dividend.
MATHEMATICAL CONCEPTS AND COMPUTER 49

(ii) Perform subtraction in which the divisor is subtracted from the


dividend.
(a) If subtraction is possible put 1 in quotient and subtract the
divisor from the corresponding digits of the dividend.
(b) Else put a 0 in the quotient.
(iii) Execute step 2 till there are no more digits from left to bring down
from the dividend.
Example: 1000011110
Then 0101 (Quotient)
(Divisor) 110) 100001 (Dividend)
110 Step 1
1000 Step 2b
~ Step 2a
100 Step 2b
.-WL Step 2a
1001 Step 2b
110 Step 2a
11 (Remainder)
Problem 12 : Explain the terms (i) computer programming and (ii) flow
charts.
[I] Computer Programming
In order to solve a problem on a computer, we have to develop an
algorithm. An algorithm is a set of instructions which if strictly followed, will
give a solution to the problem. In case an instruction is obeyed, we say it is
executed. The following activities are involved when we solve a problem on
a computer..
(a) Define the problem.
(b) Analyse the problem.
(c) Develop an algorithm or a method for solving the problem.
(e) Test and debug the program.
(f) Document the program.
There is normaIly some overlap of the above activities. For example,
with a large program, a portion may be written and tested before another
portion is written. For any problem, there will nonnaIly be more than one
algorithm (method) to solve it. Each method will have its own advantages and
disadvantages. The user will have a choice of algorithms and it will be his/her
job to decide which algorithm is the best and why this is so.
[II] Flow Charts
The algorithm uses a numbered list of instructions, the instructions
themselves are English-like statements of what must be done. Another method
which is useful for specifying smaIl algorithms is the flow chart. A flow chart
consists of 'flow chart 'Symbols , connected by arrows. Each symbol will have
information about what must be done at the point and the arrows indicate the
50 PHYSICAL CHEMISTRY-I

flow of execution of the algorithm, i.e., they indicate the order in which the
instructions must be executed. Figure below shows the symbols commonly
used in flow charting.
C) Oval, indicates Start or Stop: tenninator symbol)

[J Parallelogram, used to specify an Input or Output operatIon, ... g.,


getting data or printing data. (liO symbol)

o Rectangle, used to specify an operation or process, e.g., 'find average'


or . set F to 32 I C (process symbol)

~"
Rhombus, used to specIfy a condition. This usuall)' takes the lorm of a
quesllon with possible answers Yes or No (or True or False).
These arrows lead to the reqUIred action corresponding to the answer to
the question (the decision symbol. more commonly called the decIsIOn

o
box).

Cirde, used as a connectmg point for arrows coming from different


directions (conncctor symbol).

Flow chart symbols.


Problem 13 : Explain the following computer programming languages:
(i) FORTRAN (ii) COBOL
(iii) BASIC (iv) PASCAL
(1) FORTRAN. It means 'Formula Translation' and is the oldest high
level language. It was developed in 1956-57. It was designed to solve scientific
and engineering problems and is currently the most popular language among
the scientists and engineers. After 1957, it appears in different variations such
as FORTRAN II, FORTRAN IV and FORTRAN 77 (a version of 1977). The
updated version of FORTRAN 77 is FORTRAN 90.
(2) COBOL. It means 'Common Business Oriented Language' and was
designed to solve the data processing problems of the business community,
e.g., problems relating to pay roll, stock control, accounts received and paid,
cheque analysis etc. The activities involve little numerical work but a large
amount of data has to be processed. The processing consists in creating and
updating large files of data. Unlike FORTRAN, the COBOL does not have
all the commonly used mathematical functions such as square root, sine,
cosine, tangent, logarithms and exponentials etc. In FORTRAN, matrices
encountered in science and engineering can be manipulated easily using the
FORTRAN arrays, whereas in COBOL this cannot be done.
(3) BASIC. It means 'Beginner's All-purpose Symbolic Instruction
Code' and was developed by John Kemeny and Thomas Kurtz (1963). Basic
is easy to learn and use. It was mainly designed to be used as an interactive
language. Almost every computer manufacturer provides, BASIC on its ma-
chines. BASIC is probably the most widely used programming language in
the world because of the widespread use of microcomputers.
(4) PASCAL. It was designed by NikIaus Wirth and named after the
famous French mathematician, Blaise Pascal (l7th century). The first Pascal
compiler appeared in 1970. The language was chiefly designed as a tool for
MATHEMATICAL CONCEPTS AND COMPUTER 51

teaching structured programming concepts. Most of the colleges and univer-


sities use it to teach computer programming. Pascal has the facilities to
manipulate, not only numbers, but also vectors, matrices, strings of characters,
records and files etc. PASCAL is more readable and thus self documentary as
compared to BASIC.
Problem 14 : Discuss the operating systems as used in computers.
Operating system is a software loaded into computer memory for en-
suring the control and proper working of computer hardware. No computer
can work without an operating system. The reason for it is that the system of
working of each part of computer is defined in operating system program.

We know that many programs can be loaded in computer memory.


Different parts of computer are used to operate these programs on computer.
There are chances of clashing of instructions issued through different pro-
grams. It may create a halt to operation of the computer. If two different
programs need the service of some part of the computer, it may create a state
of confusion that whose service is to be completed first. Operating systcm
software has been assigned this responsibility to decide the priority and conro)
the entire system. Operating system is required in a computer in the same way
as a principal is required in a college or a vice-chancellor in a university, who
takes the decision above all the teaching staff.
Operating system attends to the request of different programs running
in computer memory and decides their working through the hardware. For
example, suppose you want to read the data from a floppy and execute the
command for this purpose. The operating system will read this command and
check the availability of floppy drive before reaching the data. Similarly, if
you execute the comma-v-d for printing a file through printer, the operating
system program will check the availability of printer. The complete control
of hardware is done through operating system program.
[I] Personal Computer Operating Systems
The examples of operating systems are MS-DOS, MS-WINDOWS,
UNIX, ZENIX etc. The main mode of use of a personal computer (PC) is hi


52 PHYSICAL CHEMISTRY-I

a single user. Thus, operating systems (OS) for PCs have been constructed as
a single user single task operating system, i.e., it is assumed that only a single
user uses the machine and runs only one program at a time. The operating
system of PCs consists of two parts, viz.: (i) Basic Input Output System (BIOS)
which is stored in a floppy disc or a hard disc and (ii) Microsoft Disc Operating
System (MS DOS). This type of OS is widely used. The BIOS provides basic
low level services, whereas DOS provides many user-level services. MS-DOS
provides services such as editing, filing and other utility programs.
MS-Windows is an improved operating system for more powerful per-
sonal computers which have a large memory (4 to 8 MB) and disc (200 to
400 MB). This gives a very good 'Graphical User Interface' (GUI) which
simplifies the use of the computer. It also helps us; to allow the multiple
programs to be simultaneously stored in memory and executed.
(1) UNIX operating system. This system is a very powerful operating
system used in personal computers and supercomputers. UNIX is written in
C, so it makes it portable. It is organised as a layered operating system. The
innermost portion is known as a kernel. It provides low-level services, e.g.,
device drivers and memory management. The next portion is known as a shell.
It is a command interpreter. The outermost portion is known as utilities
portion. It provides miscellaneous services such as text processing, text set-
ting, games, calender, graphics, language compilers etc. The user interacts
with the kernel using commands and utilities.

EXERCISES
[I] Numerical Problems
1. FIII Illerentl'al coe ff"IClent 0 f,log x .
' d the d'''''
x
2. Calculate the value Of: if Y! = tf-'y.

3. Evaluate f5 dx,

4. Find the value of f ..Jax + b , dx.


5. If 211 + I PII _ I : 211- I Pn = 3 : 5, find the value of n.
6 Convert (11011.101)z into decimal numbers.
7. Fmd the binary equivalent of (146)10'
8. Convert (39ho into binary equivalent.
9. Convert (O,1l5ho to octal equivalent.
10. Convert (537)8 into binary equivalent.
[II] Multiple Choice Questions
1. If nCIO = nC 15 • then value of 11 is,
(a) 5 (b) 15 (c) 20
MATHEMATICAL CONCEPTS AND COMPUTER 53

(d) 25
2. Values of 8C3 and 9P2 are respectively:
(a) 28, 72 (b) 56, 72 (c) 56, 36
(d) 28,36

f~
3
3. The value of is :
1
(a) log 3 (b) log 2
(d) log 10 x

4. The value of fax ~ b is :


I
(a) log(ax+b) (b) iog--
ax+b
(c) bI log (ax + b)
I
(d) -log (ax + b)
a
5. The probability of selecting a girl from a class consisting of.s boys and 7 girls
is:

(a) ~
5
(b) ! (c) -
6 7
7
(d) 12
6. Value of !x 72
/ is :

(a) '2 x 9/2 (b) "2X


7 5/2
2
(d) ~ log x 7/2
7. The slope of straight line 2x + 3y = 4 is :
(a) -~ (b) ~ (c) -
3
2
(d) -~
8. An equation of a straight line y = 2x means that:
(a) Une passes through the OIigin
(b) Une does not pass through the origin

(c) The slope of line is ~ (d) Every statement is wrong


9. Th I f 12! .
e va ue 0 3! x 8! IS :

(a) 2900 (b) I (c) 1980


2
(d) 198

10. The value of


5
f+-
x +4
dx is :

(a) log (x + 4) (b)

(d) * 5
log (x + 4)
54 PHYSICAL CHEMISTRY-I

11. The unit used to read the exernal data and instructions in a computer is :
(a) Input unit (b) Output unit (c) CPU
(d) None of the above
12. The computer memory is measured in terms of :
(a) Units (b) Bytes (c) Binary
(d) ALU
13. The decimal equivalent of (IOOlllh is :
(a) (19)10 (b) (29)10 (c) (39)10
(d) (49)10
14. I second consists of ...... microseconds :
(a) 10
3
(b) 106
(d) 1012
15. The following is an output device:
(a) Keyboard (b) Arithmetic logic unit (c) Monitor
(d) Scanner

[III] Fill in the Blanks


1. Differential coefficient of a constant function 'c' is .............. .
d ' .
2. The value of dx ifl (x) ·12 (x)] IS ............... .

3. The value of f (ax + b)n - I dx is ................ .

4. The value of OJ is ....................... .


5. The value of 6 P3 is ......... .
6. A mouse is ................ device generally used in graphical interface software.
7. COBOL is an example of ................ level language.
8. 8 bits are equal to .................. byte.
9. The components which we can see and touch are known as ........... .
10. 15 In decimal system is equivalent to .......... in hexadecimal system.

[IV] True or False


State whether the following statements are true (T) or
false (F) ?
1. The number of arrangements of n distinct objects taken all at a time is (n - I)!.
2n
2. It· p 3 = 2xn P4. then n =7 .

3. f2 x 3dx =
I
3l
4. The value of probability varies from - I to I.
5. For a straight line 2y = 3x - 3. the intercept on Y-axis is - 3.
6. The speed of computer is very fast as compared to a calculator.
7. A computer consists of CPU only.
8. The computer memory is measured in terms of bytes.
9. In computer memory, 0 and I are called binary digits or bits.
MATHEMATICAL CONCEPTS AND COMPUTER 55

10. Laser printers are coloured printers.


11. The octal equivalent of (10)10 is 12.
12. The binary equivalent of (0)10 is 1000.

ANSWERS
[I] Numerical Problems
1.
l-logx
2.
logx -k 2
3. 2x2 4. 3a (ax + b)3/2
x2 (I + logx)2
S. 4 6. (27.101)10 7. (IOOIOOIOh 8. (IOOlllh
9. (0.0727024)8 10. (100101111 lh

[II] Multiple Choice Questions


1. (d) 2. (b) 3. (a) 4. (d)
S. (d) 6. (b) 7. (a) 8. (a)
9. (e) 10. (d) 11. (a) 12. (b)·
13. (e) 14. (a) 15. (e)

[III] Fill in the Blanks


I
1. 0 2. j(x)J'2 (x) +hex) .f'(x) 3. - (ax + b)n
all
4. I S. 120 6. input 7. high
8. one 9. hardware 10. nc
[IV] True or False
1. (F) 2. (F) 3. (T) 4. (F)
S. (1') 6. (T) 7. (F) 8. (T)
9. (T) 10. (F) 11. (T) 12. (F).

000
GASEOUS STATE
Problem 1 : What is the nature or significance of R in general gas equation
PV = nRT? Calculate its value in different units.
Nature or significance of gas constant R. From general gas equation,
PV=nRT
R = PV = Pressure x Volume ... (1)
IlT Mole x Temperature
Force Force
Pressure =- - =
Area (Length)2

and Volume = (Length)3


So, substituting the values of pressure and volume in equation (1), we
get
R= Force x (Length)3 Work
(Length)2 Mole x Temperature Mole x Temperature

C-.. Work = Force x Length)


The work can be expressed in different units and so R can have different
units.
1
(1) Value of R in litre atmosphere deg-1 mole-
1 mole of every gas at N.T.P. occupies a volume of 22.4 litres.
.. P = 1 atmosphere, V = 22.4 litres, T = 273K

R = PV =Lx 22.4
.. T 273
= 0.0821 litre atmosphere deg-1 mole-1
(2) Value of R in erg deg- 1 mole- 1
P = 76 x 13.6 x 981dyne/sq cm, V= 22400 ml, T= 273 K
PV 76 x 13.6 x 981 x 22400
R=T= 273
= 8.314 x 107 erg deg- 1 mole-1

(56)
GASEOUS STATE 57

(3) Value of R in calorie deg-1 mole-I.


We know that, 1 calorie = 4.18 x 107 erg
7
R = 8.314 x 10 1.99 cal deg- I rnole- I
7
4.18x10
"" 2 cal deg-1 mole- I
(4) Value of R in joule deg- I mole-I.
We know that, 1 joule = 107 erg
7
I-I
.. R = 8.31410x7 10 JOU
. I d -I
e eg rno e

= 8.314 joule deg-1 mole-I.


Problem 2 : Write the postulates oj kinetic theory of gases. Derive the
kinetic equation oj gases.
Or, Prove the equation PV = 1 mnu
2

Postulates of Kinetic Theory


Kinetic theory of gases was given by Kronig, Clausius, Maxwell etc.
to explain the behaviour of gases theoretically. This theory is applicable only
to a perfect or an ideal gas. The main postulates or assumptions of the kinetic
theory are:
J. A gas consists of a large Ilwllber of small particles, called mole-
cules. The molecules are so small that their actual volume can be
neglected in comparison to the total volume oJthe gas.
2. The molecules are ill a state of collstant rapid motion in all possible
directions, colliding with each other and with the walls of the
containing vessel. i':::::'::.: : .": .' ..... .. .'. :-':::-:""'::::: '::'.: .::.: ..:.'.: ::'.:..,
3. The l110lecules are f..~ y
"':::
perfectly elastic. .. :. .: /: ::-:
Thus, there is no loss / I /

ofkinetic energy wizen .... f--~-----~


they collide with each km
other or with the walls
of the containing ves- .' A
sci. ..' -ux .r
~

4. The molecules do not


exert any appreciable :::: .. ",'0------------/---- X·.·.
lIlutual force of at- '.' ..
traction. .. Z ..
5.
~;u~~~;/~;~:~l~a~~: z:~'l=·
+H;:"-:.:?.. g=,l=.=Ki='n::;:e""tl"'c""p=jc=tu"'
=."'-
" "
.... ..............
R...
r""'~""'~=f""~a"'"~"".~" "oZ:'I: ;:e~" '~" i~" '~:;:('
ment of the moving
molecules 011 the walls of the colltaillillg l'essel,
58 PHYSICAL CHEMISTRY-I

6. The kinetic energy of the molecules is directly proportional to the


absolute temperature of the gas.
7. There is no effect of gravity on the gas molecules.
Derivation of Kinetic Equation
From the above postulates, the kinetic equation of gases can be derived
easily as follows:
Consider a gas enclosed in a cubical vessel of volume V, each side of
the cube being I cm. Let the number of gas molecules be II, mass of each
molecule be m and the velocity be u cm/sec. The velocity can be resolved
into three components ux' uy and U z along the three axes X, Y and Z, mutually
at right angles to one another. These components will be related to the velocity
(u) by the expression,
u2 =ux+uy+u
2 2 2
z .•• (1)
For the sake of simplicity, consider one molecule moving with a velocity
U x along the X-axis striking the faces, A and A'. On striking the wall A, it will

rebound in opposite direction with the same velocity. Therefore,


Momentum of the molecule before collision = mux
Momentum of the molecule after collision = m (- ux ) = -mux
:. Change in momentum per collision =(mux) - (-mu x ) = 2mux-
After striking the wall A, the molecule will have to travel a distance of
I cm before striking the wall A' the molecule will strike the wall A in II Ux
sec. In other words, the number of collisions suffered by the molecule per
second will be ux/i on the walls A and A'.
:. Change in momentum per molecule per second along X-axis
ry
Ux2mu;
=2mu x - = - -
x I I
Similarly, we can derive expressions along Y- and Z-axes.
:. Change in momentum per molecule per second along Y-axis
_ 2mu~
- I
Change in momentum per molecule per second along Z-axis
2
= 2muz

:. Total change in momentum per molecule per second


2 2 2mil:2 _ 2m 2
_ 2mux2 :::!!!!!2 2.
~
2 _ 2111u-
- I + I + I - l (u x + uy + uJ - I
_Total change in momentum per second for all the molecules
2mu2 2mnu 2
=-l-xn=-l- ... (2)
GASEOUS STATE S9

According to Newton's second law of motion, the rate of change of


momentum (i.e., change of momentum per second) is the applied force. So,
we have,
2mnu 2
Force=--
1
The gas pressure, by definition, is the force per unit area. Thus,
2 2
Pressure, P = Force = 2mnu / 1 =! . mnu
Area 6/2 3 P
2
1 m1lu 3
P="3v (As volume of cube, V=!) ... (3)

1 2
or PV=3"mIlU ... (4)

This equation is known as kinetic equation of gases. Equation (3) gives


the pressure exerted by an ideal gas.
For 1 mole of a gas, equation (4) can be written as,
1 2
PV=-mNu
3
where N = Avogadro's number.

Problem 3 : (a) Write a short account of kinetic theory of gases. Derive


the equation PV = RT from kinetic considerations. Show how the vllrious
gas laws are consistent with it?
(b) From the kinetic equation derive an expression for the kinetic energy
of one mole of a gas.
(a) [I] Short Account of Kinetic Theory of Gases
See problem 2.
[II] Derivation of Equation PV = RT
According to the postulates of kinetic theory of gases, the kinetic energy
(t mni) of the molecules and absolute temperature (n of the gas are directly
proportional to each other.
Thus, "21 mnu2 oc T or "21 mnu 2 = kT (where, k = constant).
or lx!mnu 2 =kT or !mnu2=~kT
· to2k·metIc
Accordmg 3 . equatIOn
. 0 f gases,
3 PV = 3" mnu 2 i
PV=~kT ... (1)

or PV = ~ k = constant
T 3
60 PHYSICAL CHEMISTRY-I

For 1 g mole of a gas, the value of constant is equal to R, i.e., gas


constant. Therefore,
PV =R or PV=RT.
T
[III] Consistency of Various Gas Laws
(i) Boyle's law: According to it, 'at constant temperature, the
volume of a given mass of a gas is inversely proportional to pressure', i.e.,

Voc p1 , at constant T

or PV = constant, at constant T.
From kinetic equation, we have,
1 2 2 1 2
PV=-mnu =- X-mnu
3 3 2
The kinetic energy of the gas, E = ~ mnu
2

2
PV=-·E ... (2)
3
At constant temperature, kinetic energy (E) of the gas is constant.
PV = constant.
This is Boyle's law.
(ii) Charles' law : According to it, 'at constant pressure, the volume
of a given mass of a gas is directly proportional to its absolute temperature, '
i.e.,
Voc T, at constant P
As already derived, from equation (2), we have
2 2 E
PV =3 . E or V =3 . P
At constant pressure,
V = constant X E or VocE
But we know that E oc T, where T is absolute temperature.
.. Voc T
This is Charles' law.
(iii) Avogadro's hypothesis: According to it, 'equal volumes of all
gases under same conditions of temperature and pressure contain equal
number of molecules '.
For any two gases, the kinetic equation can be written as,
1 2 2 1 2
PI VI ="3 mllllul ="3 X 2" mlll\U\
1 2 2 1 z
Pz V2 = "3 11121l 2U 2 ="3 X 2" m21l 2UZ
=G~AS~E~O~U~S~ST.~A~~=E ___________________________________ 61

When pressures and volumes of the gases are the same, i.e., PI = P2
and VI = V2 , it follows that,

2"I mInIuI2 = 2"I m2n2u22 ••• (3)

When the gases are also at the same temperature, their mean kinetic
energy will also be the same, i.e.,
121 2
2" mIuI = 2" m2u2 ... (4)

Dividing equation (3) by (4), we get,


ni =n2
This is Avogadro's hypothesis.
(iv) Graham's law of diffusion: According to it, 'the rate of diffu-
sion (r) of a gas is inversely proportional to the square root of the density (d)
of the gas, at constant pressure', i.e.,
I
r oc Td ' at constant pressure
From kinetic equation, we have

pv=imnu2 or u=~=~
mn _ Total mass of the gas - d
(As V - Vi I
.
- enslty 0
f th
e gas,
d)
oume
Therefore, at constant pressure,

The rate of diffusion (r) of the gas is directly proportional to the mean
velocity (u) of the molecules, i.e., u oc r.
I
rocTd'
This is Graham's law of diffusion.
(v) Dalton's law of partial pressures: Suppose ni molecules each
of mass m) of a gas A are contained in a vessel of ,:,olume V. Then, according
to the kinetic theory, the pressure, PA of the gas A will be given by,
1 mInIuI
PA=3"-V-
Now, suppose n2 molecules, each of mass m2 of another gas Bare
contained in the same vessel at the same temperature and there is no other
gas present at that time. The pressure, PB of the gas B is then given by,
2
1 m2n 2u2
PB=3"-V-
62 PHYSICAL CHEMISTRY-I

If both gases are present in the same vessel at the same time, the total
pressure, P is given by,
1 mllllUr 1 m2n2u~
P=3-V-+3-V-=PA+PB
Similarly, if three, four or more gases, e.g., A, B, C, D, etc are present
in the same vessel, the total pressure is given by,
P=PA +PB+PC+PD+ ...
This is Dalton's law of partial pressures.
(b) Kinetic Energy of One Mole of a Gas
According to kinetic equation of gases, for one mole of a gas,
1 2
PV = 3 mNu (where N = Avogadro's number)

2 1 2
or PV=-x-mNu ... (5)
3 2
The kinetic energy (E) of one mole of a gas is given by,
1 2
E=-mNu
2
Thus, from equation (5),
2
PV=3. E=RT

3
m E=jRT

For n mole of an ideal gas, the kinetic energy will be given by


(312)nRT.
Problem 4 : From kinetic equation, calculate the ratio of specific heat at
constant pressure to specifIC heat at constant volume in case ofmonoatomic
gases. How does this ratio vary with molecular complexity of the gas ?
Or, Explain with reason: The value ofy is 1.67 for a monoatomic gas.
(Meerut 2(05)

Or, Explain specific heat of gases at constant volume (Meerut 2(04)

Or, Why does the value of Cp differ from that of Cv?


Or, Show that for an ideal gas, Cp - Cv = R. (Meerut 2(02)

"[1] Molar Heat* of a Gas at Constant Volume (Cv)


It is defined as, 'the quantity of heat in calories required to raise the
~ temperature of 1 mole of a gas through 1° at constant volume'.

* We will use the term molar heat instead of specific heat, as in chemistry it is customary to
use the term molar heat. Morwver,
molar heat = specific heat x molecular weight.
GASEOUS STATE 63

In this case, the volume of a gas is kept constant, so there is no external


work done by the gas when it is heated. In other words, all the heat supplied
to the gas is used up in increasing its kinetic energy. If the temperature of one
mole of a gas is raised through 1°, the increase in its kinetic energy is equal
to the molar heat at constant volume.
For 1 mole of a gas, the kinetic equation is written as,

PV = 1. mNu 2 = ~ .1. mNu 2 = ~ . E = RT


332 3

where, E = kinetic energy per mole at temperature, T =~ mNu2

E=lRT
2
Similarly, kinetic energy (E1) per mole at temperature (T + 1).
3
E1=-R(T+I)
2
Increase in kinetic energy per mole for one degree rise in temperature
is given by,
3 3 3
E1-E=2 R (T+ 1)-2 RT =2 R

C v = 1 R '" 12 x 2 '" 3 calories


2
Thus, the molar heat of a monoatomic gas at constant volume should
be nearly equal to 3 calories.
[II] Molar Heat at Constant Pressure (Cp)
It is defined as, 'the amount of heat required to raise the temperature
of 1 mole of a gas through 1° at constant pressure. '
When the gas is heated at constant pressure, there will be an increase
in its volume, i.e., the gas will expand and do some external work. Thus, some
extra heat (in addition to the heat required to raise the kinetic energy of the
gas molecules) must be supplied to the gas to enable it to do the external
work. Thus, at constant pressure, the heat supplied to the gas is utilised in
two ways:
(a) In increasing the kinetic energy of the gas molecules, which is equal
to Cv.
(b) In doing external work during expansion.
So, the value of Cp is always greater than Cv•
Therefore, Cp = Cv + Work done ... (1)
The work done by the gas during expansion is calculated as follows :
* When a gas expands at constant pressure, the work done (W) is given by.
W = Pressure x Change in volume
64 PHYSICAL CHEMISTRY-I

Suppose 1 mole of a gas* is enclosed in a cylinder containing friction-


less piston. Let its pressure, volume and temperature be P, V and T, respec-
tively. For 1 mole of a gas,
PV=RT ... (2)
When the temperature is raised to (T + 1) at constant pressure P, let the
volume increases to VI' Thus,
PVI =R(T+ 1) ... (3)
From equations (2) and (3), we get
PVI - PV = R (T + 1) - RT = R
or P (VI - V) =Pressure x Change in volume = R
:. Work done, W =R
From equation (1), therefore,
3 5
Cp = Cv + R = -2 R + R = -2 R ::::: 5 calories.
Thus, the molar heat of a monoatomic gas, at constant pressure is nearly
5 calories.
The molar heat ratio (y) is defined as the ratio ofmolar heat at constant
pressure and molar heat at constant volume. For a monoatomic gas, molar
heat ratio is given by,
5
Cp 2R 5
y=-=-=-= 1.66
Cv lR 3
2
[III] Variation of y With Molecular Complexity of a Gas :
In monoatomic gases, the heat supplied to the gas is utilised only in
increasing the kinetic energy of the molecules. But in case of polyatomic
gases (whose atomicity is more than one) the heat supplied is utilised in not
only increasing the kinetic energy of the molecules but also in increasing their
rotational and vibrational energies (intramolecular energy), say x. Conse-
quently, more beat is needed to raise the temperature of the gas through 1°.
Thus,
Cp = %R + x and Cv = ~ R +x
5
C2R+x
Y=i!=-3--
v -R+x
2
The value of x varies with the complexity of the gas molecules.
For monoatomic gases, * x = 0

* Inert gases, vapours of elemenlS,


GASEOUS STATE 65

For diatomic gases,** x = R,


5 7
2R+R 2R 7
Y= = - = - = 1.40
lR+R 1R 5
2 2
. . gases *** x = -3 R
For tnatomlc , 2
5 3
2R+2R 4
y= =-= 1.33
lR+1R 3
2 2
For polyatomic gases, the value of y is still less. In fact, y decreases as
the molecule becomes more and more complex.
[IV] Difference Between the Values of Cp and Cv
At constant pressure, the heat supplied to the gas is used in increasing
the kinetic energy of the molecules as well as in doing work during expansion
of the gas against external pressure. Moreover, at constant volume, the heat
supplied is used only in increasing the kinetic energy of the molecules. Thus,
at constant pressure the gas requires more heat to raise its temperature by
1° than at constant volume. So, the value ojCp is always greater than C v by
an amount equivalent to work done by the gas. In other words,
Cp = Cv + Work done = C v + R
or ~-~=R
Problem 5: Explain the distribution of molecular velocities. How will you
verify the distribution of molecular velocities experimentally?
Or, Write a short note on Maxwell's law.
We know that all the gas molecules do not travel with the same velocity.
This is because the molecules are colliding with one another quite frequently
and so their velocities keep on changing. Maxwell worked out the distribution
of molecules between different possible velocities by using probability
considerations. According to him. the distribution of molecular velocities is
given by,
dn r _ 4 (~)312 -M/12RT . c2dc ... (1)
- 1t 2- D • e
n JIJ\.T,

*** C02, NzO, H20 vapours etc,


66 PHYSICAL CHEMISTRY-I

where, dnc is the number of molecules with velocities in the range c and
c + dc, n is the total number of molecules of the gas, M is the molecular weight
of the gas, T is the absolute temperature, c is the lower value of velocity for the
selected velocity range, dc is the velocity interval for the velocity range, e is a
mathematical constant, base of natural logarithm, whose value is 2.71.
DiVIding equation (1) by dc, we get

~ . dll c _
n
(JL) -
dc - 4n 21tRT
3/2

.e
M/I2RT
.c
2
... (2)

:: '"
<I)

"3
u
<I)

'0
'.'
:::8
.....0 :;':
r--. ;';.
<I)
.. ~
" .
E<I)

.'. ~
<I)
p...
: '-'
:
.'.
....
.'.
·1
~ :::.

ii . . . . . . . . . . . . . . . ~. 2. '"""b;~.:M:::~ V"'~ty.;if.;;..;t!
This is the usual form of the equation for Maxwell's law of distribution
of velocities.
The left hand side of expression (2) gives the probability, p of finding
molecules having the velocity c. This is virtually (not exactly) the fraction of
the molecules having the velocity c. Knowing the molecular weight of the
gas, we can calculate the fraction of the molecules having any particular
velocity c, at any desired temperature.
The general form of distribution of molecular velocities is shown in
figure 2. Fractions (percentages) of the molecules having various velocities
are plotted along Y-axis and velocities are plotted along X-axis. The actual
curve depends upon the molecular weight of the gas and temperature, but the
general form of the curve is the same for every gas.
From the curves, it is seen that the fraction of molecules having veloc-
ities greater than zero goes on increasing with increase in velocity, reaches a
maximum value and then begins to fall towards zero again for very high
velocities.
GASEOUS STATE 67

The validity of the Maxwell's distribution law can bt: checked by an


apparatus shown in figure 3. A beam of meta'i atom is created by heating a
metal like silver or bismuth in an' oven. The two discs are mounted on a
rotating axis. The slits in the discs are displaced by a known angle so that the
slit in one disc comes in the path of vapour ~olecules coming out of the slit
m second disc after an interval of time determined by the rate of rotation of
the axis. Particles moving with different velocities can be collected in the
detector by adjusting the rotation rate of the diSCS. On plottmg a function of
number of particles collected in the detector and the velocities of the partic\e~
as given by rotation rate of discs, we obtain curves as shown in figure 2.
:','y..;.:.::;.... ::;.:::':.:;.:'::::.;:-::-::': :. :':-. .. . ::.,:-:'::... .:..... .:.::.:-......:':. ::-:- '\..
'': Pin-Hole r---------------,-------, :.:'

• • I:~-v~--~ ~ol~c,:t!a! - -~;:o~~-


Velocity
------
! :Detector
0
:::; Selector I
I
.:;' I

:::: Evacuate} Chamber


C:~:.::-:::::?::.;::: .. ·...... . .... .... '" ......... "'.::.:.:::.y::':.:
.:.:.....:.:.:.:.:.:.:... ::: Fig. 3. Apparatus for testing Maxwell's distribution law. .. :: : .....
.::::.:'::::;:::;:;:;::.::;::; ... ... ......... :. '::'.;':;: :.;. :<:.

Problem 6 : Define the terms average velocity, root mean square velocity
and most probable velocity of molecules and how are they related to each
other? (Meerut 2006)
Or, Find out a relation between RMS velocity and average velocity.
(Meerut 2005)
Or, Define root mean square velocity a1ld average velocity. (Meerut 2007)

(i) Average velocity (v): It is defined as, 'the average of the


velocities of all molecules at allY time'.
If UI, u'}., U3, '" Un are the velocities of individual molecules in a gas and
n is the total number of molecules con tamed in a gas, then the average velocity
is given by,
V = UI + U2 + U3 + ... + Un
~
Il
Root mean square velocity or R.M.S. velocity (u)*: It IS
(ii)
defined as, 'the square root of the mean of squares of all velocities of the
molecules '.

* Mean square velocity. u


. 2 . .
IS given by, i = l(uh 111 + II~ + ... + u~ I!
Il
68 PHYSICAL CHEMISTRY-I

2+ 2+ 2+ ... + 2)
~(
. _ UI U2 U3 Un
RM.S. velocIty -
11

(iii) Most probable velocity (a. or u): It is defined as, 'the velocity
possessed by the l1laximum number of molecules of the gas '.
The relationship of most probable velocity to the average velocity and
root mean square velocity can be developed from the following expressions:

Average velocity, v = " ( : - )

Root mean square velocity, u


= "( 3~TJ
Most probable velocity, a. = ~( 2~TJ
Thus,
Average velocity, 'v = 0.9213 X RM.S. velocity, u
Most probable velocity, a. = 0.8164 x RM.S. velocity, u
From the above equations, it can be seen that the ratio of the three
velocities are given by :
a : v : u = I : 1.128 : 1.224
Problem 7 : From kinetic equation oj gases, how will you calculate the
root mean square velocity oj gas molecules under different conditions?
From kinetic gas equation, we can calculate the root mean square
velocity of gas molecules under different conditions of temperature, pressure,
etc., as shown below.
[I] When only temperature is given
For 1 mole of a gas we have, from kinetic equation

PV=.l mNu 2 = RT (': PV= R7)


3

or u="C::)=~(3~T)
where, \ mN = M =molecular weight

8.31~X 10 T)
7
.. u = "(3 X X

or II
4
= 1.58 X 10 X " ( ~)
Knowing the value of T, we can calculate u.
GASEOUS STATE 69

[II] When both pressure and temperature are given


From kinetic equation, we have for 1 mole of a gas,
1
PV=-mNu 2 J
=-Mu- "
(M= molecular weight)
3 3

or u=-V(3~VI ... (1)


The value of V can be determmed !rom N.T.P. considerations, i.e.,
PV _ PoVo
T-r;
(where, Po, Vo and To correspond to N.T.P. values)
PoVo T
or V=--x-
To P
Thus, the value of V can be calculated and substituted in equation (1).
[III] When both pressure and density are given
For 1 mole of a gas, the kinetic equatIon is written as,

PV=~mN1l2=~MU2 or u~-vC~V)=-V(~)
[As d = MIV = density]
Knowing the values of P and d, we can calculak u.
Problem 8 : What are real and ideal gases? In what respect does a real
gas differ from an ideal gas? (Meerut 2004)

Ideal or Perfect Gas


A gas which strictly obeys Boyle's and Charles' laws at all temperatures
and pressures is known as an ideal or perfect gas. The characteristics of an
ideal gas are as follows :
(i) The compressibility (Z) of an ideal gas is unity, i.e.,
Z= PV = 1
nRT
(ii) The product of pressure and volume of an ideal gas at constant
temperature is constant.
(iii) If an ideal gas is cooled at constant pressure, its volume decreases
continuously, till at -273°C its volume becomes zero.
(iv) If it is allowed to expand without doing any external work, it shows
no thermal effect.
(v) There is no force of attraction between molecules of an ideal gas.
Differences between an Ideal and Real Gas
(i) A real gas does not obey various gas laws particularly at low
temperatures and high pressures. Only at high temperatures and low pressures,
real gas tends towards ideal behaviour.
70 PHYSICAL CHEMISTRY-I

(ii) The volume of a real gas does not become zero at -273°C. In fact,
when cooled sufficiently, a real gas is suddenly converted into liquid state.

Problem 9 : (a) What are the limitations of the equation PV = RT? Show
in what respects vander Waals equation is an improvement over the simple
gas equation. Derive vander Waals equation and discuss it or write the
drawbacks of this equation.
(Meerut 2004, 03, 01, Garhwal 2006, 02, 2000,
Kanpltr 2005; Agra 2006, 01.2000)
Or, Discuss vander Waals equation of state in detail. (Meerut 2005)
Or, Explain vander Waals equation along with volume correction and
pressure correction. (Meerut 2007)
(b) What are the units of vander Waals cOllstants. (Meerut 2003, 2000)
(c) Show that effective volume of the gas molecules is four times
greater thall the actual volume of the molecules. (Meerut 2002)

(a) Limitations of the Equation PV = RT


The general gas equation, PV = RT derived from the postulates of the
kinetic theory is valid for an ideal gas only. Real gases obey this equation
only approximately and that too at high temperature and low pressure. The
higher the pressure and lower the temperature, the greater are the deviations
from the ideal behaviour. In general, the more easily liquefiable and highly
soluble gases exhibit larger deviations. Thus, gases like CO 2, SO:! etc. show
much larger deviations than N:!, H2o O2 etc.
-.

..

i
...'"
(1)
(1)
>, A
c:>.
'"
J
~
(1)

~
>-Po.
..
15
200 400 600 800
P-Atmosphere _
GASEOUS STATE 71

Regnault and Amagat studied the effect of pressure on volumes of gases


like H 2 , He and CO 2 etc. at constant temperature and the curves are as shown
in figure (4). According to them, if Boyle's law IS obeyed, the values of PV
for a given mass of a gas should be constant at all pressures and the graph
should be a straight line parallel to the pressure axis, as shown by dotted line.
But, it is only an ideal behaviour which no real gas will exhibit. Hydrogen
and helium at all pressures are less compressible than Boyle's law requires,
i.e., PV increases with increase in pressure at constant temperature right from
the very beginning. On the other hand, CO 2 and other gases at low pressures
are more compressible than Boyle's law requires. i.e., PV decreases wIth
increase in pressure at constant temperature. This continues with an increase
in pressure till PV passes through a minimum at a certain stage. With further
increase in pressure, the compressibility is less than expected, i.e .. PV in-
creases with increase in pressure and this contInues thereafter.
At low temperatures, the deviations are much more pronounced than at
high temperatures.
[I] Improvements by vander Waals
In order to explain the deviations of real gases from ideal behaviour
vander Waals suggested that it is necessary to modify the kinetic theory of
gases. The following two postulates of the kinetic theory, according to him.
do not appear to hold good under all conditions.
(i) The volume occupied by the gaseolls molecules is negligibly small
as compared to the total volume 0/ the gas.
This postulate can be justified only under ordinary conditions of tem-
perature and pressure. At low pressure or high temperature, the volume of the
gas is comparably large and so the small volume occupied by the gas mole-
cules can be neglected without producing any appreciable error. However, at
high pressure or low temperature, the volume of the gas becomes small and
now even the small volume occupied by the molecules cannot be neglected,
as the molecules are incompressible. Hence, under conditions o/high pressure
and low temperature, the above postulate is flOt valid.
(ii) The mutual force of attraction between gaseolls lIlolecules is neg-
ligible.
This postulate also holds good at low pressure or hIgh temperature.
because under these conditions, the volume of the gas is large and the
molecules lie far apart from one another. But at high pressure or at low
temperature, the volume of the gas IS small and the molecules lie c\o:o.er to
one another. Thus, the interlllolecularforces ofattraction canllot be neglected.
So, at high pressure and 101V temperature, the above postulate is also not
valid.
So, it is necessary to apply suitable corrections to the ideal gas equatIOn
to make it applicable to real gases. vander Waals introduced two correction
terms in the ideal gas equation to account for the errors introduced as a result
of neglecting the volume of the molecules and intermolecular forces of
attraction.
72 PHYSICAL CHEMISTRY-I

(i) Volume correction: For an ideal gas, PV = RT, where V is the


total volume occupied by 1 g mole of the gas. As the molecules are incom-
pressible, the volume occupied by them remains the same irrespective of
pressure. vander Waals suggested that a factor b should be subtracted from
V, the total volume in order to get the ideal volume, which is compressible.
Thus, the volume which is compressible is not V but (V - b). So, Ideal
volume = V-b.
The factor b is constant and is characteristic of each gas. It is known
as co-volume, effective volume or excluded volume. It has been shown that
the co-volume is nearly 4f2 times the actual volume occupied by all the
molecules contained in the gas.
(ii) Pressure correction: Consider a molecule lying somewhere in
the middle of the vessel as shown in figure (5). It is attracted uniformly on
all sides by other molecules. These forces neutralise each other and hence
there is no resultant force of attraction
on the molecule. However, as the
molecule approaches the wall of the
containing vessel, it experiences
force of attraction which tends to drag
it backwards. Hence, it will strike the
wall with a lower velocity and will
exert a lower pressure than it would
have done if there were no forces of
attraction at all. Thus, it is necessary
to add a certain quantity to the pres-
sure P of the gas in order to get the
ideal pressure. So,
Ideal pressure = P + p'
where, p' is the pressure due to intermolecular forces of attraction. The value
of p' depends upon (i) the number of molecules per unit volume in the bulk
of the gas, i.e., directly on the density of the gas and (ii) the number of
molecules striking the wall at any given time which in turn also depends upon
the density of the gas. Thus, correction factor p' is proportional to the square
of the density (d) of the gas or is inversely proportional to the square of
volume (V) [As, density oc l/Volume]. So,
I a
or p = -2
V
where, a is a constant depending upon the nature of the gas and is known as
coefficient of attraction Thus,
a
Ideal pressure = P + 7:
V
Introducing the above two corrections in the gas equation PV = RT, we
get the following equation:
GASEOUS STATE 73

(p+ ;2)(V-b)=RT ... (1)

This is known as vander Waals equation. The constants a and bare


also known as vander Waals constants.
For n moles of a gas, the vander Waals equation becomes :

n:~ }V-nb) =nRT

i
p+ ... (2)
2 2 \
':poc nd oc n naJ
v2 =7
2 2

[II] Discussion of vander Waals Equation or Drawbacks


of vander Waals Equation
We can now explain the departure of real gases from ideal behaviour
at different pressures and temperatures as shown in figure (4) on the basis of
vander Waals equation.
(i) At low pressures: When the pressure is low, the volume will
be sufficiently large and b can thus be easily neglected in its comparison.
Thus, the vander Waals equation (1) becomes,
p + ~)
2
V = RT or PV + !!:.. = RT
( V V
a a
PV = RT- V = (YV}J(leal - V
The product PV i~,less than the ideal value by an amount equal to a/V.
As P increases, V decreases or a/V increases and so PV becomes smaller and
smaller. This explains the dip in the curve of CO2 etc. at low pressures.
(ii) At high pressures: When the pressure is very high, the volume
V is quite small. It is now not possible to neglect b. But as P is large, the
factor a/V can be neglected in comparison to the high value of P. Thus,
equation (1) becomes,
P (V-b) =RT or PV- P. b=RT
or PV = RT + P.b = (PV)ideal + P . b
Thus, PV is greater than the ideal value of PV by an amount equal to
P.b. As P increases, the product P.b increases and so PV increases. This
explains why the value of PV after reaching a minimum increases with further
increase of pressure.
fiii) At high temperatures: If at a given pressure, the temperature
is very high, the volume of the gas will be sufficiently large to make the
value of a/V2 negligibly small. Under this condition, the value of b can also
be neglected in comparison to V. So, vander Waals equation approaches ideal
gas equation. This explains why the deviations are less at high temperatures.
(iv) Exceptional behaviour of hydrogen and helium: As both
H2 and He have comparatively small masses, the attractive forces of attraction
74 PHYSICAL CHEMISTRY-I

between the molecules become too small. So, the correction term alVz, due
to attraction factor is negligible even at ordinary temperature or low pressure.
Therefore, equation (1) becomes,
P (V - b) =RT or PV =RT + P.b =(PV)ideal + P.b
This explains the rise in the curves of Hz and He with an increase of pressure
right from the very beginning even at ordinary temperatures or low pressures.
(b) Units of vander Waals' Constants
From equation (3), the constant a is expressed by PVzII?, i.e., pressure
x (volume)2/moI2. If pressure is expressed in atmospheres and volume in
cm 3 the value of a will be atm cm6 moCz. If volume is expressed in litre (or
dm 3 ), the value of a will be atm L2 mol-2 or atm dm 6 mol- 2. In Sl system.
the unit of a will be
(Nm-2) (m 3)2 4-2
ry or Nm mol.
(molt
The constant b is incompressible volume per mole of a gas so, it will
have the same units as volume per mole, e.g., cm3 moC I , L mor) or
dm 3 mol-I. In SI units, the unit of b will be m 3 mol-to
(C) Effective Volume of Gas Molecules
vander Waals suggested that a
correction term nb should be sub-
tracted from the total volume V in Excluded
order to get the ideal volume which is Volume
compressible. In order to understand
the meaning of the term lib we con- ,,
I

sider two gas molecules as unpenetr-


able aM incompressible spheres each
of which has a diameter, 1", as shown
in figure (6). It is clear that the centres ....
of the two spheres cannot approach .....
each other more closely than the dis- .; ;::,:":':~
•.. ::;.;:::
.•.;:::
•.•.:;:::-.::;:
••••;•. ;:;:
.. ;:.• ;:::
.•.;:: .. +=:':::::::::::C":::::.:-:::'·.:·':'?:·:·(t.\~
.•;:;:

tance r. For this pair of molecules. ;: :;:;: :/: : ,: : : : : : :;. . ~.i~: .~. . .r:::::-:':::}:.:':':':' ..:
therefore, a sphere of radius r and of
vo Iume 3"4 1[r3 constItutes
. wh
at 'IS
known as excluded volume or co-volume. The excluded volume per molecule
is thus half the above volume, viz., ~ 1[,.3. The actual volume of one gas
· r I 'IS "3
mo IeeuIe 0 f ra dIllS 4 /1
1[r·. S 0,

4
31[r'
':1 4 (r}3 =61[1"
=31[l2
1 3
GASEOUS STATE 75

2
:. Excluded volume per molecule == 31tr
3

1 3
== 4 X -1tr
6
== 4 x Actual volume of the gas molecules.
Problem 10: What do you understand by critical temperature and critical
pressure in relation to vander Waals' equation and critical phenomenon.
Calculate the values of critical constants in terms of vander Waals' con-
stants. How are the values of critical constants determined experimentally?
(Meerut 2002; Agra 2005,03,01; Rohilkhand 2005,01; Kanpur 2006)
Or, Starting from vander Waals' equation find the values of critical con-
stants.
Or, Write a short note on continuity of state. (Meerut 2002, 2000)
Or, Define critical temperature. (Meerut 2(06)
The curves which are obtained by plotting pressure against volume at
various constant temperatures are known as isotherms (isos == equal;
therm:::: heat). Andrews obtained isotherms of carbon dioxide at different
temperatures which are shown in figure (7).
First consider the isotherm ABCD at 12.1 Dc. The point A represents
CO 2 in the gaseous state occupying a certain volume under certain pressure.
On increasing the pressure, the volume of the gas decreases along AB in
accordance with Boyle's law until at a certain pressure (at B) liquefaction
occurs. Further, decrease of volume is not accompanied by any change of
pressure, as shown by horizontal portion BC, until the vapour has been
condensed completely at C. The liquid is only slightly comp;'essible, so further
increase of pressure produces only a very small decrease in volume. This is
shown by a steep line CD which is almost vertical. Thus, along the curve AB,
CO 2 exist as a gas. Along BC, it exists partly as gas and partly as liquid.
Along CD, it exists completely as liquid.
At higher temperature (21. 1°C). a similar isotherm EFGH is obtained.
It differs from the first isotherm ABCD in two respects. viz .• (i) liquefaction
starts at a higher pressure and (ii) length of the horizontal portion of the curve
becomes shorter. With further rise in temperature, the horizonal portion grad-
ually decreases until it is reduced to a point J and the isotherm becomes
continuous as seen in the isotherm UK at 31.1°C. So. there is no horizontal
portion of the curve and no sudden change from the gaseous to the liquid
state. At this temperature (31.1 ° C), the gas passes into the liquid state without
any visible separation of one phase from the other. This is called continuous
transition of state. The idea of continuity from the gaseous to liquid state can
be explained from the isotherm UK. The change at J shows no sharp discon-
tinuity but a continuous transition occurs during the conversion. Above
31.1°C, the isotherms are continuous and there is no evidence of liquefaction
at all.
76 PHYSICAL CHEMISTRY-I

Andrews observed that if the temperature of CO2 was above 31.1°C it


cannot be liquefied, whatever the pressure may be. Other gases also behave
similarly. So, for every gas there is a limit of temperature beyond which it
cannot be liquefied, whatever high pressure may be. This limit of temperature
is known as critical temperature (Tc) of the gas. The pressure required to

liquefy a gas at critical temperature is called critical pressure (Pc)' The


volume occupied by one mole of a gas at critical temperature and critical
pressure is known as critical volume (Vc)' The point of inflexion (1) is called
the critical point. The isotherm passing through J, is called the critical
isotherm. This phenomenon is called critical phenomenon.
vander Waals' Equation and Critical State or Calculation
of Critical Constants
For one mole of a gas, vander Waals' equation,

(p+ ~2)cV-b)=RT
may be simplified as,
a ab
PV-Pb+-V- v2 -RT=O

Multiplying throughout by V2 and dividing by P, we get


GASEOUS STATE 77

V3 _ b V 2 + a V _ ab _ RW == 0
p p p
Arranging in descending powers of V, we get
V3 -(b+ RT) 2
p V +aV p == 0
p _ ab ... (1)

This equation is cubic in V and as such there may be three real roots
or one real and two imaginary roots of V for each value of P and T. In other
words, for given values of P and T, there will be either three real values or
one real and two imaginary values of V. This behaviour is not shown by
isotherms of CO2 in figure (7).
At 51° and 31.1°, there is only one volume for each pressure. At
12.1°, there are two different values of V corresponding to points Band C for
the same pr~ssure. However, the third volume predicted by equation (1) is
missing.
By substituting the experimental values of a and b in equation (1),
Thomson (1871) calculated the values of V for different values of P and T.
He plotted these calculated values of V against P and the isotherms obtained
by him are shown by dotted curves in figure (7). The isotherms for temperature
31.1° and above are exactly of the same form as obtained by Andrews.
However, the theoretical isotherms below the critical temperature differ from
experimental isotherms (of Andrews). They have no sharp breaks and the
horizontal portIons of the curves are replaced by wave like portions. For
example, the experimental isotherms ABCD and EFGH are replaced by
theoretical isotherms ABB 1B 2B 3CD and EFF j F2F 3GH, respectively. In these,
there are obviously three volumes represented by B, B2 and C (and F, F2 and
G), corresponding to one pressure as predicted by equation (1). As the tem-
perature rises, the wave portion of the curve becomes smaller and smaller and
the three values of volume get closer and closer until they merge into one
point J at 31.1 ° (critical temperature). At J, the three roots of V (say x, y and
z) of equation (1) are identical. Since the temperature is critical, the value of
V represents the critical volume of the gas, i.e., V == V,.. The three values of
V can be represented as,
(V -x) (V- y) (V - z) == 0,
At critical point, x==y==z== Vc
(V- vi ==0
Expanding and writing in decreasing powers of V, we get
• .3
V - 3 vy2 + 3 V~2 V - Vc3 == 0 ... (2)
This equation must be identical with vander Waals' equation (1) at
critical temperature and pressure, which may be written as
78 PHYSICAL CHEMISTRY-I

,.3 ( RTc) 2 aV ab
V - b + Pc V + Pc - Pc = 0 " .(3)
Since equations (2) and (3) are identical, the coefficients of equal
powers of V in the two equations must be equal to one another. Therefore,
RTc
3V(=b+ p ... (4)
r

3V2=~ ... (5)


c Pc

V3 = ab ... (6)
c Pc
Dividing equation (6) by (5), we get
Vc= 3b ... (7)
Substituting the values of Vc in equation (5), we get
P =~= a a ... (8)
c 3~ 3 X (3b)2 = 27b 2
Substituting the values of Pc and Vc in equation (4), we get
=~. PrY,. =~. (a/27b ) x 3b
2
T
(' 3 R 3 R
Sa
or T{,'= 27bR ... (9)

Determination of Critical
Constants
(a) Critical temperature and critical
pressure : These values can be determined
by a simple method which is based on the
principle that at the critical temperature, the
surface of separation, i.e., meniscus between
the liquid and the vapour phase disappears.
It is generally used when the substance is in
liquid state at ordinary temperatures.
The experimental liquid is taken in a
vessel V enclosed in a glass jacket J. The
temperature of J can be varied gradually by
circulating a suitable liquid from a thermo-
stat. The vessel V is connected to a mercury
manometer M containing air. The tempera-
ture is first lowered so that the vessel is
cooled and the surface of separation between
the liquid and its vapour becomes sharp. The ig.8. Apparatus for determining
temperature of the jacket is raised slowly. t;:l-_ _ _ T.;.:<o..;a;;;;n.;;;.d.;;.Prr.-_ _--u
This rise is continued till the meniscus be-
GASEOUS STATE 79

tween the liquid and its vapour just disappears. This temperature say t~ is
noted. The jacket is then cooled slowly till cloudiness due to the condensation
of vapours appears again. This temperature t~ is again noted. The critical

temperature will thus be C' ; (


2
).

The mean of the pressures read from the manometer M, corresponding


to temperatures t~ and t~ ; gives the value of critical pressure.
(b) Critical volume: The determination of Vc takes the advantage of
the observations made by Calliatet and Mathias that when mean values of the
densities of liquid and
saturated vapour of a
substance are plotted Density
against the correspond-
ing temperatures, a
straight line is obtained.
In figure (9), the
i
curves VC and LC show
the plots of the densities
of saturated vapours and
those of liquid against
the corresponding tem-
peratures. The point C,
where the two curves
meet gives the critical
temperature. This point
is not sharp as the curve
in this range is rather flat.
Therefore, the mean densities are then plotted against different temperatures
when a straight line MC is obtained. The point C where this line cuts the
curve VCL, gives the critical temperature as the density of the liquid now
becomes identical with that of vapour. The point C gives the critical density
of the substance. The critical volume is obtained by dividing the molecular
weight of the substance by the critical density.
Problem 11 : (a) Show in what aspects vander Waals equation is an
improvement over the simple gas laws ?
(b) Calculate the values of vander Waals constants a and b and gas
constant R in terms of critical constants..
(c) Calculate the values of constants (a, b) of vander Waals equation in
terms of critical constants and show that,
9 1 RTc
a=-RTV and b = - - (Meerut 2(}()2)
8 C C 8 Pc
80 PHYSICAL CHEMISTRY-I

(a) (i) vander Waals' equation explains the deviations of gases from
Boyle's law. Regnault and Amagat showed that gases at low temperatures and
high pressures do not obey the simple gas laws.
(ii) vander Waals' equation explains the critical phenomenon, whereas
simple gas laws do not.
(b) In problem 10, we had derived the following three equations:
RT,.
3V,.=-+b ... (1)
Pc

V~ = ab ... (3)
Pc
Dividing equation (3) by (2), we get
V,
Vc::: 3b or b=3 ... (4)

From equation (2), we get a = 3P,.v; ... (5)


Substituting the values of a and b in equation (1), we get,
RTc Vc
3V = - + -
c Pc 3
RTc V,. 8
or -=3V"--3 =-3 Vc
P"
8 Pc V,.
or R=-·- ... (6)
3 Tc
(c) From equation (6),

Squaring both the sides,


64 :z R2~
-V=- or
9 c p;c
Substituting the value of V; from equation (2), in the above equation,
we get
a 9R2r; 27 R2r;
3P = 64P; or a = 64P,. ... (7)
c

Substituting the value of V from equation (4) in equation (1), we get,


RI;.
3x3b=-+b
Pc
GASEOUS STATE 81

RTc
or 8b=- or ... (8)
Pc
From equations (4) and (8),

3RTc
or PC=8l1
c
Substituting the value of Pc in equation (7), we get
27 x 8R2~Vc
a = --:--=-==-=--
64 X 3RTc
9
or a =gRT,yc

1 RT,.
and b =- . - [From eq. (8)J
8 Pc
Problem 12 : Write short notes on the following:
(1) Various equations of state.
Or, Kammerling Onnes' equation. (Meerut 1989)
(2) lAw of corresponding states and its applications.
(Meerut 2004, 01; Kanpur 2005,02, Rohilkhand 2004,02, Garhwal2006, 03, 2000)
(3) Mean free path.
(4) Critical phenomenon and its utility.
(5) Collision frequency.
(6) lAw of equipartition of energy.
(7) Specific heat ratio. (Meerut 2004, 02)
(8) Boyle temperature. (Meerut 2(01)
(9) Continuity of state. (Meerut 2002, 2000)

1. Various Equations of State


vander Waals equation has been found to be accurate over a wide range
of pressure and temperature. Near the critical point, it does not gives satis-
factory results. Several other equations have been proposed to express
P-V-T relationship in accordance with the experimental observations. These
equations of state are:
[I] Clausius equation: This is vander Waals equation modified by
Clausius to account for the variation of vander Waals constant a with-tem-
perature. It is given by,

p+ a 2](V-b)=RT
[ T(V + c)
82 PHYSICAL CHEMISTRY-I

where. c is a new constant. This equation is fairly satisfactory but does not
hold good for all gases.
rIl] Dieterici equation: Dieterici introduced an exponential factor
to account for the effect of molecular attraction on the pressure. His equation
is given by.
P (V - b) = RT. e-aiRTV
where. e is the base of natural logarithm. This equation gives more satisfactory
results at high pressures than the vander Waals equation.
[III] Berthelot's equation: Berthelot proposed the following empir-
ical equation to explain the behaviour of real gases.

(p+ ;2)(V-b)=RT

[IV] Kammerling Onnes' equation or virial equation: Kammerling


Onnes gave an empirical equation which gives the product PV as power series
of the pressure at any given temperature. The equation is represented as :
PV = A + BP + cr
+ Dp +... 3

The factors A. B, C and D etc. are known as first, second, third, fourth
etc. virial coeffreients (Greek: virial = force).
At low pressures, the coefficient A (which is equal to RD is important
and others cancel out. At increasing pressures, other coefficients become
significant.
[V] Beattie-Bridgemann equation: This empirical equation is given
by:

PV=RT+~+~+ ~ +...
where, p, y, 0 are also called virial coefficients.
2. Law of Corresponding States
vander Waals showed that if P, V and T of a gas are expressed in terms
of critical pressure, critical volume and critical temperature of a gas, we obtain
another important generalisation, known as law of corresponding states.
P V •
Let, -=n
P('
-=cj>
, V('

and L=9
1'..
where n, cj> and 9 are known as reduced pressure, reduced volume and reduced
temperature, respectively. Therefore,
P=nPc ; V=cj>V(' and T=9T,,,
Substituting the values of P, V and T in vander Waals equation, we get,
GASEOUS STATE 83

(p+ 0 }(V-b)=RT

we get, (npc + ~ 2) «Wc - b) = RaTc


<I> V~
Substituting the values of Pc, Vc and Tc in the above equation, we get.

[
n .~
27b
+ a2 2]
<I> (3b)
8a
(<I> • 3b - b) = R a . 27 b R

or 2~~2 (n + :2)<3<1> - 1) =Ra . 2~~R


or (n+ :2}<3<1>-I)=8a ... (1)
The above equation known as vander Waals reduced equation of state
does not involve vander Waals' constants as well as gas constant, hence it is
a general equation applicable to all substances.
From equation (1), if two substances have the same reduced temperature
(a) and the same reduced pressure (n), they will have the same reduced volume
(<1». This statement is known as law of corresponding states. In other words,
two or more substances having the same reduced temperature and same
reduced pressure and thus having the same reduced volume, are said to be in
corresponding states.
Importance : While studying the relationship between physical
properties and chemical constitution of various liquids, their properties should
be studied at the same reduced temperature as pressure has practically no
effect on liquids. It is seen that the boiling point of a liquid on absolute scale
is nearly two-thirds of its critical temperature. Various liquids at their boiling
points are thus very nearly in corresponding states and to study their physical
properties at the same reduced temperature, these should be studied at their
boiling points.
3. Mean Free Path
The average distance travelled by a molecule between two successive
collisions is known as its mean free path. It is denoted by the symbol I
or A.
If v be the average velocity of the molecule and z be the number of
collisions suffered by a single molecule per second, then the average distance
covered by a molecule be~ween two successive collisions, i.e., I, is given by
1= !'..
z
The value of z has been shown to be equal to (-.1"2) nv 0'2111 ; where, 0' is
the collision diameter and nl is the number of gaseous molecules per cm 3.
84 PHYSICAL CHEMISTRY-I

1= v ... ( 1)
..J(2). nva2nl ..J(2).na2nl
For n g mole of an ideal gas

PV=nRT=!!J..RT
N
where, N = Avogadro number.

Therefore, from equation (1)


1= RT ... (2)
..J(2). na2 PVN
Thus, at constant pressure, the mean free path is directly proportional
to absolute temperature. Thus, higher the temperature, the greater is the mean
free path. Similarly, at constant temperature, the mean free path is inversely
proportional to the pressure. Thus, lower th~ pressure, the greater is the mean
free path. The mean free path is also inversely proportional to the viscosity
of the gas.
The mean free paths for H2, O2 and He are 11.2 X 10-6,6.43 X 10-6 and
18.0 x 1O-6cm., respectively.
4. Critical Phenomenon and its Utility
For critical phenomenon, see problem 10. It is utilised in the liquefaction
of gases.
5. Collision Frequency
The number of collisions between molecules per second per unit volume
(ml) of a gas is known as the collision frequency.
From kinetic theory of gases, it can be seen that if the number of
molecules per millilitre of a gas be N, then the number of molecules with
which a separate gas molecule will collide with be given by,
z' = fin v a 2N
where v = average velocity of the molecules in cm s-I; a = collision diameter
incm.
:. Total number of molecules colliding per millilitre per second,
z =z' xN= (..J2)nv~N2
As each collision involves two molecules, the number of molecular
collisions per millilitre per second will be z/2 i.e.,
Collision frequency,

N
z 1 2 2
=-=~·nvaN
2 '-12
c
So, collision frequency of a gas increases with increase in temperature,
molecular size and the number of molecules per millilitre.
GASEOUS STATE 85

6. Law of Equipartition of Energy


According to Maxwell and Boltzmann, the law of equipartition of
energy can be defined as, 'the energy given to a gas is equally distributed
amongst each degree of freedom. The degree of freedom is the system to
represent the probable velocity of a molecule.
For mono-atomic gases: In case of mono-atomic gases like He, Ar
etc. the molecules have only translational motion, i.e., they move in one
direction only. This velocity can be resolved along three axes at right angles,
so the system will have three degrees of freedom. The kinetic energy of the
gas will be ~ mu2 which can be resolved along 3 - axes, i.e., x, y, z.

1212121 2
"2 mu
=2"mux +2"muY +2"muz
According to law of equipartition of energy,
1.2 mux2 = 1.2 mti.y = 1.2 mu z2 = 1.2n
RT

(AS total kinetic energy = t mi =~ RT)

For each degree of freedom of velocity, the energy will be t RT per

mole. The total energy is ~ RT. so the molecular heat capacity will be ~ R.
For di-atomic gases: A gas where molecule has more than one atom
will have rotational energy and vibrational energy besides the translational or
kinetic energy. For di-atomic molecule, this velocity will be only in one
direction, i.e., parallel to the axis of the particle. So, such a molecule possesses
two degrees of freedom, one for potential energy and the other for kinetic
energy.
Now consider the rotational motion. Due to collision with other mole-
cules, the molecule will rotate about its own centre of gravity. Assuming the
velocity to be in one plane, it can be resolved along two axes and each axis
will have one degree of freedom.
Therefore, for a di-atomic gas molecule, the translational energy will
have four degrees of freedom, two due to rotation and two due to vibration.
If the molecule is rigid, there will be no degree of freedom due to vibration.
So, the energy will be %RT.
Molecular heat capacity = %RT "" 5.0 calories

The above value is valid at ordinary temperature when the vibrational


energy is very less. When vibration is more at a high temperature, then the
molecular heat capacity becomes nearly equal to 7.0 calories/mole.
86 PHYSICAL CHEMISTRY-I

7. Specitic Heat Ratio


See problem 4.
8. Boyle Temperature
The deviation of real gases from ideal gases is best explained by means
of compressibility factor (Z). It is defined as,
Z= PV _ PV
(PV)ideal nRT
The value of Z = 1 for an ideal gas at all temperatures and pressures.
When for a gas Z is less than or greater than 1, it shows less or more deviation
from ideal behaviour.
So, the temperature of the gas at which the value of PV remains
constant, so that Boyles' law is fully obeyed, is known as Boyle's temper-
ature (Tb)'
If the temperature is less than Boyles' temperature, the value of Z first
decreases and then reaches a minimum value and finally as the pressure is
gradually increased, the value of Z starts increasing. Different gases have
different Boyle temperatures. For hydrogen and helium, Boyle temperatures
are -80oe and -240oe, respectively. It means that at -80oe, hydrogen obeys
Boyles' law within a maximum range of pressure.
For a real gas, Boyle temperature is given by the equation,
a
Tb= bR
where 'a' and 'b' are vander Waals constants.

9. Continuity of State
See problem 10.
Problem 13 : Mention the various methods that are adopted for producing
cold and what is the lowest temperature hitherto attained? How has it been
reached? Show how these have been used in the liquefaction of gases ?
Mention the importance of liquefaction. What is inversion temperature.
(Meerut 2002,2000; GarhwaI2001; Rohilkhand 2002)

For liquefaction of any gas it is necessary to cool it below its critical


temperature and then apply high pressure. Following are the methods used
for producing cold and for liquefying gases.

1. By using freezing mixture and volatile refrigerants: This


method is based on the principle that when a volatile compound evaporates,
it absorbs its latent heat of vaporisation from the surroundings and conse-
quently the temperature becomes low. A mixture of solid e0 2 (dry ice) and
others can produce a temperature of -IOOoe by evaporating under reduced
GASEOUS STATE 87

pressure. Difluorodichloro methane or freon (CF2CI 2) is also used for pro-


ducing low temperatures.
2. By Joule-Thomson effect: When a compressed gas at a particular
temperature is allowed to expand through a porous plug the temperature falls.
During expansion, work is done in overcoming the intermolecular forces of
attraction. This work is done at the expense of heat energy and the temperature
of the gas falls. Thus, when a gas under high pressure is allowed'to expand
into a region of low pressure, it suffers a fall in temperature. This phenomenon
is known as louie-Thomson effect.
Experiments have shown that gases become cooler during loule-
Thomson's expansion only if they are below a certain temperature known as
inversion temperature (T,). The inversion temperature is characteristic of
each gas and is given by,
2a
T,= b.R
where, a and b are vander Waals' constants. At inversion temperature, there
is no louie-Thomson effect. In other words, if a gas under pressure passes
through a porous plug and expands adiabatically into a region of low pressure,
then at inversion temperature, there is neither a fall nor rise in temperature.
If the expansion occurs below the inversion temperature, there is a small fall
in temperature and if it occurs above the inversion temperature, there is a
small rise in temperature. In most cases, this temperature lies within the range
of ordinary temperatures.
Hydrogen and helium have very low inversion temperatures, i.e.,
-80°C, and -240°C, respectively. Thus, at ordinary temperatures, these gases
get heated up instead of being cooled in loule-Thomson's expansion. But, if
hydrogen is first cooled below -80°C and helium below -240°C, then these
gases also get cooled
down on louIe-Thom-
A~,.--_-,
son expansion. loule-
Thomson effect was
used by Linde in the
liquefaction of the
gases. The Linde's pro-
cess is described
below:
The gas or air
freed from impurities
and moisture is com-
pressed to a high pres-
sure and enters the
inner tubes of concen- g~rntj[F!i~g:.~l~O:::~~!J~:;~mm@tj
tric pipes at 0 as shown [
in figure (10) under a
88 PHYSICAL CHEMISTRY-I

pressure of 200 atm. The valve V in J is then opened to allow it to expand


suddently into a wide chamber C. It thus gets cooled and its pressure is reduced
to about 50 atm. The cooled gas is then made to pass through the outer tube
B and cools the incoming gas passing through the inner tube A. Thus, the
incoming gas gets further cooled on expansion into the wide chamber C. The
gas is compressed again to 200 atm, and made to pass through the inner tube
and again expanded into chamber C. This process continues till the gas or air
is liquefied and liquid gas comes out of the jet J.
3. By performance of external work: When a gas is allowed to
expand against an external pressure, it does work against pressure. If the
system is a closed one and is so insulated that neither heat can enter nor leave
the system, it is said to undergo adiabatic expansion. During such an expan-
sion, work is done by the molecules against external pressure at the cost of
their own kinetic energy and so the temperature falls.
This principle combined with Joule-Thomson effect has been used by
Claude for the liquefaction of air. Thus, Claude's process (fig. 11) is as
follows:
Air dried and freed from
CO 2 is compressed to about 200
atm, and passed througn the
pipe ABC which divides at C.
li/:":::"I
Some of the air enters the cyl-
inder D provided with aIL air j

I
y

tight pIston. Here the gas ex-


pands and does external work
in moving the piston outwards.
The temperature of the gas thus I
falls. The cooled air enters the
chamber E from below and then
goes up as shown. Thus, the gas
cools the portion of the com-
pressed air passing down the
coil CEo This chilled gas then
passes through a jet or throttle
J and is further cooled by Joule-
Thomson effect on account of
expansion. This process goes on till the gas is converted into the liquid state.
4. By adiabatic demagnetisation: It is based on the principle that if
a paramagnetic substance is magnetised and then demagnetised adiabatically,
the temperature falls. On applying the magnetic-thermal effect to the gas,
Giauque produced as Iowa temperature as 0.0033 K. By the method of nuclear
magnetls~tion it has been possible to attain a temperature of 0.001 K.
GASEOUS STATE 89

Importance of liquefaction : Liquefied gases or gases compressed


under high pressure are finding important uses in industries. These are :
(i) Liquid CO 2 is used in soda fountains.
(ii) Liquid NH3 and liquid S02 are used as refrigerants.
(iii) Liquid CI 2 is used for bleaching and disinfecting purposes.
(iv) Compressed He is used in airships.
(v) Liquid air is an important source of O2 in rockets and jet-propelled
planes and bombs.
(vi) Compressed O 2 is used in welding.
IMPORTANTFORMULAE1~______________~

1. Kinetic energy of an ideal gas, E = ~ nRT

where n = number of moles, R =gas constant, T =absolute temperature.


(Note : E will have the same unit, in which R is expressed).
2. Molecular velocity

(a) When only T is given: u = 1.58 X 10 x


4
%
where, M = molecular weight of the gas.

(b) When both P and T are given: u = ~ 3~V


PV PoVo
The value of V is calculated from, T = To
where, Po, Vo and To refer to N.T.P. values. The values are
Po =76 x 13.6 x 981 dyne cm-2, Vo =22400 cm 3, To = 273 K.

(c) When both P and d are given: u = ~


3. Molar heats of gases
(a) Molar heat = Specific heat x Molecular weight
(b) Cp=Cv+R

(c) y = ~ = 1.66 (mono-atomic), 1.40 (di-atomic), 1.33 (tri-atomic)


v
4. For n moles of a gas, vander Waals' equation is

(p + n~ ) (V - nb) = nRT.
The units of a and bare lit2 atm mole-2 and lit mole-1, respectively.

5. Critical constants, V" = 3b, P" = 2;b2 ' T" = 2~b


90 PHYSICAL CHEMISTRY-I

The value of R is generaIly expressed as 0.0821 lit atm deg- I mole-I.

6. Inversion temperature, Tj = ~

7. Diameter of a molecule is given by, b = 4N (~nr3)

where, r = radius of the molecule


:. Diameter = 2r.

NUMERICAL PROBLEMS
Ex. 1 : (a) Calculate the kinetic energy of two moles of CO 2 at 300 K,
assuming it to be an ideal gas.
(b) Calculate in ergs the kinetic energy of a mono-atomic gas (molecular
weight 20.2) molecule, moving with a velocity of 5 x 1(/ cm/s.
Solution: (a) The kinetic energy (E) of 2 moles of an ideal gas is given
by:

E=lnRT
2
3
E="2 x 2 x 8.314 x 300 = 7482.6 J.

(b) The kinetic energy


1 2
= -2 InU =-21 X 6.02320.2 42
~3 x (5 X 10 )
x 10-
= 41.92 x 10-5 erg molecule-1
Ex. 2 : Calculate the velocity of amelecule of oxygen at N.T.P. and 500·C.
Solution: The velocity of a molecule is given by (When only temper-
ature is given) :

where, M = Molecular weight of O 2 = 32.


(a) At N.T.P., T = O·C = 0 + 273 = 273 K.

U -\113"21
= 1.58 x 104 x - F273)

= 46,149.4 cm per sec.


(b) At 500·C, T= 500 + 273 = 773 K

U· 1.58 X [0' x "C3 7


;) • 77,655.4 em per sec.
GASEOUS STATE 91

Ex. 3 : The density of hydrogen at O·C and 760 mm pressure is


0.00009 g / cm3• Find the RMS velocity and the average velocity ofhydrogen
molecules (density of Hg = 13.6 g/cm 3).
Solution: In this case, the values of density and pressure are given.
Therefore, the RMS velocity (u) is given by

u=v'e:)
p = 760 mm = 76 cm = 76 x 13.6 x 981 dynes/sq cm and D = 0.00009
g/cm 3•

=- '{3 x 76 x 13.6 x 981} = 183 844 2 /


u -" 0.00009 ' . em s
The average velocity (v) is given by :
v = 0.9213u = 0.9213 x 183,844.2 = 169,375.6 em/so
Ex. 4 : (a) Calculate the RMS velocity of oxygen molecules at 2S·C.
(Meerut 2005)
(b) Calculate the RMS velocity of hydrogen gas at 27·C.

Solution: (a) We know that, u = 1.58 X 10 x


4
~(~)
4
u = 1.58 X 10 x v'( 8
2;2 )
I
= 4.8 X 104 ems-

7
_ - '(3 x 8.314 x 10 x 298)- 4 8 104 -I
U - -" 32 - . x em s
T = 25 + 273 = 298 K; M = 32
= 1.58 X 104 x ~(~) = 1.58 X 10 ~(3~)
4
(b) u x

= 19.35 x 104 em S-I


Ex. 5 : (a) Cal~te the RMS velocity ofoxygen molecules at a temperature
of 27·C and 740 mm pressure.

(b) Find out the RMS velocity for CO 2 gas at 12·C and 78 cm pressure.
Solution: (a) The RMS velocity is given by

... ( 1)

-2
p= 74 x 13.6 x 981 dyne em ; M=32
92 PHYSICAL CHEMISTRY-I

The value of V can be calculated from NTP concept, i.e.,


u =~3 x 78 x 13.6~ 981 x 22785 =40145.22 emfs
PV PoVo
T=r;
P= 74 cm, T= 273 + 27 = 300 K, V=?; Po= 76 cm,
Vo == 22400 C.c., To = 273 K.
74 x V 76 x 22400
300= 273

V == 76 x 74
22400 x 300 == 252806
x 273 • . C.c.

Thus from (1). U == ~l3 x 74 x 13.6 ~;81 x 25280.6) == 48372.6 em/so

(b) Calculate as in part (a)


For C02>M==44, P==78cm, T== 12+273=285K
V == 76 x 22400 x 285 == 22785
78 x 273 cc.

u = - /3 x 78 x 13.6 x 981 x 227850 == 40145.22 cmfJ


" 44
Ex. 6 : Calculate the temperature at which root mean square velocity of
S02 molecules is the same as that of O2 molecules at 27°C.
Solution: The RMS velocity (Ut) of O2 molecules at 27°C.

== 1.58 x 10
4
~(33020J
The RMS velocity (lI2) of S02 mblecules at T K

== 1.58 X 10
4
~F)

. 1.58 X 10' ~( ~)~ 158 X 10' ~em


or L = 300 or T == 300 x 64 == 600 K =327°C
64 32 32
Ex. 7 : Calculate the RMS velocity of O2 molecules at NTP (Density of
O2 =0.001429 g/c.c. at N.T.P., Density of Hg =13.6g/c.c.;
g =981 cm.lsec2).
Solution: The RMS velocity of O 2 molecules when P and density are
given is expressed as :
GASEOUS STATE 93

U
=-
-"
'(3P)
d
=_
-"
'(3 x 76 x 13.6 x 981) =46137 6
0.001429 • em s
-1

Ex: 8 : The density of O2 at O'C and at 1 atmosphere pressure is


1.429 g/cm 3• Calculate the mean square velocity of the gas at O·C.

S I ti •
o u on. u"
= - '(3P) =-
d -"
'(3 X 76 X 13.6 X 981) = 145899
1.429 • ems
-1

Ex. 9 : Calculate the RMS velocity of a carbon dioxide molecule at 1000·C.


(Meerut 2006, 05)

Solution: RMS velocity, u = 1.58 X 10


4
~(~)
= 1.58 X 10
4
~C~3}=84,985.5 em s-1

"" 8.5 X 104 em s-1


Ex. 10 : Calculate the root mean square velocity and average velocity in
em S-1 of H2 gas at O·C and 760 mm pressure, if its density is
9 x 10-5 g/ml.

· u = ~(3P)
· : RMS veIoClty
So Iu t Ion - = ~(3 x 76 x 13.65x 981)
d 9 X 10-
4
= 18.38 X 10 em S-1
Average velocity =0.9213 xu = 0.9213 x 18.38 x 104 em S-I
= 16.93 X 104 em S-I
Ex. 11 : Calculate the pressure exerted by 1023 gas molecules each of mass
10-22 g in a container of volume 1 Utre. The RMS velocity is Iff cm/sec.
22
Solution : Weight of 1023 gas molecules = 1023 x 10- g = 109
23
. 23 10 x 6.023 X 10
:. Weight of 6.023x 10 gas molecules = 23 = 60.23
10
:. Molecular weight, M = 60.23,
Let pressure = P cm

u=~3~V
5 /
10 cm see
=- '[3 x (P x 13.6 x 981 dynes/cm2) x 1000
-" 60.23
ml]
_ (105)2 X 60.23 _
or P - 3 x 13.6 x 981 x 1000 -15,040 em.
94 PHYSICAL CHEMISTRY-I

Ex. 12 : The specific heat at constant volume of argon is 0.075 and its
molecular weight is 40. How many atoms are there in its molecule?
Solution: Molar heat at constant volume (CI')
= Specific heat x Molecular weight
= 0.075 x 40 = 3
Cp =Cv +R=3+2=5
Cp 5
Y= C
v
=3" = 1.66
Argon is thus monoatomic. i.e., there is one atom in its molecule.
Ex. 13 : The specific heats ofa gas at constant pressure and constant volume
are 0.125 and 0.075 cais, respectively. Calculate the molecular weight of
the gas.
The specific heat and molar heat are related as :
Molar heat = Specific heat x Molecular weight
.. Cp=cpxM andCv=cvxM andCp-Cv=R
Therefore, Cp =0.125 M and C v = 0.075 M
.. 0.125M - 0.050M = R
2
or 0.050M = 2 or m = 0.05 = 40

Ex. 14 : The specific heat of a gas at constant volume is 0.152 and that at
constant pressure is 0.215. Calculate the atomicity and molecular weight
of the gas. Also name the gas.
Solution: Molar heat = Specific heat x Molecular weight
.. Cp=cpxM and Cv=cvxM
.. Cp =0.215M and C v =0.152M
Cp -C v =2
0.215M - 0.152M =2 or 0.063M =2
2
or M = 0.063 = 31.74

Y= Cp = 0.215 = 1.41
Cv 0.152
.'. Molecular weight of gas = 31.74
Atomicity = 2
Name of the gas = Oxygen.
Ex.1S : The specific heats ofa gas at constant volume and constant pressure
are 0.090 and 0.45 calories. If the atomic weight of the gas be M, calculate
its molecular weight.
GASEOUS STATE 95

We know that,
Molar heat = Specific heat x Molecular weight
Cp 0.125
Y= C = 0.090 = 1.38:::: 1.40
v
So, the gas is diatomic and so its molecular weight = 2 x M
.. Cp =0.125 x 2M =0.250M
and Cv =0.090 x 2M =0.180M
Cp-Cv=R
0.250M - 0.180M = 2 or 0.070M = 2
2
or M =0.070 = 28.57
:. Molecular weight of the gas = 28.57
Ex. 16 : Calculate the constants a and b, if Tc =31"C, Pc = 72.8 atm and
R = 0.082 lit atmldeg.
The values of Tc and Pc are given by:
P =_a_ ... (1)
c 27b 2
8a
Tc= 27bR ... (2)
Dividing (2) by (1), we get
Tc 8b 304 8b
- = - or - - - - -
Pc R 72.8 0.082
.. b = 0.0428 lit mole-1
Substituting the value of b in equation (1), we get,
72.8 = a
27 X (0.0428)2
or a = 3.6 Iit2 atm mole-2
Ex. 17 : The critical temperature and critical pressure of ethane are 305.1
K and 48.1 atm respectively. Calculate its vander Waals constants 'a' and
'b'.
Solution: As in Ex. 16, we can write,
Tc 8b 305.1 8b
- = - or - - = - -
Pc R 48.1 0.082

or b = 305.1 x 0.082 = 0 065 I·t 1-1


48.1 x 8 • I mo e

Substituting the value of b in the equation Pc = ~ , we get


27b
a
48.1 = 2'
27 x (0.065)
2 2
or a = 5.48 Iit atm mole-
96 PHYSICAL CHEMISTRY-I

Ex. 18 : The critical temperature and critical pressure of oxygen are


-U8.8°C and 49.7 atm. Calculme its vander Waals constants.
The values of Tc and Pc are given by
Sa
r..= 27bR ... (1)

P =_a_ ... (2)


c 27b2
Dividing (1) by (2),
2
Tc Sa 27b Sb
-=--x--=-
Pc 27bR a R
Tc = -l1S.S + 273 = 154.2 K; Pc = 49.7 atm.
154.2 =~ or b = 154.2 x 0.OS2
49.7 0.OS2 49.7 x S
= 0.0318 lit mole-1
Substituting the value of b in equation (2), we get
49.7 = a 2 49.7 x 27 x (0.031S)2
:::;!

27 x (0.031S)
=1.357 8tm Iit2 mole-2
Ex. 19 : The vander Waal\' constants in lit atm/mole of CO 2 are : a = 3.6,
b = 4.28 X 10-2• Calculate Tc and V(Jor the gas.
The values of Vc and Tc are given by,
Vc= 3b ... (1)
Sa
Tc = 27bR ... (2)
2
From (1), Vc = 3 x 4.2S X 10- = 3 x 0.042S = 0.1284 lit

From (2), Tc = 27 x 0.~2~·: 0.OS2 = 303.9 K = 30.9°C


Ex. 20 : Two gases obey vander Waal\" equation, their constants al and
a2 are equal but b I and b2 are different. Which of the gases under identical
conditions wiD occupy more volume?
Solution: Under similar conditions of temperature and pressure, vander
Waals' equation for two gases 1 and 2 can be written as :

P +!!l.. (VI - b l ) = RT
~
(V2 - b 2) =RT
When al = a2 , then VI - b l = \/2 - b2
The gas which has higher value of b will occupy more volume.
GASEOUS STATE 97

Ex. 21 : One mole oj NH3 is enclosed in afive litre flask at 27°C. Calculate
the pressure oj NH3 using vander Waals equation. For NH3, a = 4.17 atm
lif mole-2 and b = 0.0371 lit mole-to
Solution: vander Waals' equation is given by,
(p+ ~ )cV-b)=RT
Substituting the values, we get

. (p + 4;~7) (5 - 0.0371) = 0.0821 x 300

or (25P ;54.17 )<4.9629) = 0.0821 x 300


1 [0.0821 x 300 x 25 ]
or P = 25 0.9629 - 4.17 = 4.71 atmosphere
Ex. 22 : From vander Waals equation, show that
3
PcVc=SRTc
Solution: vander Waals equation under critical conditions is written as:

pc + ~ )(V- b) = RT('
or V3 - (RTc a V--=O
b+- V2 +- ab
Pc Pc Pc
\
At critical point, the three values of volume are identical and equal to
Vi.' Thus, .
(V- vi=o
or V3 - 3 Vc y2 + 3 ~V - V~ = 0 ... (2)
As (1) and (2) are identical. therefore.
3Vc =b+
Rt ... (3)
pc
... (4)

... (5)

Dividing (5) by (4), we get


Vc=3b

or b= Vc ... (6)
3
Substituting the value of b in equation (3), we get
Vc RTc Vc RTc
3V = - + - or 3V - - = -
c 3 Pc c 3 Pc
98 PHYSICAL CHEMISTRY-I

9Vc - Vc RTc 8 RTc


or 3 = Pc or "3 Vc = Pc
3
or PcVc=iRTc

Ex. 23 : If vander Waals' equation of a gas is given by :

(p + 0.~;86) (V - 0.00224) =0.0041 (273.16 + t)

Calculate the values of Tc and Pc of the gas.


Solution : The values of a, b, Rand T are given by :
a =0.00786, b =0.00224, R =0.0041, T= (273.16 + t)
The values of Pc and Tc are given by:
(i) P =_a_2 = 0.00786 =58.01 atm.
c 27b 27 (0.00224)2
(1·1·) T 8a 8 x 0.00786 253 5 K.
c = 27bR = 27 x 0.00224 x 0.0041 = .
Ex. 24 : 2 moles of CO2 at 27°C is filled in a five litre fliJsk. Calculate its
pressure using the following:
(i) Gas equation, (ii) vander Waals equation.
For CO2 : a =3.6 atm l;r mole-Zj R =0.082 lit atm, b =4.28 x 10-z
lit mole-I.
Solution : (i) Pressure from gas equation:

PV=nRT or P= ~T
P = 2 x 0.082
5 x 300 =9.84 a tmosph eres.

(ii) Pressure from vander Waals' equation:

(p + n;)(V - nb) = nRT n


(where = No. of moles = 2)

(p + 4 ~;.6) (5 _ 0.0428 ~ 2) = 0.082 x 300 x 2 .

or (25P + 14.4) = 0.082 x4~9~~ 25 x 2

or 25P= 0.082 x3oox25 x2 -144


4.9144 .

or P = 235.8 =9.43
25
Pressure = 9.43 atmospheres.
GASEOUS STATE 99

Ex.2S : Calculate the pressure using (i) vander Waals' equation and (ii)
ideal gas equation of 10 g ammonia contained in a 1 litre flask at O·C.
(a = 4.17 atm litr; mole-", b = 0.0371 litre mole-I)
Solution: (i) Using vander waals' equation

(p + n~: ) (V - nb) =nRT


p + ~~ 2 (4.17)]
or [ 12 [1-(!~)<0.0371)]=(~~)<0.0821) (273)
or (P + 1.442) (1 - 0.0218) = 13.1843
or (P+ 1.442) (0.9782) = 13.1843

or P = 13.1843
1442
.0.9782
= 12.036 atm
(ii) Using ideal gas equation
PV=nRT

or P x 1= (~~) (0.0821) (273)'


or P = 13.18 atm
Ex. 26: 0.5 mole of CO2 was filled in a vessel of volume 0.6 litre at 47·C.
What pressure would be expected on the basis of vander Waals equation?
(a =3.36 atm luJ mole-", b =4.27 X 10-2 lit mole-I; R =0.082 lit atm
deg- I mole-I.
From vander Waals equation,

(p + n;) (V - nh) = nRT.


or [p + (0.5)2(0.6)x ;.36] (0.6 _ 0.5 x 0.0427) = 0.5 x 0.082 x 320
or (P + 2.33) x 0.57865 = 13.12
13.12
or P =0.57865 - 2.33 =20.34 atmospheres
Ex. 27 : At 27°C, the volume of 1 mole of CO is 1.5 litre. If R is taken in
litre atmosphere and values of a and bare 1.5 atm lit mole-2 and 0.04 lit
mole-I, respectively, then what will be the pressure of the gas according to
(a) ideal gas equation and (b) vander Waals equation?
Solution: (i) From ideal gas equation:
100 PHYSICAL CHEMISTRY-I

PV==nRT
p==nRT == 1 x 0.082 x 300
V 1.5
or P == 16.4 atmospheres
(ii) From vander Waals equation:

(p+ ~2)(V-b):::::RT
or p + ~21 (l.5 - 0.04) == 0.082 x 300
( (1.5) )

or lp /5) x
-I- 1.46::::: 0.082 x 300

or P == 0.082 x 300 _ _
1 == 16.183
1.46 1.5
Pressure == 16.183 atmospheres.
Ex. 28 : For NO gas, vander Waals' constant, b == 0.02788 lit/mole, calcu-
loJe the diameter of NO molecule. (N::::: 6.023 X Itf.1)

Solution: We know that b == 4N [ 1nr3 ]

10 x ~ x 3.14 x r3
23
or 27.88 == 4 x 6.023 X (b == 27.88 cc/mole).
3
or r3 == 2.76 X 10- cm 24
or r == 1.4 X 10-8 cm.
:. Diameter of NO molecule == 2 x 1.4 X 10-8 == 2.8 X 10-8 em.
Ex. 29 : For hydrogen the vander Waals' constants a and bare 0.246 and
2.67 x 10-1 in lit atmosphere units. Calculate the inversion temperature for
hydrogen.
2a
Solution : We know that. Ti== b.R
2 x 0.246
or T, == 0.0821 x 0.0267 ::::: 223.6 K
Inversion temperature. T; == 223.6 K or - 49.4°C

MULTIPLE CHOICE QUESTIONS


1. A gas cannot be liquefied if for a gas the temperature is greater than:
(i) Critical temperature (ii) Critical pressure
(iii) Critical volume (iv) Critical density
2. At constant pressure the specific heat of a gas (molecular weight::::: 80) is 0.125.
It is :
GASEOUS STATE 101

(i) Mono-atomic (ii) Di-atomic (iii) Tri-atomic


3. The first virial coefficient of Kammerling Onnes equation at low pressure is :
(i) PV (ii) RT (iii)-b (iv) ~T

4. The unit of vander Waals constant 'Q' is :


(i) mole atm- I (ii) atm mole-I
(iii) Iit2 atm mole-2 (iv) atm 1it-2 mole 2
5. If the rms velocity of a gas is 100 cm s-I, its average velocity is :
(i) lOOcms- 1 (ii) 0.01 cm s-I
(iii) 92.l3cms- 1 (iv)81.64cms- 1
6. The conpressibility factor of an ideal gas is :
(i) Zero (ii) Infinite (iii)2.0 (iv) 1.0
7. The temperature at which the second virial coefficient of a real gas is zero. is
known as:
(i) Critical temperature (ii) Boiling point
(iii) Boyle temperature (iv) Freezing point
8. The virial state equation for an real gas is :
8' C'
(i) PV=RT+-y+ V2 + ...

. B' C'
(iii) PV=RT+Rr+R'f3 + ... (Iv)PV=RT+-y+-y+ ...
9. At 2TC the kinetic energy of 2 moles of an ideal gas is :
(i) 1800 cal (ii) 900 cal (iii) 600 cal (iv) 3600 cal
10. The following is called a permanent gas :
(i) NH3 (ii) N2 (iii) H2 (iv) CO2
11. The ratio of most probable, average and root mean square velocity is :
(i) I: 1.224 : 1.128 (ii) 1 : 1.128 : 1.224
(iii) 1 : 1.128: 1.524 (iv) 1 : 1.178: 1.524
12. The vander Waals equation for n moles of a real gas is :
(i) (p + ~) (V-b) =nRT (ii) (p + ~) (V- nb) =nRT
(iii) (p + n:~) (V -nb) = nRT (iV>(P + n:v2) (V -nb) = nRT

Fill in the Blanks


1. In general, the more easily liquefiable gases have a .............. Boyle temperature.
2. The average energy of translational motion of gas molecules is proportional only
to the .................. .
3. Real gases behave ideally at .................. temperatures and .................. pressures.
4. At 274 K, rrns velocity 'of methane molecules would be .................. ms- I . .
5. vander Waals replaced the volume term, V in the ideal gas equation by ........ .
6. The intermolecular forces which exist between the neutral molecules are known
as ................. .
102 PHYSICAL CHEMISTRY-I

7. The temperature at which the average velocity of O2 molecules would be twice


that at 20·C is .................. K.
8. Average kinetic energy of gas molecules is .................. to absolute temperature.
9. The Cp/Cv value for a gas is 1.47. If its atomic weight is x, its molecular weight
is ............ .
10. The unit of vander Waals constant a is .................. and b is ................. .
11. The pressure of real gases is less than that of ideal gases because of .............. ..

True or False
State whether the following statements are true (T) or
false (F) ?
1. PV = ±mNi is an expression for the real gases.
2. The molecules of real gases have both volume and mutual attraction.
3; Pressure depends on the number of molecules and temperature.
4. The equation (1t + :2) (341- 1) = 89 is known as the reduced equation of state.
5. The observed pressure of a real gas is less than the pressure of an ideal gas.
6. There are large number of particles in any sample of a gas for which the average
velocity must be zero, whereas the average velocity will always be final.
7. Gases which are difficult to liquefy have a low Boyle's temperature.
8. The vander Waals constant b is the actual volume of gas molecules.
9. Root mean square velocity of gas molecules is always greater than the average
velocity under similar conditions.
10. The ratio of the average molecular kinetic energy of CO2 to that of SF6 both at
300 K is one.

ANSWERS
Multiple Choice Questions
1. (i), 2. (i), 3. (ii), 4. (iii), 5. (iii), 6. (iv), 7. (iii), 8. (i), 9. (i),
10. (iii) 11. (ii) 12. (iii)
Fill in the Blanks
1. high 2. absolute temperature 3. high, low
4.652 5. (V-b) 6. vander Waals forces
7.586 K 8. directly 9. directly proportional
10. x 1l.lit2 atm mole-2, lit mole- 1
12. mutual attraction between gas molecules
True or False
1. (F) 2. (1) 3. (1) 4. (T) 5. (T) 6. (1)
7. (1) 8. (F) 9. (T) 10. (1)

.. 000
CHEMICAL AND PHASE EQUILIBRIUM
CHEMICAL EQUILIBRIUM
Problem 1: Exp1o.in 1o.w of mass action and equilibrium constant.
Guldberg and Waage (1867) found the effect of reacting substances on
a reversible reaction and gave a law known as the law of mass action.
According to this law,
"The rate at which a substance reacts is directly proportional to its
active mass and the rate ofa chemical reaction is proportional to the product
of the active masses of the reactants."
The term active mass of a reactant is the molecular concentration,
i.e., the number of gram moles per unit volume. It is expressed by enclosing
the formula of the substance within two square brackets, e.g., [H2], [HI] etc.
Consider a reversible reaction, A + B ~ C + D
According to the law of mass action,
Rate of forward reaction oc [A][B] = kJ [A][B]
where kJ is the velocity constant for the forward reaction.
Rate of backward reaction oc [C][D] = k2 [C][D]
where k2 is the velocity constant for the backward reaction.
At equilibrium: Rate of forward reaction = Rate of backward reaction
or kJ [A][B] = k2 [C][D]
[C][D] _~_
or [A][B] - k2 - Kc ... (1)

where Kc is known as equilibrium constant in terms of concentration.


In a gaseous reaction, the partial pressure (P) of the gas is proportional
to its molecular concyntration. Therefore, the equilibrium constant in terms
of partial pressures, will be

K =PCXPD ... (2)


P PA XPB

where PA,PB,PC andpD represent the partial pressures of the respective con-
stituents.
(103)
104 PHYSICAL CHEMISTRY-I

Equations (1) and (2) are the mathematical forms for the law of mass
action.
In general, for a reaction
n 1A+n2B + ... ~ m 1C+m2D + '"

we can write,
K = [C]m. [D]mZ ...
C [A]n. [B ]n2 ..•
m. X m,x
Similarly, K = !....P"'-C_P!....D/.!.-_ _
• '-'
P n. X n2 X
PA PH •••

Problem 2: Write short notes on the following:


(i) Workfunction (ii) Free energy
[I] Work Function
We know that the concept of entropy is the fundamental consequence
of the second law of thermodynamics. There are two other functions, which
utilize entropy in their derivations. These two functions are 'free energy
function' and 'work function.' These functions like the internal energy, heat
content and entropy are fundamental thermodynamic properties and depend
upon the states of the system only.
We know that energy can be converted into work, but it is not always
necessary that all the energy may be converted into work. So, any kind of
energy which can be converted into useful work is called 'available energy',
such as the operation of an engine or a motor. But the energy which cannot
be converted into useful work is known as 'unavailable energy'.
:. Total energy = Isothermally available energy
+ Isothermally unavailable energy
The isothermally available energy present in a system is called its free
energy. It is denoted by the symbol G. Free energy is also known as thermo-
dynamic potential or Gibbs' function.
It has also been explained before, that a part of internal energy of a
system can be utilized at constant temperature to do useful work. So, this
fraction of internal energy which is isothermally available is called the 'work
function' of the system. It is denoted by the symbol A. The exact nature of
the functions will be clear as follows :
The function, G is defined by the equation,
G = E - TS + PV ... (1)
where, P, V, T as usual refer to the pressure, volume and temperature of the
system, E is the energy content of the system whose entropy is S. As
E. T and S depend only on the thermodynamic state and not on its previous
history, it is clear that the same views must apply to G, the free energy
CHEMICAL AND PHASE EQUILIBRIUM 105

function. Therefore, G may be regarded as single valued function of the state


of the system. Hence dG, will be a complete differential. As E and S are
extensive properties, therefore. G will also be extensive in character or in
other words, the value of G will depend on the quantity of matter specified
in the system. As E + PV = H, Equation (1) reduces to.
G=H- TS ... (2)
The work function, A is defined by,
A=E-TS ... (3)
where, all the symbols have their usual significance. Like the free energy, the
work function A is also seen to be a single valued function of the state of the
system. Hence, likewise dA is also a complete differential. Similarly. as
E and S are extensive properties, A will also be extensive in nature.

[II] Physical Significance of Work Function


In order to obtain some understanding of the physical significance of
the work function, consider an isothermal change from the initial state to the
final state. The two states are represented by the subscripts 1 and 2, respec-
tively. In such a case, (3) can be written as,
Al = EI - TS I ••• (4)

and ... (5)


The temperature remains the same as the process is an isothermal one.
Subtracting equation (4) from (5), we have,
A2 - AI = (E2 - EI) - T (S2 - SI)
or M=AE-T6.s ... (6)

where, M is the increase in the work function, AE is the corresponding


increase in internal energy and AS is the increase in entropy of the system.
Suppose the change is brought about reversibly at a constant tempera-
ture T, then the heat absorbed will be equal to Qrev (say). Thus, the increase
in entropy is given by,
AS= Qrev
T
Substituting this in equation (6), the result is
M = AE- Qrev ... (7)
From first law of the thermodynamics,
AE=Qrev- W
or - W=AE-Qrev
Combining this equation with equation (7), we have
W=-M ... (8)
106 PHYSICAL CHEMISmY-1

Since the process is carried out reversibly, W represents the maximum


work. It is clear from equation (8) that the decrease in work function (- M)
gives the maximum work that can be obtained from the system at a constant
temperature. It is this fact, which justifies the use of the concerned term as
work function.
[III] Physical Significance of Free Energy
Consider an isothermal change from the initial state to the final state at
constant temperature T. The two states are represented by the two subscripts
1 and 2, respectively as before.
Equation (2), therefore, reduces as follows for both the states:
G 1 = HI - TS, ... (9)
G 2 =H2 - TS2 ... (10)
Subtracting equation (9) from (10), we have
(G2 - G,) = (H2 - H,) - T(S2 - S,)
or !lG = !lH - T !lS ... (ll)
where, !lG and !lH represent the change in free energy and change in heat
content, respectively.
At constant pressure, we know that,
!lH=M+P!lV
!lG = !lE + P !l V - T !lS ... (12)
Combining equations (6) and (12), we have
!lG=M +P!lV
or -!lG = W - P!l V [From equation (8)] ... (13)
Since P!l V represents the quantity of work done by gas on expansion
against the external pressure, therefore, - !lG gives the maximum work which
can be obtained from the system other than that due to change of volume, at
constant temperature and pressure. The work other than that due to change
of volume is caned the net work. Hence,
Net work = W - P !lV
Therefore, -!lG is a measure of the net work. In other words, the
decrease in free energy (- !lG) is a measure of maximum net work that can
be obtained from a system at constant temperature and pressure. G can also
be defined as, 'the fraction of total energy which is isothermally available
for converting into useful work'. This decrease in free energy is of great
importance in chemistry, especially in physical chemistry.
[IV] Variation of Free Energy With Pressure and Temper·
ature
From equation (1), we have
CHEMICAL AND PHASE EQUILIBRIUM 107

G=E+PV-TS
On differentiation, we have,
dG =dE+PdV+ VdP-SdT-TdS ... (14)
From first law of thermodynamics, we have
dE=dQ-dW
If the work is only due to expansion, then dW = P dV
Hence, dE =dQ - P dV
or dQ=dE+PdV

Now we know that, dS= d~rev


or T dS =dQrev =dE + P dV ... (15)
Combining (14) and (15), we have
dG= VdP-SdT ... (16)
A constant temperature, dT = 0
dG T = VdP

or
(OG)
oP
=V
T
.... (17)

Equation (17) represents the variation offree energy with pressure.


When pressure is constant, dP = 0 ..10 such a case (16) reduces to,
dGp=":'-SdT

or (~~)p = - S ... (18)

Equation (18) represents the variation of free energy with temperature.


[V] Change of Free Energy With Pressure for a Perfect
Gas
For an isothermal change, dT= 0, hence equation (16) reduces to
dG=VdP
On integration, we have,
(G2 dG= J,P2 VdP
JGI PI

RT
For 1 mole of perfect gas, V = P

1,G2 dG= J,~2 RT dP


G. PI P
P2
G2 -G 1 = I1G =RTloge PI
108 PHYSICAL CHEMISTRY-I

For n moles of perfect gas,

... (19)

Equation (19) applies to a change occurring at constant temperature for


a perfect gas. It also applies, whether the process is reversible or otherwise.

Problem 3: Give thermodynamic derivation of law of mass action.


Consider the gaseous reaction
A+B~C+D
Suppose there are two large vessels at the same temperature, in each of
which the four substances A. B, C and D are always in equilibrium. Suppose
each of the four walls of the vessels is permeable to only one of the substances.
LetPA,PB,pcandpD be the partial pressures of A, B, C, D in the first
vessel, whereas P'A, P' B, P' c and P'D that of A, B, C, D in the second vessel.
The vessels are supposed to be so large that the transfer of relatively small
quantities of the material from one vessel to the other results in no appreciable
change in the concentrations or partial pressures of the substances. It is also
assumed that an substances are ideal gases.
First a reversible and isothermal process is carried out. Suppose 1 mole
of A is transferred isothermally and reversibly from the first vessel to the
second. This is done by means of the wall permeable to A. If PA and P'A
represent the initial and final pressures, respectively, then the change in free
energy (~G) is given by,
P'
llG=
JPA
A VdP

SO, increase of free energy of the substance A is given by,

llGA =
J~
PA
VdP= J~ -dP=RT
RT
PA
J~ -~
P PA P
=RTlOgp'A IPA
Similarly, for the transfer of 1 mole of B from PB to P'B in the same
direction, the change in free energy is given by,
llGB = RT log P' B IPB
During the time A and B are transferred in one direction, 1 mole of C
and 1 mole of D are transferred isothermally and reversibly in the opposite
direction.
Thus, the free energy change for the transfer of 1 mole of C from
P' c to Pc is given by,
llGc=\RT logPc1p'c
For the transfer of 1 mole of D, we have,
llG D = RTlog PD Ip'D
CHEMICAL AND PHASE EaUILIBRIUM 109

The total change of free energy is given by.


IlG = IlGA + IlG B + IlG c + IlG D
'P'A P' B Pc PD
= RTlog- + RTlog-+ RTlog -,- + RTlog -,- ... (1)
PA PB PcP D
As the vessels are large and the system is in equilibrium at constant
remperature T. the total change of free energy at constant temperature is zero.
i.e .• IlG =0.
, ,
log P A + log P B + log P,C + log P,D = 0 (From equation 1)
PA PB PcP D
, , , ,
PA
or log - + log -PB = log Pc PD
- + log-
PA PB Pc PD
P'AP'B I P'CP'D
or Io g - - = o g - -
PAPB PCPD

or
=
P'A P'B P'c P'D
PAPB PCPD
, ,
PCPD PcPD
or - - = -,-,- = Constant (Kp) ... (2)
PAPB PAPB
As the partial pressure is proportional to the molar concentration. equa-
tion (2) becomes.
CC' CD [C] [DJ _
... (3)
CA' CB [AJ [BJ - K("
The equations (2) and (3) are different forms of law of mass action
which have been deduced thermodynamically.

Problem 4: Gil'e the thermodynamic deril'ation of maximum work ob-


tainedfrom gaseous reactions or l'an't Hoff isotherm.
When a small change is considered occurring in a reversible reaction.
it is assumed that the system is at equilibrium. So. the change in free energy
(1lG) can be put to zero. If. however. the constituents are not at eqUilibrium.
but at certain arbitrary concentrations. there is a definite change of free energy•.
as the reactions get converted into the products. This change in free energy
can be determined by the usual equilibrium box.
Imagine a large box in which the various gases taking part in the reaction
A + B ~ C + D are present at equilibrium. Let the P' terms represent the
equilibrium pressure. It is assumed that all gases are perfect. Then the fol-
lowing steps are carried out :
(i) Transfer 1 mole of A isothermally and reverSibly at arbitrary pressure
PA. into the equilibrium box through the wall permeable to A. Let the final
pressure be p'A- Then the change in free energy of the substance A is given
by.
110 PHYSICAL CHEMISTRY-I

AGA = P'A VdP=RT JP' dP-


- -RTIog- P'A
JPA PA
A
P PA
(ii) Similarly, the change in free energy for the transfer of 1 mole of B
isothermally and reversibly into the box, from pressure PB to final pressure
P' B is given by,
AGB = RTlogp'B IpB
(iii) With the passage of the reactants A and B into the box, it is assumed
that (he products are removed or withdrawn. .
The free energy change in the transfer of 1 mole of C from the box
isothermally and reversibly, when the pressure of the gas C is reduced from
P'c ro Pc is given by,
AGe = RTlogpc Ip'c
(iv) For the transfer of 1 mole of D, the free energy change when the
pressure is reduced from P'D to PD is given by,
AGD= RTlOgPD Ip'D
The rotal change of free energy (AG) for the whole process is given by,
AG=AGA +AGB+AGc+AGD
P'A P'B Pc PD
= RTlog - + RT log - + RTlog -,- + RT log -,-
PA PB Pc PD
, , I ,
PA PB Pc PD
= RT log - + RT log - - RT log - - RT log -
PA PB Pc PD

or aG = RTlog Pc ,PD _ RTlog '!:c ,P:D ... (1)


PA,PA PA,PB
, ,
-u
AG-RTI
-
PCPD - RTIo g
og -,-,- PC,PD
--
PAPB PA,PB
PC,PD
or - AG =RTlog Kp - RT l o g - - ... (2)
PA,PB
where, Kp is equilibrium constant, as the fraction in the second term of
equation (1) on R.H.S. is obviously equal to Kp. The first term on R.H.S. is
also of the same form, but it involves arbitrary partial pressures instead of
'equilibrium partial pressures.
Consider the general equation,
aA + bB + cC + .. , ~ pP + qQ + rR + ...
for which equation (2) can be written as,
p'f,POPR ...
- AG =RTlog Kp - RTlog a b ,.
PAPBPC ...
or - aG = RTlog Kp - RTI.lIlogp ... (3)
whe:-e. I.n log P represents the algebraic sum of all the II log P terms, those
for products being taken as positive and for reactants as negative. Equation (3)
CHEMICAL AND PHASE EQUILIBRIUM 111

is one of the forms of reaction isotherm deduced by van't Hoff (1885). If all
the partiaJpressure terms are replaced by concentration (c) terms, then equa-
tion (3) can be written as,
-I1G = RTlog K,. - RTLn loge. . .. (4)
Similarly, if partial pressure or concentration terms are replaced by mole
fraction (x) tenns, then equation (3) or (4) becomes at constant pressure,
-I1G = RT log Kx - RT Ln log x, ... (5)
So, van't Hoff isotherm gives the increase offree energy change ac-
companying the transfer ofreactants at any arbitrary concentration (or par-
tial pressure) to products at any arbitrary concentration (or partial pressure).
Equations (4) and (5) are other fonns of van't Hoff isothenn. The
maximum work obtained is also given by equations (3), (4) and (5), and often
-I1G is called the affinity of the reaction.

Problem 5: Give the thermodynamic derivation ofvan't Hoffisochore or


van't Hoff equation. Mention its applications also.
[I] Thermodynamic Derivation
The vant Haff isotherm is given by, -I1G = RTlog Kp - RT Ln log p
where all letters have their usual significance.
For a homogeneous gaseous reaction in which the products and reac-
tants are at unit pressures, the last equation becomes
I1G = - RTlog Kp ... (1)
Differentiating equation (1) with respect to temperature, at constant
pressQre, the result is
a(I1G)] = _ R Iog Kp _ RT d log Kp
[ aT p dT

On multiplying equation (2) by T, we get


a(I1G)] ...2. d log Kp
T [ aT p = - RTlog Kp - Rl dT ... (2)

Combining equations (1) and (2), the res~lt is,


a (I1G)] _ _ r d log Kp
• T [ aT p -I1G R dT

or Rrdl~~Kp=I1G-T[a~~G)t ... (3)

From Gibb's-Helmholtz equation, we have


Standard free energy: Standard free energy of a reaction gives the increase of free energy
when the reactants, all in their standard states of unit pressure or unit concentration are converted
at constant temperature (1) into the products, which are also in their standard states. The standard
free energy (AG"T) is given by the expressions,
AGOT = - RTiog K,J
112 PHYSICAL CHEMISTRY-I

AG =AH + T[d ~ATG) 1 .. (4)

From Gibbs-Helmholtz equation (4), equation (3) reduces to,

Rrdl~~Kp AH

dlogKp AH
or ... (5)
dT Rr
Equation (5) represents the variation of equilibrium constant with tem-
perature at constant pressure. This equation is referred to as van't Hoff
reaction isochore (Greek: isochore = equal space), as it was first derived by
van't Hoff for a constant volume system. Since AN is the heat of reaction at
constant pressure, the name isochore is thus misleading. Therefore, equation
(5) is also called as van's Hoff equation.
An alternative form of equation (5) showing the variation of K(. with
temperature and involving the heat of reaction at constant volume is obtained
as follows:
We know that,
... (6)
where, An is the increase in the number of molecules in the gaseous reaction.
Taking logarithms of equation (6) and differentiating it with respect to
temperature, we have,
log Kp = log K(. + An log RT
d log Kp d log Kc + An
... (7)
dT dT T
Combining equations (5) and (7), we have
dlogKc AN An
dT = Rr-T'
d log Kc AN - An . RT
or
dT Rr
dlog Kc !J.E
or ... (8)
dT =Rr
[c.f. thermodynamics, AE = AH - An R71
where, !J.E is the increase of energy or heat of reaction at constant volume.
Equation (8) is another form of van't Hoff isochore or equation.
Integrated Form of van't Hoff Isochore
If AH remains constant over a range of temperature, then on integrating
equation (5), the result is,
CHEMICAL AND PHASE EQUILIBRIUM 113

till t1H
log., Kp =f Rr dT =- RT + constant " .(9)

t1H
10gIO Kp = - 2.303 RT + constant ... (10)

From equation (9), it is clear that there is a linear relation between


log Kp and liT.
On integrating equation (5) between two temperatures T j and T2 at
which the equilibrium constants are Kp and K,:. we have,

10geK'p-loge Kp = - ~ (;2 -;1)


On converting loge to 10gIO, we get

IOglO K'p -IOglO Kp = - 2.:R (;2 -;J .. (11)

Equations (9), (10) and (11) are integrated forms of van't Hoff
isochore.
[II] Applications of van't Hoff Isochore
(i) To fmd the heat of reaction (MJ): It can be obtained by either
of the following two methods :
(a) If values of IOglO Kp are plotted agairist liT, we get nearly a
straight line. The slope of this line at a given point will be equal
to - MJI2.303R. The value of till can thus be measured by
measuring this slope.
(b) The second way to calculate t1H is from equation (11). Knowing
the equilibrium constants at two given temperatures, we may find
the value of t1H.
(ii) van't Hoff isochore is also applicable to find the heat of solution of
a substance from the temperature coefficient of the solubility, the heat of
dissociation of a weak electrolyte from the temperature coefficient of the
dissociation constant etc.
According to equation (5) the equilibrium constant of a gaseous reaction
increases with temperature, if till is positive. An increase in the equilibrium
constant means an increase in the proportion of the resultants, so that if heat
is absorbed in the reaction, an increase in temperature favours the formation
of the resultants. On considering the reverse case, if heat is evolved in the
reaction, an increase of temperature displaces the equilibrium in the direction
ofthe reactants. The behaviour is again in accordance with the Le-Chatelier's
principle.

Problem 6: Give the thermodynamic derivation of Clapeyron equation


and Clausius-Clapeyron equation. Discuss their applications also.
114 PHYSICAL CHEMISTRY-I

[I] Clapeyron Equation


Suppose a single substance exists in two phases A and B in equilibrium
with each other at constant temperature and pressure. If one mole of a
substance is transferred from one phase A to the other phase B without altering
the temperature and pressure, then there will be no work done other than that
of expansion. So, from equation (dG = V dP - S d1), we get,
dG = 0, as dP = 0 and dT = 0
i.e.,
In other words, the molar free energy of a substance is the same in the
two phases which are in equilibrium.
In a phase change, dG = VdP - SdT can be written as
dGA = VA dP-SA dT ... (1)
and ... (2)
Subtracting equations (1) from (2), we get,
dGB-dGA = (VBdP-SBdn - (VA dP-SA dn
or
or VBdP-SBdT= VAdP-SAdT
or (VB - VA) dP = (SB - SA) dT
or l::.VdP=l::.SdT
-dP -l::.S
-
or
dT l::.V·
where, l::.S and l::. V have their usual significance.
Recognising further that at equilibrium we have the change in entropy
(t::.S), given by
l::.S=l::.H [As l::.G = 0, at equilibrium]
T
dP t::.H
Therefore, ... (3)
dT Tl::.V
Equation (3) was first derived by a French engineer Clapeyron in 1834.
It gives the variation of the equilibrium pressure (P) with temperature (n for
any two phases of a given substance.
[II] Application of Clapeyron Equation
Equation (3) is ~pplicable to various equilibria such as solid-liquid
equilibria, liquid-vapour equilibria and equilibria between two solid modi-
fICations. The Clapeyron's equation for these various equilibria can be easily
obtained as follows :
(1) Solid-liquid Equilibrium
We know that solid and liquid forms of a substance can exist in equi-
librium only at the freezing or melting point. Hence, in equation (3), Twill
CHEMICAL AND PHASE EQUILIBRIUM 115

be the freezing point and P will be the external pressure exerted on the system.
So, equation (3) can also be written in a reversed form as :
dT T~V
dP= MI ... (4)
If Vs and VL represent the molar volumes of the solid and the liquid
phases, respectively at temperature T and pressure P, then,
~V=VL-VS
where, ~ V represents the increase in volume in transferring 1 mole from solid
to liquid phase, MI the amount of heat absorbed may be replaced by
~Hf' i.e., molar heat of fusion. So making these changes in equation (4), we
get the following Clapeyrons's equation.
dT T(VL - Vs)
... (5)
dP= Ml
f
(2) Liquid-vapour Equilibrium
In this equilibrium, the increase in volume (~V) accompanying the
transfer of one mole of liquid to the vapour state will be equal to Vv - VL.
where Vv and VL represent the molar volumes of the vapour and liquid,
respectively.I1H may then be replaced by i1Hv, i.e., molar heat of vaporisa-
tion.
From the above considerations, equation (4) becomes:
dT _ T(Vv - VL)
... (6)
dP - Mlv
It is a well known fact that the boiling point of a liquid is the temperature
at which its vapour pressure becomes equal to the external pressure. So,
equation (6) represents the variation of boiling point of a liquid with pressure
P. Thus equation (6) can also be written as :
dP Mlv
-
dT T(V v - VJ
... (7)

Equation (7) represents the rate of change of vapour pressure (P) of the
liquid with temperature.
Note: The volume measured at the same temperature and pressure,
i.e., Vvand VL are sometimes known as 'orthobaric volumes'.
[III] Clausius-Clapeyron Equation
If the temperature of the liquid is not too near the critical point then we
can easily neglect the volume of the liquid (VL), compared to that of the vapour
(Vv). In such a case equation (7) reduces to equation (8) on replacing
Pby p. So,

... (8)
116 PHYSICAL CHEMISTRY-I

Furthermore, under such conditions the vapour pressure is very samll,


so it may be assumed that the vapour behaves as an ideal gas to which the
equation PVv = RT is applicable. So,
dp AHv
dT=P· RT-

:. From equation (8),


1 dp AHv
or p. dT= RT-
dlogp AHv
or ... (9)
( i f = RT-
Equation (9) is sometimes known as Clausius-Clapeyron equation and
is generally spoken to as first latent heat equation. It was first derived by
Clausius (1850) on the thermodynamic basis of Clapeyron f?quation.
Equation (9) is valid for evaporation and sublimation processes, but not
valid for transitions between solids or for the melting of solids. Clausius-
Clapeyron equation is an approximate equation because the volume of the
liquid ~as been neglected and ideal behaviour of the vapour is also taken into
account.
[IV] Integrated Form of Clausius-Clapeyron Equation :
Assuming the heat of vaporisation to be independent of temperature, if
we integrate equation (9) between the limits TI to T2 (for temperature) and
PI to P2 (for vapour pressure) we get,

fPI
Pz 1
d log P = T A ~ dT
TI
z

Rr

or log P2 = AHv Tz dT r
e PI R JTI T-

P2 AHv (T2 - TI)


or 10glO PI =2.303R T) T2 ... ( 10)
If AHv is expressed in cal/mole and R = 1.987 cal/degree/mole, then
equation (10) becomes
pz AHv (T2 - TI)
loglo PI = 4.576 TIT2 ... (11)
Equation (11) is an integrated form of Clausius-Clapeyron equation.
If the integration is carried out indefinitely (without limits) then we can write
the vapour-pressure equation (9) as,
AHv
10& P = - RT + C (constant) ... (12)
CHEMICAL AND PHASE EQUILIBRIUM 117

[V] Applications of Clausius-Clapeyron Equation


(i) Latent heats from vapour pressure data: Suppose the vapour
pressures PI and P2 are determined at two temperatures T. and T2 respectively;
then from (11) it is possible to calculate the molar or specific heat of vaporisa-
tion. It is also possible to calculate the value of Hv graphically. It is clear
from equation (12) that if log P is plotted against the reciprocal of the absolute
temperature, i.e., liT, then a straight line is obtained. The slope of this line
will be equal to - AHIR. So, knowing the value of R, the value of AH can
thus be easily determined after measuring the slope.
(ii) Deduction of Trouton's law: Equation (9) can be written in the
form,
1 dp AHv
p' RT= Rr
For liquid-vapour equilibrium ANv = Le , i.e., molar heat of evaporation.
Hence, the last expression becomes,
1 dp Le
p' dT= Rr
or
1..!!P- _ Le ... (13)
p dT- RT
We know that,
TITc = e and PIPe = 1t,
where Pc, Tc represent the critical pressure and critical temperature, respec-
e
tively and 1t, are the reduced pressure and reduced temperature, respectively.
So (13) \>ecomes,
e .d1t
- - =Le
- ... (14)
1t de RT
vander Waals (1888) suggested the empirical relationship,

P
log-=k ( I -Tc)
-
Pc T
where k is a constant which is equal to 3 for many substances. Hence in
reduced terms it can be put as,
log 1t = 2.3k (1 - lie) ... (15)
Differentiating equation (?) with respect to e, we get,
dlog1t =2 3~
de . e2

or .!. . d1t =2.3 ~2


1t de e
e d1t k
or n' de =2.3 9 ... (16)
118 PHYSICAL CHEMISTRY-I

From equations (14) and (16), we conclude that,


Le k
RT=2.3 e
Le R
or T=6.g e [As k = 3]

If the temperature is taken to be the boiling point Tb , then e, which is


Tb/Tc is nearly equal to 0.6 and, therefore,
Le
-:::: 23 [As R = 1.99/degree/mole] ... (17)
Tb
So, the molar heat of vaporisation of a liquid divided by its boiling
point 011 absolute scale, i.e., Le/Tb is constant and is approximately equal to
23, provided the latent heat is expressed in calories. This fact was first
observed by Pictet (1876) and rediscovered by Ramsay (1877) and Trouton
(1884) and is commonly known as Trouton's law.

Problem 7: State and explain Le-Chatelier-Braun principle and mention


its applications to different equilibria. (Meerut 2006)
Le-Chatelier and Braun (1888) put a generalisation which helps us to
predict the effect of changes of pressure, temperature and concentration on
the course of two opposing processes at equilibrium. It is known as Le-Chatel-
ier-Braun or simply Le-Chatelier's principle. It is widely applied to physical
and chemical processes. According to this principle,
'If a system is in equilibrium and one of the factors, i.e., pressure,
temperature or concentration, involved in the equilibrium is altered, the
equilibrium will shift so as to tend to annul the effect of the change.'
(1) Effect of temperature change on the position of equilibrium
In reactions which proceed entirely in the gas phase, phosphorous
pentachloride dissociates to phosphorous trichloride and chlorine or hydrogen
iodide dissociates to hydrogen and iodine in a reversible reaction as follows:
PCI5(g)~ PCI 3 (g)+CI 2 (g); DoH = Qkcal.
2HI (g) ~ H2 (g) + 12 (g); DoH = x kcal.
In the above, forward reaction (dissociation of PCI 2 or HI) is accom-
panied by the absorption of heat. If the temperature is increased, the equilib-
rium will be disturbed. Le-Chatelier's principle requires the reaction to
respond to oppose this change, that is to lower the temperature. This can be
achieved if the forward reaction which is endothermic, is allowed to predom-
inate over the backward reactIon, which is exothermic. In such a case, the
position o~ balance of the reaction is disturbed and we say that the position
of equilibrium has been shifted from left to right, In other words, the disso-
ciation of PCI5 or HI increases.
CHEMICAL AND PHASE EQUILIBRIUM 119

We may summarise the effect of temperature on a chemical equilibrium


as follows:
Forward reaction Change in
(left to right) Effect on position of equilibrium
temperature
Exothermic Increase New equilibrium has more of substances on
left (reactants in forward reaction).
Decrease New equilibrium has more of substances on
right (products in forward reaction).
Endothermic Increase New equilibrium has more of substances on
right (products in forward reaction).
Decrease New equilibrium has more of substances on
left (reactants in forward reaction).

(2) Effect of pressure change on the position of equilibrium


Consider the gas phase reaction involving the decomposition of
dinitrogen tetra-oxide into nitrogen dioxide.
N 20 4 (g) ~ 2N02 (g)
Other reaction is: PCIs (g) ~ PCl 3 (g) + Cl2 (g)
At equilibrium, the mixture will contain the two compounds in a definite
proportion. If the pressure is increased, Le-Chatelier's principle demands that
the equilibrium position of the reaction should change in order to restore the
balance and this can occur by a decrease in volume (since the total capacity
of the reaction vessel is fixed, a decrease in volume of the gases is equivalent
to a decrease in pressure). An increase of pressure will thus shift the eq~ilib­
rium to the left, i.e., dissociation of N20 4 or PCls is decreased.
However, pressure will have no effect on those reactions in which there
is no change in the number of'molecules as a result of the reaction, i.e., in
the reaction 2HI ~ H2 + 12,
The effect of pressure on an equilibrium system may be summarised as
follows:

Type of reaction Effect of increase in Effect of decrease in


pressure pressure
I. Increase in number of Position of equilibrium Position of equilibrium
molecules, left to right, e.g., moves to the left, i.e., less moves to the right, i.e.,
PCI 5 ~ Pel 3 + C12· dissociation of Pe1 5. more dissociation of Pe1 5.
2. Decrease in number of Position of equilibrium
molecules left to right, e.g., moves to the right, i.e.,
Position of equilibrium
moves to moves to the
I
N2 + 3H2 ~ 2NH 3· more NH3 will be formed. right, i.e., more NH3 will
be formed.
3. No change in number of No effect No effect.
molecules, left to right, e.g., Position of equilibrium Position of equilibrium
H2+12 ~ 2HI. maintained. maintained.

(3) Effect of concentration change on the position of equilibrium


If the concentration of one of the substances present in an equilibrium
reaction is changed without change in any of the other conditions, then by
120 PHYSICAL CHEMISTRY-I

Le-Chatelier's principle, the position of equilibrium will move to decrease


the concentration of the added substance. Thus, in the reaction
N2 (g) + O2 (g) ~ 2NO (g),
at a given temperature, adding N2 or O2 would shift the equilibrium from left
to right, i.e., more nitric oxide will be formed.
The effect of changes in concentration of substances on the position of
equilibrium in a chemical reaction may be summarised as follows:
Change in concentration of EtTect on eqUilibrium position of reaction
substance A+B~C+D
Increase in concentration of A or B Proportion of C and D increased, i.e., equilibrium
shifts to right
Decrease in concentration A or B Proportion of C and D decreased, i.e., equilibrium
shifts to left
Increase in concentration C or D Proportion of A and B increased, i.e., equilibrium
shifts to left
Decrease in concentration C or D. Proportion of A and C decreased, i.e., equilibrium
shifts to right.

Applications of Le-Chatelier's Principle


(1) Physical Equilibria
(a) Melting point of ice: Ice melts with decrease in volume ,as well
as absorption of heat, e.g., H 20 (s) ~ H20 (I). It is represented as :

Ice Water - Heat


(More volume) (Less volume)
Increase of pressure or temperature will shift the equilibrium from left
to right. In other wards, melting point of ice is lowered by an increase of
pressure or temperature.
(b) Vaporisation of water : The equilibrium between water and steam
is represented as :
Water ~ Water vapour - Heat
(Less volume) (More volume)
On increasing the temperature, the equilibrium will shift in that direction
in which heat is absorbed, i.e., forward reaction. So, more steam will be
produced. Similarly, on increasing the pressure, the equilibrium will shift in
that direction in which volume is decreased, Le., backward reaction. So, steam
will condense into liquid. In other words,formation of steam will befavoured
by increase of temperature and decrease of pressure.
(c) Solubility o/substances: Certain substances like sugar, NaCI etc.
dissolve with an absorption of heat, e.g.,
Sugar + aq ~ Sugar (aq) - Heat
So, increase of temperature will shift the equilibrium to the right. So,
the solubility of such substances increase on increasing the temperature.
CHEMICAL AND PHASE EQUILIBRIUM 121

Certain substances like Ca(OHh etc. dissolve with an evolution of heat,


e.g.,
Ca(OH)z + aq ;=::: Ca(OHh (aq) + Heat
So, increase of temperature will shift the equilibrium to the left, i.e.,
direction in which heat is absorbed. So, the solubility of such substances
decrease on increasing the temperature.
(d) Solubility of gases in liquilfs: Consider the solution of a gas in
equilibrium with the gas.
The equilibrium can be represented as :
Gas + Solvent ~ Solution of gas
I I (Less volume)
(More volume)
If pressure is increased, volume will be reduced without affecting the
pressure and some of the gas will dissolve in the solvent. Thus, the solubility
of the gas increases on increasing the pressure.
[2] Chemical Equilibria
(a) Synthesis of ammonia by Haber's process: Haber's process
involves the reaction
Ml=-22 kcal
1 vol. 3 vol. 2 vol.
(i) Effect oftemperature,' If the temperature of the reaction is lowered,
the equilibrium must shift so as to tend to raise the temperature again (Le-
Chatelier's principle). That is, heat must be liberated by the production of
ammonia. That is, low temperature favours the formation of ammonia. But
lowering of temperature reduces the rate of reaction, so it is necessary to use
a catalyst which will give a sufficient reaction rate inspite of a relatively low
temperature.
(ii) Effect of pressure. Ammonia is produced from its elements with
reduction of volume. Therefore, if the system is in equilibrium and the
pressure is then raised, the equilibrium must shift so as to tend to lower the
pressure (Le-Chatelier's principle). To do this, the volume must be reduced
by the production of more ammonia. That is, high pressure favours the
formation of ammonia.
(iii) Effect of concentration. If the system is in equilibrium and more
N2 is added to increase its concentration, then according to Le-Chatelier's
principle, the equilibrium will shift so as to tend to reduce the N2 concentra-
tion. That is, more ammonia will be produced to use up N 2. This increases
the yield of ammonia relative to H 2, and vice versa if the H2 concentration is
increased.
The formation of ammonia is favoured by :
(i) Low temperature
(ii) High pressure, and
122 PHYSICAL CHEMISTRY-I

(iii) High concentration of the reactants.


(b) Formation of sulphuric acid by the contact process: The first
step in the production of sulphuric acid is the conversion of sulphur dioxide
into sulphur trioxide according to the reaction
2S0 2 (g) + O 2 (g) ~ 2S0 3 (g); tJ.H = - 47 kcal
2 vol. 1 vol. 2 vol.
This reaction is just similar to the synthesis of ammonia described
above. So, the effect of pressure, temperature and concentration will oe the
same as mentioned in the synthesis of ammonia. Low temperature, high
pressure and increased concentrations of S02 and O2 wiU favour the for-
mation of sulphur trioxide.
The S03 is removed from the equilibrium mixture by dissolving it in
fairly concentrated sulphuric acid, forming oleum which is then diluted to
get the acid of the required concentration.
(c) Formation of nitric oxide: The reaction is represented as
N 2(g) + 02(g) ~ 2NO(g); tJ.H = + 43.2 k. cal.
I vol. I vol. 2 vol.

(i) Effect of pressure: As no change of volume occurs during the


formation of nitric oxide, there will be no effect ofpressure on the equilibrium.
Oi) Effect of temperature: If the temperature is increased then the
equilibrium will shift in that direction in which heat is absorbed, i.e., in the
forward direction. So, high temperature favours the formation of nitric oxide.
(iii) Effect of concentration: If to the system in equilibrium N2 is
added, the equilibrium will shift in that direction so as to reduce the concen-
tration of N2• So, more nitric oxide will be formed. Similar is the effect of
adding oxygen.
So, the formation of nitric oxide is favoured by
(i) high temperature and
(ii) high concentrations of N2 or O2,
Problem 8: Explain with reasons that high pressure and low temperature
are favourable for the high production ofammonia gas by Haber's process.
(Meerut 2006)
See Problem 7.

NUMERICAL PROBLEMS
Problem 1: In the formation of silver chloride from its elements under
normal conditions, tJ.G is - 26.3 k .. cals. and m is -30.3 k. caL per mole at
18°C. What is the corresponding entropy change?
Solution: From the following, we have equation
tJ.G = tJ.H - TtJ.S
or -26.3 = -30.3 - 291 x IlS
CHEMICAL AND PHASE EQUILIBRIUM 123

or AS = -30'~9~ 26.3 =0.0137 k. cal mole-1


Problem 2: Calculate the change infree energy (in cals.) which occurs
when 2 g moles ofa perfect gas expands reversibly and isothermally at 37°C
from an initial volume of 55 litres to 1000 litres.
Solution: The change in free energy for an isothermal expansion is
given by,
VI
I1G = 2.303 nRT log 10 V
2
VI = 55 litres; V2 = 1000 litres;
T= 310 K; R = 1.987 cal. deg-I mole-I; n =2.
55
I1G = 2.303 x 2 x 1.987 x 310 loglO 1000
= 2.303 x 2 x 1.987 x 310 (log 55 -log 1000)
= - 3.574 cals.
Problem 3: At 300 K and 1 atmosphere pressure N 20 4 is 20% dissociated
to N02 • Calculate the standard free energy change for the reaction.
Solution: N20 4 ~ 2N02
1 0 [Initially)
(1 - 0.2) 0.4 [At equilibrium)

Total number of moles = 1 - 0.2 + 0.4 = 1.2.

and

On applying the law of mass action,


2
K = PNo! = 119 =1..
p PN 0
2 4
2/3 6
Standard free energy change (I1GO) is given by,
I1Go = - RT loge Kp = - 2.303 RT loglO Kp
=- 2.303 x 1.98 x 300 log 116 =1075.22 cals.
Problem 4: At 1000 K water vapour at 1 atmosphere pressure has been
found to be dissociated into hydrogen and oxygen to the extent of
3 x 10-5%. Calculate thefree energy decrease of the system in this reaction.
(R = 1.98 callmole/degree).
Solution: The partial pressures in the equilibrium mixture are thus
given as,
124 PHYSICAL CHEMISTRY-I

7 7
PeHp = 1; PeH 2 = 3 X 10- ; PeO! = 3/2 X 10-
as one molecule of water yields one molecule of hydrogen and half a molecule
of oxygen.
For the reaction, 2H2 + O2 = 2H20
2
PeH,O 1
We have, K - - ----:-;------=
14 7
p - P;H! x PeO! - 9 X 10- x 1.5 X 10-

1
=----=
1.35 X 10-20
The change in free energy is given by,
AG = - RT loge Kp = - 2.303 RT log 10 Kp
1
= - 2.303 x 1.98 x 1000 log 20
1.35 x 10-
= - 2.303 x 1.98 x 1000 (log 1020 -log 1.35)
= - 2.303 X 1.98 X 1000 X 19.8697 = -90,610 cals.
:. Decrease in free energy of the system is 90,610 cals.
Problem 5 : The equilibrium constant of the reaction
2S02 + O2 ~ 2S03> at S28°C is 98.0 and at 680°C is 10.5. Find out the
heat of the reaction.
Solution: We know that,
loge Kp (I) - loge Kp (2) = - ~ UI -~)
where, Kp is the equilibrium constant and All is the heat of reaction.
log Kp (I) = 98.0 Kp (2) = 19.5 = 10.5
TI = 528 + 273 = 801 K T2 = 680 + 273 = 953 K

or loge 98 -loge 10.5 = - 1~~7 (8~1 - 9~3)


953 X 801)
All =- 1.987 X 2.303 (loglO 98 -loglO 10.5) ( 953 _ 801
953 X 801
= - 1.987 X 2.303 X 0.97 X 152
=- 22,290 cals.
Problem 6: The vapour pressures of water at 9S0C and 100°C are 634
and 760 mm., respectively. Calculate the latent heat of evaporation of water
per gram between 9S0C and 100°C.
Solution : log P2 = Le
W PI R TI
[1-_ 1-]
T2
= Le [T2 - TI]
R TI T2
P2 Le [T2 - TI]
or 2.303 log]() PI = Ii TI T2
CHEMICAL AND PHASE EQUILIBRIUM 125

where, PI and P2 are the vapour pressures at temperature TI and T2, respec-
tively and L. is the latent heat of evaporation/mole. It is given that:
PI =634 m.m.
P2 = 760 m.m.
TI = 95 + 273 = 368°K T2 = 100 + 273 = 273°K

760 Le [373 - 368]


So, 2.303 IOglO 634 = 1.987 368 x 373
or Le = 9886 cals/mole = 9886/18 = 549.2 caVg.
Problem 7: At what height must the barometer stand in order that water
may boil at 99°C? Given that latent heat of vaporisation of water per gram
is 536 cal
Solution As known, the integrated form of Clapeyron-Clausius
equation is,
Iog -P2 -Le-[T2
-- -
TI]
e PI - R TIT2
PI =76 cm. mercury P2 =?
TI = 100 + 273 =273°K T2 =99 + 273 = 372°K
Le = 536 cals/g = 536 x 18 cals/mole
P2 536 x 18 [372 - 373]
2.303 IOglO 76 = 1.987 372 x 373
536 X 18 xl
= 1.987 x 372 x 373
536 x 18 xl
loglo P2 -loglo 76 = - 1.987 x 372 x 373 x 2.303
536 x 18 xl
or loglo P2 = loglo 76 - 1.987 x 372 x 373 x 2.303
= 1.8808 - 0.0159
=1.8659
Taking antilog we get, P2 = 73.43 cm. of mercury
Problem 8: The dissociation pressure of magnesium sulphate hydrated
is 35.6 m.m. at 35°C and 47.2 m.m. at 40°C, the hydrated salt being in
equilibrium with the anhydrous salt. Calculate the heat of dissociation of
the hydrate.
Solution: We know that,
dlogp _JL.
dT - Rr
where, Q is the heat of dissociation per mole of salt. Integrating this between
proper limits, we find,
126 PHYSICAL CHEMISTRY-I

or

or

Substituting the values. we get.


47.2) 313x308
Q = 2.303 x 1.987 ( loglO 35.6 x 5
= 13,610 cals.
Problem 9 : Cakulate the equilibrium constant for a reaction in which
AGo value is - 22 k cal at 25° C. (Meerut 2006,2007)
Solution. From vant Haff isotherm. in the standard state
AGo = - RTlog K
10 K=_~G· =_ (-22 k cal)
or
g RT (2 x 10 - 3 k cal)(298)
22 = 36.9
2 x 10- 3 x 298
Taking anti log. K = 0.7943 X 1037
36
or K = 7.943 X 10

PHASE EQUILIBRIUM
Problem 1 : Explain and illustrate the terms phase, component and degree
offreedom. (Meerut 2006, 2005)

In order to deal effectively with the heterogeneous equilibria, W.J. Gibbs (l873J
gave a generalised rule in the form of phase rule. Its utility was not immediately known
until several scientIsts like Ostwald. Roozehoom applied the rule to several well known
heterogeneous equilibria.
Before defining phase rule one must clearly understand the definitions of three
basic terms, which are frequently used In this connection. The three terms are :
(1) Phase, (2) Number of components, (3) Degree of freedom.

[I] Phase
A phase is defined as "any homogeneous, physically distinct part of
a system which is mechanically separable and bounded by a definite surface.
A phase can exist in either state of matter. viz .• solid. liquid or gas. A system
may consist of one phase or more than one phases.
CHEMICAL AND PHASE EQUILIBRIUM 127

Examples:
1. Pure substances: A pure substance (s, / or g) made of one chemical
species only has one phase, e.g., oxygen (02), ice (H20), alcohul
(C 2H50H) etc.
2. Mixture of gases: A mixture of gases, say H2, N2 and O2 contributes
one phase only as all gases mix freely to form a homogeneous mixture.
3. Miscible liquids: Two or more completely miscible liquids give a
uniform solution. e.g., a solution of water and ethanol has one liquid phase.
4. Non-miscible liquids: A mixture of non-miscible liquids forms as
many number of liquid phases as that of liquids, because on standing they
form separate layers, e.g., a mixture of water and chloroform forms two liquid
phases.
5. Aqueous solutions : An aqueous solution of a solid substance such
as NaCI is uniform throughout. So, there is only one liquid phase.
6. Mixture of solids : A mixture of two or more solid substances
contains as many phases. Each of these substances have different physical
and chemical properties and form a separate phase. Thus, a mixture of calcium
carbonate and calcium oxide has two solid phases.
7. In the dissociation of calcium carbonate there will be three phases,
viz., two solid phases (CaC03 and CaO) and one gaseous phase (C02),
8. In water system, there are three phases, viz., ice (solid), water (liquid)
and vapours (gas). Similarly, in sulphur system, there are four phases, viz.,
rhombic sulphur and monoclinic sulphur (solids), liquid sulphur (liquid) and
vapour sulphur (gas).
[II] Components
The number of components of a system is defined as, "the smallest num-
ber of independently variable constituents by means of which the composi-
tion of each phase can be represented by means of a chemical equation."
Constituents can either be elements or compounds. While writing the
chemical equations, we can use zero as well as negative quantities of the con-
stituents, besides the positive ones (as is the convention).
Examples:
1. Water system: We know that water system consists of three phases,
viz., solid (ice), liquid (water) and gas (water vapours). Each of the three phases
is nothing else but water. Hence, all the three phases can be represented in terms
of the composition of only one constituent, water i.e., by the formula H20.
Ice (s) = H 20; Water (/) = H 20; Vapour (g) = H 20
So, water system is a one component system.
2. Dissociation of calcium carbonate: The case of dissociation of cal-
cium carbonate is rather complicated. Its equilibrium can be represented as :
CaC0 3(s) ~ CaO(s) + CO2(g)
The composition of all the three phases can be expressed in terms of
either of the two components. Any two out of three substances can be chosen
as the two components. This is clearly understood as follows :
128 PHYSICAL CHEMISTRY-I

(a) When CaC03 and CaO are the two components :


Phase Components
CaC03 = CaC03 + O.CaO
CaO = O.CaC03 + CaO
CO 2 = CaC0 3 - CaO
(b) When CaC03 and CO2 are the two components :
CaC03 = CaC03 + O.C0 2
CaO = CaC03 - CO2
CO2 = O.CaC03 + CO2
(c) When CaO and CO2 are the two components :
CaC03 = CaO + CO2
CaO = CaO + O.C02
CO2 = O.CaO + CO2
Hence, from the above three cases, it is crystal clear that only two
constituents are needed to express the composition of each of the three phases.
Hence, it is a two component system.
3. Dissociation of ammonium chloride : The case of dissociation of
ammonium chloride is very interesting. It dissociates as follows :
NH4Cl(s) ~ NH 3(g) + HCI(g)
There are two phases, viz., one solid and one gas. The system wiII be
a one component or two component system depending upon the relative
quantities of HCl and NH3 formed.

(a) When NH3 and HCI are in equivalent quantities :


Gaseous phase Component
NH3 + HCI = NH4CI
Solid phase
NH4CI = NH4CI
So, we see that the composition of both the phases has been expressed
in terms of only one substance, viz., ~Cl. So, the system is a one component
system.
(b) When NH3 and HCI are not in equivalent quantities :
Suppose:
~CI(s) ~ x NH 3(g) + y HCI(g) (x> y)
The composition of the solid phase can only be represented by
~CI, but the composition of the gaseous phase cannot be represented by
~CI, but in terms of NH3 and NH4CI as follows :
Gas phase: xNH3 + yHCI = yNH3 + yHCI + (x - y) NH3
= y~CI + (x - y) NH3
Solid phase : ~CI = ~CI + O.NH3
CHEMICAL AND PHASE EQUILIBRIUM 129

Hence, in this case the two components are NH4CI and NH 3. If HCI is
present in excess over NH 3 , then the two components will be NH 4CI and HCI.
However, the system remains a two component system.
[III] Degree of Freedom or Variance
There are three variable factors, vi;::., temperature, pressure and concen-
tration, on which the equilibrium of a system depends. In some cases, we
have to mention only one factor to define the system completely, sometimes
two or three. So, the degree of freedom (or variance) of a system is defined
as, the least number of variable factors such as temperature, pressure or
• concentration which must be specified so that the remaining variables are
fIXed automaticaUy and the system is completely defined.
System having degrees of freedom three. two, one or zero are known
as trivariant, bivariant, univariant (or monovariant) and non-variant sys-
tems, respectively.
Examples:
1. For ice-water-vapour system, F =0 : In the system, ice ~ water
~ vapour, the three phases co-exist at the freezing point of water. As the
freezing temperature of water has a fixed value, the vapour pressure of water
also has a definite value. The system has two vaJiables (Tand P) and both these
are already fixed. So, the system is completely defined automatically and there
is no need to specify any variable. So, it has no degree of freedom, i.e., F = O.
2. For saturated NaCI solution, F = 1 : The saturated solution of
sodium chloride in equilibrium with solid NaCI and water vapour, i.e.,
NaCI(s) ~ NaCI (solution) ~ Water vapour
is completely defined if we specify temperature only. The other two variables,
i.e., composition of NaCI solution and vapour pressure have a definite value
at a fixed temperature. So, the system has one degree of freedom.
3. For a pure gas, F = 2 : For a sample of pure gas, PV = RT. If the
values of P and T are specified, volume (\I) can have only one definite value
or that the volume, i.e., third variable is fixed automatically. Any other sample
of the gas under the same pressure and temperature as specified above will
be identical with the first one. So, the system containing a pure gas has two
degrees of freedom.
Problem 2: (a) Define phase rule. (Meerut 2(04)

(b) Give the thermodynamic derivation of phase rule.


[I] Statement of Phase Rule
Phase rule as given by W.J. Gibbs, is defined as follows:
"If any heterogeneous system in equilibrium is not affected by
electrical or magnetic forces or by gravity, then the degrees of freedom
(F), number of components (C) and number of phases (P) are connected
by means of the equation,
F=C-P+ 2."
130 PHYSICAL CHEMISTRY-I

The mass of the phase does not enter into the equation. as it has no
effect on the state of equilibrium.
(II] Thermodynamic Derivation of Phase Rule
Consider a heterogeneous system in equilibrium consisting of C com-
ponents distributed in P phases. The degree of freedom of the system is equal
to the number of independent variables which must be fixed arbitrarily to
define the system completely. The number of such variables is equal to the
total number of variables minus the number of variables which are defined
automatically because of the system being in equilibrium.
At C(quilibrium. each phase has the same temperature and pressure. so
there is ,one temperature variable and one pressure variable for the whole
system. So. these variables total two only. The number of composition (or
cOl\~ntration) variables. however. is much more. In order to define the
composition of each phase. it is necessary to mention (C - 1) composition
yariables. because the composition of the remaining last component may be
. obtained by difference. Thus. for P phases, the total number of concentration
or composition variables will be P (C - 1).
Total number of variables = P (C - 1) + 1 + 1
for composition for temperature for pressure
= P (C -1) + 2
According to thermodynamics, when a heterogeneous system is in
equilibrium. at constant pressure and temperature, the chemical potential
(~) of any given component must be the same in every phase. Therefore. if
there is one component in three phases x. y and z and one of these phases.
say x is referred to as standard phase. then this fact may be represented in the
form of two equations :
~l (x) = ~l (y)

~l (x) = ~l (z)
So. for each component in equilibrium in three phases. two equations
are known. In general, therefore, for each component in P phases. (P - 1)
equations are known. For C components, thus the number of equations or
concentration variables that are known from the conditions of equilibrium
are C (P - 1). Since chemical potential is a function of pressure. temperature
and concentration. it means that each equation represents one concentration
variable.
Therefore. the number of unknown variables (which should be fixed)
or degree of freedom.
F = (Total number of variables) - (Number of concentration variables
which are already fixed)
or F=[P(C-l)+2]-[C(P-l)]
or F=C-P+2
This equation is the phase rule as given bv Gibbs.
CHEMICAL AND PHASE EQUILIBRIUM 131

Problem 3 : Explain with reason:


Can all the four phases in a one component system co-exist in equi-
librium?
No, all the four phases in a one component system cannot co-exist in
equilibrium.
Reason : In a one component system, C =1 and, therefore, the phase
rule equation (F = C - P + 2) becomes:
F = 1 - P + 2 =3 - P.
The minimum degree of freedom in any system can be zero. i.e.
F= 0 and so,
0=3 - Pmax or Pmax = 3.
Thus, not more than three phases can co-exist in equilibrium in a one
component system.
Problem 4 : Discuss the application ofphase rule with a neat and labelled
diagram of water system. . (Meerut 2006, 2005, 2002, 2(00)
Water exists in three phases, viz. (a) solid-ice, (b) liquid-water,
(c) gas-vapour.
The system is a one component system, as the composition of all the
three phases can be expressed in terms of only one constituent, H 20. The
equilibrium diagram for the water system is as shown in figure (l)

218
f atm.
~
~~ Ice
a..
4.58 Vapour
mm

0.0075 100 375


Temperature (OC)_

Fig.l

The salient features of the phase diagram are as follows :


(1) The curves AO, OB, oe.
132 PHYSICAL CHEMISTRY-I

(2) The triple point, O.


(3) The areas AOB, BOC, AOC.
The significance of each of these features is discussed below.
(i) Curve AO : It is known as vapour pressure curve or vaporisation
curve !!f liquUJ water. The two phases in equilibrium along AO are water and
water vapour. Hence, the curve is univariant. It also follows from the phase
rule,equation,
F=C-P+2= 1-2+2= 1.
From the curve it is also clear that in order to define the system along
it, we have to mention either temperature or pressure. This is because for one
value of temperature there can only be one value of pressure. The curve AO
terminates at 0 the critical point of water (374°C).
(ii) Curve OB : It is known as the vapour pressure curve or subli-
mation curve of ice. The two phases in equilibrium along OB are solid ice
and water vapour and so the curve is univariant (F = C - P + 2 1 - 2 =
+ 2 = 1). This also follows from the curve, because there is only one value of
pressure for any value of temperature.
The curve OB extends to B, which is nearly absolute zero where no
vapour exists.
(iii) Curve OC : It is known asfreezing point curve of water orfusion
curve of ice. The two phases, ice and water, are in equilibrium along OC. So,
the curve is univariant (F - C - P + 2 = 1 - 2 + 2 = 1).
This curve shows the effect of pressure on the melting point of ice* or
freezing point of water. As the curve OC slopes towards the pressure axis,
the melting point is lowered as the pressure is increased. This fact can also
be predicted by Le-Chatelier's principle, as ice melts with decrease in volume.
(iv) Triple point: The curves OA, OB and OC meet at point 0, where
all the three phases, viz., liquid water, ice and water vapour exist in equilib-
rium. So, it is known as a triple point.
Applying phase rule to triple point, we have
F =C - P + 2 = 1 - 3 + 2 =o.
So, this point is non-variant, i.e., in order to define the system at 0, we
have not to mention any variable factor, i.e., the system is self-defined.
The triple point, posseses fixed values for pressure and temperature,
i.e., 4.58 mm. of Hg and +O.OO75°C, respectively. (It is clear that 0 is not the
actual melting point of ice, i.e., OoC. Its value has been increased due to the
fact that OoC is the normal melting point of ice at 760 mm of Hg and decrease
of pressure will increase the melting point of ice. Since a decrease of pressure
by 1 atmospheric pressure or 7(fJ mm of Hg increases the melting point by
O.OO8°C, therefore, a decrease of pressure to 4.58 mm wilt raise the melting
point to +O.OO75°C.)
* Normal melting point of a substance is defined as the temperature al which the solid and liquid
are in equilibrium at atmospheric pressure.
CHEMICAL AND PHASE EQUILIBRIUM 133
-
(v) Areas: The diagram consists of three areas viz., AOB, BOe and
AOe, which show the conditions of temperature and pressure under which a
single phase-water vapour, ice and water respectively, is capable of stable
existence.
Applying phase rule to different areas, we have
F = C - P + 2 = 1 - 1 + 2 =2,
i.e, the systems are bivariant. This also follows from the diagram, because in
order to define the system completely at any point within the area, we have
to express both the variable factors-pressure and temperature-making the
system bivariant.
Metastable Equilibrium : Fahrenheit observed that under certain
conditions, water can be cooled to -9°C, without the separation of ice at oDe.
Similarly, every liquid can be cooled below the freezing point without the
separation of the solid phase. So, water cooled below its freezing point is
known as supercooled water. But as soon as the equilibrium is disturbed
either by stirring or by adding a small piece of ice, supercooled water im-
mediately changes into ice. Therefore, it can be said that such water is in itself
stable, but becomes unstable on disturbing the equilibrium. Such an equilib-
rium is known as metastable equilibrium. It can be defined as, "an equilib-
rium which in itself is stable but becomes unstable on being disturbed by
stirring or adding a piece of the solid phase."
From the diagram, it is clear that if water is cooled along AO undis-
turbed, then at 0, no ice separates out, as should have happened. In such a
case, the curve AO merely extends to A'. The phases along OA' will be water
and vapour, in metastable equilibrium, as water normally should not exist
below O. The curve OA' which is known as metastable curve will be
univariant. As soon as the equilibrium is disturbed by stirring or adding a
piece of ice, the curve OA' immediately merges into OB with liquid water
changing into solid ice. The curve OA' is in accordance with the general
principle that the vapour pressure of the metastable phase is always higher
than the stable phase. This is evident because the curve OA' lies above the
curve OB. Below 0, ice is the stable phase and water is the metastable phase.
Salient Features of water System

Curve/Regionl Phases : Solid (S),


Name Liquid (L) and Variance
Point Vapour (V)
I. Curve OA Vaporisation curve of water L --" V I
..-
2. Curve OB Sublimation curve of ice S --" V I
..-
3. Curve OC Fusion curve of ice L --"S I
4. Region AOB - Vap;;;;;:- 2
5. Region AOC - Liquid 2
6. Region BOC - Solid 2
134 PHYSICAL CHEMISTRY-I

Phases: Solid (S),


Curve/Regionl Name Liquid (L) and Variance
Point Vapour (V)
7. Point '0' Triple point S~L~V 0
8. Curve OA' Metastable vaporisation L~V I
curve of water

Problem 5 : Apply phase rule to system of one component comprising of


more than one solid phase or apply phase rule to sulphur system.
(Meerut 2004, 2001)
Sulphur exists in two crystalline forms, viz., rhombic and monoclinic.
Normally, sulphur exists in rhombic form which is octahedral in shape. When
sulphur is heated to 95.6°, the rhombic sulphur changes into monoclinic
variety which is prismatic in shape. Above 95.6°, monoclinic sulphur exists
as the stable phase. If monoclinic sulphur is cooled then at 95.6°, it changes
into rhombic type. Therefore, it is clear that below 95.6°, rhombic sulphur
exists as a stable phase, whereas above 95.6°, only monoclinic variety occurs.
At 95.6°, these two crystalline forms are in equilibrium with one another.
Hence, 95.6° is the transition temperature of SUlphur.
95.6'
Rhombic sulphur ~ Monoclinic sulphur.
Besides the above two crystalline forms, sulphur also exists as :
(1) Liquid sulphur and (2) Vapour sulphur.
Monoclinic sulphur when heated to 120°, melts into liquid form which
is pale yellow. On heating gradually the colour of the liquid sulphur changes
and the viscosity increases. At 444°, liquid sulphur begins to vaporise. The
complete equilibrium is represented as follows :

Rhombic sulphur
(SR)
95.6"
~Monoclinic
(SM) J
120"
sulphur---7 Liquid sulphur
(SL)
Changes
in colour
and viscosity
Vapour sulphur
(Sv) 444'

As evident, not more than three phases can co-exist in equilibrium at


anyone time in a one component system. In sulphur system, there are four
phases, therefore, all the phases can never co-exist in equilibrium. Only three
out of the four phases can exist in equilibrium at anyone time.
Equilibrium Diagram : The salient features of phase diagram repre-
sented in figure (2), are as follows :
(i) The six curves, AB, BC, CD, BE, CE and EP.
(ii) The three triple points B, C and E.
(iii) Tlte four areas ABCD, ABEF, BCE and DCEP.
.....", .... ,.
CHEMICAL AND PHASE EQUILIBRIUM 135

r"""''''''''''''''''''''''''''''''''''''''''''''''''''''""""""".,.".""""".,.,', "" """"::"""""'~"""'\I:I

I:
..•• i
o
SR I$~ . ~~~

i r

I T.:~~mW~'~~O::~~) - ,I:
::;:;';;';;~;·;:;~i·;;;r~~·~1;;;;;;;;;;;;!
The significance of the features is discussed below.
(1) Curve AB : It is known as vapour pressure or sublimation curve
of Sib as it gives the vapour pressure of solid SR at different temperatures.
Two phases in equilibrium are SR and Sy. The system is, therefore, univariant
as,
F=C-P+2
F=I-2+2=1.
At B, SR changes reversibly into SM'
(2) Curve BC : It is known as vapour pressure or sublimation curve
of SM as it gives the vapour pressure of solid SM at different temperatures.
The two phases in equilibrium are SM and Sy.
The system is again univariant. At C, i.e., at 120°, SM changes into SL'
(3) Curve CD: It is known as vapour pressure curve ofSL, as it gives
the vapour pressure of SL at different temperatures. The two phases in equi-
librium are SL and Sy. The system is univariant, as
F=C-P+2=1-2+2=1.
i 136 PHYSICAL CHEMISTRY-I

(4) Curve BE : It is known as transition curve and the two phases in


equilibrium are solid SR and solid SM' The system is univariant.
The curve indicates the effect of pressure on the transition temperature
of SR to SM' As the curve BE slopes away from the pressure axis, an increase
of pressure will increas;! the transition temperature. The curve BE terminates
at E. beyond which SM disappears.
(5) Curve EF : It is known as melting point or fusion curve of SR' The
two phases co-existing in equilibrium are SR and SL' The system is univariant.
(6) Curve CE : It is known as melting point or fusion curve of SM-
The two phases in equilibrium along it are SM and Sv The system is univariant.
The curve CE shows the effect of pressure on the melting point of SM' As
this curve slopes slightly away from the pressure axis. the melting point of
SM increases with an increase of pressure. This is in accordance with Le-
Chatelier's principle. as melting of SM is accompanied by a slight increase of
volume.
(7) Triple Points (B, C and E).
(i) Triple point B .' It is the meeting point of the three curves AB, BC
and BE. Three phases, viz., solid SR' solid SM and Sv exist in equilibrium at
point B. Thus, it is a non-variant point (95.6°C).
F=C-P+2=1-3+2=O
At B, SR is changed to SM and the process is reversible.
(ii) Triple point C.' It is the meeting point of the three curves BC, CD
and CEo Three phases, viz., solid SM' SL and Sv exist in equilibrium at point
C. So, it is a non-variant point (l20°C).
(iii) Triple point E .' It is meeting point of the three curves BE, CE and
EE Three phases, viz., solid SR' solid SM' and SL exist in equilibrium at point
E. It is a non-variant point (150°C).
(8) Areas or Regions (ABCD, DCEF, BCE and ABEF) : Any point
within the area ABCD, DCEF, BCE and ABEF gives the conditions of
temperature and pressure for the stable existence of only one phase, i.e.,
Sv, SL solid SM, solid SR, respectively. The areas have two degrees offreedom
as shown below.
F = C - P + 2 = 1 - 1 + 2 = 2.
(9) Metastable Equilibria: The change of SR to SM occurs very slowly.
If enough time for the change is not allowed and SR is heated rapidly, it is
possible to pass well above the transition point (B) without obtaining SM' In
that case, the curve AB extends to O. The curve AD is known as metastable
vaporisation curve of SR' The phases SR and Sv will be in metastable
equilibrium along this curve. It is a univariant system.
On super-cooling along DC, the curve CO is obtained. It is, infact, the
back prolongation of DC. The curve CO, known as vaporisation curve of
supercooled Sv represents the metastable equilibrium between supercooled
SL and Sv. It is also univariant.
Similarly, the two metastable phases SR and SL exist in equilibrium
along EO, which is known as metastable fusion curve of SR' It is also
univariant.
CHEMICAL AND PHASE EQUILIBRIUM 137

At point 0, three metastable phases SR' SL and Sy are in equilibrium.


It is known as metastable triple point. It is non variant, as
F=C-P+2=1-3+2=O
Salient Features of Sulphur System
System
Curves I Phases in Variance
Regions! Name of the system (F=C-
equilibrium
Triple points P+2)
I. Curve AB Vapour pressure curve of SR SR~Sy I
2. Curve BC Vapour pressure curve of SM SM~ Sv 1
3. Curve CD Vapour pressure curve of SL SL ----" Sy 1
~

4. Curve BE Transition curve SR~ SM I


5. Curve CE Fusion curve of SM SM----" SL I
~

6. Curve EF Fusion curve of SR SR~ SL I


7. Region ABCD - Sy 2
8. Region DCEF - SL 2
9. Region BCE - SM 2
10. Region ABEF - SR 2
II. Point B First triple point (95.6", SR~SM~SV 0
0.006 m.m.)
12. Point C Second triple point (\20", SM ' SL~Sy 0
0.04 m.m.)
13. Point E Third triple point (15 J", S~S~SL 0
1288 cm)
14. Curve BO Metastable sublimation SR~SV I
curve of SR
15. Curve CO Metastable vaporisation SL~Sy I
curve of SL
16. Curve EO Metastable fusion curve of SR~SL I
SR
17. Point 0 Metastable triple point SV~SL< 'Sv 0
(114°,0003 m.m.)

Problem 6 : Write short notes on the /oUowing :


(i) Non-variant system in phase rule studies.
(ii) Triple point.
(iii) Transition point.
(i) Non-Variant System in Phase Rule Studies
A system for which degree of freedom is zero is called non-variant
system. Under such conditions, there can be no change in temperature,
pressure and concentration, because if any change is made in either of them,
one or more phases may disappear. Such systems are also called self defined
systems.
138 PHYSICAL CHEMISTRY-I

In water system, the three phases viz., ice (s), water (/) and vapours
(g) remain in equilibrium at a fixed temperature (O.0075°C) and fixed pressure
(4.58 mm). At no other temperature and pressure, all the three phases can
coexist in equilibrium. From phase rule equation (F = C - P + 2), we can also
show that value of F of the system is zero, i.e.,
F= 1-3 +2=0.
(2) Triple Point
Triple point is that point where all the three phases in a one component
system exist in equilibrium. At this point, both the variables, e.g., temperature
and pressure are fixed, i.e., they have definite values. Thus, triple point is a
non-variant point, i.e., degree of freedom is zero. It is also clear from phase
rule equation i.e.,
t' = c - P + 2 = 1 - 3 + 2 = O.
The system at triple point is self defined. If any change is made in either
of the variable factors, one of the phases of the system disapppears. In water
system (figure I), 0 is the triple point where ice, liquid water and water
vapours exist in equilibrium. In sulphur system (figure 2), there are three
triple points namely B, C and E.
(3) Transition Point
It is defined as that temperature at which one allotropic form of a
substance is converted into another allotropic form of the same substance.
The point where this change occurs is called the transition point.
Example:
(i) At 95.6°C, rhombic sulphur is changed into monoclinic sulphur.
Below 95.6°C, rhombic sulphur can exist and above 95.6°C monoclinic
sulphur exist. Thus, 95.6°C is the transition temperature of sulphur (See
problem 3).
(ii) On cooling, white tin is converted into grey tin at 18°C (transition
temperature).
Problem 7 : What is a two component system and how it is graphically
represented? Define reduced phase rule equation and condensed state.
[I] Two Component System
A two component system is that system in which the composition of
each phase present in it can be represented in terms of two substituents,
e.g., lead-silver system, potassium iodide-water system etc.
Phase rule when applied to a two component system becomes
F=C-P+2=2-P+2=4-P
Since the minimum number of phases in any system is one, it is evident
from the above equation that the maximum degree of freedom in a two
component system is three. Therefore, in addition to temperature and pres-
sure, a third variable, namely composition, has also to be taken into account.
In order to represent such an eqUilibrium graphically, it is, therefore, necessary
to have three coordinate axes at right angles to one another. This will lead to
CHEMICAL AND PHASE EQUILIBRIUM 139

three dimensional or space models which cannot be easily represented on


paper (figure 3-a). Therefore, it is customary to choose any two of the three
:
/'
:
~

T P
C constant
t t
P
-P -T
I
C

: (a) (b)

T P
P constant T constant
t t
P P
-c -c
(c) (d)

:::::::::::: : '::':':':':':':':':':';'::}}}}i
d
:::::::
Fig. 3
.:.:.:. : :::::::::::::::::::::::::::::::::.:::::::::::::::::::::::

variables for graphical representation, assuming the third variable to remain


constant. Thus, we get three types of curves as shown in (figure 3 b, c, d).
[II] Condensed Phase Rule Equation
When in a system, one variable out of pressure, temperature or concen-
tration is taken as constant, the number of degrees of freedom is reduced by
one. In that case, phase rule equation (F = C - P + 2) becomes,
F=(C-P+2)-1 or F=C-P+ 1
This equation is known as condensed or reduced phase rule equation.
[III] Condensed State or Phase
In solid-liquid system, a negligible change in pressure produces no
change in equilibrium as the vapour pressure of solid is negligible. Therefore,
in such a system, the pressure variable may be taken as nearly constant. So,
a system in which vapour phase is ignored is known as a condensed system.
For such a system, we apply the condensed phase rule equation,
F=C-P+l.
140 PHYSICAL CHEMISTRY-I

Problem 8: Apply phase rule to lead-silver system. Mention the effect of


cooling and also explain the Pattinson's process for desilverisation of lead
ores.
The two metals lead and silver mix together in the liquid form but do not
form any chemical compound. The phase diagram is as shown in figure (4).
It is explained as follows :
1. Curve AC : Pure lead melts at 327°, while silver at 961°. Point A
represents the melting point of pure lead. By the gradual addition of silver,
the freezing point of lead decreases along AC. The curve AC is thus known
as freezing point or melting point curve of lead. The phases present along
AC are solid lead and its solution with silver. Thus, it is a univariant curve as,
F=C-P+l=2-2+1=1
2. Curve BC : Similarly, B represents the melting point of pure silver.
By the addition of lead to it, the freezing point of silver decreases along BC
and we get a solution of it with lead. The curve BC is known as thefreezing
point or melting point curve of silver. The two phases present along it are
solid silver and a solution of it with lead. It is, therefore, a univariant curve as,

Unsaturated
solution
'X
I

1 I
Liquid + Solid silver

::: ~ Liquid of': y


~ ~~
~----~~----------------~D~' 3030

Solid
lead + Solid silver + Eutectic
Eutectic

F=C-P+l=2-2+1=1.
3. Point C : The two curves AC and BC meet at a common point C.
Therefore, C gives conditions of temperature and composition under which
three phases, viz., solid silver, solid lead and solution co-exist in equilibrium.
It is thus a non-variant point as,
F = C - P + 1 = 2 - 3 + 1 = O.
CHEMICAL AND PHASE EQUILIBRIUM 141

The point C is known as eutectic point of the system. The temperature


and composition corresponding to eutectic point are known as eutectic tem-
perature and eutectic composition, respectively. The eutectIc temperature and
0
composition are 303 and 2.6% Ag and 97.4% Pb, respectively.
4. Areas: In the area above the curves AC and BC, the two components
are present in the form of one homogeneous liquid phase. The system becomes
bivariant as,
F = C - P + 1 = 2 - 1 + 1 = 2.
In other areas the phases are present as shown in the figure.
Effect of cooling: Consider the phase changes which occur on cooling
a liquid mixture. Suppose a liquid mixture of composition x is cooled at
constant temperature. The temperature will fall without any change in the
composition until the point y on the curve yC is reached. At this point, I&d
will begin to separate out. The system becomes univariant, as it consists of
two phases. The temperature will now fall with a change in the composition
of the liquid mixture along AC. As cooling continues, Pb keeps on separating
out while the solution becomes richer and richer in silver. When the eutectic
temperature (303~ is reached, the second solid phase, viz., silver begins to
crystallise out. The system thus becomes non-variant at C. The two solids
lead and silver will separate out together in a fixed ratio on further cooling,
so that the composition of the solution remains constant as shown by point
C. The temperature also remains constant. When the solution phase has
completely disappeared as solid, the system consists only of a mixture of solid
Ag and Pb. The system becomes univariant and further cooling will lower
the temperature below solidus DD'. In area below DD' two solid phases Pb
and Ag will co-exist.
If a liquid solution of composition represented by a point x' is cooled
its temperature will fall without change in the composition along x'y'. At y',
solid Ag begins to crystallise out and the system becomes univariant. Further
cooling will shift the equilibrium along y'C, when Ag goes on separating out
and the solution becomes richer and richer in Pb. When the eutectic temper-
ature is reached, Pb also begins to crystallise out. Further cooling will not
change the temperature as well as the composition as long as three phases are
present at C. When solution phase solidifies, only then the temperature falls
below solidus DD' within which two solid phases Pb and Ag co-exist.
Pattinson's process of desilverisation of lead : This process consists
of increasing the relative percentage of silver in ores of lead called argenti/-
erous lead ores containing very small amounts of silver, say 0.1 %. Its relative
content can be increased by taking a liquid solution of the ore and increasing
it to a high temperature. It is then cooled and the temperature falls along
xy. At y, solid lead begins to crystallise out which can be removed by ladels.
Further cooling will shift the equilibrium along ye, making the solution richer
and richer in Ag. Along ye, lead will go on separating out which is -:ontinu-
142 PHYSICAL CHEMISTRY-I

ously removed. At C, the percentage of silver increases to 2.6% (starting from


0.1%).
Problem 9: Apply phase rule to potassium iodide and water system. Explain
the effect of cooling on the system.
A careful study has shown that water and potassium iodide system is a
simple eutectic system. The phase diagram is as shown in figure (5) (One
essential feature of salt-water system is that the melting point of the salt is
usuaUy very high, even higher than the critical temperature of water. It is,
therefore, not possible to represent the melting point of the salt in the
equilibrium diagram). The phase diagram is discussed as follows :
:

,
Unsaturated solution x0
I B

i
~

~
Q)
a. SolidKI
E +
t! Solution
Ice + Solution
-230
Ice + Eutectic Solid KI + Eutectic

0 52 100
Composition (% KI) _

1. Curve A C : Point A represents the melting point of ice or the freezing


point of water which is 0° at 1 atmospheric pressure. With the gradual addition
of KI, some of it dissolves in water and remains in contact with ice. The
melting point of ice decreases along the curve AC by the addition of KI. The
curve AC is known as freezing point curve of water or fusion curve of ice.
Along this curve, solution of KI in water is in contact with ice, hence it is a
univariant curve. It is also seen from condensed phase rule equation:
F=C-P+l=2-2+1=1
2. Point C : At point C, a new solid phase, potassium iodide also
separates out. Thus, three phases, viz., ice, potassium iodide and liquid, exist
at C. Hence, it is a non-variant point as,
F=C-P+l=2-3+1=0
CHEMICAL AND PHASE EQUILIBRIUM 143

The point C is known as the eutectic point or cryohydric point of the


system. It gives the lowest temperature which can be attained in the system,
i.e., _23°. At the cryohydric point the solution freezes at constant temperature
without change of composition. The eutectic composition is 52% KI and 48%
ice.
3. Curve CB : If the system is heated at C, ice will melt and potassium
iodide will pass into the solution in the same ratio in which it is already present
in the solution, so that the composition of the solution remains unchanged.
The temperature and composition do not change, as long as three phases are
present at C. The heat supplied to the system is utilised in transforming ice
. into water. If heating is continued and potassium iodide is present in excess
then ultimately all ice will disappear. Then there will be two phases and the
system will become univariant. At this stage, addition of more potassium
iodide will bring about a change in the temperature also and then the curve
CB is traced.
The curve BC is also traced in an alternative way. If we add ice to solid
potassium iodide then we get a solution of potassium iodide in water in
equilibrium with solid potassium iodide. In other words, the addition of ice
to potass!um iodide decreases the melting point of solid potassium iodide
along Be. The curve BC is known as the solubility curve ofpotassium iodide,
as KI is in equilibrium with its solution. The nature of the curve BC shows
that the solubility of potassium iodide increases slowly with rise of temper-
ature. At point C, a new solid phase, viz., ice begins to separate out.
Effect of cooling : If we cool a solution of composition x, then the
temperature will fall along xy without any change in the composition, as the
system in bivariant. At y, solid ice will separate out. The system becomes
univariant. The change in temperature will now be followed by a change in
composition. Hence, the temperature will fall along yC when at C, potassium
iodide will separate out as a new solid phase. Similarly, we can predict the
effect of cooling a solution of composition x'.
Problem 10 : Determine the number of phases and components in the
foUowing systems:
(i) Water in a beaker at room temperature.
(ii) A solution of ethanol with water which is in equilibrium with
vapours.
(iii) NH4CI(s) ~NH4Cl(vap) ~NH3(g) + HCI(g)
(iv)HCI gas is passed from outside in equilibrium (iii).
(i) Components = 1; Phases = 2
(ii) Components = 1; Phases = 2
(iii) Components = 1; Phases = 2
(iv) Components = 2; Phases = 2.
Problem 11 : What will be the degree of freedom in the foUowing closed
systems?
(i) liquid water and water vapours.
144 PHYSICAL CHEMISTRY-I

(ii) Liquid water and water vapours at 2S·C.


(iii) Dilute solution of NaCI fiUed partially in a closed vessel.
(iv) Saturated solution of Naafilled partially in a closed vessel.
(v) A gaseous mixture of N:z, O2 and H2o
(i) One(F=I-2+2=1)
(ii) One (F = 1 - 2 + 2 = 1)
=
(iii) Two (F 2 - 2 + 2 = 2)
(iv) One(F=2-3+2=1)
(v) Four(F=3-1+2=4).
Problem 12 : Determine the number ofphases, components and degree of
freedom of the following systems:
(i) Sodium chloride and water
(ii) A mixture of nitrogen and hydrogen gases in a vessel.
(iii) 2HzS(g) ~ 2HzO(g) + Sz(g)
(iv) H20(s) ~H20(g)
(v) Na~04. 10HzO(s) ~Na~Ois) + 10HzO(I)
(vi) CaCOis) ~ CaO(s) + CO2(g)
(i) Sodium chloride-water system
Phases = 2; Components = 2 (NaCI and H 20)
Degree of freedom = 2
(ii) System of Nz and Hz gases
Number of phases = 1; Components = 2 (H2, N+)
Degree of freedom = 3
(iii) Phases = 1, Component = 1, Degree of freedom = 2
(iv) Phases = 2, Component = 1, Degree of freedom = 1
=
(v) Phases 3, Component = 2, Degree of freedom = 1
(vi) Phases = 3, Component = 2, Degree of freedom = 1

SYSTEMS OF LIQUIDS IN LIQUIDS


Problem 13: What are ideal solutions? Explain the vapour pressure of
ideal solutions.
Before defining ideal solutions, we must understand Raoult's law.
Raoult measured the vapour pressures of a number of binary solutions of
volatile liquids and made the important generalisation, known as Raoult's
law. It can be stated as follows:
The partial pressure of any volatile component of a solution at any
temperature is equal to the product of the vapour pressure of the pure
component and the mole fraction of that component in the solution.
Consider a binary solution made of nA moles of a volatile liquid A and
ns moles of another volatile liquid B. If PA and Ps be the partial pressures of
the two liquid constituents, then according to Raoult's law,
PA =XAPAo
Ps=xsPso
CHEMICAL AND PHASE EQUILIBRIUM 145

where, XA is the mole fraction of the component A, given by ~ ,


IlA +IlB

XB is the mole fraction of the component B, given by ~,


+ liB nA

PA° and PB° are the vapour pressures of the pure components A and B.
respectively and n term represents the number of moles.
If vapour behaves as an ideal gas then according to Dalton's law of
partial pressures, the total pressure (P) is given by
P=PA +PB
o
or P=XAPAo+XBPB ... (1)
Experiments show that Raoult's law is obeyed only approximately for
a number of binary solutions. It is obeyed perfectly only in case of ideal
solutions. So, a solution of two or more components is said to be ideal if it
obeys Raoult's law perfectly at all temperatures and concentrations.
Similar liquid pairs are generally found to form ideal solutions, e.g.,
binary mixtures of ethylene bromide and ethylene chloride, benzene and
toluene, n-heptane and n-hexane etc.
Vapour Pressures of Ideal Solutions
The vapour pressure of an ideal binary solution of two components A
and B is shown in Fig. 6. It is clear from the graph that the curve of the partial
pressure of each component against its mole fraction in the solution is a
straight line and the total vapour pressure of the solution for a given concen-

Total ~
~, apOur pressur;
..... , ..... , e of solution P O.4PA + O.6PB
..... ,
..... ,oq P =X pO
, ..... :'1<1/ A A + XaPa
IJ ..... , IJre8s ,~
,
:.II ~.t'''''' lire , ,
, , "
I1IJ..qo'''''',~-1 , ,
,
, ,
..... ... ...... ' "
,
,,."" ..."'........ ,
..... ,
sS\l{e " , 0
o\~" " ..... ,
.....
~&tU.&\ ~{~, , ~ , ::. 'J..-o'Y-O ..... ,
..... ,
, , ", -0 ..... ,
, ..... ,
, , "

---. Mole fraction


146 PHYSICAL CHEMISTRY-I

tration is equal to the sum of the partial vapour pressures of the two constit-
uents. The total vapour pressure (P) of the solution is given by equation (1).
Thus, when the mole fraction of B is say 0.60, the total vapour pressure (P)
of the solution is given by
P = 0.4 PAc + 0.6PBo

Activity of a Component in an Ideal Solution


Consider a binary solution of two components A and B, forming an ideal
solution. Let aA and aB be the activities, and XA and XB be the mole fractions
of the constituents A and B, respectively. According to Raoult's law, the partial
vapour pressure (PA) of constituent A is given by
PA =XAPAo
PA
or -=XA
PAc
As vapour pressures of solid or liquid components of a solution at
ordinary temperatures are usually low, lying well within the range in which
the fugacity if) may be taken as equal to pressure, therefore,
As known from thermodynamics,
PA fA
-=-=XA
PAc lAO
As we know from thermodynamics,
PA
-=-=aA
IA
PAc lAO
where aA is the activity of the component A.
So, XA =aA
and similarly x--B = aB,
where aB is the activity of the component B.
In other words, activities of components forming ideal solutions are
equal to their mole fractions. Thus, ideal solution may also be defined as
follows:

aB =1
aA =1
< m
'0 '0
~ f
llC..-______---""t
XA = 1 Mole fraction =
XA 0
XB=O =
XB 1
CHEMICAL AND PHASE EQUILIBRiUM 147

A solution in which the activity of each component is equal to its mole


fraction under all conditions of temperature, pressure and composition, is
said to be an ideal solution.
The relation between
activities and mole fractions
of the constituents is shown
graphically in figure (7) So, If
Xi is the mole fraction of a
component i, then its activity
a, is given by
ai=xi

Problem 14 : What are


non-ideal or real solutions?
Explain the vapour pressure
curves of completely miscible
binary solutions.
Those solutions which
show appreciable deviations
(positive or negative) from
the ideal behaviour are
known as non-ideal or real
solutions.
Real solutions may be
divided into the following 800~---------------.

three types :
Type I. Non-ideal so-
lutions of this type show small
deviations from ideal behavi-
our and total pressure remains
always within the vapour
pressures of the pure compo- ~
~ 400
nents, as shown in figure (8), f/)

in which the dotted lines rep- ~


~ 300
resent ideal behaviour. It i&
observed that the total pres-
~
&200
sure of each component
shows a positive deviation 100
from Raoult's law. However,
,,
the total pressure remains o "------,---..--r---r--.....
within the vapour pressures of o 0.2 0.4 0.6 0.8 1.0
the pure constituents A and B. Ethanol Chloroform disulphide
Example of such a type of so- - - . Mole fraction
lution is furnished by carbon
tetrachloride and cyclohexane
system.
148 PHYSICAL CHEMISTRY-I

Type II. Solutions of this type show large positive deviations and
the total vapour pressure curve rises to a maximum which is above the vapour
pressure of either of the two pure constituents A and B as shown in figure
(9). Mixture of ethyl alcohol and chloroform is a system which belongs to
this type.
Type III. Solutions of this type show large negative deviations and
the total vapour pressure curve dips to a minimum, for some of the concen-
trations. The total vapour pressure of the mixture may be below the vapour
pressure of either of the two pure constituents as shown in figure (10). Mixture
of acetone-chloroform is a system which belongs to this type.

~;'::":-:-:':""\I
• 1: 1

I~ ::
;:::::-.
0 0.2
A~ Mole tractionChlorotorm
0.4 0.6 0.8 ,.01
J:
:;:;:;:;:;:;:;~: ;: :;:;:;:;:;:;:;:;: :;:;: :;:;:;:;: : ~ !~: .~:~: : ;:;: : :;: :;:;:;: : : : : : : : : :~:;:;: : : : :
Vapour Pressure (or Boiling Point) Composition Curves
of Completely Miscible Binary Solutions
Consider a binary solution of two components A and B which are
completely miscible with one another. On heating under constant pressure, it
will boil at a temperature at which its total pressure becomes equal to the
atmospheric pressure. If PA and PB represent the partial pressures of the two
components A and B, then conditions for boiling is that
P=PA +PB
where, P is the atmospheric pressure.
CHEMICAL AND PHASE EQUILIBRIUM 149

Solutions of different compositions have different vapour pressures, so


they will boil at different temperatures. Therefore, a solution of lower vapour
pressure will boil at a higher temperature and vice versa. This helps us in
drawing boiling temperature-composition curves from the corresponding va-
pour pressure composition curves as shown in figure 6. As illustrated, there
are three types of mixtures.
Type I. Mixtures in which the vapour pressure changes continuously
with composition of the mixture.

T = Constant P = Constant

~A
::l
1ij
Q;
a.
Type I E
$ B
OJ
~
·0
co
0 1 0
Mole fraction of B Mole fraction of B
Composition (i) Composition (iv)

T=Constant P = Constant

C ~
~ ::l
::l
I/) ~A
I/) Q)
~ B a.
a. Type II E
$ B
OJ
~
·0
co
0 0
Mole fraction of B Mole fraction of B
Composition (ii) Composition (v)

T= Constant P= Constant

~ D
~ ::l
::l B ~
~ Q)
~ g-A
a. Type III
SA $
OJ
8. ~ B
~ ·0
co
0
Mole fraction of B Mole fraction of B
Composition (iii) Composition (vi)
150 PHYSICAL CHEMISTRY-I

Type II. Mixtures in which the vapour pressure shows a maximum in


the vapour pressure-composition curve.
Type III. Mixtures in which the vapour pressure shows a minimum
in the vapour pressure-composition curve.
In type I, the vapour pressure of pure A is the lowest and that of pure
B is the highest, therefore, the boiling point of pure A will be highest and that
of pure B will be lowest. Since the vapour pressures of the mixtures of A and
B lie in between the vapour pressures of pure components, their boiling points
will also lie in between as shown in figure 11 (iv). At a given temperature,
the vapour pressure will be richer in the more volatile component B, the
composition of the vapour phase will be always richer in B than that of the
liquid phase. Thus, the vapour composition curve will lie above the liquid
composition curve.
In type II, the vapour pressure shows a maximum for a definite com-
position, say C, as shown in figure 11. The solution of that composition will
boil at the temperature. Thus, there will be a minimum in the boiling temper-
ature-composition curve, as shown in figure 11 (v).
In type III, the vapour pressure shows a minimum for a definite com-
position, say C, as shown in figure 11 (iii). The solution of that composition
will boil at the highest temperature. Thus, there will be a maximum in the
boiling temperature-composition curve, as shown in figure 11 (vi).
Problem 15: Discuss the theory offractional distillation of binary solu-
tions.
As the boiling point curves of the three types of solutions are different.
the behaviour of these solutions on distillation will be different.
1. Solutions
of type 1. Mix-
tures which shows
neither a maximum
nor a minimum on
the vapour pressure-
composition curve
or boiling tempera-
ture-composition
curve are known as
zeotropic mixtures.
Consider the given
figure (12). Suppose
a solution of compo-
sition X is heated.
The boiling will start o 1
when temperature T - - . Mole fraction of B
is reached. At this
temperature, the va-
pour coming off will
CHEMICAL AND PHASE EQUILIBRIUM 151

have the composition X since it is richer in B, than the composition of the


residual liquid will become richer in A. Let it be shown by Y. This liquid,
having different composition cannot boil at T. Instead, it will require a higher
temperature, say T I • The vapour coming off now will again be richer in B, as
represented by point X2• Thus, the composItion of the residual liquid will be
further enriched in the liquid A. The temperature of this residual liquid will
have to be increased again to make it boil again. If this process is continued,
the boiling point of the solution will go on increasing from Tto TA , the boiling
point of pure liquid A. The composition of the residual liquid will also become
richer in A and ultimately only liquid A remains in the; liquid phase.
Regarding the vapour phase, it has been stated t~at the vapour coming
off from the original solution at its boiling point T will have the composition
shown by point XI. If these vapours are condensed and the liquid distilled
again, the new boiling point will be T2 and now the composition of the vapours
distilling over will be given by Z. Therefore, the distillate is now richer in
component B than before. If this process of condensing the vapours and
redistilling the liquid obtained is continued, the distillate obtained ultimately
will consist entirely of pure component B. In other words, by carrying out
fractional distillation of solutions of type I, it is possible to isolate 'both the
constituents from each other.
2. Solutions of type II. The boiling temperature-composition curves
of the liquid and vapour phases meet at a maximum boiling point fig. (13).
In other words, the liquid and vapour phases at this point have the same
composition. Thus, the liquid mixture represented by the point M, will boil
at a constant temperature and will distil over completely without change of
composition. Such mixtures which like pure chemical compounds, boil at
: : :

100
B
90
U
0
e. 80
!~
-e
~ 70
E
~

r 50
10 20 30 40 50 60 70 80 90 100
--+ Percent A
152 PHYSICAL CHEMISTRY-I

a constant temperature and distil over completely at the same temperature


without change in composition, are known as constant boiling mixtures or
azeotropic mixtures.
On applying phase rule to azeotropic mixtures, we should remember
that there is only one restriction, viz., compositions in liquid and vapour phase
should be the same.
H~c~ F=C-P+l=2-2+1=1
i.e .• azeotropes should behave as univariant systems, so that the boiling point
is constant, if pressure is fixed. This is what we will see in the next cases.
The characteristics of azeotropic mixtures are shown in figs. (14) and (15).
The boiling tem-
perature composition
diagram is shown in
figure (14). The con-
stant boiling mixture
has the maximum boil-
ing point, i.e., it is least
volatile. The vapour
phase for any mixture
lying between A and
M will, therefore. be
richer in A and any
mixture lying between
Band M, will be richer I
z :y
I

--+ Composition
boiling mixture M.
Suppose a mix-
ture of composition X
is distilled. The first
fraction distilled will have the composition indicated by Xl' Evidently, it is
richer in A. The composition ofthe residual liquid, thus shifts towards constant
boiling mixture M. As the distillation proceeds. the composition of the dis-
tillate changes towards A and that of the residue towards M. Ultimately, a
distillate of pure A and a residue of constant boiling mixture M will be
obtained.
Similarly, a mixture of composition lying between Band M, say Y will
ultimately provide on distillation, a distillate of pure B and a residue of
constant boiling mixture M.
It is, therefore, evident thaI any binary solution of this type, on complete
fractional distillation, can be separated into a residue of composition M and
a distillate of either A or B depending upon whether the initial composition
lies between A and M or between Band M, respectively. Thus, it is not possible
to completely separate such a binary mixture into pure components A and B
on distillation.
CHEMICAL AND PHASE EQUILIBRIUM 153

The mixture with the maximum boiling point is called maximum boil-
ing azeotrope and behaves as if it is a pure chemical compound of two
components, because it boils at a constant temperature and the composition
of the liquid and vapour is the same. But the azeotrope is not a chemical
compound, because its composition is not constant under conditions and rarely
corresponds to stoichiometric proportions.
Pure water and hydrogen chloride boil at 100° and -85°. while their
constant boiling mixture (azeotropic mixture) containing 20.25% of hydrogen
chloride boils at 108.5°, under a pressure of 1 atmosphere. If a solution
containing less than 20.25% of HCl is distilled, (i.e., between points A and
M), water will pass over as the distillate and the residue left behind in the
flask will consist of 20.25% solution of HCl in water. Thus, pure HCl can not
be obtained. Similarly, if a solution containing more than 20.25% HCl is
distilled, then pure HCl will pass over as distillate and the residue left behind
in the flask contain a mixture of the same constant composition, viz., 20.25%
HCl in water.
3. Solutions of type III. The boiling temperature-composition curve
for the liquid and vapour phases for such type of solutions is shown in
figure (15). The constant boiling mixture in this case has the minimum boiling
point, i.e., it is highly volatile.
Consider
distillation of compo-
the m;:========z======mrm
sition represented by
X (fig. 15). Then the A
first fraction collected
will have the compo-
sition X \. It will be
richer in the constant I

boiling mixture. The


composition of the re- Cl I
: (.'9(.'
"'et
sidualliquid will shift ~
'0
------r-------T----
I I

towards A. As distilla- !XI I


I
I
I

tion continues, the


i
I I
I I I I

composition of the x: Z: :Y1: Y


distillate and residual 100%
liquid changes to-
wards M and A, re-
spectively. By
repeating this pro-
cess, the mixture of
minimum boiling point of composition M will be obtained as distillate, while
the residue left over in the distillation flask will contain only pure liquid A.
If we distil a liquid of composition represented by Y, then the compo-
sition of the first fraction will be represented by Y \. Evidently, it will be richer
in the constant boiling mixture. The composition of the liquid will become
richer in B. As the distillation proceeds, the distillate and the residual liquid
154 PHYSICAL CHEMISTRY-I

will become richer and richer in constant boiling mixture and pure B. respec-
tively. Finally. the distillate will contain only the constant boiling mixture and
the residual liquid in the distillation flask will contain only B. There will be
no pure A in this case. If the mixture has the azeotropic composition (say Z),
it will distil unchanged.
In the system of water-ethanol. the point M corresponds to a minimum
boiling temperature of78.13° and a composition of95.57% ethanol by weight.
If any solution of composition of pure water and 95.57% ethanol is distilled,
then we get a residue of pure water and a constant minimum boiling mixture
of 95.57% alcohol in the distillate. No pure ethanol can be recovered. On the
contrary. if a solution of composition between pure alcohol and 95.57% ethyl
alcohol is distilled. then we get a mixture containing 95.57% ethanol and pure
alcohol. No pure water will be recovered. .
Solubility of Partially Miscible Liquid Pairs
In the study of miscibility of partially miscible liquid pairs, the exterpal pressure i
kept constant and, therefore, the vapour phase is Ignored. The mutual solubilities ar(
epresented by means of temperature-composition diagram.
On the basis of mutual solubility of two liquids, there are three types of liquid pair
such as :
Type I: Tlwse pairs whose mutual miscibility increases with increasing temper-
lature.
Type II: Those pairs whose mutual miscibility decreases with increasing tem-
'perature.
Type Ill: Those pairs wlwse mutual miscibility both increases and decreases
with increasing temperature.

Problem 16: Discuss the theory of partially miscible liquid pairs with
special reference to :
(i) Phenol-water system
(ii) Triethylamine-water system
(iii) Nicotine-water system.
Define the upper and lower critical solution temperature. What is the
effect of impurities on them? (Meerut 2(04)
If we consider two liquids A and B and shake together, then some of A
dissolves in B. while some of B dissolves in A. We then have two saturated
solutions-one of A in B and the other of B in A. On increasing the temper-
ature. the solubility of A increases in B and also drat of B in A. in this particular
case. As an example. we take the familiar phenol-water system.
[I] Phenol-Water System
Phenol and water are partially mi'scible at ordinary temperature. When
we add phenol to water phenol gets dissolved in water. till its concentration
reaches 8%. The addition of more phenol will give rise to two liquid layers.
One layer will consist of water in phenol system and the other that of phenol
in water system. Such solutions of different compositions co-existing in
equilibrium with one another are known as conjugate solutions.
CHEMICAL AND PHASE EQUILIBRIUM 155

As the temperature is
raised, the solubility of phe-
nol in water increases,
whereas that of water in Homogeneous
phenol also increases. Ulti-
mately at a certain tempera- ,,
ture, the two conjugate ,
solutions change into one :Tie line b
homogeneous solution. This ----t------
temperature is known as
critical solution tempera- I Heterogeneous
ture or consolute tempera- 200 A : C
ture. The value of consolute
temperature for this system B
is 68.3°, and the composi-
tion is 33% phenol and 67% o 203340 60 80 100
water. Above 68.3°, the two --+ Percent phenol
liquids are completely mis-
cible in all proportions. The
variation of mutual solubil-
ity of water and phenol with temperature is shown in fig. (16). The solubility
of phenol in water increases with rise of temperature along the curve AB,
while the solubility of water in phenol increases along CB. The two curves
do not intersect each other, but meet 'at a certain point B, known as C.S.T.
In between the area ABC, the system will be heterogeneous. If we
prepare a mixture of phenol and water of composition and temperature rep-
resented by any point within the area ABC, it will separate into two liquid
layers or binary conjugate solutions. Any mixture of composition represented
by a point outside ABC will give a homogeneous solution.
At any fixed temperature say to, the composition of each layer is fixed
as shown by the points a and b, a gives the composition of the aqueous layer,
while b gives that of phenol layer. The line ab is known as tie line.
The amounts of the conjugate solution may be easily calculated. For example,
suppose we have a mixture of phenol and water, whose total composition is 50% phenol
at a temperature of 40°C. Then the data of the table of fig. (14) indicates that the two liquids
formed have the respective compositions 7.5% and 67% phenol. Let x be the weight of
aqueous layer in 100 g of the mixture and, therefore, (loo-x) g will be the weight of
phenolic layer. Then,

XU~)+(l00-X>(I~ )=50
x=28.5
Therefore, lOOg of total mixture give rise to 28.5 g of aqueous layer and 71.5 g of
phenolic layer, or 285 x 0075 = 213 g of phenol is present in aqueous layer and
715 x 067 = 4790 g of phenol is present in the phenolic layer.
Other examples are (i) Water-aniline (167°), (ii) Benzene-aniline (59.so) (iii)Methyl
Alcohol-cyclohexane (45.5°), (iv) Bi-Zn (Metallic .\ystem) (85.0")
156 PHYSICAL CHEMISTRY-I

[II] Triethylamine-Water System


There are some cases, when solubility of one liquid in another decreases
with the rise in temperature. 9prrIT:2TI:ITrrillIT:2TI:ITTIillEJ]
The temperature I:::
composition curve for tri- ~
ethylamine and water sys- .a C
I!!
CD
tem is shown in fig. (17). Co
Triethylamine and water E
~
mix together in all propor-
tions below 19 0 but on rais-
ing the temperature above
i
19°C -------"'---,
19°, the system separates B
into two liquids layers. One phase
The curve AB shows 0% Amine . . 100% Amine
the decreasing solubility of 100% Water -+ ComposItIon 0% Water

triethylamine in water ':::JWB~i42~~ii14m.2.22EWJ


while curvers CB shows the ~~ilij[lli2iliili~~;;;±ilill2lligili]
decreasing solubility of Ii;;
water in triethylamine. The
two curves meet at B, which is the lower critical solution temperature or lower
consolute temperature of the system. Any point within the area ABC will gi ve
two liquid layers, while any point outisde the area ABC will give a homoge-
neous solution.
Methyl ethyl ketone and water has a lower consolute temperature of
lO°e.
[III] Nicotine-Water System
In some cases, it BIZJJZ:SEEZZ=ITEEZZ:==81
has been observed that
the mutual solubility One phase
curve is a closed curve
with both the types of
consolute temperatures.
The temperature Two
phases
composition curve for
this system is shown in
fig. (18). The system of
nicotine and water be-
i
60.BOC -----
One phase
longs to this class. At
temperatures below 100% Nicotine
0
60.8° and above 208 ,
the two liquids are com- Wg¥.Ig¥.[!4¥~~~@g¥.Ig4.4.4~
pletely miscible in all &lli22lli2llib~~ti2iliJ22bBd
CHEMICAL AND PHASE EQUILIBRIUM 157

proportions giving a homogeneous solution. Between 60.8° and 208°, they


are only partially miscible and give two layers. Hence, the solubility curve is
a closed curve. The upper and lower consolute temperature are 208° and
60.8°, respectively.
[IV] Influence of Impurities on Critical Solution Temperature
The critical solution temperature has a fixed value for a given system
and is completely defined. Its value is very sensitive to the presence of
impurities present in either or both the components. Hence, the determination
of C.S.T. gives us an accurate method of determining the presence of impurit-
ies qualitatively as well as quantitatively.
The effect of dissolved impurities on C.S.T. was observed by Crismer.
The C.S.T. of ethanol petroleum system was raised by 17°, by the presence
of only 1% of water in ethanol. The C.S.T. of methanol and cyclohexane
system is 45.55°. The presence of 0.0 1% of water in methanol raises the C.S. T.
of the ssytem to 45.65°. The presence of armoatic hydrocarbons in petrol can
be detected and estimated by determining the C.S.T. of petrol-aniline system.
Similarly, the amount of ceresin in wax can be determined. The biological
importance of C.S.T. is in testing the functioning of kidney. A kidney produc-
ing urine which raises the C.S.T. of urine-phenol system by 8° is in good
order. The kidney is exceedingly well if the C.S.T. is raised by 12° to 16°.
In general, the C.S. T. of the system is raised if the impurity is soluble
in only one of the two components and the C.S. T. is lowered if the impurity
is soluble in both the components.
Advantage is taken of the above principle in the preparation of well
known disinfectant lysol. Lysol is a system of cresols and water. These two
components do not mix completely at ordinary temperatures, but the addition
of soap to the given mixture-soap is soluble in both cresol and water-lowers
the C.S.T. to such an extent that the two components readily mix with one
another at ordinary temperature to form homogeneous solution.

DISTRIBUTION LAW
Problem 1 : State and explain Nernst's distribution law. What are the
limitations of this law? Give some important applications of distribution
law with special reference to extraction process.
(Meerut 2004, 20(H, Kanpur 2(05)
Or Define and discuss distribution law. (Meerut 2(05)

[I] Distribution Law


Nemst found that if a substance is present in two immiscible solvents
in a system at equilibrium then the solute distributes itself between the two
immiscible solvents in such a way that at constant temperature, the ratio of
its concentrations in the two solvents is constant, whatever the total amount
of the solute may be.
158 PHYSICAL CHEMISTRY-I

If C 1 and C2 represent the concentration of a solute in the two solvents,


then
C1
-C = constant = K •
2
where K is known as distribution or partition coefficient.
Nemst found that the ratio C 1/C 2 is constant only when the solute has
the same molecular species in both the solvents. If a solute associates to form
double molecules in one solvent and not in the other, the equilibrium cannot
exist between double molecules present in one solvent and single molecules
present in the other. The law is valid only if the ratio of concentrations of
single molecules in the two solvents is taken into consideration. The distri-
bution law as enunciated by Nernst does not hold good in cases where the
solute undergoes dissociation or association in any of the solvents. For ex-
ample, if a solute remains unaltered in one solvent and undergoes partial
dissociation in another, the ratio of total concentrations in the two solvents
will not be constant, but the ratio of concentration of undissociated molecules
in the two solvents would be constant.
Thus, the distribution law in its proper form may be stated as,
'When a solute distributes itself between two immiscible solvents in
contact with one another, there exists for similar molecular species at a
given temperature, a constant ratio of concentration between the two sol-
vents, irrespective of any other species which may be present.'
Thus, in the above equation, C 1/C 2 == K, the terms C 1 and C2 refer to
concentrations of similar molecular species in the two liquids at constant
temperature.
As the solubility of a solute changes with temperature, and as the
magnitude of the change in the two solvents may not be the same, the
distribution coefficient (K) is found to vary with change in temperature.
[II] Limitations of Distribution Law
The important conditions to be satisfied for the application of the
distribution law are as follows :
(i) Constant temperature: The temperature should be kept constant
throughout the experiment.
(ii) Same molecular state : The molecular state of the solute should
be the same in both the solvents. The law fails if the solute dissociates or
associates in one of the solvents.
(iiii) Non-miscibility of solvents : The two solvents should be non-
miscible or only slightly soluble in each other. The extent of mutual solubility
of the solvents remains unchanged by addition of solute to them.
(iv) Dilute solutions : The concentration of the solute in the two
solvents should be low. The law fails when the concentrations are high.
CHEMICAL AND PHASE EQUILIBRIUM 159

(v) EquiHbrium concentrations : The concentrations of the solute


should be noted after the equilibrium has been established.
[III] Applications of Distribution Law
Distribution law helps us in calculating the degree of dissociation or
association of a solute. Distribution indicators also involve the principle of
distribution law.
1. The process of extraction : The most common and important ap-
plication of distribution law is the solvent extraction of substances by sol-
vents. Organic compounds are more soluble in organic solvents like
CCl4 • C6Hc; etc., than in water and so in the laboratory, this principle is used
for the removal of a dissolved substance from aqueous solution by using
organic solvents. Since organic compounds have their distribution ratio
largely in favour of organic phase, "most of the organic substances would pass
into non-aqueous layer. Finally, this non-aqueous layer is removed and dis-
tilled to obtain the pure compound.
In solvent extraction, it is advisable to use a given volume of an
extracting liquid in small lots in successive stages rather than in one single
operation at a time. The greater the distribution ratio is in favour of the
organic solvent, the greater wiU be the amount extracted in anyone oper-
ation.
Suppose a solute A is present in 100 c.c. water and that 100 C.c. of
benzene is used for its extraction. Let the distribution coefficient of A between
benzene and water be 4.
K =Concentration of A in benzene 4
.. Concentration of A in water
(i) When the whole of benzene (100 c.c.) is used at a time for
extraction, suppose Xl g of solute pass into benzene layer and x2 g is left in
aqueous layer, so
xI/lOO
x2/ 100 =4;
Xl XI 4
i.e., -=4 or - - = -
X2 XI +X2 5
In other words, 100 c.c. benzene has separated 4/5 or 80% of the solute
originally present.
(ii) Now let us use 100 c.c. benzene in two successive extractions,
using 50 c.c. each time. Then in the first extraction,
Xl/50
X2/ l00 =4;

i.e.,
160 PHYSICAL CHEMISTRY-I

In other words, in the first extraction (2/3)rd, i.e., 66.6% is extracted.


Hence, (l/3)rd or 33.4% of the original amount is still retained in aqueous
medium. In the second extraction using 50 C.c. of benzene, we shall further
extract (2/3)rd, of (l/3)rd, i.e., (2/9)th of the original amount. So, in both the
extractions, using 100 c.c. benzene we can separate (2/3 + 2/9) = 8/9 or
88.9% of the original amount of the solute. Thus, a two stage extraction is
more efficient. If we use four or five extractions, the operation wiIi still be
more efficient.
Derivation of General Formula
.It is possible to derive a general expression for the amount remaining
unextracted after a given number of operations. Let V C.c. of a solution
containing W g of the substance be extracted with v C.c. of a solvent. Let
WI g of substance remain unextracted in aqueous layer. Then
Concentration of substance in solvent
W-WI
v
Concentration of substance in water
WI
- V
:. Distribution coefficient
K= WI/V
W-WI
v
KV(W- WI) KV
or WI = V
=W·--
KV+v
If W2 be the amount remaining unextracted at the end of the second
extraction with v C.c. of the solvent, then

w2 -W KV -W KV KV _W(~)2
- I KV + v - KV + v . KV + v - KV + v
Similarly, after nth extraction, the amount remained unextracted will
be given by,

Wn = W ( KVKV+ vIn ... (8)


If the entire quantity of the extracting solvent is used in one lot, then
unextracted amount (W) will be given by,

W=W(K~~V) ...(9)
Since the quantity within the bracket is less than unity, (8) is smaller
than (9) and Wn will be smaller, the greater the value of n. Hence the efficiency
of extraction increases by increasing the number of extractions using only a
CHEMICAL AND PHASE EQUILIBRIUM 161

small amount of the extracting solvent each time. It must be remembered that
the value of K, the partition coefficient in equations (8) and (9) is that of the
substance between the solvent (A) in which the substance is already dissolved
and the solvent (B) which is used for extraction.
2. Determination of association: When a solvent is associated in
solvent 1 and exists as single molecules in solvent 2, the distribution law is
written as,
'vc;- = K
Cz
where n = number of molecules which combine to form an associated mole-
cule. Thus, knowing the values of C 1, Cz and K, we can calculate the value
ofn.
3. Determination of dissociation: Suppose a solute is dissociated in
aqueous solvent 1 and exists as single molecule in solvent 2. If a is the degree
of dissociation of the solute, the distribution law is written as :

Thus, a. can be calculated.


4. Confirmatory test for bromide and iodide: The salt solution is
treated with chlorine water, when a small quantity of Br2 or Iz is liberated.
This solution is then shaken with CCl 4 or CHCI 3. On standing, the CCl 4 or
CHCl3 layer forms the lower layer. The free Br2 or 12 being more soluble in
CCl4 or CHCl 3 concentrates into the lower layer, making it brown for bromine
and violet for iodine.
S. Determination of solubility : If the concentration of a solute in
solvents 1 and 2 be C 1 and C2, then distribution coeffici~nt (K) is given by,
K=C1/C z·
As concentration and solubility (S) are proportional to each other, we
can write,
K=S I /S2 ·
If solubility (SI) of a solid in one solvent is known then solubility
(S2) in other solvent can be calculated.
6. Distribution indicators: In iodine titrations, the end point is indi-
cated by adding starch solution which turns blue. A greater sensitivity is
obtained by adding a distribution indicator. A few drops of an immiscible
organic solvent say CCl4 is added to the solution. The bulk of any iodine
present passes into CCl 4 layer making its colour more intense.
Besides, distribution law is applicable in deducing the formula of a
complex ion, in desilverisation of lead, partition chromatography etc.

Problem 2 : State and explain Nernst's distribution law. How it is modified


when the solute undergoes association or dissociation in one o/the solvents?
162 PHYSICAL CHEMISTRY-I

[I] Nernst distribution law


See problem 1.
[II] Dissociation of the solute in one of the solvents
Let A represents the normal
formula of the solute. It is not disso-
ciated in solvent I, but dissociates Solvent I
into X and Y in the second solvent No dissociation
II (figure 19). Let C 1 and Cz be the Conc. = C1
concentrations in solvents I and II,
respectively. The distribution law is
valid only for the ratio of concentra-
t~--->. X + y Solvent II
Dissociation
tions of similar molecular species in Occurs
both the solvents. Total conc. =C2
The equilibrium is represented
as:

A~X + Y
C2 (1 - a) C2a C2 a
where, a is the degree of dissociation of solute A in solvent II.
According to modern distribution law.
K = Concentration of A in solvent I = C1
Concentration of A in solvent II C2 (1 - a)
Such a case is observed in distribution of oxalic acid between water
and ether.
[III] Association of the solute in one of the solvents
Let A represents the normal
formula of the solute. It does not
associate in solvent I, but associates Solvent I
in solvent II to give molecules of No association
Conc. = C1
the type An (figure 20). Let C 1 be
the concentration of solute A in sol- I~ Solvent II
vent I and C2 be its total concentra- nA~A"
Association
tion in solvent II. occurs
In solvent II, the associated
. molecules exist in equilibrium with Total conc. <:2
single molecules, viz.,
An~nA
According to the law of mass
action,
CHEMICAL AND PHASE EaUILIBRIUM 163

or ... (1)
If the solute exists largely as associated molecules in solvent II, the
concentration of the associated molecules (An) may be taken to be equal to C2,
i.e.,
[An] = C2
Therefore, from equation (1), we get
[A] = constant x (C2)lIn = kC 2 l1n .
Applying the distribution law to similar molecular species, we have,
K = Concentration of A in solvent I
Concentration of A in solvent II
or K= Cj
k (C2)1I2
C1
or -vi" = K . k =constant.
(C2)
Such a case is observed in the distribution of benzoic acid between
water and benzene.

NUMERICAL PROBLEMS
Problem 1 : The following data shows the distribution of phenol between
water and chloroform :
Concentration (cJ) in water 0.094 0.163 0.254
0.436
Concentration (C2) in chloroform 0.254 0.761 1.85
5.43
Calculate the partition coeffr.cient between water and chloroform. What
conclusions can be drawn from these results concerning the molecular
condition of phenol in chloroform layer?
Solution : Phenol in chloroform may be present either as normal
molecules or in the form of associated molecule», In the former case, Cl/c2
should be constant, whereas in the latter cl/c211n should be constant, where
n is the number of molecules of phenol associate to give a single associated
molecule.
The value of c 1/c2 in the four cases are given as follows:

(i) ~: = ~:~~: =0.3701, (ii) ~~ = ~:~~i = 0.2142,


( "') Cj
111 C2
= 0.254 = 0 1373
1.85 . , (iv) ~: = ~~36 = 0.0833.
Since values of cil C2 are not constant, so phenol does not exist as single
molecules in chloroform.
164 PHYSICAL CHEMISTRY-I

CI
Now, the values of t= are given as follows :
VC2

. CI 0.094 CI 0 163
(I) -:rc;=";(0.254) 0.1865, (ii)-;rc;=,,;(O.761) =0.1868,

(
... ) CI 0.254 0 1867 (.) CI 0.436 01870
11l -rc;=";(l.85) . , IV -rc;=";(5.43) . ,
Since c l /";C2 values are constant, therefore, phenol exists as double
molecules in chloroform.
Problem 2 : In the distribution ofsolute between water l ) and chloroform eC
(C2 ) the following data were obtained:
CI C2
0.163 0.761
0.436 5.43
What information do you gather regarding the molecular state of solute
in chloroform?
Solution: When the given solute exists as simple molecule in both
the phases, then C I /C 2 = K.
CI
C1 C2 -=K
(In water) (In chloroform) C2
0.163 0.761 0.163 = 0 214
0.761 .

0.436 5.43 ~~36 =0.083


As the values of K are not constant, the distribution law in its simple
form is not applicable. It means that the molecular state of the solute in both
the phases is not the same. It may be either associated or dissociated.
Assuming the association of the solute in chloroform as double molecules,
then C j /";eC2) = K:
C
Cj C2 :.rc;=K
I

0.163
0.163 0.761 ~(0.761) = 0.1868
0.436
0.436 5.43 ~(5.43) = 0.1870
As the values of K are constant, the solute exists as double molecules
in chloroform layer.

Problem 3 : (Jllculate how much succinic acid would be extracted from


100 C.c. of water containing 5 g of succinic acid if extracted with 50 C.c. of
CHEMICAL AND PHASE EQUILIBRIUM 165

ether. The partition coefficient of succinic acid between water and ether is
5.5. Succinic acid has normal molecular weight in both the solvents.
(Meerut 2005)
Solution: Let x g of succinic acid passes over into 50 c.c. of ether, then
the amount left unextracted in 100 c.c.water will be (5 - x) g.
:. Concentration of solute dissolved in ether
=x g per 50 c.c.
Concentration of solute left in water
= (5 - x) g per 100 c.c.
From the distribution law,
CI
K=-
C2
= (5 -x)/lOO
or 5 .5 x/50
or x =0.4166 g.
Amount of succinic acid extracted = 0.4166 g
Problem 4: At 20·C, S02 was permitted to distribute between 200 C.c. of
chloroform and 75 c.c. water. At equilibrium, the chloroform layer con-
tained 0.14 mole ofS02 and water layer 0.05 mole. Calculate the distribution
coefficient of S02 between water and chloroform.
'
o ution
SI : Q~ mo Ies per c.c.
Cw = --:ys

0.14 I
CCHCI3 = 200 mo es per c.c.

K=~= 0.05 x 200 =0.953


CCHCI, 75 x 0.14
Problem 5 : If the distribution coefju:ient of benzoic acid between water
and C6H6 is 0.304 at 20·C, calculate the number of moles of benzoic acid
which may be extractedfrom 100 C.c. of 0.02 molar aqueous solution by 10
C.c. of C6H6'
Solution: Let x moles of acid remain unextracted. The concentration
of the acid in benzene will be
0.02-x
10 perc.c.
Also the concentration of the acid in the original aqueous solution is
x 1100.
x/100
K = (0.02 - x)/l0
x 10 x
or 0.304 = 100 x (0.02 - x) 0.02 - lOx
x = 0.015 mole
166 PHYSICAL CHEMISTRY-I

:. Number of moles of acid extracted


= (0.02 - 0.015)
=0.005
Problem 6 : The solubility 0/ a substance is twice as great in ether as in
water. Compare the quantities extracted/rom 100 C.c. 0/ aqueous solution
by using:
(i) 100 c.c. ether in single operation.
(li) 50 c.c. ether in successive operations.
Solution: (i) By using 100 c.c. ether at a time.
Let 1 g of substance be present in 100 c.c. aqueous layer and let by
using 100 c.c. ether at a time, x g of it passes over into ethereal layer.
Then from distribution law,
CI
K=-
C2
or 2=_X_ (': K=2 given)
I-x
x= 0.66 g
(ii) By using 50 c.c. ether in two instalments.
Illfirst instalment: Let by using 50 c.c. ether Xl g of substance passes
over from 100 c.c. aqueous layer.
Concentration of substance dissolved in ether
= Xl g per 50 c.c.
Concentration of substance left in aqueous layer
= (1 - Xl) g per 100 c.c.
Hence from distribution law,
CI
K=-
C2
Xl

2:::: 50
(1- Xl)
100
or Xl =O;Sg
III second instalment: The amount of substance now left in aqueous
layer is 1~.5 g, i.e., 0.5 g. Consider by using next 50 c.c., X2 g of substance
passes over from 100 c.c. aqueous layer.
Then. concentration of substance dissolved in ether
= X2 g per 50 c.c.
Concentration of substance left in aqueous layer
= (0.5 - x~ g per 100 c.c.
CHEMICAL AND PHASE EQUILIBRIUM 167

Hence, from distribution law, K = C I /C 2


X2

or 2= 50
(0.5 -x2)
100
or X2 = 0.25 g
So, total amount of substance extracted in two instalments
= 0.5 + 0.25 = 0.75 g
Thus, it is clear that it is more advantageous to use 50 c.c. ether in two
instalments (as it extracts 0.75 g) rather than 100 c.c. 'in one instalment (as it
extracts only 0.66 g).
Problem 7: From the following data for the distribution of C~sCOOH
between water and benzene at 20·C :
(a) Show that C~sCOOH is associated as double molecules in benzene,
and
(b) Caku1ate the partition coefficient on the basis ofthis assumption. The
dissociation of C6H sCOOH in water may be neglected.
CHp = 0.015, 0.0195, 0.0289
Cell, = 0.242, 0.412, 0.970.
Solution: If benzoic acid remains normal in both the solvents, then,
K = CCH20 = 0.0 15 = 0.0619 ... (1)
C.H6 O. 24 2
= 0.0195
0.412
=0 .0461 ..,
(2)

= 0.0289
0.970
= 0.2980 ..,
(3)
Since the values of K are not constant, it is clear that benzoic acid does
not exist as normal molecules in both the solvents.
Suppose benzoic acid associates as double molecules in benzene, then
K = Cwater/'l/(Cbenzene)
(i) K = 0.0151"(0.242) =0.304
(ii) K = 0.0195/'1/(0.412) = 0.303
(iii) K =0.0289/"(0.970) =0.293
.. ff' . 0.304 + 0.303 + 0.293
S0, partitIon coe tClent = 3
=0.300
Problem 8 : 1 g of 12 is in 50 ml of CS2• 1000 ml of water is added into it.
Cakulate the amount of h extracted into water. Partition coe/ficiellt .?( h
in CS2 and H 20 is 200.
Solution : Suppose x g of h is extracted into water. Then, amount of
12 left behind in 50 m1 of CS 2 will be (1 - x) g.
168 PHYSICAL CHEMISTRY-I

~
Conc. of 10 In CSo
K == - -_._--._':"
50
=: - -
Conc. of 12 In H20 X

1000
~~
200==~0-- or x == 0.0909 g
x
1000
So, amount of 12 extracted in waler == 0.0909 g
Problem 9 : At 25°C, the distribution coefficient of iodine between CCl4
and water is 85. If at 25°C, the solubility of iodine in water is 0.33 gllitre,
determine the solubility of iodine in CCI,Jo
Conc. orI z In CCI 4 Solubility Ofl2 in CCI 4 (S])
Solution: K - - - - -=----
- Conc. of 12 in H 20 Solubility Ofl2 In H 20 (S2)
Sl
or 85=---
0.33 g/litre
or Sl .= 85 x 0.33 g/litre
...: 28.05 g/Iitre

MULTIPLE CHOICE QUESTIONS


1. The vapour pressure of water at lOOT I~ 760 mm. The molar heat of vaporisation
of water is 41.27 kJ mar I The vapour pressure of water at 95"C is :
(a) 63.43 mm (b) 760 mm (c) 740.6 rum (d) 634.3 rum
2. The maximum work done In a reactIOn is given by:
(a) van't Hoff isotherm (b) van't Hoffisochore
(c) Clausius-Clapeyron equatIOn (d) None
3. The variation of equilibrium constant with temperature is given by :
(a) van't Hoff isotherm (b) van't Hoff isochore
(c) Law of mass action (d) Le Chatelier's principle
4. The variatIOn of vapour pressure with temperature is given by :
(a) van't Hoff isotherm (b) van't Hoff isochore
(c) Law of mass action (d) Clausius-Clapeyron equation
5. In the equilibrium A + 2B ~ C + D, increase in concentration of A will give
more:
(a) B (b) A (c) C (d) No effect
6. In the equilibrium 2A + 3B ~ 3C 1 2D, an increase of pressure will :
(a) shift the equilibrium towards the left
(b) shift the equilibrium towards the nght
(c) have no etl'ect
(d) may shift the equiliblium In both directions
7. In a one component system, the maximum number of phases which can exist in
equiliblium at a point is
(a) 0 (b) I (c) 2 (d) 3
8. In water system, the triple potnt has the values of :
CHEMICAL AND PHASE EQUILIBRIUM 169

(a) O·C, 1 atm (b) O·C, 4.58 mm


(c) 0.OO75"C, 1 atm (d) 0.OO7S·C, 4.58 mm
9. At congruent melting point, the system is :
(a) Non-variant (b) Monovariant
(c) Bivariant (d) Trivariant
10. At eutectic point, a system has:
(a) Only two phases (b) Highest melting point
(c) Minimum melting point (d) Uncertain composition
11. At congruent melting point, Zn - Mg system is a :
(a) One component system (b) Two component system
(c) Three component system (d) Three phase system
12. Henry's law is not applicable to :
(a) H2 (b) O2 (c) CO2 (d) He
13. In lead-silver system, the percentage of silver at the eute~tic point is :
(a) 0% (b) 2.6% (c) 97.4% (d) 100%
14. The dissociation of CaC03 is a :
(a) One component system (b) Two component system
(c) Three component system (d) Two phase system
15. For an ideal solution:
(a) Mlmix -70 (b) M-Imix < 0 (c) ilHmix = 0 (d) ilHmix "? 0
16. A mathematically constant ratio exists between concentrations of similar molec-
ular species in any two phases in contact with each other at a constant temper-
ature. This law is known as :
(a) Boyle's law (b) Charle's law (c) Distribution law (d) Dilution law
17. Which one of the statements is false for distribution law?
(a) Temperature must remain constant
(b) The two solvents must be mutually immiscible
(c) The concentration of the solute in two solvents must be kept very high
(d) The concentration of the solute in two solvents must be very low
18. When a solute is shaken with water and a solvent X in which it forms a dimer.
the concentrations were found to be CIV and C., in water and solvent, respectively.
Which one of the following expression will be correct between water and X ?
(a) C, (b) Cw (c) C.,. (d) -rc;
Cw rc; C" C.,.
19. Distribution law cannot be applied for the system in which 12 is distributed
between:
(a) H20 and CS 2 (b) H20 and CCl 4
(c) H20 and ether (d) H20 and alcohol
20. In a pair of immiscible liquids a common solute dissolves in both and eqUilibrium
is reached. then the concentration of the solute in the upper layer is :
(a) In fixed ratio with that in the lower layer
(b) Same as in lower layer
(c) Less than in the lower layer (d) None of these
21. A 9S·C, an aqueous solution of iodine containing 0.0156liC! is in equilibrium
with a CCl4 solution containing 4.412 g lie!. If the solubility of 12 in water at
9O"C is 0.34i lie l , then its solubility in CCl4 is :
(a) 4.412 x 0.0516 (b) 0.0516 x 0.34
0.34 4.412
170 PHYSICAL CHEMISTRY-I

() 4.412 x 0.34 (d). 0.0516


c 0.0516 4.412 x 0.34
22. The solubility of iodine in water is 0.8 gllitre. Ifthe partition coefficient of iodine
between water and carbon tetrachloride is 82, the solubility of iodine in carbon
tetrachloride is :
(a) 102.5 g/Iitre (b) 65.6 gllitre (c) 0.009 gllitre (d) 81.2 gllitre
23. The condition under which Nernst distribution law will not hold true is :
(a) Temperature is constant
(b) Molecular state of the solute is same in both solvents
(c) The solute does not cause any change in the mutual solubility of the two
solvents
(d) None of them
24. You are given 100 mL of CCl4 to extract iodine from 200 mL of its aqueous
solution for extracting maximum amount of iodine. Which one of the following
processes would you use ?
(a) All 100 mL of CCl 4 (b) 50 mL of CCl 4 twice
(c) 10 mL of CCl 4 ten times (d) 25 mL of CCl 4 four times

Fill in the Blanks


1. van't Hoff equation gives the variation of ........ with temperature.
2. The equilibrium constant ........... with increase of temperature.
dP MI' .
3. dT = T!1V IS k nown as ........... equatIOn.

4. Kp and K,. are related by the equation .............. .


5. Phase rule was given by ............... .
6. In ferric chloride-water system, the number of hydrates formed are ..... in number.
7. Dissociation of NH4Cl in a closed vessel is a .......... component system.
8. In Zn-Mg, the intermetallic compound has the composition ............ .
9. Phase rule equation is given by .............. .
10. In a two component system, the maximum degree of freedom is ......... .
11. In a system of H 2, 02, N2 gases, the number of phases will be ............. .
12. The eutectic temperature of Pb-Ag system is ............. .
13. The solubility of triethylamine in water ., ....... with increasing temperature.
14. Benzoic acid ............... as double molecules in benzene.
15. The temperature during the distribution law experiment should be ......... .
16. Distribution law was given by ........... .
17. When iodine is distributed between water and CCI 4 , the solute exists in the ......... .
molecular state in both the liquids.

True or False
State whether the following statements are true (T) or
false (F) ?
1. P2 !1Hv
10glO PI = 2.303 R
(I 1"
T2 - TI represents integrated formofClausius-Claperyon

equation.
2. Clapeyron equation can be applied to an equilibrium between any two phases
of the same substance.
CHEMICAL AND PHASE EQUILIBRIUM 171

3. van't Hoff isochore gives the temperature dependence of the vapour pressure of
a substance.
4. In the equilibrium N2 + 02 ~ 2NO, !l.H = x kJ, an increase of temperature will
shift the equilibrium towards the right.
[At' [Btl
5. In the reaction, nlA + n~B ~ mlC + m2 D, K~ = .
- , [C]m, [D]m 2
~. Nicotine-water has only one consolute temperature.
7. There is only triple point in sulphur system.
8. Henry's law is applicable to the solubility of H2 in water.
9. The eutectic point in KI - H 20 system is also known as cryohydric point.
10. In a two component system, three phases exist in equilibrium at the eutectic
point.
11. NaCI - H20 system belongs to a system in which the compound has an incon-
gruent melting point.
12. For non-ideal solutions, !l. Vmix is greater or less than zero.
13. HCl - H20 system has a minimum boiling azeotropic mixture.
14. Critical solution temperature does not change with pressure.
15. The two solvents should be soluble in each other, if distribution low is valid.
16. The solutions should be concentrated in distribution experiment.
17. The process of extraction of a solute is based on distribution law.
18. The degree of dissociation of NaCI can be calculated from distribution law
experiment.
19. If K = -rc;;ICB,then solute exists as dimer in solvent B.
ANSWERS
Multiple Choice Questions
1. (d), 2. (a), 3. (b), 4. (d), 5. (c), 6. (c), 7. (d), 8. (d), 9. (a), 10. (c)
11. (a) 12. (c) 13. (b), 14. (b), 15. (c), 16. (c), 17. (c), 18. (b), 19. (d), 20. (a),
21. (c), 22. (c) 23. (d) 24. (c)
Fill in the Blanks
1. equilibrium constant 2. increases 3. Clapeyron
4. Kp = Kc (RT)'~N 5. Gibbs 6. four
7. one 8. MgZn2 9. F=C-P+2
10. Jhree 11. one 12.303"C
13. decreases 14. associates 15. constant
16. Nemst 17. same
True or False
I. (T) 2. (T) 3. (F) 4. (T) 5. (F) 6. (T)
7. (T) 8. (F) 9. (F) 10. (F) II. (F) 12. (F)
13. (T) 14. (T) IS. (F) 16. (F) 17. (T) 18. (F)
19. (T)

000
(OllODIAl STATE
Problem 1 : Explain the terms colloidal state and colloidal solution. De-
scribe various methods used in the preparation of colloidal solutions. Also
mention the different methods used for the purifICation of colloidal solu-
tions. (Meerut 2002; Agra 2005,2003,2001; Kanpur 2005,2000)

Thomas Graham (1861) divided soluble substances into two classes.


(i) Crystalloids, i.e., salt, urea, sugar and other crystalline substances
which could rapidly diffuse through vegetable and animal membranes. Such
solutions are called true solutions in which the diameter of the dispersed
particles ranges from 1 Ato 10 A.
(ii) Colloids, i.e., starch, gelatin, gum, proteins and other amorphous
substances which could not diffuse through vegetable and animal membranes
(Greek; kola = glue and eiodos = like).
In a suspension, as sand stirred into water, the dispersed particles are
aggregates of millions of molecules. The diameter of these particles is of the
order of 2000 A or more.
A colloidal solution is regarded as an intermediate between a true
solution and coarse suspension.
When the diameter of the particles of a substance dispersed in a sol vent
ranges from about 10 A to 2000 A, the system is called colloidal solution,
colloidal dispersion or simply a colloid. The material with particle size in the
colloidal range is said to be in the colloidal state.
CoUoidal solution is a heterogeneous system consisting of two phases.
(a) Dispersed phase: The substance whose particles are distributed in
a medium is called the dispersed phase.
(b) Dispersion medium: The medium in which the particles of the
dispersed phase are distributed is called the dispersion medium.
Preparation of Colloidal Solutions
Colloidal solutions (also known as sols) can be prepared by different
methods depending on the nature of the substances. Many substances., e.g.,
gelatin, starch etc. form colloidal solutions by merely dissolving them in
water. Metals and inorganic substances are brought into the colloidal state by
special methods. Two types of methods are mainly used:
(172)
COLLODIAL STATE 173

(i) Dispersion methods.


(ii) Condensation methods.
1. Dispersion methods are those in which the particles of bigger size
are broken into smaller colloidal particles. The different ways are :
(i) Mechanical dispersion : The substance to be dispersed is sus-
pended in dispersion medium to form a coarse suspension. This suspension
is passed through a colloid mill which consists of two metal discs rotating in
opposite directions at a speed of about 7000 revolutions per minute.
(ii) Electro-dispersion (Bredig's arc method) : This method is gener-
ally employed in preparing hydrosols of metals, i.e., Ag, Au, Cu. Pt etc. Two
rods of metal to be dispersed are kept immersed in cold water containing
some KOH and a direct electric arc is struck between them [Fig. (1)]. The
vapours of the metal condense to form colloidal particles. The traces of alkali
act as stabilising agent.

/+---- Metal Electrodes

De-Ionised Water +
r----- Traces of Alkali

::;;E-=--~:ij:~t-- Spark

(iii) Peptisation: If a freshly prepared Fe(OHh precipitate is treated


with a small quantity of FeCl3 solution, it immediately forms a dark reddish
brown colloidal solution of Fe(OHh. Similarly, a colloidal solution of
AI(OHh is obtained when freshly precipitated Al(OHh is treated with a small
quantity of dil. Hel, the amount of acid added being insufficient to convert
the hydroxide completely into chloride.
The process of transferring back a fresh precipitate into colloidal form
is known as peptisation.
It is the converse of coagulation or in other words, it is the re-dispersion
of a coagulated sol. Substances like FeCl 3 and HCl which bring about I-~P~
tisation are known as peptising agents.
174 PHYSICAL CHEMISTRY-I

Cause of peptisation : The peptising action is due to the preferential


adsorption of one of the ions of the electrolyte. which then gives to the
colloidal particle a positive or negative charge. according to the charge on
the adsorbed ion. For example. Fe(OHh adsorbs Fe3+ ions from FeCI 3 (pep-
tising agent) and thereby gets a positive charge on the surface. Similarly.
charged particles get separated yielding smaller sized colloidal particles of
the type [Fe(OHhJ Fe 3+.
Similarly. AS 2S3 precipitate obtained by pasing H2S through arsenious
oxide solution peptises on treatment with excess of H2S and yields a nega-
tively charged sol of [AS2S3J S2-.
2. Condensation methods are those in which the substances forming
a true solution are converted into colloidal form. These are as follows:
(i) By chemical reaction: The methods are :
(a) Double decomposition: It is used in the preparation of sols of
sulphides. e.g.• sols of AS 2S3 and HgS are obtained by passing H2S through
their salt solutions.
A solution of 1.0 g AS20 3 in 500 ml or'boiling water is obtained. It is
cooled and H2S gas is allowed to pass slowly. When the intensity of yellow
colour of the sol does not increase further. the passing of H2S is stopped.
ASP3 + 3H2S -7 As2S3 + 3H20
The excess of H2S is removed either by boiling the colloidal solution
or by bubbling H2 through it. when a yellow sol of AS2S3 is obtained.
(b) Hydrolysis: A colloidal solution of ferric hydroxide is obtained by
adding a freshly prepared saturated solution of ferric chloride (2 or 3 c.c.)
drop by drop to 500 ml of boiling distilled water. Ferric chloride hydrolyses
to give a brown sol of ferric hydroxide.
FeCl 3 + 3H20 -7 Fe(OHh + 3HCl
The hydrochloric acid formed and unhydrolysed ferric chloride are
separated by dialysis to get pure Fe (CHh sol.
(c) Reduction: A colloidal solution of gold (or purple of Cassius) is
obtained by reduction of auric chloride with stannous chloride or by HCHO.
2AuCl 3 + 3SnCl2 -7 2Au + 3SnCl4
2AuCl 3 + 3HCHO + 3H20 -7 2Au + 3HCOOH + 6HCl
A trace of K2C03 is added to neutralise the acid formed whose presence
renders the sol unstable.
(d) Oxidation : A colloidal solution of sulphur is obtained by the
oxidation of HzS with dil. HN03 or sulphurous acid.
3H zS + 2HN03 -7 2NO + 4H20 + 3S
2H2S + H2S03 -7 3HzO + 3S
The colloidal solution of sulphur so formed is treated with saturated
salt solution when sulphur is precipitated. It is filtered. washed with water to
COLLODIAL STATE 175

remove salt and then washed further when the precipitate of sulphur peptises
and forms a colloidal solution.
(ii) By exchange of solvent, i.e., by lowering of solubility : Colloidal
solution of a substance can be prepared by dissolving it in one solvent and
pouring this solution ill another solvent in which it is less soluble, e.g., a
sulphur or phosphorus sol can be prepared by adding their saturated alcoholic
solution in water.
(iii) Passing vapours of an element into a liquid: When the vapours
of a boiling element are passed through a liquid, condensation takes place
sometimes with the formation of a soluble sol, e.g., mercury sol can be
prepared by passing a stream of vapours from the boiling element into cold
water containing a suitable stabilising agent.
Purification of Colloidal Solutions
The colloidal solutions prepared by the above methods contain impurit-
ies. These impurities must be removed in order to make the sol stable. This
is done by either of the following methods.
1. Dialysis : The purification of colloidal solution by this method is
based on the inability of the sol particles to pass through an animal membrane
or a parchment paper which allows only the molecules or the ions to pass
through. The vessel in which dialysis is carried out is known as dialyser
[Fig. (2)]. A dialyser consists of a special type of vessel open at both the
ends. To the lower end a membrane is stretched. This membrane allows only
the solvent and other molecules to pass through it, but it is impermeable to
the colloidal particles. The dialyser is then suspended in a larger vessel

::+---1: lied to stand


r--- Dialysis bag ~
~------~ ~ ~~--------
/·e \ I
~ ,'. • ~Sol particle +
"e " ~
e "e +' ~. Molecule
Water +,Ions +
Molecules

~
'.
:e • ·e-
.' R'
+ Ions

·.x·"--1--/~,"
~ -e
~
Fresh
water.--
e .~

~----------------------------~
Fig. 2
176 PHYSICAL CHEMISTRY-I

containing pure distilled water. The distilled water is periodically renewed by


allowing water to flow into the outer larger vessel and removing it by means
of a syphon. Dialysis is continued till no further contamination of the washing
liquid occurs.
2. Electrodialysis: In this process, dialysis is carried out under the
influence of electric field (figure 3) Potential is applied between the metal
screens supporting the membranes. This speeds up the migration of ions to
the opposite electrodes. So, dialysis is greatly accelerated.

~n
DiaIYSiS bag
Olecule
Sol
particle " + '. +
~,'-.\ - I
Impure
colloidal (f) /"e - \ +
solution e ",/ Positive-ion '. -
Gi....-..
e Water + Ions
+ Molecules
I-&--- e
Anode
I ...e. , Cathode
'. Negative ion "
\ I
rD,
~ + •__ -...... ~~
' . . ....... I

Fresh
water. Watere
------------------------~
Fig. 3

Electrodialysis is not meant for the removal of non-electrolytic impurit-


ies like sugar, urea etc.
3. Hot dialysis: In hot dialysis, a temperature of 65-90°C increases
the rate of dialysis by about three times. Heating is done by placing an
electrical heating unit in the dialyser. All hydrophobic sols cannot withstand
the high temperature and the membranes may give off undrained colloidal
impurities.
4. Ultraf"dtration : Sols pass through an ordinary filter paper. Its pores
are too large to retain the colloidal particles. However, if the filter paper is
impregnated with collodion and then dipped in HCHO or glacial acetic acid,
the pore size is much reduced. Such a modified filter paper is called an
ultra..Jilter*.
Preparation of ultrafilters : The preparation invol ves the imp~gnation of special filter paper
with collodion (5% solution of nitrocellulose in a mixture of alcohol and ether) and subse-
quently hardening by dipping it in a solution of HCHO and finally drying. The size of pores·
of ultrafilter may be altered by varying the concentration of collodion solution. The pore size
of ultrafilters can be determined by the rate of penetration of water or by forcing air through
the wetted membrane. During ultrafiltration, the sol particles are left behind on the ultrafilter,
which can be redispersed in the pure dispersion medium.
COLLODIAL STATE 177

The separation of the sol particles from the liquid medium and
electrolytes by filtration through an ultrafilter is called ultrafiltration. It is a
slow process. Gas pressure or suction is applied to speed it up. The colloidal
particles are left on the ultrafilter in the form of slime. The slime is then
stirred into fresh dispersion medium to get the pure sol. By using graded
ultrafiIters, the technique of ultrafiltration can be used to separate sol particles
of different sizes.

Problem 2 : (a) Mention the differences between a true solution, colloidal


solution and suspension.
(Meerut 2004)
(b) Explain the types of colloidal systems with examples.
(a) The differences between a true solution, colloidal solution and
suspension are given in the table shown below.

S.N. Characteristic
property Suspension Colloidal True solution
solution
I. Nature Heterogeneous Heterogeneous Homogeneous
2. Size range Particles are greater Particles are Particles are less
than 10-5 cms (or between 10-5 and than 10-7 ems (or
0.1 ~ Of 100 m~) 10-7 ems (or 0.1 i.! I m~) in diameter.
in diameter. to I m~) in
diameter.
3. Visibility Particles are viSIble Particles are Particles are not
under a generally VIsible visible even under
microscope or even under ultramicroscope.
with a naked eye. Ultramicroscope.
4. Diffusibility Do not diffuse Diffuse slowly Diffuse rapIdly
5. Filtrability Can be filtered Can be filtered Cannot be filtered
even by an through an animal
ordinary fi Iter paper. membrane, through
which the colloidal
particles do not
pass.
6. Molecular weight Low High Low
7. Osmotic High Low High
pressure

(o.P. oc M~W.J
8. Colour - Depends upon the Depends upon the
shape and size of nature of the ion.
the particle

9. Tyndall effect Do not exhibit Exhibit Do not exhibit


178 PHYSICAL CHEMISTRY-I

S.N. Characteristic Suspension Colloidal


True solution
property solution
10. Brownian Do not exhibit Exhibit Exhibit
movement

II. Electrophoresis Do not exhibit Exhibit Exhibit


12. Coagulation - Can be coagulated Can be
by adding precipitated by
electrolytes. adding suitable
electrolytes.

13. Presence of - Particles carry Particles do not


electric charge either positi ve or carry any charge.
negative charge.

(b) A colloidal system is a biphasic system consisting of (i) dispersed


phase and (ii) dispersion medium. Therefore, the two phases can either be
solid, liquid or gas. Thus, different types of colloidal system depending upon
the nature of the dispersed phase and dispersion phase are possible, which
can be summarised in the following table with examples.

Dispersed Dispersion Name of the


S.N. phase medium colloidal system Examples

I. Solid Solid Solid sol. Gems, precious stones


etc., alloys.

2. Liquid Solid Gels or jellies Jelly, butter, cheese.

3. Gas Solid Solid foam Pumice stone, cork,


foam rubber.

4. Solid Liquid Colloidal solution, sol Sols of metals, sulphur


etc., paint, ink.

5. Liquid Liquid Emulsion Milk.

6. Gas Liquid Foam Soap lather, shavfng


cream, whipped cream.

7. Solid Gas Aerosol Smoke, dust.

8. Liquid Gas Liquid aerosol Fog, cloud, mist.

A gas dispersed in a gas forms a homogeneous mixture and so do not


form a colloidal solution.
Problem 3: How will you prepare the colloidal solutions o/the/ollowing:
Arsenic sulphide,/erric hydroxide, gold, sulphur, silicic acid, carbon,
iodine, mastic. (Meerut 2000 )
1. Arsenic sulpbide sol: A solution of 1.Og AS 20 3 in 500 ml of boiling
water is obtained. It is cooled and H2S gas is allowed to pass slowly. When
COLLODIAL STATE 179

the intensity of yellow colour of the sol does not increase further, the passing
of H 2S is stopped.
AS 20 3 + 3H2S ~ AS 2S 3 + 3H20
The excess of H 2S is removed either by boiling the colloidal solution
or by bubbling Hz through it, when a yellow sol of ASZS3 is obtained.
2. Ferric hydroxide sol: A colloidal solution of ferric hydroxide is
obtained by adding a freshly prepared saturated solution of ferric chloride (2
or 3 ml) drop by drop to 500 ml of boiling distilled water. Ferric chloride
hydrolyses to give a brown sol of ferric hydroxide sol.
FeCl3 + 3HzO ~ Pe(OHh + 3HCl
The hydrochloric acid formed and unhydrolysed ferric chloride are
separated by dialysis to get pure Fe (OH)3 sol.
3. Gold sol: A colloidal solution of gold (or purple of Cassius) is
obtained by reduction of auric chloride with stannous chloride or by HCHO.
2AuCl3 + 3SnCz ~ 2Au + 3SnCl 4
2AuCl3 + 3HCHO + 3H20 ~ 2Au + 2HCOOH + 6HCl
A trace of KZC03 is added to neutralise the acid formed whose presence
renders the sol unstable.
4. Sulphur sol : A colloidal solution of sulphur is obtained by the
oxidation of HzS with dil. HN0 3 or sulphurous acid.
3H zS + 2HN03 ~ 2NO + 4H 20 + 3S
2H2S + H ZS03 ~ 3HzO + 3S
The colloidal solution of sulphur so formed is treated with saturated
salt solution when sulphur is precipitated. It is filtered, washed with water to
remove salt and then washed further when the precipitate of sulphur peptises
and forms a colloidal solution.
5. Silicic acid sol: It is obtained by adding a solution of sodium silicate
to a solution of 4N HCI with constant stirring till the acid is in excess. The
mixture is diluted to about twice its volume and then dialysed against running
water till it is free from HC!.
6. Carbon sol : It is obtained by rubbing lamp black with water in
presence of little protective colloid like gum arabic, when a colloidal solution
of carbon (Indian ink) is formed.
7. Iodine sol: It is prepared by oxidising hydroiodic acid with iodic
acid, when a purple colloidal solution of iodine is obtained.
5HI + HI0 3 = 3HzO + 312
8. Mastic sol: It is a resin type hydrophilic colloid. Its colloidal solution
is obtained when it is dispersed in only pure distilled water. As such it carries
a very little charge, but when dispersed in acidic solution and alkaline solution,
a positively charged and negatively charged sol, respectively is obtained.
180 PHYSICAL CHEMISTRY-I

Problem 4 : Define and discuss or write short notes on the following:


(i) Lyophilic and lyophobic colloids· (Meerut 2001)
(U) Peptisation (Meerut 2002)
(iii) Dialysis (Meerut 2002)
(iv) Ultramicroscope
(v) Tyndall effect
(vi) Brownian motion
(vii) Electrophoresis
(viii) Electro-osmosis
(ix) Coagulation (Meerut 2006, 2001)
(x) Hardy-Schulze law (Meerut 2007,2006,2000)
(xi) Protection (Meerut 2007)
(xii) Gold number (Meerut 2007, 2005, 2003, 2000)
(xiii) Stability of lyophobic colloids
(xiv) Iso-electric point
(xv) Emulsions and characteristics (Meerut 2006, 2004, 2003)
(xvi) Gels (Meerut 2002)
(xvii) Electrical double layer (Meerut 2007,2005,2002,2000)
Or Zeta potential (Meerut 2006)

1. Lyophilic and Lyophobic Colloids


A substance which passes into the colloidal state, simply by bringing it
in contact with water is known as hydrophilic colloid (hydro = water, philic
= loving). But if any solvent like organic liquid is used instead of water, then
the more general term lyophilic colloid (lyo = solvent; philic = loving) is used.
Gum, starch, soap are lyophilic colloids. These colloids when once precipi-
tated can again be brought back directly into the colloidal state. Hence, they
are also known as reversible colloids. (Colloidal state precipitate). Re-
versible colloids are also termed as resolllble or non-electrocratic colloids.
Substances like metals, metal sulphides cannot be brought into the
colloidal state simply by bnnging them in contact with water and, therefore,
special methods are devised for the purpose. Hence, they are known as
hydrophobic colloids (hydro = water; phobic = hating). In case of solvent
other than water, the general term lyophobic is used. Further, if these colloids
are precipitated, then it is not very easy to reconvert the precipitate directly
into the colloidal state. Hence, they are termed as irreversible colloids (col-
loidal state ~ precipitate),'irresoluble or electrocratic colloids.
Difference between lyophilic and lyophobic sols
S.N. Lyophilic sols Lyophobic sols

L Prepared by direct mixing wIth Not prepared by direct mixing with the
dispersion medium, medium,

2. Viscosity higher than dispersion Vi,cosity almost the same as of


medIUm. set to a gel. medium. do not set to a gel.
COLLODIAL STATE 181

3. Little or no charge on particles. Particles carry positive or negative


charge.

4. Do not show Tyndall effect. Show Tyndall effect.

5. Particles migrate to anode or cathode Particles migrate to either anode or


or not at all. cathode.

6. Particles are generally solvated. No solvation of particles.

7. Precipitated by higher concentration of Precipitated by low concentration of


electrolytes. electrolytes.

8 Reversible. Irreversible.

2. Peptisation
See problem 1.
3. Dialysis
See problem 1.
4. Ultramicroscope
Zsigmondy and Siedentopf (1903) devised ultramicroscope to study
Tyndall effect. It consists of an ordinary microscope with a special arrange-
ment for focussing a beam of light. The light from an arc lamp S is allowed
to pass through a projection lens Ll and objective lens L z and focussed into
a vessel containing a colloidal solution C. The light scattered from colloidal
particles now enters the microscope M placed at right angle to the path of
light. The colloidal particles look as pin points of light. We do not see the
colloidal particles in this ultramicroscope but see the light scattered by them.
As each particle can be detected as 'spot' of light, therefore, slit ultra-
microscope helps to determine the number of particles in a definite volume.
The great value of the slit consists in the ability by its use to suction off a
portion of the fluid which is being examined. Zsigmondy (1912) devised an
immersion ultramicroscope, which was used with strongly coloured liquids.
In it, illumination objective and microscope objective are brought as close as
possible, and the liquid to be investigated can be used as a drop between the
two lenses.
The ultramicroscope does not give any information regarding the shape
and size of colloidal particles.
5. Tyndall Effect
When a strong beam of light is passed through a true solution, it cannot
be seen unless the eye is kept directly in the path of light, but when the same
beam of light is passed through a sol and viewed at right angles, the path of
the light becomes illuminated. This is due to the fact that sol particles absorb
182 PHYSICAL CHEMISTRY-I

f!F""""":;':'"''''''''''''''''''''''''''''''''''''''''"""""""""""""""""""""""","','""""""'""""""""""'''''''''''''''''''''\!J

M Microscope
Knife

I
Objective Projection edge Projection
lens lens slit lens
o C:>Arc
o ~Iight

;;};;;:;;:~~::~~;i':;;':!
light energy and then emit it in all directions in space. This scattering of light
illuminates the path of the beam in the colloidal solution. The phenomenon
of the scattering of light by the colloid particles is called Tyndall effect. The
illuminated beam or cone formed (figure 5) by the scattering of light by the
sol particles is often referred to as Tyndall beam or Tyndall cone.
If the sol particles are large enough, the sol may even appear turbid in
ordinary light. as a result of Tyndall scattering. As ions or solute molecules
are too small to scatter light, the beam of light passing through a true solution
is not visible when viewed from the side. So, Tyndall effect can be used to
distinguish a colloidal solution from a true solution.
The blue colour of sky, hazy illumination of the light beam from the
film projector in a smoke filled theatre or light beams from the head-lights
of a car on a dusty road are common examples of Tyndall effect.

M~SCOpeO
A
Halo
around
sol particle
*
~ • ~
scattered

View under
microscope
',',
'.'
COLLODIAL STATE 183

6. Brownian Motion
R. Brown (1827) observed that
• .• ?"""".".,."'
• . """"'."'., ,""""'·'·'·"'··""?·ll::
when a sol is examined with an ultrami-
croscope, the suspended particles are :::,
~ ~

seen as shining specks of light. By fol- ....


:;:;
lowing an individual particle, it is ob-
served that the particle is undergoing a
constant rapid motion in straight lines in
all possible directions [Fig. (6)].
The continuous rapid zig-zag
movement executed by the colloidal
particles in a dispersion medium is
known as Brownian movement.
Suspensions and true solutions do
not exhibit Brownian movement. It was seen that Brownian motion was
independent of the nature of the colloidal particles but was more rapid when
the particles are smaller and the solution is less viscous.
Explanation: It was proved that the motion of the particles is due to
the unequal bombardment of the suspended particles by the molecules of the
dispersion medium in which they are dispersed [Fig. (7)]. On this basis,
Einstein suggested that colloidal particles must behave like dissolved mole-
cules and gas laws should apply to these systems, just as they were applied
to true solutions. As the particles increase in size, the probability of unequal
bombardment diminishes and eventually the collisions on different sides

equalise each other. As the size is reduced, the probability of unequal bom-
bardment rapidly increases, whereby the Brownian movement becomes more
and more violent.
Applications and Importance
(i) Confirmation of kinetic theory: It is a direct demonstration of
ceaseless motion of molecules as pictured by kinetic theory of motion.
(ii) Determination of Avogadro's number: With the help of ultrami-
croscope, the number of particles in a given mass of the gas can be counted
and the Avogradro's number, i.e., the number of molecules in one mole of
gas can be calculated.
(iii) In stabiliSing colloidal solution: The zig-zag motion prevents the
184 PHYSICAL CHEMISTRY-I

setting of colloidal particles by gravity and thus helps in stabilising the


colloidal solutions to some extent.
7. Electrophoresis
The sol particles carry a
charge either positive or nega- :{"""";.'."""'X."""'' ' ' ' ' ' ';. ,.,""""""""""""""""'\:.•'.•;
tive. It was first observed by Lin-
der and Picton (1892) and is used
to indicate the migration of col- :::;
loidal particles in an electric '.'

field. When the particles move


towards the cathode, the phe- ",
nomenon of migration of .::: Water
particles is known as cataphore-
sis and migration towards the :::: ::;:
anode is known as allapllOresis.
But in practice, a more general ','
'.'
term electroplzoresis is used. So, ....
electrophoresis is defined as the ..'
movement of colloidal particles :::;::-t·?~S7:;zz:::::::::~h::Gs:d
towards oppositely charged
electrode under the influence of
electric field.
Electrophoresis can be studied by the simple apparatus [Fig. (8)] which
consists of a U-tube fitted with a funnel shaped reservoir and a stop cock. A
small amount of water is placed in the V-tube and then some quantity of the
sol is taken, so as to form a layer under the pure dispersion medium. An
electric current is then applied, by connecting the two electrodes dipped in
the solution. It will be seen that the particles begin to migrate towards the
oppositely charged electrode. If the sol is coloured, then the movement of the
particles can be observed directly by naked eyes. But if the sol is not coloured,
then the electrophoretic migration can be observed by microscope or ultra-
microscope. When the colloidal particles reach the electrode they lose their
charge and generally coagulate into coarse particles.

Applications of Electrophoresis
(i) In determining the charge : The nature of the cha~ge on a
colloidal particle can be ascertained by its migration in an electriC field.
(ii) In electro-deposition of rubber: The latex obtained from the sap
of certain trees is an emulsion of negatively charged rubber particles dispersed
in water. This can be deposited on any substance which is made the anode
during electrolysis.
(iii) In the removal of colloidal smoke from outgoing gases : The
removal of negatively charged carbon particles from smoke can be done by
passing the smoke between positively charged metal electrodes. The principle
has been utilised in the construction of Cottrell precipitator.
COLLODIAL STATE 185

(iv) Electrophoresis has proved to be of great importance in medicine,


industry etc. e.g., in characterising proteins causing diseases as well as in the
isolation of enzymes. Flavin enzyme was similarly isolated from yeast by
Theorell (1932).
(v) Lyophobic sols show electrophoresis: This is because the col-
loidal particles are electrically charged, which move under the influence of
electric field.
8. Electro-osmosis
A sol is electrically neu-
tral. So, the dispersion medium
carries an equal but opposite
charge to that of the dispersed
particles. Thus, the medium
will move in opposite direction
to the dispersed phase under the
influence of applied potential.
When the dispersed phase is
kept stationary, the medium is
actually found to move to the
electrode of sign opposite than
its own. Plug of
moist clay
The movement ofthe dis-
persion medium under the in-
fluence of applied potential is
known as electro-osmosis. ~~~~~~
Electro-osmosis is a di- WB2ill22t::z:::z:::z:z:±2Bili211iU
rect consequence of the existence of zeta potential between the sol particles
and the medium. When the applied potential exceeds the zeta potential, the
diffused layer moves and causes electro-osmosis.
The phenomenon of electro-osmosis can be studied by using aU-tube
[fig. (9») in which a plug of moist clay (a negative sol) is fixed. The two limbs
of the tube are filled with water to the same level. The platinum electrodes
are dipped in water ar,d potential applied. It is observed that water level rises
on the cathode side, while it falls on the anode side. This motion of the medium
towards the negative electrode, shows that the charge on the medium is
positive. Similarly, for a positively charged sol, electro-osmosis will occur in
the reverse direction.
Applications of Electro-osmosis
(i) In the preparation of pure colloids. The case in point is the prep-
aration of colloidal silicic acid of low molecular weight, i.e., in a state of fine
sub-di vision.
186 PHYSICAL CHEMISTRY-I

(ii) In the tanning of hides and impregnation of similar materials.


(iii) In the manufacture of gelatin for photographic emulsions. The
object is to get a gelatin free from fat, minerals and reducing constituents.
This phenomenon is also applied in the manufacture of high grade glue.
(iv) In the dying of peat, a process due to von Schwerin (1903).
9. Coagulation
It has been known for some time that on the addition of a very small
amount of suitable electrolytes to a sol, the corresponding substances are
thrown out of the solution and form a precipitate. If a suitable quantity of
sodium chloride is added to a ferric hydroxide sol, the sol becomes turbid
and ultimately precipitates. The electrolyte thus causes the colloid particles
to coalesce. So, the process by means of which the particles of the dispersed
phase in a sol are precipitated is known as coagulation or flocculation.
It has been observed that :
(i) All iOIl having a charge opposite to that of the sol is responsible
for coagulation. This ion is known as active ion, e.g., ill the coagulation of
negatively charged sols the cations are the active ions and anions are the
active ions in the coagulation of positively charged sols.
Oi) The coagulating or.flocculating power of the active ion increases
with increase in the valency of the active ion.
Hardy-Schulze law
Hardy and Schulze gave a law, after observing the regularities concern-
ing the sign and valence of the active ion. This law is known as Hardy-Schulze
law. It can be defined as,
'Greater the valency ofthe active ion, greater is the coagulating power
of the active ion. '
Thus, in the case of a positively charged sol the coagulating power of
anions is in the order of PO~- > SO~- > CC whereas the coagulating power
is in the order of A1 3+ > Bi+ > Na+ in the case of negatively charged sols.
The precipitate after being coagulated is known as 'coagulum'.
The amount of electrolyte required to produce coagulation depends
upon the total surface exposed by the colloidal particles. Hence, more con-
centrated sols require more electrolyte for coagulation. The minimum concen-
tration of an electrolyte required to bring about coagulation or flocculation
of a sol is known as coagulation or flocculation value. It is evident, therefore
that polyvalent ions are the most effective active ions in causing coagulation.
The coagulation of one sol may be effected by mixing an oppositely
charged second sol. Thus, when a positive sol of Fe(OH)3 is mixed with an
equi valent amount of a negative sol of AS 2S3• both get coagulated and separate
out. This process is known as mutual coagulation of sols.
COLLODIAL STATE 187

10. Hardy-Schulze Law


See part 9 of this problem.
11. Protection
Lyophilic sols are more stable towards electrolytes. It is due to the fact
that these particles are highly solvated and the electrolyte does not penetrate
easily to coagulate lyophilic sols. On the other hand. lyophilic sols are readily

--:.
~ 0
Hydrophilic - - H2 0 Hydrophobic
sol particle :. _ : Layer _ _ sol particle

~j j
.0-
- -
Dehydrating
---,s-,-ub-,s-,-ta_nce,,--_~..
0 Sol particle

precipitated by small amounts of electrolytes. However. these sols are often


stabilised by the addition of lyophilic sols. So, the property of lyophilic sol
to prevent the ceaguiation of a lyophobic sol is called protection. The
lyophilic substance which is used to protect a lyophobic sol from coagulation
is called aprotective colloid. This protection is due to the fact that the particles
of the lyophobic sol adsorb the particles of the lyophilic sol. Thus. the lyophilic
colloid forms a coating around the lyophobic sol particles The lyophobic
colloid thus behaves as a lyophilic sol and is precipitated less easily by
electrolytes [Fig. (10)).
Thus. if a little gelatin (protective colloid) is added to a gold sol the
latter is protected. The protected gold sol is no longer precipitated on the
addition of an electrolyte. say NaCl.
12. Gold Number
Different protective colloids have different protective powers.
Zsigmondy showed that protective power of a protective colloid can be
measured in terms of its gold Ilumber.
Gold number may be defined as 'the number of milligrams of a protec-
tive colloid which will just stop the coagulation of 10 ml of a given gold sol
on addillg 1 ml of 10% sodium chloride solution'.
188 PHYSICAL CHEMISTRY-I

If no protective colloid is present in the gold sol, it will turn from red
to blue. The smaller the gold number of a protective colloid, the greater is its
protective power.
Gold Number
Colloid Gold number
Casein 0.1
Gelatin O.OOS-O.OI
Dextrin 6.20
Starch (potato) 2S.0
Colloidal Si02 oc

Gum arabic 0.IS-D.2S

Thus, starch has a high gold number, which shows that it is an ineffective
protective colloid, while gelatin has a small gold number and is thus an
effective protective colloid.
(problem : Gold number of starch is 25. Calculate its amount to be added intd
100 ml of gold sol so that it is not coagulated in presence of 10 ml of 10%
NaCI solution.
Solution : Goid number of starch is 25 means 25 mg of starch IS added to
prevent coagulation of 10 ml of gold sol by I ml of 10% NaCI solution. For 100 ml
of gold sol, 250 mg of starch is needed to prevent coagulation by 1 ml of 10% NaCI
solution. But for 10 ml of 10% NaCI solution, only 251~ 1 mg or 25 mg of starch is
I[Cquired.
13. Stability of lyophobic Colloids
Preventing coagulation of a colloidal solution by any means is known
as stabilising the colloid. Bancroft (1915) gave the position with regard to
stability. He said that 'allY substance may be blVugllt into a colloidal state,
plVvided the particlt:.s of the dispersed phase are so small that the Brownian
movement keeps the particles suspended and plVvided the coagulation ofllie
particles is prevented by a suitable sUlface film '.
It was observed that lyophobic sols are stable due to their electric charge.
As most of the lyophobic sols are prepared by preferential adsorption of ions,
the mutual repulsion of particles is responsible for their stability. The sols
become unstable, as soon as they are robbed off their charge. However, in
case of lyophilic sols, the stability is due to electric charges as weIl as
solvation-a phenomenon in which the colloid particle is surrounded by a
thin film of the solvent. The layer of the solvent forms an envelope around
each colloidal particle and thus pre-forms an envelope around each colloidal
particle and thus prevents the particles from coming together. Groups like
COLLODIAL STATE 189

-COOH and -NH2 in proteins and -OH in hydroxides and polysaccharides


can bind water molecules. Lyophilic sols are coagulated when first the solvent
layer and then the charge of the ion are removed.
Factors Affecting Stability: There are numerous factors which affect
the stability of a colloidal system.
(i) Brownian movement: As a result of this movement, the particles
are in constant rapid motion, whereby aggregation of particles is prevented.
So, the sol remains stable. But as soon as Brownian motion ceases, the sol
becomes unstable. It is, therefore, expected that a sol will be more stable, the
higher the dispersity and greater the charge on the particles.
(ii) Addition of electrolytes : As described before, a sol becomes
unstable by the addition of suitable quantity of electrolytes. It is now estab-
lished that the presence of electrolytes has a powerful effect on the potential
difference between the particles of the dispersed phase and the dispersion
medium and that this potential difference is closely connected with the sta-
bility of sols.
(iii) Effect of concentration of the sol: Ghosh and Dhar (1927)
showed that a number of positively charged sols follow the rule that, greater
the concentration of the sol greater the amount of electrolyte required to
coagulate the sol. In other words, the greater the concentration of the sol,
greater is its stability.
(iv) Effect of dilution: Chaudhury (1928) showed that the dilution of
a sol also affects the stability, which is decreased. The reason is that the
decrease of the charge and total surface of sol particles with dilution decrease
the stability of the sol. Mukherjee (1930) attempted to find out a relation
between dilution and stability of a sol by measuring the migration velocity
of the particles, but found the relation to be complicated.
(v) Rate of addition of electrolyte : Dhar (1925) showed that the
amount of electrolyte required to confer stability on a sol depends upon the
ra1e at which it is added. This phenomenon is known as 'acclimatization of
sols '. There are two types of acclimatizations: in some cases, less electrolyte
is required, when it is added slowly or in small quantities at a time, and in
other cases, the amount required is more. The former and latter are known as
positive and negative acclimatizations, respectively. This phenomenon is
probably due to charge occurring in the stabilisation by electrolytes.
(vi) Temperature: It is seen that decrease of temperature confers
more stability on the sols. Reid and Burton (1928) showed that heat alone is
sufficient to cause coagulation.
(vii) Mechanical agitation: It has been seen that mechanical agitation
decreases the stability of sols. Freundlich and Loebmann (1922) showed that
the sol of CuO was found to be coagulated by mechanical agitation. The rate
of coagulation is proportional to the square of the rate of stirring.
(viii) Ultra-violet light and X-ray radiations: Many lyophohc sols
are coagulated by U-V light and X-rays or rays even from radiations of
190 PHYSICAL CHEMISTRY-I

radium. It is seen that the coagulation effect of the rays is independent of the
sign of the colloid. Lal and Ganguly (1930) studied the coagulating influence
of U-V light on sols of AgI, Au, Ag, V205' Th(OHh, As 2S3 etc.
14. Isoelectric Point
Isoelectric point is that point at which the concentration of the positi ve
ions in solution becomes equal to that of the negative ions. Similarly, in case
of colloidal solutions, a sol may be positively charged in presence of an acid,
due to the preferential adsorption of H+ ions. On the contrary, a sol may be
negatively charged in presence of an alkali, due to the preferential adsorption
of OH- ions. However, there must be an intermediate H+ ion concentration
at which the colloidal particles are neither positively nor negatively charged.
Hence, the isoelectric point in case of colloidal solution, is that point at which
the colloidal solutions have no charge at all.
IsoeleclI"ic point plays an important part in the stabilization of a sol.
The stability of a sol is due to the presence of zeta potential (see subsequent
pages) which is zero at the isoelectric point. At this point, the colloidal
particles will not move towards any electrode under the influence of electric
field. Hardy said that at isoelectric point, the colloidal particles are electro-
phoretically inert. Isoelectric point also occurs in protein sols. A protein
solution is amphoteric. In acidic medium, a protein sol is positively charged,
while in an alkaline medium, it is negatively charged. At a certain point or at
a certain pH, the particles will have equal positive and negative charges.
Different protein sols have different isoelectric points, e.g., isoelectric point
for a gelatin sol occurs at pH == 4.7. Below pH 4.7, the colloidal particles of
gelatin move towards the cathode, while at pH greater than 4.7 the particles
move towards the anode.
15. Emulsions and Characteristics
Emulsions are liquid-liquid colloidal systems. In other words, an emul-
sion may be defined as a dispersion of finely divided liquid droplets in
another liquid.
Generally, one of the two liquids is water and the other. which is
immiscible is designated as oil. Either liquid can constitute the dispersed
phase. .
Types of Emulsions : There are two kinds of emulsions:
(i) Water in oil type: In it water is the dispersed phase and oil acts
as dispersion medium, e.g., butter etc. It is designated by WIO or w-in-o.
(ii) Oil in water type: In it oil particles form the dispersed phase and
water is the dispersion medium, e.g., milk, vanishing creams etc. It is desig-
nated by O/W or o-in-w.
Tests for Types of Emulsion : The two types of emulsions can be
experimentally identified in a number of ways, which are as follows:
COLLODIAL STATE 191

(i) Conductivity method: If the conductivity of the emulsion is large,


the emulsion is of O/W type, as these emulsions are more conducting. But if
the conductivity is low, it is of W/O type, as they are less conducting.
(ii) Filter paper method: In this method, a drop of emulsion is kept
on a piece of filter paper. If the liquid spreads easily and readily, le~ving a
spot at the centre, then the emulsion is of O/W type. In case the emulsion
does not spread, the emulsion is of W/O type.
Preparation of Emulsions: Emulsions can be prepared by shaking or
stirring the two phases with the addition of a suitable emulsifier. The type of
emulsion formed, depends on the angles of contact of the two liquids with
the solid emulsifier.

=::=::=:=:=::=:=:Na+ =:=:=1-_-=:,"",_:=-_-=:,-'_==t--Oil droplet

~~~~!i~;~~~~~~~--=~~~:=::-~~~~~~~~
:::::.:::::::
------- ------ B-::- -=:=--------
-- -- - :=:=:=
:~;:~~~~~~~~
-----t---
~~-~~~~~~~:~;~~-
--:~-----
Polar head

~~~~:~:~;:=~
----~:~~:~:~~~;-~~
Hydrocarbon
tail
---------- --- --- - ----
~~~~~~+:~.:::~ ~::._:N~:~~~~
----- - --------
----- ------- - -----
------- -------- ------
---------------
-------------- -----------------
---------------
-------------- ---------------
--------------- ----------- ---- Water

Fig. 11

The liquid to be dispersed is added in small quantities to the dispersion


medium, where it is spread into a thin, unstable film which spontaneously
breaks up into droplets under the influence of surface tension. Many
emulsifying machines are used, such as homogenizers, in reducing the size
of globules.
A condensation method is used t t epare concentrated O/W emulsions.
It consists in allowing vapours of oil to pass through an aqueous solution
containing an emulsifying agent.
Emulsifiers or Emulsifying Agents : The two types of emulsions
i.e., W/O or O/W type, do not remain stable and after sometime, the two
layers separate. It means that an emulsion formed by merely shaking two
liquids is unstable. But in order to get stable emulsions it is necessary to add
a third substance known as emulsifier or emulsifying agent in suitable small
192 PHYSICAL CHEMISTRY-I

quantity. By adding an emulsifier, a stable emulsion of high concentration


can be obtained.
The emulsifiers are generally long chain compounds with polar groups.
such as soaps of various kinds, long chain sulphonic acids and alkyl sulphates.
The function of an emulsifier is to lower the interfacial tension between the
dispersed phase and the dispersion medium so as to facilitate the mixing of
the two liquids.
In compounds like soap, the aliphatiC portion is soluble in oil. while
the end group (sulphonic acid etc.) called a polar group (because its unsym-
metrical grouping contributes a dipole moment to the compound) is soluble
in water. The soap molecules get concentrated at the interface between water
and oil in such a way that their polar end (-COONa) and hydrocarbon chain
(R-) dip in water and oil, respectively as shown in figure 11. This allows the
two liquids to come in close contact with each other.
Soluble substances like iodine also act as emulsifiers, in the case of
etherlwater emulsions. Some mixed stabilising agents also act as emulsifiers,
e.g., ethyl alcohol and lycopodium powder, acetic acid and lamp black, clay
and sodium oleate etc.
Properties of Emulsions: Since an emulsion is a colloidal system of
liquid dispersed in liquid, therefore, its properties are common to those of
colloidal solutions.
(a) Concentration and size of the particle : In an emulsion, the
amount of solid substance dissolved is very small as compared to that of the
dispersion medium. An emulsion contains droplets of diameter 0.008-0.05
mm.
(b) Electrical conductivity: It has been found that emulsions of OIW
type are characterised by higher electrical conductivity, while emulsions of
WIO type have a lower or no electrical conductivity.
(c) Electrophoresis: Like colloidal particles, the droplets of emulsion
are also electrically charged. Hence, they migrate towards the oppositely
charged electrode under the influence of electric current.
(d) Dilution: On increasing the amount of dispersion medium, a
separate layer is formed.
(e) Brownian motion: Just like colloidal particles, droplets of emul-
sion are also in a state of constant rapid zig-zag motion.
(f) Optical properties: Droplets of emulsion scatter light to different
extent, depending upon their size.
Reversal of Phase: The change of an emulsion of OIW type to W10
type or vice-versa is known as 'reversal of phase'. An OIW emulsion. e.g.,
olive oil in water, containing a sodium or potassium soap as emulsifying agent,
may be converted into WIO type by the addition of salts of bivalent or trivalent
cations.
COLLODIAL STATE 193

Breaking of Emulsions: Emulsions can be broken or demulsified to


get the constituent liquids by heating, freezing, centrifuging or by addition of
appreciable amounts of electrolytes. They are also broken by destroying the
emulsifier. For example, an oil-water emulsion stabilised by soap is broken
by the addition of a strong acid. The acid converts soap into insoluble free
fatty acids.
Uses of Emulsions : Emulsions find numerous applications in daily
life, medicine, industry and cosmetics. They may be described as follow& .
Daily articles of life: Milk is an emulsion of fat dispersed in water
stabilised by casein and as all know is practically a complete food. Ice cream,
is an emulsion, in which ice particles are dispersed in cream, stabilised by
gelatin. Artificial beverages, coffee, fruit jellies are all emulsions in nature.
The cleansing action of ordinary soap is due to a large extent on the production
of O/W emulsion.
A number of medicines and pharmaceutical preparations are emulsions
in nature. It is assumed that in this form, they are more effective. Cod-liver
oil, castor oil, petroleum oil are used as medicines which are all emulsions.
Asphalt emulsified in water is used for building roads, without the necessity
of melting the asphalt. Most of the cosmetics used are emulsions as they
permit uniform spreading and promoting the penetration into the skin. Van-
ishing cream is an O/W type emulsion. Hair creams, cold creams are W/O
type emulsions.
A variety of emulsions of oils and fats are used in leather industry to
make leather soft and pliable and also to make it waterproof. Emulsions are
also used in oil and fat industry, paints and varnishes, plastic industry, adhe-
sives, cellulose and paper industry etc.
16. Gels
Graham applied the term gel to any coagulum from a sol, whereas
according to Ostwald, a gel is a jelly like colloidal system in which a liquid
is dispersed in a solid medium.
For example, when a warm sol of gelatin is cooled, it sets to a semi-solid
mass, which is gel. The process of conversion of a sol to a gel is known as
gelation. Gelation may be brought about by either of the methods, (a) cooling
the sol, (b) evaporating the sol, (c) addition of electrolytes.
Classification of Gels
(i) On the basis of dispersion medium: Gels produced as a result
of coagulation are known as coagels. When the dispersion medium in the gel
is either water, alcohol or benzene, Qlen the gel formed is known as hydrogel,
alcohol or benzogel, respectively. They may be cut, bent or broken as desired.
If gels are subjected to prolonged pressure they begin to flow, assuming the
shape of the containing vessel.
194 PHYSICAL CHEMISTRY-I

(ii) On the basis of chemical composition: Gels are called inorganic


and organic according to their chemical composition. e.g .. gels containing
inorganic solvents are known as inorganic gels. Similarly, organic gels are
those gels in which organic solvents are used.
(iii) On the basis of size of particles: Gels may be colloidal or coarse
depending upon whether the size of particles is small or comparatively bigger.
Common jellies, agar-agar solution are colloidal gels.
(iv) On the basis of their properties: Such gels are classified on the
basis of thei~ mechanical properties. Hence, there may be elastic gels, non-
elastic gels, rigid gels etc.
(q) Heat reversible or elastic gels: Such gels when partially dehy-
drated'change into a solid mass which, however, change back i~lto the original
form on simple addition of water followed by slight warming. if necessary.
Gelatin, agar-agar and starch form reversible gels.
(b) Heat irreversible or non-elastic gels: Such gels when dehydrated
become glassy or change into a powder which on addition of water and
followed by warming do not change back into the original gel. Silica, alumina
and ferric oxide gels form irreversible gels.
Preparation of Gels : Gels may easily be prepared by any of the
following methods :
(i) By cooling of colloidal solutions: Certain substances form gels
when their hot solutions are cooled, e.g., gelatin, agar-agar etc. The setting
or gelation of such a solution is characteristic of: (a) temperature of gelation,
(b) time of gelation, (c) viscosity of the medium, (d) minimum cOllcentration
of the substance at which the gelation may occur.
It has been found that substances like sulphates, tartarates, acetates,
citrates promote the rate of gelation, whereas certain substances like chlorides,
nitrates etc. retard the rate of gelation. The gelation of proteins is retarded by
acid and alkalies.
When pure alcohol is added to the aqueous solution of calcium acetate,
the whole of the salt goes into alcohol, which then sets into a gel containing
the liquid.
(ii) By double decomposition: The gels of some sols are prepared
by the process of double decomposition. On adding water to sodium silicate
we get a gel of silicic acid. During the reaction, silicic acid is prepared in the
free state which rapidly sets to a gel.
(iii) By exchange of solvents : Sometimes gelation may occur due to
the exchange of solvent in which the sol is insoluble.
(iv) By chemical reactions : Gels can be prepared by this method
from concentrated solutions, if one product of the reaction is insoluble and
the particles have a tendency to form linear aggregates. On shaking concen-
trated solutions of barium thiocyanate and manganous sulphate, we get a gel
of barium sulphate. Gel of aluminium hydroxide can be prepared by mixing
concentrated solutions of aluminium salt and ammonium hydroxide.
COLLODIAL STATE 195

(v) By coagulation or by decrease of solubility: Many gels can be


prepared by the coagulation of colloidal solutions. Sols of aluminium hydrox-
ide or ferric hydroxide can be converted into gels when some coagulating
agents are added to the colloidal solutions and provided the concentration of
the sol is sufficiently high.
Properties of Gels
(i) Swelling of gels: Hydrophilic gels or elastic gels when placed in
water absorb a definite amount of the liquid, whereby the volume increases.
The process is known as swelling or imbibition.
(ii) Optical properties: Gels show the phenomenon of double re-
fraction. It may be present in the gel from the start or it may be produced by
pressure or tension. e.g .• a dry gel of gum shows double refraction like that
of the glass under tension and compression.
(iii) Electrical properties: It has been seen that the electrical conduc-
tivity does not change during the transition from the sol to the gel state.
(iv) Syneresis: Syneresis is phenomenon of the excluslOll of the liquid
constituent of gels on standing, whereby the gel contracts or shrinks. This
may be regarded as reverse of swelling. Gelatin and agar-agar shows syneresis
at low concentration, while silicic acid shows it at high concentration.
(v) Some of the gels, particularly gelatin and silica liquefy on shaking,
changing into the corresponding sol. The solon standing reverts back to the
gel. This phenomenon of reversible sol-gel transformation is generally
refen'ed to as thixotropy.
(vi) Colloidal system usually shows a slow spontaneous aggregation.
This is known as ageing. In gels, ageing results in the gradual formation of
a denser network of gelling agent.
(vii) Gels also show the mechanical properties of rigidity, tensile
strength and elasticity that are characteristic of solids.
Structure of Gels : There is still a difference of opinion about the
structure of gels. Various theories have. however, been proposed to explain
the structure of gels.
(i) Honey-comb theory: Butschli (1897-1900) concluded as a result
of investigations of gels under a microscope that many gels have fine honey-
comb structure.
(ii) Martin-Fisher solvation theory: According to this theory, a gel
may be regarded as a system of two components. one liquid solute and the
other solid solvent. X-ray studies prove that the structure of gels is miscellar
in nature.
(iii) Zsigmondy's theory: Zsigmondy and his co-workers as a result
of ultra-microscopical observations with gelatin gel could not confirm the
honey-comb structure and suggested that the structure of the gel is much finer
than that assumed by Butschli. Therefore. according to Zsigmondy and Bac-
hmann. gels have granular fine structure.
196 PHYSICAL CHEMISTRY-I

(iv) Fibrillar theory : It may also be assumed that in gelatin, the


particles arrange themselves into fibres, which are then inter-twined. These
threads cause cohesion and great elasticity of gels.
(v) von Wiemarn's theory: According to von Wiemarn's concept,
gels may be classified according to the degree of dispersion of their primary
structural elements.
(vi) Thomas-Sibi-theory: Thomas and Sibi (1928-1930) conducted
experiments and suggested view close to von Wiemarn's theory. They as-
sumed that solon cooling forms cluster of needles which interpenetrate to
give a firm mass. They further showed that units of structure may be inter-
twining hair instead of straight needles.
Uses of Gels: Gels have found several applications. Silica gel is used
in laboratory and in industry and is also used to support the platinum catalyst,
when this is used in the contact process of H2S04 manufacture. It is resistant
to catalytic poisoning. Solidified alcohol (gel) is used as fuel in picnic stoves
and is made from alcohol and calcium acetate.
17. Electrical Double Layer or Zeta Potential
In order to explain the origin of charge on a colloidal particle, von
Helmholtz postulated the existence of an electrical double layer of opposite
charge at the surface of separation between a solid and liquid, i.e., at solid
liquid interface.
According to modern views, when a solid is in contact with a liquid, a
double layer appears at the surface of separation. One part of the double layer
is fixed on the surface of the solid. It is known as fixed part of the double
layer and it consists of either negative or positive ions. The second part of
the double layer consists of a mobile or diffuse layer of ions which extends
into the liquid phase. This layer consists of ions of both the charges but the
net charge is equal and opposite to that on the fixed part of the double layer.
This arrangement is shown in figure (12). In figure (a), the fixed part of the
double layer comprises of positive charges, while in figure (b), the fixed part
of the double layer consists of negative charges.
The presence of charges of opposite signs on the fixed and diffuse parts
of the double layer produces a potential between the two layers. This potential
is known as electrokinetic potential or zeta potential. It is represented by
S(zeta). It is therefore the electromotive force which is developed between
the fixed layer and the dispersion medium.
The ions which are preferentially adsorbed by the sol particles, are held
in the fixed part of the double layer. It is these ions which give the charac-
teristic charge to the sol particles.
It is supposed that the charge on a colloidal particle is due to the
preferential adsorption of either positive or negative ions on the particle
surface. If the particles have a preference to adsorb negati ve ions they acquire
negative charge or vice-versa. The negative charge on As 2S3 sol is due to the
preferential adsorption of sulphide ions on the particle surface. The sulphide
COLLODIAL STATE 1-97

ions are given -by the ionisation of hydrogen sulphide, which is present in
traces. Similarly, the negative charge on metal sol particles obtained by
Bredig's method is due to the preferential adsorption of hydroxyl ions given
by the traces of alkali present. The positive charge on sol of Fe(OH)3 prepared
by the hydrolysis of ferric chloride is due to the preferential adsorption of
ferric ions on the surface of the particles. The ferric ions are obtained by the
dissociation of ferric chloride present in traces in the sol.

!·.:ii·················++:··:~:::~::~··;;:::=I~'i
:.I..

+:+
+· -- + +•
I
I
--

+ +
+ +::::
::::
','

+
+·--
+ +I

+
+ + :}
+:+
+ + +
I
I
I
--
+
+ +
--.,.....,
I
I
, --
-........-. \
+
y

Fixed Difuse layer Fixed Difuse Layer ':-:


',' layer layer" .
t ;
: ·:·;:i:;:;:;: ~:~[; :;~: i:i;:~; :~:;i~:;:~: :;: :~:;:;: :~: :;: ;~; : ;~:~:;: :~';:;:~;~; : :' ~~~~: ~~. ·: :;: i~;~: :;·~;: ;i:~; : : : :·;: ·;:;: ·: : ;: : ;: ~; : :i·: : :; ~.:i:.:·: ~l:~: .:·: : ·:
From the above, it is clear that the ion which is more nearly related
chemically to the colloidal particle is preferentially adsorbed by it. So, in .
AS ZS 3 sol, sulphide and not hydrogen ion is preferred and in Fe(OHh sol,
ferric and not chloride ion is preferred as shown below :
[As S ]Sz-12H+ and [Fe(OHh]Fe 3+ 13CI-
Z 3

Now consider stannic oxide sol. If a freshly prepared precipitate of


stannic oxide is peptised by a small amount of HC!. the colloidal solution
carries a positive charge. If the. precipitate is peptised by a small amount of
NaOH, the colloidal solution carries a negative charge. In the first type of
sol, a small amount of stannic chloride SnCI4 is formed and Sn 4+ ion is
preferred over CC ion. So, the sol gets positively charged. In the second type
of sol, a small amount of sodium stannate Na2Sn03 IS formed and now the
SnO~- ion is preferred over Na+ ions. So, the sol gets negatively charged.
The structure of the sol particles in the two cases can be represented as :
Positive sol: [Sn02J Sn4+ 14CI-
Negative sol: [SnOzJ SnO~-12Na+
The chloride and sodium ions form the diffuse part of the electrical
double layer.
198 PHYSICAL CHEMISTRY-I

Another case is the formation of positively and negatively charged sols


of silver IOdide. If a dilute solution of AgN0 3 is added to a slight exces~ of
KI solution. a negative sol of AgI is obtained, due to the adsorption of iodide
Ions. The structure of the particles is represented as rAgI] C I K-'- If a dilute
solutIon of KI is added to a slight excess of AgN0 1 solutIon. a positive sol
of AgI is obtamed, due to the adsorption of silver ions. The structure of the
particle is shown as [AgIJAg + / NO). If on the contrary, equivalent amounts
of AgN0 3 and KI are mixed, there is complete precipitation of silver iodide
and no sol is formed.
Problem 5 : (a) Discuss the origin of charge 011 the colloidal particle.
Explain its significance also. (Meerut 2004, 2001)
(b) Classify the following sols according to their charge :(Meerut 2002)
(i) gold (ii) ferric hydroxide
(iii) gelatin (iv) blood
(v) sulphur (vi) AS1 S 3
(c) What will be the charge on the following:
(i) AgJ in AgN03 (ii) AgJ ill KJ
(iii) As 2S3 ill H 2S (iv) Fe(OHh ill FeCl 3
(1) Origin of Charge
All the dispersed particles of a sol carry a positive or negative charge.
In order to explain the origin of charge on a colloidal particle. following Vlew&
have been gIven.
(i) Charge is ionic ill nature: It has been shown that colloids, e.g.,
sodium palmitate and soap dissociate and produce ions. So, the charge is
produced a~ a result of the formation of ions. But this view does not hold
good for non-electrolytic colloids such as clay, smoke etc. as they also carry
a charge.
(ii) Charge is frictional in nature: In earlier days. it was regarded
that the source of the charge on colloid particles is essentially physical and
it was suggested that the charge resembled the frictional electricity brought
about by the contact of such substances as glass and silk in this case, substance
with hIgher dielectric constant will render the sol negatIvely charged.
(iii) Association of electrolytes ; It has been observed that small
quantities of electrolytes are associated with colloidal systems and that if they
are removed by persistent dialysis or by other methods, the sols become
unstable and the particles grow in size and are finally precipitated. It is
probable that in many cases, traces of ions present in the sol are re~ponslble
for the charge and the stability of colloidal particles. It is seen that stable sols
of gold, silver etc. can be obtained by Bredig's arc method by passing an
electric current between electrodes of the metals under water. Water contam~
a little KOH or any other alkali. One ion is retained by the collOIdal particles,
giving the same charge to the colloidal particles and the other ion is retained
by the dispersion medium.
COLLODIAL STATE 199

(iv) Formation of electrical double layer: See problem 4 (16).


Significance of charge: The charge on a colloidal particle is of great
importance as shown below.
(i) In industrial applications of colloids: A number of processe& like
electro-deposition of rubber, removal of smoke, punficatlOn of water. tanning
of leather depend upon the presence of charge on a colloidal particle.
(ii) In stability of sols: As all colloidal particles have the same charge.
they are kept apart due to mutual repulsions and so they do not coalesce with
one another to form bigger particles. If the charge is removed, they come
closer together and get coagulated.
(2) Classification of Sols
Positive sol: Fe(OHh.
Negative sols: Gold, gelatin. blood, sulphur. AS 2S 1.
(3) Charge on Sols
(i) Positive (ii) Negative (iii) Negative (iv) Positive.
Problem 6: Explain thefollowingfacts:
(a) A sulphur sol is coagulated by adding a little electrolyte, whereas (I

gelatin sol is apparently unaffected.


(b) What happens when a collOidal solution of gold IS brought Hilder The
influence of electric field?
(c) What happens when an electrolyte is added to colloidal solution of
gold?
(d) What happens when a beam of light is passed through a colloidal
solution of gold?
(e) A colloidal solution is stabilised by addition of gelatin.
(/) Presence of H 2S is essential ill AS2S3 sol though H 2S iOllises alld should
precipitate the sol.
(g) Why ferric chloride or alum is used for stoppage of bleeding?
(a) A sulphur sol consists of negatively charged particles dispersed in
water. When an electrolyte is added to it, the sol being lyophobic in nature I~
easily coagulated.
On the other hand; gelatin sol is lyophilic in nature and gelatm particles
are heavily hydrated and so water envelopes around them and prevents their
coming in contact with electrolyte. Hence, it is not easily coagulated on
addition of electrolyte.
(b) When colloidal gold solution is brought under electric field, the
gold particles move towards anode and lose their charge and get coagulated.
This shows that gold particles carry a negative charge (electmpllOresis).
(c) When an electrolyte is added to gold sol. the cation of the electro-
lyte neutralises the negative charge on the collOidal particles and the sol get!'>
coagulated.
200 PHYSICAL CHEMISTRY-I

(d) When a beam of light is passed through a colloidal solution, the


path of the light becomes visible when viewed through an ultramicroscope,
due to scattering of light by colloidal particles (Tyndall effect).
(e) A colloidal solution is stabilised by the addition of gelatin. This is
due to the fact that gelatin being a lyophilic colloid forms a protective layer
around the colloidal particles. This protective layer prevents the precipitating
ions from reaching the sol particles and in this way they are prevented from
being precipitated.
(1) According to modem view, at the surface of separation of solid
and liquid. an electrical double layer IS formed. One of the layer is called
fixed layer which is towards the particle and the other layer called the mobile
layer points towards the dispersion medium. The fixed layer contains the Ions
of one charge, either positive or negative and those ions are preferentially
adsorbed which are chemically alike. So. to stabilise AS ZS 3 sol, S2- ions given
by H 2S from the fixed layer give negative charge to AS 2S3 particle, i.e.,
[AS ZS3]S2-12H+. Thus, because of the same charge, AS ZS3 particles do not
come closer to get coagulated. So, unlike other electrolytes, HzS stabilises
AS 2S3 sol.
(g) Blood is a negatively charged sol in which albuminoid substance
is dispersed in water. When FeCl 3 or alum is added, the trivalent ions, Fe3+
or A1 3+ coagulate the blood. Thus, bleeding is stopped.
Problem 7. Describe the applications of colloids in chemistry.
The applications of colloids can broadly be divided into three types:
(i) Natural applications,
(ii) Technical applications,
(iii) Analytical applications.
[A] Natural Applications:
(i) Rain: Cloud is a colloidal system in which water particles are
distributed in air. When air which has become saturated with water vapours,
reaches the cooler parts of the atmosphere. cloud is formed as a result of
condensation. Further cooling and condensation form bigger drops of water
which fall due to gravity in the form of rain.
(ii) Blood: Blood is a collOIdal system, having albuminoid substance
as the dispersed phase carrying a negative charge. The stoppage of bleeding
by the application of alum or ferric chloride can be explained on the basis of
coagulation, as in this case A1 3+ acts as active ion for the coagulation of
negatively charged albuminoid particles.
(iii) Bille colour of the sky: This is an application of Tyndall effect.
The upper atmosphere contains colloidal dust or ice particles dispersed in air.
As the sun rays enter the atmosphere [Fig. (13)], these strike the colloidal
particles. They absorb sunlight and scatter light of blue colour (4600-51 00 A).
The light that is incident at earth's surface is considerably reddened due to
the removal of most of the blue light in the upper atmmiphere (figure 13).
COLLODIAL STATE 201

IY"'e.~u~::~:+! .'.
" .
.'.

i;;;;;;~~. ;~ . '··;1
(iv) Articles ofdaily use: Milk which is a complete food is an emulsion
of oil in water (OfW) type stabilised by casein. Butter is an emulsion of water
dispersed in fat (O/W) type. Fruit juice is a colloidal system having juice
dispersed in the solid tissue of the fruit. Ice cream is ice particles dispersed
in cream.
(v) Formation of deltas: The river water contains colloidal particles
of sand and clay which carry negative charge. The sea water on the other

IY;':::::::::;;:~~:::::::+

• ~=~<~~ ~
.'.
~
e J:
~

~ R~~ j
~ 4
: : :· i:~:~ ~ :~·: :;~;~; :;: ~:; :i~: ~ ~;~;:~:~;i~.~;~;·.·.~ ~.:·.·.~.~.~:;~i:;·:.;:i.:i~:i:~:; ;·i.~. ;~;t;i;·. ;:·:l:·~.: ~ !
hand, contains positive ions such as Na+, Mg2+, Ca:!+. As the river water meets
sea water, these ions coagulate the sand or clay particles which are precipitate<!
as delta [Fig. (14)].

\
202 PHYSICAL CHEMISTRY-I

[B] Technical Applications: These are numerous technical applica-


tions of colloids. but we shall explain only the important ones.
(i) Rubber industry: Rubber is obtained from the sap of certain trees.
The sap--a1so known as latex-is a colloidal system of colloidal particles
dispersed in water. stabilised by protein contents. On coagulating the latex
by various methods. collOIdal gel. known as rubber is obtamed.
The rubber particles carry a negative charge. When rubber is to be
deposited on any material. then two electrodes are made. The anode is made
of that substance on which rubber has to be deposited. On passing an electric
current. rubber thus deposits on the substance.
(ii) Tanning ofleather : Both hide and leather are colloidal substances
and possess gel structures. When hide is soaked in any tanning agent which
consists of tannin (a sol containing negatively charged particles). mutual
coagulation of the two takes place due to oppostie charges. This process is
known as talllling and the hide becomes more and more leather like. imparting
hardness to it.
(iii) Soils and clays : The value of a soil. i.e .• its ability to retain
moisture is dependent on the colloidal propertIes of humus-an important
constituent of the soil. When soil is completely saturated with H+ ions, it is
known as 'clayic acid' and the salts derived from this acid are known as
'clayates'. There are colloids. e.g., Fe(OHh, AI(OHh. amorphous silicates
and bacteria which are present in soils.
(iv) Purification of sewage and water: Sewage consists of impure
particles dispersed in water and is thus colloidal in nature. carrying a negative
charge. The dirt particles carrying a negative charge are coagulated on the
oppositely charged positive electrode and are thus removed. The deposit is
then used as manure. The method for sewage purification is known as 'acti-
Wifed sludge process '.
(v) Photography: Photographic plates are thin glass plates coated
with a fine suspension of silver bromide and gelatin. Silver bromide dispersed
in gelatin gel forms photographic emulsion. The AgBr sol may be produced
by mixing gelatin solution containing dilute solution of AgN0 3 with dilute
alkali bromide. whereby insoluble AgBr is formed in a fine suspension.
stabilised by gelatin which is then painted on thin glass plates.
(vi) Metallurgy of alloys: The technical property of an alloy depends
upon the size of the subdivided particles in it. Thus. alloys which are brittle
and non-elastic have a coarse structure. As the degree of dispersion is in-
creased. the hardness of the alloy is also increased.
(vii) Colours of various materials: The colour of various materials is
due to colloid metal and colloid metallic compound. The red colour of ruby
glass is due to colloidal gold. Yellow glass owes its colour to colloidal silver.
Colloidal chromium imparts colour to artificial rubies.

!
COLLODIAL STATE 203

(viii) Dyeing: Cotton, silk and wool are all colloidal in nature having
a gel structure. The dyestuffs include a number of substances having different
propertIes; their degree of dispersion is colloidal.
preCip~~i:n~O:;ok:~~ a:;s7e: " .: " :; : -:." ' ':' ' .:.;' ' ': : ~.::-;.:"':':.~::""':"::-:"'~'.,"'-:''''''''~-'''''D:-'u-'s:-t~fr"":e"":~'"'·::··"":-;:=.i.:
in which carbon particles are dis- :::: gases
persed in air carrying a negative ):
charge. In large industrial cities '. ir
:yO~~:~h~:sa~~:;a~i~~:sc;::~ i:l!
ent in it are very injurious to ::::
human health. So, to remove car- :::
bon particles, use is made of :-:
'Cottrell electrical precipitator' ..:.::'
[Fig. (15)] in which the smoke is '.' Point electrode
made to pass through a positively.;.;
Plate electrode
charged anode in the chimney,
when carbon particles settle f Gases carrying
dust or smoke
down. According to a modern :;::

]~--
I •

~~~~~~ t~:te~m~:h i: l:det:~li~ :!::


knob charged to a very high pos- :;:.
itive potential (50,000 volts). The p.r.~~#~t~Q
gases free from smoke, pass from A~f~r::~~r .
the top of the chimney.
(x) Medicines: Most of H..~:-~~~~~
the medicines which are used, are l·~··::;Lii:~=~=::::::;:::::::c;::"::~~2~i3
colloidal and are effective due to their easy assimilation and ab~orption by
the human system. Colloidal antimollY is effective in curing kalazar. Colloidal
sulphur is effective in curing skin diseases and in kilhng germs. Colloidal
calciltm and gold are used as intramuscular injections to increase the vitality
of human system in serious diseases, Colloidal silver or argyrol is used for
curing gmnulatio~. Milk of li!agllesia, an emulsion. is used for stomach
troubles,
[C] Analytical Applications:
(i) Qualitative and quantitative analysis: A knowledge of the behavi-
our of colloids and their mode of formation play an important role in analytical
problems.
In volumetric analysis, hydrophilic colloids alter the end point, e.g .. in
a titration of HCI and NaOH, the amount of deviation in the end point is
increased with increasing amounts of coJloids. In the volumetric estimation
of silver by Mohr's method, the phenomenon of adsorption comes into being.
204 PHYSICAL CHEMISTRY-I

In gravimetric analysis, definite crystalline precipitates are desired and


the procedure thus adopted aims to get sufficiently bigger particles, because
extremely small particles may pass through the filter paper. To prevent
adsorption of the undesirable molecules with the desired precipitate, it is
necessary to precipitate the solution at the boiling point and fairly dilute
solution should be used.
(ii) Detection of natural from artificial honey: Ley's test for detect-
ing the natural honey from artiticial honey consists in treating few drops of
the honey with an ammoniacal silver salt solution. If the honey is natural or
pure, then the metallic silver produced assumes a reddIsh yellow colour due
to the traces of albumin or ethereal oils, which act as protective colloids and
maintain the colloidal silver in a high degree of dispersion. A dark yellow or
greenish precipitate is formed with artificial honey.
(iii) Identification of traces of noble metals : Noble metals when
present in the colloidal form produce very bright and intense colours. Cassius
purple test indic:ates the presence of colloidal gold.
Problem 8: Give an elementary idea about sol-gel transformation. Men-
tion the physical changes occurring therein. What is the effect of dissolved
substance on sol-gel transformation?
Sol-gel transformation is a phenomenon in which a gel is formed from
a sol. Fernau and Pauli showed that a freshly prepared sol of CeOz containing
about 10 g. per litre was transformed into a gel by coagulation with electrolyte.
But if the sol of CeOz was kept for 200-300 days, then it lost its power and
gave a precipitate instead of a gel, by the addition of electrolytes. But in the
case of some lyophilic sols, such as gelatin in water, agar in water, the sol-gel
transformation is well known and the transformation is reversible within
certain limits. For example, if a gelatin sol (of not too low concentration) is
prepared by heating gelatin with water upto 70°C, and if it is then cooled,
then it is seen that the sol sets to a gel at low temperature. If gel is again
warmed, it liquefies to a sol and so on. This process can be repeated as and
when desired. It appears that an irreversible chemical reaction is continuously
taking place in gelatin sol, which can be easily recognised. If a gelatin sol is
kept at a high temperature at which it does not gelatinise, the viscosity
decreases continuously and reaches a considerable lower end value than its
original value. The sol thus formed loses the property of gelatinizing and the
gelatin thus transformed is known as f3-glutin or galactose. In other words,
the sol ~ gel transformation is practically actually reversible, as long as
the temperature 65-70° is not exceeded.
This sol-gel transformation proceeds perfectly continuously, because
there are no singular points or rapid changes in direction in the temperature-
time curve.
COLLODIAL STATE 205

[I] Physical Changes Occuring in Sol-Gel Transformation


(i) Thermal: The cooling curves of solutions of low molecular weight
substances show usually pronounced breaks or arrest points when solute
begins to separate. With sols the onset on gelatin produces a small break,
showing that the sol-gel transformation is accompanied by the liberation of
heat. Similar behaviour is seen with soap sol when it is changed into gel.
(ii) Optical: Arsiz (1915) showed that there is a complete continuous
change in the Tyndall cone ofIight during the sol-gel transformation. A gelatin
sol in water or water containing glycerine, shows a weak Tyndall cone at
70°C. But as the sol is cooled, the Tyndall cone becomes stronger, and
becomes weaker on warming again. It alters in its intensity during the sol-gel
transformation taking place at constant temperature. In other words, the
Tyndall light increases and decreases with the viscosity of the solution.
(iii) Electrical conductivity: No change is observed in the electrical
conductivity during the sol-gel transformation. If the sols contain electrolytes,
a slight increase in conductivity is observed, which may be ascribed to
secondary effects during the transformation.
(iv) Viscosity: Viscosity is the property which changes remarkably
during the sol-gel transformation. The relationships are clear and well defined
in the case of glycerosols of gelatin as studied and examined by Arsiz.
(v) Diffusion: It has been found that the rate of diffusion of substances
having low molecular weights is similar to the rate of diffusion of pure solvent.
(vi) Volume changes: During the sol-gel transformation a change in
volume occurs. It was found that in some cases expansion of volume takes
place, as in the case of methyl cellulose-water system, while in some cases
contraction of volume takes pa1ce, e.g., gelatin-water system. In the gelation
of 9% sol of ferric hydroxide, no change in volume was observed. The change
in volume can be ascribed to either the process of crystallisation or dissolution.
(vii) Other properties: As regards a number of other properties, sols
and gels appear to be exactly alike, e.g., in refractive index and vapour
pressure etc.
[II] Effect of Dissolved Substances on Sol ~ Gel Trans-
formation
Dissolved substances have a marked effect on the sol-gel transforma-
tion. Paschels (1890-1900) examined the changes in the solidifying point of
gelatin sols in presence of some substances. The effect of anions appears to
be in the order of S04> tartarate > acetate> halogens> thiocyanates. The
first three groups favour gelatinization and thereby raise the solidifying point
and thus shortens the time of gelatinization. The rest of the groups lower the
l>olidifying point and thereby lengthen the time of gelatinization.
Non-electrolytes also influence the time of gelatinization. Lewites
found that substances such as sugar, polyhydric alcohols etc. shorten the time
206 PHYSICAL CHEMISTRY-I

of gelatinization and substances like urea. urethane, thiourea etc .. lengthen


the time of gelatinization.
Problem 9: mite a note on thixotropy.
Or What is thixotropy ? (Meerut 2006)
When suitable quantities of electrolytes are added to a concentrated sol
like Fe(OHh. A1 20 3, V205' Zr02, Sn02 etc. a pasty gel is formed which has
a remarkable property of being liquefied when shaken. only to set again to
gel form. on standing. So. in many cases. gels can be liquefied either by
shaking or any other mechanical action. but returns to the gel state more or
less rapidly as soon as the disturbing action is stopped. Peterfi (1927) sug-
gested the name of 'thixotropy' (change by touch) for this type of phenome-
non. It is also known as 'isothermal reversible sol-gel transformation'.
Systematic observations on thixotropy were made by Szegvari and Shalek
(1923), when they studied sols of Fe:P3 and many dilute sols of high poly-
mers.
Goodeve and Whitfield (1837-39) studied the equilibrium between the
spontaneous building of an internal structure and its breakdown. The apparent
viscosity of a thixotropic system was measured at different rates of shear. If
11n 11r represent the apparent viscosity and extrapolated residual viscosity,
respectively, then we have,
8
11n = 11r + S
where 8 is the coefficient of thixotropy and S is the shear.
If 11n and lIS are plotted as ordinate and abscissa. we get curves of
different types. The intercept on the Y-axis will give the value of 11m i.e.,
resistance of flow at extremely high shear ratio. It has been observed that
thixotropy of carbon black dispersion is reduced on adding 3% linoleic acid.
These curves also show the effect of thixotropy in the absence and presence
of linoleic acid, respectively.
[I] Theories of Thixotropy
Several theories have been proposed to account for thixotropy which
are given below:
(i) Solvation or hydration theory: According to this theory, thick
envelopes called lyospheres of oriented water molecules are formed round
the colloidal particles and these eventually become so large that freedom of
movement is lost and the system becomes a gel. It is also regarded that
lyospheres are destroyed on shaking, but partially, so that the gel is then
liquefied. It is seen that finely divided inorganic substances suspended in
inorganic solvents, also exhibit thixotropy. This fact goes against this theory.
(ii) Oriented coagulation theory : According to this theory, the
particles of sol capable of forming a thixotropic system are believed to be
COLLODIAL STATE 207

anisotropic and anisometric or both, so that the electrical charge and water
of hydration, if any, are unequally distributed. When the correct amount of
electrolyte is added to reduce the zeta potential to a sufficient extent, the
particles tend to coalesce, but owing to their shape they can form a stable gel
only if correctly oriented. The extent of the surfaces of the particle in contact
is probably limited, and as a consequence. any stress will tend to reduce this
and destroy the gel structure. It is interesting to note in connection with this
theory, that nearly all thixotropic sols show marked streaming double refrac-
tion, so that the particles are probably anisotropic or anisometric.
(iii) Three dimensional theory: This theory is based on the hypothesis
of three dimensional gel structure as already described before. It is supposed
that when a gel is thixotropic, shaking is only sufficient to break the cross
linking, thus re-forming the sol. The particles would then be linear, thus
accounting for the double refraction of now suggested in the second theory.
[II] Influence of Foreign Substances on Thixotropy
Thixotropic gelation may be considered as a form of nocculation and
is sensitive to all kinds of additions. The time of solidification (1) is strongly
dependent on the concentration (c) of the electrolyte, as seen in the empirical
equation given by Schalek and Szegvari (1923),
log T=A - B.c
It has been shown that thixotropic gelation is a slow coagulation. Time
of soliditication is also changed with a change in the pH of the solution as
mentioned by Freundlich. It is seen that alcohol promotes the thixotropic
gelation and after evaporating the alcohol. the sol returns to the gel state.
[III] Influence of Temperature on Thixotropy
As seen from Schalek and Szegvari's experiments, the increase of
temperature will decrease the time of solidification.
[IV] Applications of Thixotropy
The phenomenon of thixotropy has found several applications in bio-
logical fields, technology and in articles of daily use. Protoplasm has thixo-
tropic properties. Myosin sols are known to form strongly thixotropic gels,
which might indicate that thixotropy has a significance for muscular action.
Most of the quicksands are thixotropic in nature. The appl ication of thixotropy
i!> much used in drilling muds used in drilling oil wells. Drilling muds always
contain a certain amount of plastic clays responsible for its thixotropy. Paints,
varnishes and printing inks are all thixotropic in nature. A paint having a
greater time of solidification is considered to be its good property. A suspen-
sion of graphite in a mineral oil shows a thixotropic gelation.
208 PHYSICAL CHEMISTRY-I

MULTIPLE CHOICE QUESTIONS


1. The following will have the maximum coagulating power:
(i) Na+ (ii) Sn4+ (iii) Bi+
2. Milk is .
(i) Gel (ii) Sol
(iii) Aerosol (iv) Emulsion
3. The size of colloid particles is :
(i) > 0.1 Il (ii) 1 mll- 0.1 Il (iii) > I mil (iv) > 5000 mil
4. A freshly prepared precipitate ofSn02 peptised by HCI will carry the follOWIng
charge:
(i) Neutral (ii) Positive
(iii) Negative (iv) Amphoteric
5. Every colloid system is :
(i) Homogeneous (ii) Heterogeneous
(iii) Contains one phase
(iv) Homogeneous and heterogeneous.
6. The following is a hydrophobic COllOId:
(i) Gelatin (ii) Gum (iii) Starch (iv) Sulphur
7. Gold sol prepared by different methods will have different colours. It is due to :
(i) Difference in size of colloid particles (ii) Gold shows variable valency
(iii) Different concentrations of gold (iv) Presence of impurities.
8. Following is an emulsifier:
(i) Oil (ii) Water
(iii) NaCI (iv) Soap
9. The sky appears blue. It is due to :
(i) Reflection (Ii) AbsorptiQn
(iii) Scattering (iv) Refraction
10. Which of the following reaction will give colloidal solution:
(i) Cu + HgCl 2 --t Hg + CuCl 2
(ii) Cu + CuCI 2 --t CU2CI2
(iii) 2Mg + CO 2 --t 2MgO + C
(iv) 2HN0 3 + 3H2S --t 3S + 4H20 + 2NO
11. Fat is:
(i) Emulsion (ii) Gel
(iii) Colloidal solution (iv) Solid sol
12. Colloidal solutions cannot be purified by :
(i) Dialysis (ii) Electrodialysis
(iii) Ultrafiltration (iv) Electrophoresis
13. The charge on AS2S 3 sol is due to the adsorption of:
(i) H+ (ii) OW (iii)Sz- (iv) O2
14. Surface tension of lyophilic sols is :
(i) Lower than water (ii) More than water
(iii) Equal to water
(iv) Sometimes lower and sometimes more than water
15. On addition of 1.0 ml of 10% NaCI to 10 rnl gold sol in the presence of 0.0250
gm of starch, the coagulation is just stopped. The gold number of starch is :
COLLODIAL STATE 209

(i) 0.025 (ii) 2.5


(iii) 2.5 (iv) 25.0
16. Which of the following is a lyophilic colloid?
(i) Milk (ii) Fog
(iii) Blood (iv) Gelatm
17. At critical micelle concentration, the surfactant moiecules undergo:
(i) Association (iJ) Aggregation
(iii) Micelle formation (iv) All the above
18. Which of the following electrolytes is least effective in causmg coagulatIOn of
Fe(OHh sol?
(i) K3Fe(CN)6 (ii) K2S04 (iii) KCl (iv) K2Cr04
19. The ability of an ion to bring about coagulation of a given colloid depends on :
(i) Its charge (il) Sign of the charge only
(ii) Magnitude of charge (iv) Both charge and magnitude
20. Which of the following statement is false?
(i) Elastic gels undergo imbibition
(ii) Non-elastic gels are reversible in nature
(iii) Inorganic gels usually exude solvent on standing
(iv) Both elastic and non-elastic gels show thixotropy.

Fill in the Blanks


1. True solutions are ............... systems.
2. In a colloidal system .......... phases are present.
3. In aerosol ............ is the dispersed phase and ............. is the dispersIOn medium.
4. Hair cream is an example of .............. system.
5. The process of converting a fresh precipitate into a colloidal solution is known
as .......... ..
6. Tyndall effect is due to ............. of light.
7. When placed in water ............... gels ............. This phenomenon is called ........ .

8. Breaking of emulsions is also known as .......... .


9. The' emulsifier in milk is ................. .
10. Formation of deltas ~s an example of .................. .

True or False
State whether the following statements are true (T) or
false (F) ?
1. The lower the gold number, the better is the protective power of a protective
colloid.
2. Lyophobic sols are irreversible in nature.
3. The process of passing a precipitate into colloidal solution on adding an elec-
trolyte is called electrophoresis (Meerut 2(02)
4. Colloidal system is biphasic in nature.
5. Colloidal solutions exhibit Brownian motion.
6. The soap may be considered as a surfactant.
7. Blood carries a positive charge.
8. Lyophilic colloids can act as protective colloids.
210 PHYSICAL CHEMISTRY-I

9. The role of an emulsifier is to increase the surface tension between the two
phases.
10. Elastic gels are reversible in nature.
11. Gum arabic is a hydrophobic colloid. (Meerut 2(03)
12. The colour of a colloidal solution depends on the shape and size of the particles.
13. Avogadro's number can be determined with the help of Brownian motion.
14. Out of NaCI, BaCI 2, AICI 3 solutions, the coagUlating power of NaCI is maxi-
mum for coagulating a As 2S3 sol.
15. The difference in potential between the fixed and diffuse layers of an electrical
double layer is called streaming potential.
16. Boot polish is not a colloidal system. (Meerut 2(01)
ANSWERS
Multiple Choice Questions
1. (b), 2. (d), 3. (b), 4. (b), 5. (b), 6. (d), 7. (a), 8. (d), 9. (c),
10. (d) 11. (a) 12. (d) 13. (c), 14. (a), 15. (d), 16. (d). 17. (d). 18. (c),
19. (d). 20. (b),

Fill in the Blanks


1. Homogeneous 2. two 3. liquid, solid
4. emulsion 5. peptization 6. Scattering
7. elastic, swell, imbibition 8. demulsification 9. casein
10. coagulation
True or False
1. (T) 2. (F) 3. (F) 4. (T) 5. (T) 6. (T)

7. (F) 8. (T) 9. (F) 10. (T) 11. (F) 12. (T)


13. (T) 14- (F) 15. (F) 16. (F)

• DOD
CHEMICAL KINETICS &CATALYSIS
1. CHEMICAL KINETICS
Problem 1: (a) Define and discuss the following terms:
(i) Rate of chemical reaction
(ii) Velocity coeffICient
(iii) Molecularity of a reaction
(iv) Order of reaction (Meerut 2007, 2006, 2005)
Or, Write a note on order and molecularity of reactions.
(b) What is the difference between molecularity and order of reac-
tion?
(Meerut 2002 2004. 2000)
(C) Explain why reactions of higher orders are rare?
(d) What are the factors which affect reaction rates? (Meerut 2003)

(a) [I] Rate of Chemical Reaction


It is defined as, 'the rate at which the concentration of a reactant
changes with time', i.e.,
. _ Change in concentration
R eactton rate - T" I
tme mterva
The rate at which a reaction proceeds can be followed by measuring
the concentration of either the reactant or product. If dx represents the amount
of the reactant changed during a small interval of time dt, then the reaction
rate is represented by dxl dt. If, on the other hand, de represents the concen-
tration of the reactant left behind after a short interval of time dt, then the
reaction rate is also represented by - del dt. The negative sign implies that
the concentration of the reactant decreases with time or rate of reaction
decreases with time. The unit of reaction rate is mol L -1 S-1.
For a reaction, A + B ~ C + D, the reaction rate can be expressed as
_ d [A] or _ d [B] or + d [C] or + _ d [D]
~ ~ ~ ~
C,onsider the reaction, 2N20 S (g) ~ 4N02 (g) + O2 (g). The rates of
reaction can be expressed as,
d [N 20 S] d [N0 2] d [0 2 ]
- dt or+ dt or +~
All the rates can be equated as,

(211)
212 PHYSICAL CHEMISTRY-I

1 d [N20 5 ] 1 d [N02 ] d [0 2]
-"2 dt =+4" dt =+~
[II] Velocity Coefficient
Consider the following reaction :
A + B ~ Products
If a and b are the initial concentrations of the reactants A and B and if x
be the number of moles of each reactant undergoing reaction after time t, then
the concentrations A and B after time t will be (a - x) and (b - x) respectively.
According to the law of mass action, the reaction rate (dxldt) is given by
dx oc (a - x)(b - x) or dx = k (a - x) (b - x)
dt dt
where, k is a constant known as velocity constant, rate constant, velocity
coefficient or specific reaction rate. It is characteristic of the reaction.
If (a -x) = 1 and (b -x) = 1, then
k=: = Rate of reaction
So, velocity coefficient is defined as, the rate of reaction under con-
ditions when the molecular concentration of each reactant is unity.
[III] Molecularity of Reaction
It is defined as, the total number of molecules of all the reactants
taking part in a chemical reaction as represented by a simple equation.
Examples. (i) Inversion of cane sugar (Molecularity = 2)
H+ ions
C 12H 220 11 + H 20 --7 C6H 120 6 + C 6H I2 0 6
(ii) Dissociation of ammonia (Molecularity = 2)
2NH3 ~ N2 + 3H2
(iii) In general, for a reaction, nlA + 1I2B ~ 113C + 11l2D the molec-
ularity will be nl + 112'
So, reactions having molecularity 1,2, 3 etc., are known as unimolecu-
lar, bimolecular, trimolecular reactions, respectively.
[IV] Order of Reaction
Reactions are generally classified on the basis of their order of reaction.
It is defined as,
'The total number ofreacting molecules whose concentration changes
during the chemical reaction. '
In other words, it is also defined as,
'The total number of reacting molecules whose concentration deter-
mines the rate of reaction '.
(a) If A ~ Products and (dxldt) = k [A], then
Order of reaction = 1
CHEMICAL KINETICS & CATALYSIS 213

(b) If A + B ~ Products and (dx/dt) = k [A][B], then


Order of reaction = 2
(c) If nlA + n2B + n3C + ... ~ Products, and
: = k [A)"' [Btl [C]"3 ... , then
Order of reaction = 11\ + 112 + 1I3 + ...
dx
(a) If A + 2B ~ Products, and - = k then
dt '
Order of reaction = 0
In general, order of reaction is also defined as, the sum of the powers
to which the concentration (or pressure) terms of the reactants are raised
in order to determine the rate of a reacton.
(b) Relation between Molecularity and Order of Re-
action
Till recently, the molecularity and order of reaction were regarded as
synonymous. But, it has now been found that in certain cases the two are not
identical. For example, during the inversion of cane sugar into glucose and
fructose or the hydrolysis of ethyl acetate in presence of acid catalyst, appar-
ently two molecules take part, making the reactions bimolecular. But actually,
the concentration of only one of reactants changes, i.e., concentration of water
does not appreciably change. Hence, the reactions though bimolecular are of
the first order.

H+ ions
CH3COOC 2H 5 + H 20 - - 7 CH300H + C 2H 50H
Thus, often molecularity of a reaction coincides with its order, but the
two need not be always identical. The molecularity must be an integral value,
but the order of reaction may be zero, whole number or even fractional.
Whereas molecularity can be given on the basis of some proposed theoretical
mechanism so as to satisfy the experimental findings, the order of reaction
can be obtained from experimental results.
For complex reactions, occurring in steps, the molecularity of each step
will be different, while the order of reaction will be determined by the slowest
step.
Table-I. Difference between order of reaction and molecularity
Molecularity Order of reaction
l. It is equal to the number of molecules of J. It is equal to the sum of the powers of the
reactants which take part in a single step molar concentrations of the reactants mJhe
chemical reaction. rate expression.
2. It is a theoretical concept which depends 2. It is an expermientally determined quantity
on the rate determining step in the reaction which is obtained from the rate for the
mechanism. overall reaction.
214 PHYSICAL CHEMISTRY-I

3. It is always a whole number. 3. It may be whole number. zero or fractIOnal


value.
4. It is obtained from a single balanced 4. It cannot be obtained from a balanced
chemical equation. chemical equation.
5. It reveals some basic facts about 5. It does not reveal anything about
reaction mechanism. reaction mechanism.
(c) Most of the reactions are of the first and second orders. According
to kinetic theory of gases, the reaction occurs due to collisions of molecules.
For a second order reaction, two molecules have to collide which is reasonably
possible. But the chances of three, four or even more molecules colliding
simultaneously are very remote and so reactions of higher orders are very
rare. There are a number of complex reactions involving a number of mole-
cules. But in such cases, the reactions occur in several stages and slowest
stage is the rate determining step and it gives the order of reaction.
The following reaction, though involving ten molecules is of the second
order.
KCI0 3 + 3H2S04 + 6FeS04 ~ 3Fe2(S04h + KCI + 3H20
(d) Factors which Affect the Rate of Reactions
The following factors influence the rate of reactions.
(i) Effect ofconcentration: We know that the rate of a reaction falls
with time, so it is clear that the rate of reaction is directly proportional to the
concentration of reactants.
(ii) Effect of temperature: The rate of reaction increases with rise
in temperature. In most cases, a rise of lOoC in temperature doubles and in
some cases even trebles the rate of a reaction. The ratio of the velocity
constants of a reaction at two temperatures differing by lOoC is called tem-
perature coefficient of the reaction.
. . Rate constant at 35°C _ k35
Temperature coeffiCient = R 250C
ate constant at k25

(iii) Effect of nature of reactants: Rates of reactions are effected


considerably by the nature of reactants. For example, consider the two well
known oxidation reactions taking place in aqueous solution. One is the oxi-
dation of ferrous ion, Fe 2+ and the other of oxalate ion, C:p/-, by perman-
ganate ion in acid medium.
(a) 5Fe2+ (aq) + Mn04- (aq) + 8H+ (aq) ~ 5Fe3+ (aq)
+ Mn2+ (aq) + 4H 20
(b) 5C20/- (aq) + 2Mn04- (aq) + 16H+ (aq) ~ IOCOz (g)
+ 2Mn-'+ (aq) + 8H20
The first reaction is much faster than the second. As Mn04" ion is
common in both the reactions, the difference clearly lies in the nature of
ferrous and oxalate ions. Fe2+ ion is a simple ion, whereas C 20/- ion is a
poly atomic ion and contains a number of covalent L:mds which have to be
broken in the oxidation reaction.
CHEMICAL KINETICS & CATALYSIS 215

(iv) Effect of catalyst: A catalyst generally increases the rate of a


reaction at a given temperature. A catalyst is generally specific in its action,
i.e., it may affect the rate of one particular process only. In very few cases, a
catalyst may decrease the rate of a reaction.
(v) Effect of surface area of reactants : In heterogeneous reactions,
particle size decreases and so the surfae area for the same mass increases.
The smaller particles, therefore, react more rapidly than the larger particles.
(vi) Effect of radiation: We know that the energy associated with
each photon of radiation is given by hv, where h is Planck's constant and V
is the frequency of the radiation concerned. The rate of certain reactions may
be speeded up by the absorption of photons of certain radiations. Such
reactions are called photochemical reactions.
Problem 2: What do you understand by a zero order reaction? Derive
the rate expression for it. What are the characteristics of zero order reac-
tions? Given an example of such a reaction.
Or, Write a short note on zero order reaction. (Meerut 2003)

[I] Zero Order Reaction


A reaction in which the concentration ofthe reactants does not change
with time and the reaction rate remains constant throughout is said to be
a zero order reaction.
[II] Rate Expression for Zero Order Reaction
Consider the following zero order reaction, A -----7 Products
dx
:. Reaction rate, dt = k (k = velocity constant)
or dx =k dt

On integration, we get

or x == kt + 1 where, 1 == integration constant ... (1)


When t=O, thenx=O.
1=0
:. From equation (1), x == kt or k=~ ... (2)
t
Equation (2) is known as zero order rate expression.
[III] Characteristic
(i) The dimension of velocity constant is concentration x time-lor
mole liC! time-! or mole liC! s-t, if time is expressed in second.
[IV] Example
(i) Several photochemical reactions and heterogeneous reactions (en-
zymic) are of zero order, e.g., the photochemical combination of Hz and
Cl 2 in presence of sunlight to give HCl is of zero order.
216 PHYSICAL CHEMISTRY-I

Sunlight
H2 (g) + CI 2 (g) ----t 2HCI (g)
Problem 3: What is a half order reaction? Derive rate equation for it
and discuss its characteristic.
[I] Half Order Reaction
A reaction in which the reaction rate depends on half power of concen-
tration of a reactant is known as half order reactioll, i.e .•
A ~ Products
. cbc
.. ReactIOn rate. dt = k [A] 112
[II] Rate Equation for Half Order Reaction
Consider the reaction.
A ~Products
(Initial COliC.) a
(COliC. at time t) (a-x)

dx-
- _ k (a _ x )112
dt
dx
or ---l/-~ = k dt
(a - x) ~

On integrating,

dx 112 =fkdt
f (a -x)

1 112
or --(a-x) ===kt+I ... (1)
2
where I = integration constant.
When t = O. x = O. so
-! a 112 = I
2
Putting the value of J in equation (1). we get.

- 2"1 (a - x)
112
= kt - 2"1 a 112

or k= - ;t [(a - x) 112 - a 112]

... (2)

Equation (2) is the required rate equation for a half order reaction.
CHEMICAL KINETICS & CATALYSIS 217

[III] Characteristic of Half Order Reaction


(1) The unit of rate constant will be _._1_ x conc l12 or
time
(mol L-I)1I2 (sri or mol 112 L -112 S-I.

Problem 4: What is a first order reaction? Derive the rate expression


for it and discuss its characteristics. Give some examples of first order
reaction.
(Meerut 2(02)

[I] First Order Reaction


A reaction in which the reaction rate depends only on one concentra-
tion term is said to be a first order reaction.
For example, if A ~ Products

then, reaction rate = ~ =k [A]

[II) Rate Expression for First Order Reaction


Consider the following simplest first order reaction.
A ~ Products
Let a mole/litre be the initial concentration of the reactant A. Suppose
x mole/litre of A decomposes in time t, leaving behind (a - x) mole/litre of
it. For a first order reaction, the reaction rate at any time t is given by :

: :::: k {A] = k (a - x)

where, k is the velocity constant or rate constant.


dx
or --=kdt ... (1)
a-x
In order to get the value of k, we integrate equation (1). So,
- loge (a - x) == kt + I ... (2)
where, I is the integration constant.
When t =0, x :::: 0, therefore, from equation (2),
I =-loge a
Substituting the value of I in equation (2),
- 10& (a - x) = kt - loge a
1 a
or k::::-Io&-- ... (3)
t a-x
2.303 I a
k ::::-- oglO-- ... (4)
I a-x
(As loge x == 2.303 10gIO x)
Equations (3) and (4) are known asfust order rate expressions. The
constant k is called the first order rate constant.
218 PHYSICAL CHEMISTRY-I

[III] Characteristics of First Order Reaction


(i) The value of velocity constant is independent ofthe units in which
concentration ofthe reactant is expressed. If unit of concentration is changed
to say n times its original value, then equation (4) becomes
2.303 I n.a 2.303 I
k =- - oglO
t n.a - n.x
=- t
- oglO 11 (ana- x)
= 2.303 10glO _a_
t a-x
This is the same as equation (4).
(il) The dimension offirst order rate constant is reciprocal of time
i.e., t-1• From equation (4), we have
k = _1_ . concentration = _1_
time concentration time
If time is expressed in second or minute, k is expressed in second-lor
minute -1, respectively.
(iii) The time taken to complete a certain fraction of a reaction is
independent of the initial concentration of the reactant. Suppose tl/2 is the
time taken to reduce the concentration of reactant A to half, i.e., from a to
al2, then x:::: al2. From equation (4), we get
2.303 a 2.303
k :::: - - 10glO (/2) :::: - - 10glO 2
tl/2 a- a tl/2

_ 2.303 I 2 - 2.303 03010 _ 0.693 ... (5)


or tl/2 - k oglO - k x. - k
Equation (5) is thus independent of a, i.e., the initial concentration of
the reactant.
[IV] Examples of First Order Reaction
(i) Decomposition of hydrogen peroxide in aqueous solution.
I
H 20 Z - - ? H 20 + "2 O2
(ii) Hydrolysis of methyl acetate in presence of acid.
Acid
CH3COOCH3 + H 20 --? CH3COOH + CH30H
(iii) Hydrolysis of cane sugar in presence of acid.
Acid
C I2H 220 Il + H20 --? CJI120 6 + C6H 1Z0 6
Problem 5: (a) What are pseudQ-unimolecular reactions? (Meemt 2000)
(b) How will you study the kinetics of the hydrolysis of methyl ace-
tate?
(c) The rate of first order reaction increases with the concentration
of the reactant. Explain with reason. (Meenlt 2005)
CHEMICAL KINETICS & CATALYSIS 219

(a) Pseudo-Unimolecular Reactions


Reactions which are not unimolecular, but obey the first order rate
expression are known as pseudo-unimolecular reactions. For example. hy-
drolysis of methyl acetate. inversion of cane sugar etc. are pseudo-unimolecu-
lar reactions. In general. when the order of reaction is generally less than the
molecularity of a reaction, it is said to be a pseudo order reaction.
H+
CH3COOCH3 + H 20 ~ CH3COOH + CH30H
The reaction is bimolecular but the rate of reaction is given by,
dx
-dt = k [CH3COOCH.31

The concentration of water does not change during the course of the
reaction, as it is taken in excess.
Similarly, for the reaction,
H+
C 12H 220 Il + H20 ~ C6 H 120 6 + C6H 120 6,
the rate of reaction is given by
dx
dt = k [C12H220111

because the value of [H20] remains constant as it is taken in excess.


(b) Kinetics of Hydrolysis of Methyl Acetate
(Experimental Determination)
The hydrolysis of methyl acetate is catalysed by hydrogen ions and
occurs as follows :

It is a pseudo-unimolecular reaction, for although two molecules take


part in the change, the concentration of water does not appreciably change
because of its being present in large excess. As acetic acid is produced in the
reaction, the reaction can be followed by withdrawing a definite quantity of
the reaction mixture after definite intervals of time and titrating it against a
standard alkali solution, say NaOH. The amount of alkali used is equivalent
to the total amount of HCI present originally and the amount of acetic acid
formed in the reaction. As the amount of HCl originally present is known or
can be determined by titrating against the same alkali at the start of the
reaction, the amount of acetic acid formed (x) after different intervals of time
(t) can be determined. The amount of acetic acid formed at the end of the
reaction is equivalent to initial concentration (a) of the esier.
5 ml of reaction mixture (10 ml CH3COOCH3 + 100 ml 0.5 N HCl) IS
withdrawn into a conical flask at the start of the reaction and some pieces of ice
220 PHYSICAL CHEMISTRY-I

are added to freeze the equilibrium. It is titrated against N/1O NaOH, using
phenolphthalein as an indicator. Similarly, 5 ml of the reaction mixture is with-
drawn after say 5,10,20,30 minutes and also at the end of 24 hours, when the
reaction is supposed to be complete. The solution is then titrated as usual.
Suppose the volumes of standard NaOH solution required for the neu-
tralisation of 5 ml of the reaction mixture, at the start of the reaction, after
time t and at the end of the reaction, are VO,VI and V~, respectively. Then
x, the amount of acetic acid formed after time t is proportional to
(VI - Yo), while a, the initial concentration of the ester is proportional to
(V~ - Yo). Thus, (a - x), the concentration of the ester left behind unreacted
after time t is proportional to (V~ - yo) - (VI - yo) or (V~ - VI)' SubstitutIng
the values of a and (a - x) in the first order rate equation,
k - 2.303 10 _a_
- t g,oa-x
we get, k - 2.303 log V~ - Vo
- t 10 V~ - VI

The values of k at different intervals of time come out to be constant.


This shows that the reaction is of the first order.
(C) For a first order reaction, the rate of reaction is given
by,
dx dx
dt = k [Reactant] or dt oc [Reactant]
So, the rate of reaction will increase with an increase in the concentration
of the reactant.
. Problem 6: (a) Show that the time required for the completion of a
definite fraction (say one halj) of the reaction is independent of the initial
concentration for a reaction of the fust order.
(b) Prove that if the half life period of a first order reaction is 30
minutes, the reaction will be 75 percent complete in 60 minutes
(Meerut 2004)
(8) Suppose is the time taken to complete a definite fraction, say
tl/2
one half of a reactant from initial concentration a to a12, i.e., x = a12. For a
first order reaction,
2.303 I a 2.303 I a
k =- - oglo - - =- - oglO--
t a-x tll2 a
a-2"
2.303 a 2.303
or tll7 =- k - loglO (aI2) =-k-1oglO 2
2.303 x 0.3010 0.693
= k =-k-

Thus, tll2 is independent of initial concentration, a.


CHEMICAL KINETICS & CATALYSIS 221

(b) 75 percent completion of a reaction involves two half life periods,


so the time taken for the reaction to be 75 percent complete will be
2 x t1l2 =2 x 30 minutes =60 minutes.
Problem 7: What is a second order reaction? Derive the rate expression
for a second order reaction. Discuss the characteristics of a second order
reaction. Write some examples of second order reactions and study the
kinetics of anyone reaction. (Meerut 2002, 2000)
Or What is the unit of k in a second order reaction? (Meerut 2006)

[I] Second Order Reaction


A reaction is said to be of the second order if its reaction rate depends
on two concentration terms of reactants.
For example,
2A ~ Products.
The rate of reaction at any given time is given by
dx = k [A]2
dt
[II] Rate Expression for Second Order Reactions
(a) When there is only one reactant or both reactants have equal
concentrations : Consider the following reactions :
2A ~ Products.
or A + B ~ Products.
Let a be the initial concentration of each of the two reactants and
(a - x) be their concentration after any time t. Then, according to the law of
mass action, the reaction rate is given by
dx 2
- = k (a - x) (a - x) = k (a - x)
dt

or dx = k dt
(a _x)2
where k is the velocity constant.
In order to get the value of k, we integrate the above expression, i.e.,

dx 2 =fkdt
f (a -x)
1
or --=kt+1
(a -x)
(I = integration constant) ... (1)

When t = 0, x = 0, therefore, from equation (1),

* It is assumed that the concentration of the substance which is taken in excess does not
appreciably change during the course of the reaction.
222 PHYSICAL CHEMISTRY-I

1=1..
a
Substitutmg the value of 1 in equation (1). we get
-1-=kt+1..
a-x a
1 1
or ----=kt
a-x a

or k=1..[_l _1..]
t a-x a

or k=1... x ... (2)


t a (a - x)
Equation (2) is known as second order rate expression.
(b) When the initial concentrations of both reactants are not equal :
Let a and b be the initial concentrations of reactants A and B and let (a - x)
and (b - x) be their respective concentrations after any time t. The reaction
rate will thus be given by

: = k (a -x)(b -x) or _---=dx=-__ =k tit


(a -x)(b - x)

or _1_ [_1___1_]
(a - b) (b -x) (a -x)
dx = k dt .. ,(3)

In order to get the value of k, we integrate equation (3), i.e.,

_1_ f[-I- -_1_] f k


(a - b) (b -x) (a -x)
dx - dt

or (a~b)[f (b~X) -f (a~x)l= f kdt

1
or (a _ b) [-loge (b - x) + lo~ (a - x)] = kt + 1 ... (4)

(l = Integration constant)
When t = 0, x = 0, therefore, from (4),
1 1 a
(a-b) [-Iogeb + loge a] =1 or 1 = (a _ b) loge b
Substituting the value of 1 in equation (4), we get
_1_ (a-x) _ _1_ ~
(a-b) loge (b-x) -kt+ (a-b) loge b

or k= 1 10 b (a -.i2 ... (5)


t (a - b) ge a (b - x)
CHEMICAL KINETICS & CATALYSIS 223

Equation (5) is known as second order rate expression.


[III] Characteristics of Second Order Reactions
(i) The rate constant is not independent of the unit in which the
concentration is expressed.
Let the new unit be 11 times the first value. So, from equation (2),
k,=lx nx _1 x .1
t na x n (a - x) t a (a - x) 11
The new value of k, i.e., k' is lin times the original value.
(ii) Unit of rate constant.
From equation (2), the rate constant will be expressed in terms of
(litime) x (conc/conc 2) or (concr l (timer'. If concentration is expressed in
mole L-I and time in second, then k will be expressed in (mol L-Ir l (srI or
mol- I L S-I.
(iii) The time taken to complete a certain fraction of the reaction is
inversely proportional to the initial concentration of the reactant.
Let tll2 be the time required for the completion of half the reaction,
i.e., x = O.5a. Substituting this value in equation (2), we get
k =_1_ . O.5a =_1_ . O.5a 1
tll2 a (a - O.5a) tl/2 a X O.5a a tl/2
1 1
or tll2 = -k or tll2 oc -
.a a
(iv) If one of the reactants is present in excess, the second order
reaction becomes of the first order.
For different initial concentrations of the reactants, we have from equa-
tion (5),
k= 1 10 b (a - x)
t (a - b) ge a (b - x)
When one of the reactants, say A is present in very large excess, we
can neglect x and b in comparison to a. The above equation then reduces to :
1 ba l b
k=-log =-log
ta e a (b - x) ta e b ~x
Since a remains constant throughout the change, we have
1 b
k.a=-lo~--
t b-x
k,=l.log _b_
t e b-x

This equation is identical with first order rate expression.


[IV] Examples of Second Order Reactions
(i) Hydrolysis of an ester by an alkali.
CH3COOC 2H5 + NaOH ~ CH3COONa + C 2H50H
(ii) Thermal decomposition of acetaldehyde.
224 PHYSICAL CHEMISTRY-I

2CH3CHO ~ 2CH4 + 2CO


(iii) Decomposition of ozone into oxygen.
203 ~ 302
[V] Study of Kinetics of Hydrolysis of Ethyl Acetate
(Experimental Determination)
The reaction between ethyl acetate and alkali also known as saponifi-
cation of ester occurs as follows :
CH3COOC 2H5 + NaOH ~ CH3COONa + C 2H50H
Take 50 ml each of 0.01 M ester solution and 0.01 M NaOH solution
in a flask. The reaction mixture is kept in a constant temperature bath. 5 ml
of the reaction mixture is withdrawn at regular intervals, say at the start and
after 5, 10, 20, 30, 40 minutes. Ice is added to the reaction mixture to freeze
the equilibrium. The mixture is titrated against a standard acid say 0.05 M
HCI. The reaction mixture is then kept for 24 hours when it is supposed to
be complete and 5 ml of it is titrated as before. The volume of acid in each
case corresponds to the amount of unchanged caustic soda or ethyl acetate,
i.e., (a - x) at that time. The volume of acid required initially corresponds to
the original concentration, a. Thus, x will be given by a - (a - x).
Inserting the values of a, x and (a - x) in second order rate expression,
k=!. x
t a (a -x)
we find that the values of k come out to be constant. This shows that the
reaction is of the second order.
Problem 8 : Show that the time required for the completion of a definite
fraction (say one half) of the reaction is inversely proportional to the initial
concentration for a second order reaction. (Meerut 2(03)
Suppose the initial concentration of the constant is a. Let tl/2 be the
time required to complete a definite fraction, say one half of the reaction,
then x = a12. For a second order reaction,
k =! . x =_1_ . al2
t a (a - x) 11/2 a (a - al2)

or
1 _! ~ __1_
112 -k . a (aI2) - k . a
1
or tl/2 oc-
a
i.e., the time is inversely proportional to the initial concentration of the
reactant.
Problem 9: What are third order reactions? Derive the rate expressions
and d~cuss their characteristics. Give examples of third order reactions.
CHEMICAL KINETICS & CATALYSIS 225

[I] Third Order Reaction


Reactions of third and higher orders are rare, but there are in fact
reactions which are definitely of third and sometimes of higher order. This is
due to the fact that the probability of three molecules coming to a single point
simultaneously, i.e., probability of trimolecular collisions is much less as
compared to unimolecular or bimolecular collisions.
So, a third order reaction is one in which the reaction rate depends
on three concentration terms of the reactant/so
For a third order reaction :
A + B + C - 7 Products
The reaction rate is given by,
dx
dt = k (a - x)(b - x)(c - x)

[II] Rate Expressions for Third Order Reactions


Case I. When a =b =c, we have
dx 3
dt = k(a -x)

or dx kdt
(a _x)3
On integration, we get

f(a :t X
)3 =f k dt

1
---=kt+1 ... (1)
2 (a _x)2
where 1 = integration constant.
When t =0, x =0, we then have
_1_=1
2a 2
Substituting the value of 1 in equation (1), we get
1 1
----=-2 = kt +-2
2(a -x) 2a
1
kt=---,.
2(a -xi - 2a2
=1. [a 2- (a - X)2] =! x (2a - x)
2 a2 (a _x)2 2 a2(a _x)2

or k =.l . x2(2a -x)2 ... (2)


2t a (a -x)
Case II. When a -:t b -:t c, we have
226 PHYSICAL CHEMISTRY-I

dx
dt = k(a - x) (b - x) (c - x)
dx
ill =k&
(a - x) (b - x) (c - x)
Breaking into partial fractions, we get
dx + dx + dx =k~
~-~~-~~-~ ~-~~-~~-~ ~-~~-~~-~
On integrating, we have

k =..!.l(b-c) log~+(c-a) IOg¥+(a-b) 'Og~l


... (3)
t 1 (b - c)(c - a)(a - b)
Case III. When two of the molecules are identical as in
2A + B -----7 Products
the concentrations at any time are (a - 2x) and (b - x). The rate equation then
becomes
: = k (a - 2x)2 (b - x)

On integrating, we get
k= 1 [2x
(2b - a) + In b (a - 2x)]
t (2b - a)2 a (a - 2x) a (b - x)

[III] Characteristics of Third Order Rate Expressions


(1) From equation (2), we have
k = _._1_ . (concentration x concentration)
tIme (concentration)2 x (concentration)2
1 1
=-.-x 2
tIme (concentration)
The unit of k will be (timer l (concentrationr 2• If concentration is
expressed in mole/litre, then k will be expressed in (timer l (mole/litrer2.
(2) The time taken to complete a certain fraction of a reaction is
inversely proportional to the square of the initial concentration of the
reactants.
The time (to.5) taken for the completion of half the reaction is given by
x = a12. Therefore,
k =-_,_ al: (2a - a12)
2t05 a~(Il-aI2)
1 (,':~. 3a!2 3 1
or to.5 -= - . - '---2-- =- . ---;;-
2t a-. a 14 2k a-

i.e.,
CHEMICAL KINETICS & CATALYSIS 227

[IV] Examples of Third Order Reactions


The following reactions belong to third order reactions:
(i) 2NO + O2 ~ 2N02
(ii) 2NO + Cl 2 ---7 2NOCI
(iii) 2NO + Br2 ---7 2NOBr
(iv). keaction between p-nitro benzoyl chloride and n-butyl alcohol.
(v) Reaction between silver acetate and sodium formate.
HCOONa + 2CH3COOAg ~ 2Ag + CO 2 + CH3COOH + CH3COONa
Problem 10: What are nth order reactions? Derive the rate equation and
discuss the characteristic.
[I] Definition
A reaction in which the reaction rate depends on nIh power of the
concentration of the reactant is known as nth order reaction, e.g.,
nA ~ Products
dx =k [At
dt
[II] Rate Equation
In general, a reaction of the nth order where all the initial concentrations
are the same is represented as,
nA ~ Products
(Initial cone.) a
(Cone. at time t) (a - x)

dx =k [At =k(a - xt
dt
On integrating, within the boundary conditions, t =0, x =0, we get
fo x dx =fl kdt
(a-xr 0

or
[(n-I)(:_X)(n-I)I~: =k[t(~
or _1_[ "I
(n-I) (aJx)(n-l)- a(n-l) -
_I_]-kt
\"

or k ~ I. [ 1(
t(n-l) (a-x)n-
I) --( 1 I)]
a n-
... (1)
This equation is applicable for all orders except when n = 1, because
when n = 1, n - 1 = 0.
So,
k 0 t
1 [
X0
1
(a _ x)' - :0 j
..,
228 PHYSICAL CHEMISTRY-I

=-7 (~) =Indeterminate


Therefore, the equation becomes indeterminate.
[III] Characteristic
Halflife value. The time taken to complete half reaction is inversely
proportional to (n - l)th power of initial concentration of the reactant. For
half change,
a
x ="2' t= t1l2

Putting the values in equation (1),

'112 = k(n ~ 1) [(a _ i{" - "J l


a }-

1 [2(n
= k(n-l) a(n-I) - a(n-I)
-I) 1]
1 [2(n -I)
= k(n-l) a(n-I) - a(n-I)
1]
= 1 [2(n-l) _ 1]
k(n-l)a(n-I)
(2(n-I)-1) 1
t1l2 = k(n _ 1) . a(n - 1) .•• (2)

1
or t 112 = Constant X (i1=1)
a
1
or t 112 oc (i1=1)
a
Problem 11: Discuss the methods employed in determining the order of
a reaction. (Meerut 2005, 2001)
There are several methods to determine the order of a reaction.
[I] Integration Method (Hit and Trial Method)
In this method, the initial concentrations of all reactants are determined.
The concentration of the reacting substances is then determined by analysing
the reaction mixture at different intervals of time. The different values of a
and x are then substituted in rate expressions of the first, second and third
order reactions. The order of reaction is given by that equation which gives
a nearly constant value of k. This method, therefore, involves the trial of one
equation after another till the correct one is found. This method is extensively
used for simpler reactions.
CHEMICAL KINETICS & CATALYSIS 229

[II] Fractional Change Method or Half-change Method


(Method of equi-fractional parts)
We know that the time required to complete a certain fraction (say one
half) of the reaction. is independent of initial concentration for a first order
reaction, is inversely proportional to the initial concentration for second order
reaction, is inversely proportional to the square of the initial concentration
for a third order reaction. In general. the time required for the completion of
the same fraction of the reaction is inversely proportional to the initial con-
centration raised to the power which is One less than the order of reaction,
i.e.,
1
toc--n-I
a
where, n is the order of reaction.
If tl and t2 be the times for the completion of the same fraction of
reaction with different initial concentrations al and a2 and if n is the order of
reaction. then

or

or

or

For gaseous systems, if PI and pz be the pressures at times tl and t 2, then


log (t1/t2)
n = 1 + -"-'--=---=-
log (P2/PI)
So, in this method, we start with two different concentrations of the
reactants and note the time for the completion of any fraction, say, half change
in each case. By substituting these values in the last equation, we can calculate
the order of reactiOIf. n.
[III] Ostwald's Isolation Method
This method is applicable particularly for those reactions where two or
more reactants are involved. The experiments are carried out by taking all
reactants except one in excess. The reactant which is not taken in excess is
said to be isolated*. The order of reaction is then determined with respect to
the isolated reactant. The order of the whole reaction will then be the sum of
these individual orders of reaction.

* It is assumed that the concentration of the substance which IS taken in excess does not
appreciably change during the course of the reaction.
230 PHYSICAL CHEMISTRY-I

Consider a general reaction


nlA + n2B + n3C ~ Products
Let the order of reaction be nl when A is isolated, i.e., Band Care
taken in excess, n2 when B is isolated, i.e., A and C are taken in excess and
n3 when C is isolated, i.e., A and B are taken in excess. The total order of the
reaction will thus be given by nl + n2 + n3.
[IV] Graphical Method
In this method, the values of x (amount decomposed) are plotted against
t (time). The value-Of dxldt at any time is determined from the graph (Fig.
1) by measuring the angle e, since dxldt =tan 8.
::::::::··::t:·:·:·:·:::-::·::::·::::·::::·;':·:·:·:::::::::::::::·::::::::::::::::::\l!·~

( '5 :;.:
.... ...... ·o~

U1<-_~.,{?_i'._O"_~0_1. q;:_0~
:::: ~ ~
I

:dx Slope = tan 9 = dx


I dt
C ..... !S
dt
_ _ _--,

.... (a-x)_
: : :. ~========~:

The various values of dxl dt (Y-axis) are plotted against the correspond-
ing concentrations (a - x) on X-axis as shown in figure (2). If we get a straight
line, the reaction is of the first order. In case, a straight line is obtained by
plotting dxl dt and (a - x)2 or (a - x)3, the reaction is of second or third order,
respectively.
Problem 12: Di1icuss the energy of activation and temperature coeffi-
cient. (Meerut 2000)
Or Di1icuss the effect of temperature on reaction velocity in the light of
Arrhenius theory. How the energy of activation of a chemical reaction can
be determined experimentally?
[I] Temperature Coefficient
See problem 1 (d).
[II] Energy of Activation
Arrhenius (1889) put forward the hypothesis that all the molecules of
a reactant do oot take part in the chemical reaction. It is only a certain number
of molecules known as active molecules that may take part in the reaction.
CHEMICAL KINETICS & CATALYSIS 231

The molecules not capable of taking part in the reaction are known as passive
molecules.
On raising the temperature, the equilibrium between active and passive
molecules is supposed to shift over rapidly, so that not only we have an
increased collision frequency, but still more, we have a largely increased
number of active molecules ready to take part in the reaction. The equilibrium
is represented as
Mpassive ~ Mactive

K = [M active]
a [M passive)

Rate of the reaction oc [M active] = Ka [M pa"ive]


According to this concept, passive or non-reactive molecules can be
activated by absorption of energy. An expression connectmg the temperature
with the equilibrium constant (K) of a reversible reaction was given by van't
Hoff on thermodynamical grounds. The relation is given by
dlogeK !ill
... ( 1)
dt - RT-
where, aE is the heat of reaction or energy change.
For a reversible reaction :
A+B ~C+D
k) [CUD]
we have, K,. = k2 = [A][B]
where k) and k2 are the velocity constants for forward and backward reactions,
respectively.

From van't Hoff isochore, we have


dlog e K,. _ aE d loge (k)/k2) aE
or
dT - RT- dT Rr
---_.

It follows that ... (2)

and ... (3)

where E) - E2 = !ill and B is a constant.


It has been found that B is independent of temperature and is equal to
zero, so that
d loge k) E)
dT -RT-
d loge k2 E2
and
dT -RT-
232 PHYSICAL CHEMISTRY-I

dlogek EI E
In general, dT = Rr or d log e k = ---
Rr . dT ... (4)

On integration, we get
E
loge k = - RT + constant ... (5)

or more general, k = Ae-E1/<F ... (6)


where A = constant, known asfrequency or Arrhenius factor of the reaction
or collision number. It is also known as pre-exponential factor. £ is a term
which has the dimensions of energy. If R is measured in calories, then £ is
measured in calories per mole. Equation (6) is the famous Arrhenius equation
for the rate constant.
On taking logarithms, equation (6) can be written as follows:
£
loge k = loge A - RT
Differentiating it with respect to temperature, we get
£
dlog k=-~ dT ... (7)
e Ri
Integrating equation (7) between proper limits, we have

k'- IT'- £
fk
I
dlogek=
T
I
-ry

RJ'
dT

k? £ ( 1 1)
or 10glO k~ = 2.303 R TI - Tz ... (8)

[III] Determination of Activation Energy


If we measure the velocity constants of a reaction at two different
temperatures, it is possible to calculate the value of activation energy (E) from
equation (8). The value of E can also be evaluated graphically.
If 10glO k is plotted against liT, we get a straight line, the slope of
which is equal to - E/2.303 R or - £/4.576. Knowing the slope, E can be
easily evaluated. The intercept on loglo k axis gives the value of log A.
The Arrhenius equation holds equally good for homogeneous and het-
erogeneous reactions. Heterogeneous and catalytic reactions also give straight
lines over a very wide range of temperatures.
If there is a marked deviation from a straight line on plotting log k and
liT, then it gives an indication that the observed reaction is a composite one
made up of two or more concurrent reactions influenced by one of the
reactions and it may predominate, so that the slope of the curve corresponds
to the value of E proper to this reaction.
CHEMICAL KINETICS & CATALYSIS 233

For many reactions taking place at ordinary temperatures, the energy


of acti vation is of the order of 20,000 cal/mole and under such circumstances
the temperature coefficient (tc) is found to satisfy van't Hoff rule, e.g., at
300K, the value of tc is given by
kT + 10 d loge k E
log t = log - - =
e c e kT dt
X
Rr
10=-x 10

10 kT + 10 = 20,000 X 10 "" 1
ge kT 2 X (300)2

kTk+TIO "" e "" 2.7

Problem 13 Discuss activation energy, potential energy barrier and


Arrhenius law.
We know that for a chemical reaction to occur, collisions between
reactant molecules must occur. It has been postulated that collisions between
those reactant molecules only which are associated with a certain minimum
amount of energy can cause a chemical reaction. The minimum amount of
energy which must be associated with molecules so that mutual collisions
may result in chemical reaction is termed as threshold energy. Molecules
possessing energy less than threshold energy will not react on collisions.
Molecules possessing energy equal to or in excess of threshold energy con-
stitute only a small fraction of the total number of molecules. This explains
why a small fraction of the total number of collisions result in chemical
reaction. The number of molecules possessing energy equal to or in excess of
threshold energy increases appreciably even with a small rise in temperature.
So, the rate of reaction increases considerably even with a small increase of
temperature.
We can thus conclude that most of he molecules have much less kinetic
energy than the threshold energy. The excess energy that the reactant mole-
cules having energy less than the threshold energy must acquire in order to
react and give the final products is known as activation energy. Therefore,
Activation energy = Threshold energy - Energy possessed by the mol-
ecules initially.
In other words, passive or non-active molecules, i.e., molecules pos-
sessing energy less than threshold energy, can be activated by absorption of
excess energy known as activation energy.
As already stated, the reactant molecules have to acquire a minimum
amount of energy before they can yield products on mutual collisions, i.e.,
there is an energy barrier placed between reactants and products (Fig. 3).
This barrier has to be crossed before the reactants can yield products. It
determines the magnitude of threshold energy which the reactant molecules
must acquire before they can react to give products.
234 PHYSICAL CHEMISTRY-I

.,., ... ",. ',','.,.,.,',' ....'.'.'. """"""'."""';"""")11

.....
-::::
Activation Activation
~{ energy energy of
.... >. reverse process

:I w
~.

!
E1 Reactants '
A ~-~-.------- - - t_ - - - -- t- --__
AE = Heat of reaction
~-------------L----------~----------~--B
Products }~

p~~g~~~.S reacti~n.
!li:·\::-:................................................................ .. Of =. .. . . . . . . . . .....

);l..;.
:.:::::::::::::::::::::::::::::::::::::::::::::::::::::\::::::::::;:::-:::::::.:.::: :/.:.: ::::. ~i~..3.. :::::::;::::::::::;:::::::::::::}}::::::::::;::::::::::::::::::::::::::::::::::::::\\:

Further, different reactions require dillerent amounts of activation en-


ergy. The concept of activation energy gives us an idea whether a given
reaction is slow or fast at a given temperature. A reaction which has lower
activation energy will proceed at a faster rate at a given temperature or
vice-versa. The differences ill activation el!el~f?Y are mainly responsible for
observed difference in the rates of reactio/l.
The concept of activation energy as applied to chemical reaction can
be explained by plotting energy against the progress of the reac-
tion (A ~ B) as shown in figure (3). The point A represents the initial
energy (E 1) of the reactants, AC represents the energy barrier which the
reactants have to cross and B represents the products having energy, E 2 . At
the beginning of rising portion of the curve AC, the molecules come very
close to each other, but they cannot react as they possess energy less than the
threshold energy. If, however, sufficient energy is given to them, they can go
up and reach the summit C of the energy barrier. The molecules are then said
to be in an activated state. In this state, the molecules are under conditions
of acute strain. The bonds between atoms of the reacting molecules become
very feeble. Now, the condition is such that the probability of formation of
new bonds between atoms of the molecules of the reactant is fairly strong.
This probability is shown by the curve CB moving down the barrier.
CHEMICAL KINETICS & CATALYSIS 235

In the above case, the reactants are shown to possess a total energy
higher than the products. So, according to the law of conservation of energy.
there will be a release of energy, i.e., heat will be evolved. The reaction is
exothermic and the amount of heat evolved gives the heat of reaction. This
release of energy is shown by llE. Evidently, llE = EI - E2, where EI is the
normal energy of the reactants and E2 that of the products. In this case, llE
is negative as energy is released and not absorbed. An exothermic reaction
has thus a lower activation energy or a faster rate of reaction. Now, consider
the reverse reaction (B ~ A). Since the energy (E2) of molecules of B is
less than the energy (E)) of A, as shown in the figure, energy will be absorbed
and not released, i.e., the reverse reaction will be endothermic. An endother-
mic reaction has always a greater activation energy and hence has a slow~r
rate of reaction than the opposing reaction.

Problem 14. Discuss the collision theory for llnimolecillar reactions.


(Meerut 2007, 2005)
It seems to be difficult as to how the collision theory could possibly be
used to explain the mechanism of unimolecular reactions. In unimolecular
processes only one molecule takes part in the reaction, then a question arises:
How do molecules in unimolecular reactions attain their energy of
activation? Lindemann (1922) suggested its answer by pointing out that the
behaviour of unimolecular reactions can be explained on the basis of bimo-
lecular collisions provided we postulate that a time lag exists between acti-
vation and reaction during which activated molecules may either react or be
deactivated to ordinary molecules. Thus, the rate of reaction will not be
proportional-to all the molecules activated, but only to those which remain
active. Lindemann suggested that the above reaction takes place as follows :
kj
First stage: A+A A*+A

* k3
Second stage: A ~ Products
The first stage involves collision of reacting molecules forming a few
activated molecules represented by A*, with a velocity cQnstant k b after which
time-lag occurs. During this time-lag the activated molecules lose their excess
energy and revert to the original state, with a velocity constant k 2•
Alternatively, the activated molecules may decompose into products,
with a velocity constant k 3•
2
Rate of activation =k l [AJ
Rate of deactivation = k2 [A*] [A]
236 PHYSICAL CHEMISTRY-I

Rate of decomposition = k3 [A *]
According to the stationary or steady state principle, whenever a
short-lived reaction intermediate occurs in a system, its rate of formation can
be taken as equal to its rate of disappearance. Applying this principle, we have
kl [A]2 = k2 [A'] [A] + k3 [A']
or = [A·] {k2 [A] + k3}
, kl [A]2
[A ] = k2 [A] + k3
Since the rate of reaction is proportional to the concentration of actI-
vated molecules, we can, therefore, write that,

Rate of reaction = - d ~~] = k3 [A']

d [A] klk3 [Af


or
-~= k2 [A] +k3
(i) At ~igh pressure: At sufficiently high pressure, the term k2 [A]
is far greater than k3 , which can thus be neglected. Equation (1) then reduces
to

_ d [A] = klk3 [A] = k'[A]


dt k2
Hence, the reaction is of the first order, as the rate of reaction is
proportional to the concentration of only one molecule of the reactant. Thus,
if concentration of A is high, the reaction should be of the first order.
(ii) At low pressure: At low pressure, k3 > > k2 [A], therefore,
k2 [A] can be neglected in comparison to k 3 • Therefore, equation (1) reduces
to

... (2)

Hence, the reaction is of the second order. Thus, if the concentration of


A is low, the reaction becomes of the second order.

Problem 15. Discuss the mathematical treatment of transition state the-


ory. Compare this theory with collision theory.
According to transition state theory, the rate of a reaction is the number
of activated complexes passing per second over the top of potential energy
barrier. This rate is equal to the concentration of activated complex times the
average velocity with which a complex moves across to the product side. The
activated complex is not in a state of stable equilibrium, since it lies at a
maximum potential energy.
CHEMICAL KINETICS & CATALYSIS 237

Postulates of Transition State Theory


(i) As the reacting molecules approach each other there is a contin-
uous series of changes in bond distances. These changes are
accompanied by energy changes.

Activated complex

--- 1- - - - - - - - - - --------- -I --~, = _ 0 0 eneraVl::::1

EI of reactants
E2
E2 = Activation ",",2rn'",':'"
of products

E
E' -------------------------
Products
Reaction path_

(ii) The reactant molecules are changed into an energy rich interme-
diate called activated complex or transition state (fig. 4).
(iii) The activated complex may be formed by some loose association
or bonding of reactant molecules with necessary rearrangement
of valence bonds and energy. If it is a unimolecular reaction, the
reactant molecule may produce the activated complex by rear-
rangement of atoms and redistribution of energy.
(iv) The activated complex, though unstable, has a transient existence.
It is treated formally as a definite molecule with an independent
entity. The activated complex remains in equilibrium with the
reactants and its potential energy is maximum. Finally, the acti-
vated complex decomposes into products.
(v) The activation energy of reaction in the light of this theory, is the
additional energy which the reacting molecules must acquire to
form the activated complex.
Thermodynamic or Mathematical Treatment of Transition
State Theory
Consider a bimolecular reaction between reactants A and B. According
to transition state theory,
238 PHYSICAL CHEMISTRY-I

A + B ~ x* ~ Products
Reactants Acti vated
complex
The equilibrium constant (K) for the formation of activated complex
is,
_ [x*]
K - [A] [B] or [X*] = K [A] [B] ... (1)

According to transition state theory, the rate of reaction is the number


of activated complexes which pass over the potential energy barrier per unit
time. This, in turn, is equal to the concentration of activated complex multi-
plied by the frequency at which the complex would decompose into products.
Mathematically,
~ = [X*] x Rate (or frequency) of dissociation of activated complex ... (2)
From equations (1) and (2), we get

~=K [A] [B] x Rate (or frequency) of dissociation of activated complex.

The activated complex would decompose only if enough vibrational


energy is supplied to the system, so that the atoms vibrate with certain critical
frequency, leading to bond breaking. Therefore,
Frequency of dissociation of activated complex = Evib1h ... (3)
where, Evib = average vibrational energy at temperature Tand h =Planck's
constant.
RT
But, EVib=kT=/i (.: k= RIN) ... (4)

From equations (3) and (4) frequency of dissociation of activated com-


plex =RTINh
~ = K* [AJ [B]. Z~ ... (5)

For conversion of reactants into products,


dx
-=k[AJ [BJ (k = rate constant) ... (6)
dt
From equations (5) and (6),
k [A] [B] = K [A] [B]. RTINh
k=K. RTINh ... (7)
Equation (7) is the mathematical statement of transition state theory.
According to thermodynamics, K can be correlated with ilG* through
the following relation,
ilG* = - RTln K
where ilG* = (Free energy of activated complex) - (Free energy of reactants).
ilG* is known as standard free energy change
CHEMICAL KINETICS & CATALYSIS 239

AG' == AH· - TAS'


- RTln K' ~ AF - TAS~
u* == - (AF - TAS'l
or Inl\. RT
' - (M!' - TtS')/RT
or K =- e ... (8)
From equations (7) and (8) we get.
k RT - (!lH' - TtS' )IRT
=- Nh . e

k == ~~. e-!lH'IRT. etS'IR •.• (9)

where, AFt = standard enthalpy change, i.e., standard heat of activation,


As" =standard entropy change, i.e., standard entropy of activation.
Equation (9) can be applied not only to bimolecular, but also to uni-
molecular and trimolecular processes. Morever, it can be applied to reactions
in solution also.
Comparison of Transition State Theory With Collision
Theory.
(i) From transition state theory, i.e., according to equation (9)
k == RT . e -W'IRT • etS'lR
Nh
From collision theory, k = PZe - EIRT
where P = probability factor or steric factor
Z = collision number or collision frequency. Comparing both equa-
tions, we have

or (:. E=AJt)

or

or
The steric factor P is thus related to entropy of activation.
(ii) In collision theory, no account is taken of the internal motions of
the reactant molecules, whereas in transition state theory, account is taken of
the internal degrees of freedom of reactant molecules and the changes these
undergo on reaction.
(iii) The concept of entropy of activation in transition state theory is
very useful for qualitative purposes. Thus. for bimolecular processes, this is
an advantage over the colliSIOn theory.
240 PHYSICAL CHEMISTRY-I

Important Formulae to Remember


1. Unit of reaction rate = mol L -I S-I
20 Units of rate constant
(i) Zero order reaction = c~nc = (conc) (timer l = mol C l S-I
time
(If concentration is expressed in mol L-I and time in second)

(,0,0) FO,rst ord o=l


er reaction -0- . )-1 = s-I
= ( time
tIme
000) S econ d
(III Ofd o
er reaction l
=- --2 0
1
- 0 -
0 )-1
= (conc )-2 ( time
(conc) tIme
= (mol L-lr2 (sri = mol-2 L 2s-1
30 Rate expressions
(i) Zero order reaction: k = :!.
t
00 k =-
(1 1) FOIrst order reaction:
0 - IoglO--
2.303
t
a
a-x
where, a = initial concentration of reactant, x = concentration of reac-
tant undergoing reaction in time to
(iii) Second order reaction
(i) k=lo x [Whena=b]
t a(a-x)
(ii) k = 20303 log b(a - x) [When a :;t: b]
tea - b) 10 a(b -x)
40 Order of reaction (n) can be calculated by half change method by the
following formula:
log t, - log t2
n = 1 + .,.---"--'---:-"-
log P2 -log PI
where,PI =pressure of the system at time tl' P2 = pressure of the system
at time t20
50 The activation energy (E) can be calculated from the following formula:
k2 E (1 1)
loglO kl = 2.303 R TI - T2

where, kl and k2 are the rate constants at absolute temperatures TI and


T20
60 For a reaction, ilIA + n2B ~ n3C + f!4D, we can equate the reaction
rates as :

ReactIOn rate =- -
o 1 d[A]
- d[B]
- =- -I - -
III dt 112 dt
CHEMICAL KINETICS & CATALYSIS 241

=+ ~ d [C] =+ ~ d [D]
n3 dt n4 dt

7. Rate expression for third order reaction


(i) k = J... . x(2a - x) [When a = b = c]
2t a 2(a - x)2

k=1.[(a-b) log~+ (b-c) IOg~+ (c -a) IOg¥Jl


(ii)
t (a - b)(b - c)(c - a)
[When a * b * c)
NUMERICAL PROBLEMS
Ex. 1: The decomposition of hydrogen peroxide was studied by titrating
it at different intervals of time with potassium permanganate. Calculate the
velocity constant for it from the following data, if the reaction is of the first
order.
t (sec) o 600 1200

KMn04 (ml) 22.8 13.8 8.2

. IS:
SoI. The fiIrst ord er rate expressIon . k 2.303
=- - Iog--
a
t a-x
The amount of KMn04 is proportional to the amount of H 20:! present,
so the volume ofKMn04 used at zero time corresponds to initial concentration
(a) and the volume used after time t, corresponds to (a - x) at that time.
Inserting these values in the above equation, we get

k 600 = 2.303 I 22.8 =0 000837 -1


600 og 13.8' s

k 1200 = 2.303 I 22.8


1200 og 8.2 =0000852-
. s
1

The average value of velocity constant,

k = 0.000837 + 0.000852 = 0.000844 sec- I


2
Ex. 2: The optical rotation of sucrose in presence of dil. HCI at various
intervals is given in the following table:
t (min.) 0 10 20 40 100 00

Rotation (degree) 32.4 28.8 25.5 19.6 6.7 -14.1

Show that reaction is of the first order.


Sol. The inversion of sucrose will be a first order reaction if the above
data conforms to the equation,
242 PHYSICAL CHEMISTRY-I

2.303 I a 2.303 I ro - r~
k = - - oglo--=-- o g - -
t a-x t rt-r~

where ro, rl and r~ are the optical rotations at the start of the reaction, after
time t and at the completion of reaction, respectively. Inserting the values in
the above equation, we get

k = 2.303 I 32.4 - (- 14.1) = 2.303 I 46.5 = 0008060 ·-1


10 10 og 28.8 _ (_ 14.1) 10 og 42.9' mm

k20 = 2.303 I 32.4 - (- 14.1) = 2.303 I 46.5 = 0 008037 ·-1


20 og 25.5 _ (_ 14.1) 20 og 39.6' mm

k = 2.303 I 32.4 - (-14.1) = 2.303 I 46.5 = 0 008054 ·-1


40 40 og 19.6 _ (_ 14.1) 40 og 33.7' mm

k = 2.303 I 32.4 - (- 14.1) = 2.303 I 46.5 = 0 008040 ·-1


100 100 og 6.7 _ (-14.1) 100 og 20.8' mm

The constancy in the value of k shows that the reaction is of the first
order.
Ex. 3: Decomposition of diazobenzene chloride was foUowed at constant
temperature by measuring the volume of nitrogen evolved at different times.
The data is given below:
t (min) o 20 so 70
o 1 10 2S 33 162

Calculate the specific reaction rate and order of reaction.


Sol. Diazobenzene chloride deomposes as :
CJIsN = NCI ~ CJIsCI + N2
Thus, the amount of N2 evolved will be a measure of diazobenzene
chloride decomposed; i.e., x The total N2 evolved at infinite time will thus
give the initial concentration i.e., a. Now, substituting the values in the first
order rate expression,
k - 2.303 10 _a_
- t glo a-x
2.303 I
k20 = -W 162 2.303 I 162 000322 ·-1
ogIO 162 _ 10 = -W ogIO 152 =. mm
2.303 1 162 2.303 I 162 000336
kso = ---so ogIO 162 _ 25 = ---so ogIO l37 =. mm
·-1

2.303 I
k 70 = -m 162 2.303 I 162 000326
ogiO 162 _ 33 = -m oglo 129 =.
·-1
mm
CHEMICAL KINETICS & CATALYSIS 243

The constancy in the values of k shows that the reaction is of the first
order. The specific rate constant, k is thus given by :

k = 0.00322 + 0.00336 + 0.00326 mm. -I 000328 .-1


=. min
3
Ex. 4: A first order reaction is 15% complete in 20 minutes. How long
will it take to be 60% complete?
Sol. For a first order reaction, we have
2.303 I a
k = - - oglO--
t a-x
2.303 I al 2.303 I a2
or - - oglO--=-- oglO-- ... (i)
tl al -XI t2 a2 -X2
15
Here, XI = 100 al = 0. 15a1 tl = 20
60
x2 =100 a2 =O. 6a l t2 =?
Inserting these values in equation (i), we get
2.303 I al 2.303 I a2
-W oglO al - 0. 15a l = ~ oglo a2 - 0.6a2

1 al 1 a2
or loglo -085 = - . IOglO -04
20 . al t2 . a2

or _ 20 log (1/0.4) _ 20 log (100/40)


t2 - x log (1/0.15) - x log (100/85)
t2 = 112.76 minutes.
Ex. 5: Show that the case of unimolecular reaction, the time required for
99.9% of the reaction to take place is ten times that required for half of the
reaction. (Meerut 2005)
Sol. For unimolecular reaction, the rate expression may be written as
k = 2.303 log _a_
fl 10 a-x

For half the reaction, let the time taken be fl, then X == 0.5a.
2.303 a
.. k == - - loglO 05
tl a- . a
0.693
or fl ==-k- ... (i)

Suppose the time taken for 99.9% of the reaction is t2' then
x=0.999a.
2.303 a
k == -t2- loglO a - 0999 ... (ii)
. a
244 PHYSICAL CHEMISTRY-I

or k = 2.~03 log 103 or t2 = 2.30: x 3 = 6.~09


Dividing (ii) by (i), we get
!1 = 6.909 + 6.693 = 10 or t2 = 10 t 1•
tl k k
Ex. 6: For a reaction A + B ~ C + D, the initial rates for different
initial concentrations of reactants have been found as follows:
IniJial conc. (mollir 1) Initial rate

[A] [B)
I 2.0 2.0 2.0 x 10-3
II 4.0 2.0 4.0 x 10-3
III 6.0 2.0 6.0 x 10-3
IV 2.0 4.0 2.0 x 10-3
V 2.0 6.0 2.0 x 10-3
VI 2.8 8.0 2.0 x 10-3

Give:
(a) Order of reaction with respect to A (Meerut 2007)
(b) Order of reaction with respect to B (Meerut 2007)
(c) Overall order of reaction (Meerut 2007)
(d) Rate law equation
(e) Rate constant
Sol. (a) According to sets I, II, III, we can infer that order of reaction
with respect to A {when [B] is kept constant} will be 1.
(b) According to sets IV, V, VI, we can infer that order of reaction
with respect to B {when [A] is kept constant} will be O.
(c) Overall order of reaction = 1 + 0 = 1.
(d) The rate law equation will be

dt = k [AJ
dx ... (.)
1

(e) From set I, we can substitute the values of dx/dt and [AJ in
equation (i). Therefore,
3
20 X 10-3 = k x 2.0 or
k = 2.0 X 10- = 10-3 s-1
2.0
Ex. 7: 1 ml of methyl acetate was added to 20 ml of N/20 HCI at 2SOC. 2
ml of the reaction mixture was withdrawn at different times and titrated
with a standard alkali.
Time (millutes)
ml qf alkali used

.-
CHEMICAL KINETICS & CATALYSIS 245

Show that the hydrolysis of methyl acetate is a pseudo u1limolecular


reaction.
Sol. The reaction will be of the first order or pseudo unimolecular
reaction if the data conforms to the first order rate expression
k = 2.303 log _a_ = 2.303 log V= - Vo
t a-x t V=-Vt
where Vo, Vt and V= are the volumes of alkali used at the start of the reaction,
after time t and at the end of the reaction, respectively. Thus,
k = 2.303 I 42.03 - 19.24 = 2.303 I 22.79 = 0 00327 ·-1
75 75 og 42.03 _ 24.20 75 og 17.83' mm
k = 2.303 I 42.03 - 19.24 = 2.303 I 22.79 = 0 00327 " - I
119 119 og 42.03 _ 26.60 119 og 15.43" mm
2.303 I 42.03 - 19.24 2.303 I 22.79 000319 " - I
k 183 = 183 og 42.03 _ 29.32 = 183 og 12.71 =. mm
As the values of k are nearly constant, the reaction is pseudo unimolecu-
lar.
Ex. 8: The rate of the reaction aA + bB ----7 cC + dD is given by the
foUowing expression :
-d [A] =k [A] [B]2
dt
where, k is the velocity constant. What is (;) order of reaction if A is present
in excess (ii) order of reaction if B is present in excess (iii) total order of
reaction (iv) molecularity of the reaction?
Sol. (i) Order of reaction when A i£ in excess = 2
(ii) Order of reaction when B is in excess = 1
(iii) Total order of reaction = 2 + 1 = 3
(iv) Molecularity of the reaction = a + b.
Ex. 9: The half life periods for the thermal decomposition of phosphine
at three different pressures are given below:
-
Initial pressure (mm) 707 79 37.5

Half life (sec) 84 84 84

Calculate the order of reaction. (Meerut 2000)


Sol. 1st Method: Since the half life periods are the same. irrespec-
tive of the initial concentrations or pressures, the reaction is of the first order.
2nd Method: We know that:
log (t1/t2)
n = 1 + -"'--'--'--=-
log (a2/al)
log (tl/t2)
or 11=1+ (":aocp)
log (P2/PI)
For first set:
246 PHYSICAL CHEMISTRY-I

_ log (84/84)
.. n - 1 + log (79/707) 1 + 0 or n = 1
For second set:
_ log (84/84) _ _ _
n - 1 + log (07.5/79) - 1 + 0 - lor n - 1
:. Order of reaction = 1.
Ex. 10 : The timefor half change (t)for a gaseous reaction was measured
for various initial pressures (p). The following data were obtained :

Calculate the order of reaction. (Meerut 2006)


Sol. 1st method: We know that
1
tll2 oc - or p. tll2 = constant
p
(i) p x tll2 = 200 x 150 = 30000 mm min
(ii) p x tll2 = 300 x 99·8 = 29940 mm min
(iii) p x tll2 = 400 x 75.3 = 29120 mm min
As the values of ptll2 are constant, the reaction is of the second order.
(.: tll2 oc lip)
2nd Method: We knew that:
log (t)/t2)
n = 1 + ---':::....:.-'--..:::....
log (P2/P)
For first set:
_ 1 log (150/99.8) _ 1 0.1769
•. II - + log (300/200) - + 0.1760
= 1 + 1.004 = 2.004 :::: 2
For second set:
_ 1 log (99.8/75.3) _ 1 0.1223
/1- + log (400/300) - + 0.1249
= 1 + 0.979 = 1.979:::: 2
So, the order of reaction is 2.
Ex. 11: Time for half change (t) of a gas undergoing thermal decompo-
sition for various initial pressures was found as :
PreHure (mm) 750 500 250

1¥me (mill utes) 105 235 950

Find the order of the reaction.


Sol. The order of reaction is given by
log (t)/t2)
II = 1 + _=-,--=--..:=.o...
log (P2/P)
CHEMICAL KINETICS & CATALYSIS 247

For first set:


_ 1 log (105/235) _ 1 0.3498
.. n - + log (500/750) - + 0.1760
= 1 + 1.98 =2.98 "" 3
For second set:
_ 1 log (235/950) _ 1 0.6066
n - + log (250/500) - + 0.3010·
= 1 + 2.01 =3.01"" 3
:. Order of reaction = 3.

Ex. 12: The following data were obtained for a gaseous reaction :
Initial pressure (mm) 200 300 400

Half period (min) 150 99.8 75.3

Calculate the order of reaction.


log (tJ/t2)
Sol. n = 1 + ---'''---'-''---'''-
log (P2/PJ)

=1 log (150/99.8) =1 0.1769 =1 1 =2


n + log (300/200) + 0.1761 +

Similarly. =1 log (150/75.3) = 1 0.2992 = 1 1= 2


n + log (400/200) + 0.3010 +
Order of reaction = 2.
Ex. 13: Dinitropentaoxide decomposes as follows :
- d [N20SJItit = kl [N20S]
d [N02J1tit = k2 [N20s1
d [02J1dt = kJ [N2 0 s1
What is the relation between k., k2 and k J ?
Sol. The decomposition of dinitropentaoxide is a first order reaction,
so

According to stoichiometry, the rate pf the above reaction can be


expressed by the following relations :
d [N 20 51 I d [N0 21_1 d [0 21
dt 2 fit 1 dt
2
. 1
or kJ [N 20 51= 2" k2 [N 20 51= 2k3 [N 20 5]
248 PHYSICAL CHEMISTRY-I

k2
or kl =2= 2k3 ,.. (i)

or 2kl = k2 = 4k3 ... (ii)


So, relation between k 1, k2 and k3 can be expressed by either equation
(i) or (ii).
Ex. 14: The first order reaction has k = J.5 X 10-6 per second at 200C. If
the reaction is allowed to run for 10 hours, what percentage of the initial
concentration would have changed into product?
Sol. For a first order reaction,
k - 2.303 10 _a_
- t glOa-x
-6 2.303 100
or 1.5 x 10 = 10 x 60 x 60 log (l00 -x)

100 1.5 X 10-6 x 10 x 60 x 60 = 0 023447


log (100 - x) 2.303 .
100
or 100 - x = 1.055
or 100 = 105.5 - 1.055x

or 5.5 5 3
x = 1.055 = .21
:. 5.213% of the initial concentration had changed into product.
Ex. 15: An acid solution of sucrose was hydrolysed to the extent of 57%
after 66 min. Assuming the reaction to be of first order, calculate the time
taken for 75% hydrolysis.

k = 2.303 10gIO _a_


t a-x
For 57% hydrolysis, x = 0.57 a,
2.303 a 2.303 100
.. k = ~ 10gIO a _ 0.57a = ~ 10gIO 43
For 75% hydrolysis, x = 0.75a,
k = 2.303 10 a = 2.303 10 0 100
t glO a - 0.75a t ",10 25

2.303 I 100 _ 2.303 I 100


66 oglO 43 - t og 25
100
log 25 0.6020
or t = --100 x 66 = 0.3665 x 66
log--
43
CHEMICAL KINETICS & CATALYSIS 249

or t = 108.4 min.
Ex. 16: For a first order reaction, the rate constant is found to be 7.0
x 10-7 at 'PC and 9 x 10-4 at 5'PC. Calculate the energy of activation and
its specific reaction rate at 12'PC.
, k2 E
Sol. ,We know that, log k; = 2.303 R T\ - T2
[1 1]
where, E=activation emvgy, T\=273+7=280K,k l =7xl0-7,
4
T2 = 273 + 57 = 330 K, k2 = 9 X 10- .
So, from equation (8), we get
7
log (9 x 10-4) - log (7 x 10- ) = 2.303 ! 1.987 [ 2!0 - 3~0 ]
E [330-280]
= 2.303 x 1.987 280 x 330
-4 -7 Ex50
or log (9 x 10 ) -log (7 x 10 ) = 2.303 x 1.987 x 280 x 330
- - Ex50
or 4.9542 -7.8451 = 2.303 x 1.987 x 280 x 330

x 330 x 3.1091 -~ 26•29 x 103 ca II moIe.


- 2.303 x 1.98750
or E -

Spccific reaction rate at 127°C, i.e., 400 K is given by

26290 [1 1]
log k400 - log k280 = 2.303 x 1.987 280 - 400

26290 [400 - 280]


2.303 x 1.987 400 x 280
7 26290 x 120
or log k400 - log (7 x 10-) 2.303 x 1.987 x 400 x 280 = 6.153.
or log k400 = log (7 x 10-7) + 6.153
=-7.8451 x 6.153 =- 1.6921
or k400 = 0.02032 min-to
Ex. 17: Bodenstein by studying the kinetics ofdecomposition ofa gaseous
hydrogen iodide gave the values of specific reaction rates to be
3.517 x 19-7 and 3.954 x 10-2 at 556 K and 781 K. Cakulate the energy of
activatfo'n of the reaction and frequency factor.
k2 E [ 1 1]
(a) We know that log k\ = 2.303 R T\ - T2
2
10 3.954 X 10- _ E [_1___1_]
g 3.517 x 10-7 - 2.303 x 1.987 556 781
250 PHYSICAL CHEMISTRY-I

E [781 - 556J
=2.303 x 1.987 557 x 781
3.954 x 10-2
log 7X 2.303 X 1.987 X 556 x 781
E= 3.517 x 10-
225
= 44610 cal/mole.
where, A = frequency factor.
Hence, 3.517 x 10-7 = A.e-44610/1.987 x 556
-7 44610
or log 3.517 x 10 = log A - 1.987 x 556
or - 6.4538 = log A - 40.2910 or log A = 33.8372
or A = 6.874 X 1034•
Ex. 18: In the saponification of ethyl acetate by sodium hydroxide using
equal concentrations the progress of the reaction was foUowed by titrating
25 ml of the reaction mixture at regular intervals against standard acid.
The foUowing data were obtained:
Time (min) 0 15 25 35
Volume of acid used (ml) 16 6.13 4.32 3.41

Show that the reaction is of the second order.


Sol. We have, a oc Vo and (a - x) oc VI' Therefore, x oc (Vo - VI)' For a
second order reaction, when reactants are of equal concentrations,
k =! . x =!. (Vo - VI)
t a (a - x) t Vo . VI
Substituting the different values, we get
1 (16 - 6.13)
k 15=15' 16x6.13 =~ .
00067 . -I -1
mIn cons

1 (16 - 4.32)
k25 = 25' 00069 . -I -I
16 x 4.32 =. mm cons

k - 1 (16 - 3.41) 0.0068 min- 1 cons-I


35 - 35' 16 x 3.41
The constancy in the values of k shows that the reaction is of the second
order.
Ex. 19: Decomposition of a gas is of second order when the initial
concentration of the gas is 5 x 10-4 mole per litre. It is 40% decomposed in
50 minute~. What is the value of velocity constant?
Sol. For a second order reaction,

k=!. x
t a (a - x)
CHEMICAL KINETICS & CATALYSIS 251

Here, a =0.00005; x =0.0005 x (40/100) =0.0002


1 0.0002 26 661't I -1 ·-1
. . k = 50 x 0.0005 (0.0005 _ 0.0002) = . I mo e mID .

Ex. 20: A second order reaction where a = b is 20% completed in 500


seconds. How long wiU it take for the reaction to go to 60% completion?
Sol. For a second order reaction, where a = b,

..
k=.!.. x
t a (a - x)
When the reaction is 20% complete, x = 0.2a
k=_l_. 0.2a 1. 0.2a _ __
.. 500 a (a - 0.2 a) 500 a X 0.8a 2000 a
Suppose it takes t\ seonds for the reaction to go to 60% completion.
N k =1. . 0.6a _ 1 0.6a =1. . l
ow, tl a (a - 0.6a) t 2 · a X OAa tJ 2a
1 .-
- 3 =--
or
tl 2a 2000a
t) = 3000 sec.
Ex.4: A second order reaction with two reactants is started with O.IM
concentrations of each reactant. It is 20% completed in 500 seco~ds. How
long will it take the reaction to go to 70% completion?
For a second order reaction when the concentrations are equal, we have
k=.!.. x
t a (a - x)
For 20% completion: a =0.1, x= 0.2 X 0.1 =0.02; t =500
k = _1_ X 0.02 =_1_ X 0.02 =_I_
.. 500 0.1 (0.1 - 0.02) 500 0.1 x 0.01 200
For 60% completion: a = 1; x =0.6 X 0.1 = 0.06; t = ?
k=! X 0.:.::.06=--_
.. t 0.1 (0.1 - 0.06)
1 1 0.06
or -=-x
200 t 0.1 - 0.04
200 x 0.06
or t =Q.l X 0.04 =3000 seconds.

2. CATALYSIS
Problem 1. Define catalyst and catalysis. Mention the types and classifi-
cation of catalysis. Discuss the characteristics of catalytic reactions.
[I] Catalyst and CatalysiS
Berzelius (1835) found that the speed of a number of reactions is in-
creased due to the presence of a small quantity of a foreign substance. He also
252 PHYSICAL CHEMISTRY-I

found that these substances remain chemically unchanged at the end of the
reaction. He termed these substances as catalysts and the phenomenon itself is
known as catalysis. A familiar example is that of the decomposition of
KCI03 . The decomposition of KCl0 3 is very slow even at high temperature,
but a small quantity ofMn02 increases the rate of decomposition to a very great
extent and Mn02 remains chemically unchanged at the end of the reaction.
2KCl03 + [Mn02] ~ 2KCl + 302 + [Mn02]
But later on it was observed that there are certain substances which can
retard the rate of a chemical reaction. Hence. Ostwald defined that.
"A catalyst is a substance which influences the speed of a 'chemical
reaction without itself undergoing any chemical change at the end of the
reaction. "
Catalysis is mainly divided into two types. viz .• homogeneous catalysis
and heterogeneous catalysis.
[A] Homogeneous Catalysis
When the catalyst is present in the same phase as that ofthe reactants,
the phenomenon is known as homogeneous catalysis.
(a) Examples of homogeneous catalysis in gas phase
(i) Oxidation of sulphur dioxide (S02) to sulphur trioxide (S03) with
nitric oxide (NO) as catalyst.
2S02 + O2 + [NO] ~ 2S03 + [NO]
Gas Gas Gas Gas

(ii) Decomposition of acetaldehyde (CH 3CHO) with iodine (1 2) as cat-


alyst.

Vapour Vapour Gas Gas


(b) Examples of homogeneous catalysis in solutiop-pbase
Many reactions in solutions are catalysed by acids (J-r) and bases
(OH-).
(i) Decomposition of hydrogen peroxide (H20 2) in the presence of
iodide ion (r) as catalyst,

I
2H20 2 ~ 2H20 +02
(ii) Hydrolysis of cane sugar in aqueous solution in the presence of
mineral acid as catalyst.

Cane sugar Glucose Fructose


(iii) Hydrolysis of an ester in the presence of acid or alkali.
+ -
H/OH
CH3COOC 2Hs + H20 --~ CH3COOH + C2H50H
Ethyl acetate Acetic acid Ethanol
CHEMICAL KINETICS & CATALYSIS 253

[8] Heterogeneous Catalysis


When the catalyst is in a different phase than that of reactants, the
phenomenon is known as heterogeneous catalysis.
Some examples of heterogeneous catalysis with reactants in the gas,
liquid or the solid phase are given below.
(a) Heterogeneous catalysis with gaseous reactants (contact
catalysis)
(i) Oxidation of ammonia to nitric oxide (NO) in the presence of a
platinum gauze (a stage in the manufacture of nitric acid),
4NH3 + 502 + [Pt] ~ 4NO + 6H20 + [Pt]
Gas Gas Solid
(ii) Combination of sulphur dioxide (S02) and oxygen in the presence
of finely divided platinum or vanadium pentoxide, V 20 5, (contact process for
sulphuric acid).
2S02 + O2 + [Pt] ~ 2S03 + [Pt]
Gas Gas SolId

(iii) Hydrogenation reactions of unsaturated organic compounds are


catalysed by finely divided nickel.
H2C=CH2+ H2 + [Ni] ~ H3C-CH3 + [Ni]
Ethene (gas) Gas Solid Ethane
Vegetable oils are tri-esters of glycerol with higher unsaturated acid
(oleic acid). When hydrogen is passed through the vegetable oils in presence
of nickel, the carbon-carbon double bonds of the acid portions are hydrogen-
ated to yield solid fats (vanaspati ghee).
(iv) Combination of nitrogen and hydrogen to form ammonia in the
presence of finely divided iron, (Haber's process for ammonia).
N2 + 3H2 + [Fe] ~ 2NH3 + [Fe]
Gas Gas Solid
(b) Heterogeneous catalysis with liquid reactants
(i) The decomposition of aqueous solutions of hydrogen peroxide
(H20 2) is catalysed by manganese dioxide (Mn02) or platinum in colloidal
form.
2H20 2 + [Pt] ~ 2H20 + O2 + [Pt]
Liquid Solid
(ii) Benzene and ethanoyl chloride (CH3COCI) react in the presence
of anhydrous aluminium chloride to form phenyl methyl ketone
(CJI5 COCH3)·
C614 + CH3COCI + [AICI3] ~ C6H5COCH3 + HCI + [AICI3]
Liquid Liquid Solid
(c) Heterogeneous catalysis with solid reactants
The decomposition of potassium chlorate (KCI0 3) is catalysed by man-
ganese dioxide (Mn02).
254 PHYSICAL CHEMISTRY-I

2KCI03 + [Mn02J ----) 2KCI + 302 + [Mn02J


Solid Solid
The above reaction is heterogeneous. though both reactants are in the
same phase (solid). because the solid forms a new phase.
[II] Classification of Catalysis
Catalytic reactions are of the following four types:
(a) Positive catalysis
(b) Negative catalysis
(c) Auto-catalysis
(d) Induced catalysis
(a) Positive Catalysis: When the catalyst used accelerates the speed
ofa chemical reaction, it is known as a positive catalyst and the phenomenon
is known as positive catalysis. For example. the rate of decomposition of
hydrogen peroxide increases in the presence of colloidal platinum as catalyst.

Catalyst
Other reactions are :
Mn°2
(ii) 2KCI03 --~) 2KCI + 302

PtorNO
(iv) 2S02 + O2 ) 2S03
(b) Negative CMaJysis: When the foreign substance retards the
speed of a chemical reaction, it is known as a negative catalyst and the
phenomenon is known as negative catalysis. The following are examples of
this type.
(i) Decomposition of hydrogen peroxide
H,P04
2H20 2 ~ 2H20 + O 2
(ii) Oxidation of chloroform
C,H,OH
4CHCl3 + 302 ) 4COCI 2 + 2Cl2 + 2H20
-
(iii) Tetraethyllead as antiknock
When tetraethyllead. Pb(C2H5)4 is added to petrol. it retards the too
rapid or explosive combustion of the fuel which is responsible for the working
of the engine.
Explanation of Negative Catalysis
The mechanism of negative catalysis could be different for different
reactions, e.g .•
(1) By poisoning a catalyst. A negative catalyst may work by poi-
soning a catalyst which already happens to be present in the reaction mixture.
CHEMICAL KINETICS & CATALYSIS 255

For example, the traces of alkali dissolved from the glass of the container,
catalyse the decomposition of hydrogen peroxide (H20 2). However, the ad-
dition of an acid would destroy the alkali catalyst and thus prevents decom-
position.
(2) By breaking a chain reaction. In some cases, negative catalysts
are believed to operate by breaking the chain of reactions. For example, the
combination of H2 and C1 2, which is a chain reaction, is negatively catalysed
by nitrogen trichloride (NCI 3).
Cl2 ~ Cl'+Cl'
Free radicals

H' + Cl2 ~ HCl + Cl'


.NCI3 breaks the chain of reactions by absorbing the propagating species
(CI') and the reaction stops.
I
NCl3 + CI· ~ "2N2 + 2Cl2

(c) Auto-Catalysis: When one of the products formed in the reac-


tion itself acts as a catalyst, the
substance is known as an auto-
catalyst and the phenomenon is
known as auto-catalysis.
tI ---------------------
Completion of reaction

In auto-catalysis the initial c


.Q
rate of the reaction rises as the
catalytic product is formed, in-
stead of decreasing steadily (See
i
Sigmoid curve
fig. 5). The curve plotted between
reaction rate and time shows a
maximum when the reaction is
complete.
Time---+
(i) For example, hydrolysis
of ethyl acetate by water is an Fig. S : Curve showing the rise
auto-catalytic reaction, since ace- of rate of reaction with time.
tic acid liberated in this reaction
acts as a catalyst.
CH3COOC2Hs + H20 ~ CH3COOH + C2HsOH
Auto-catalyst

(ii) The oxidation of oxalic acid by acidic KMn04 is catalysed by the


presence of Mn2+ ions formed in the solution. In the beginning, the colour of
KMn04 disappears slowly, but as Mn2+ is formed in the solution, the colour
discharges rapidly. So, Mn2+ ions acts as auto-catalyst.
256 PHYSICAL CHEMISTRY-I

2Mn04" + 5C20~- + 16W ~ 2Mn2+ + lOC0 2 + 8H20


Violet Colourless

(iii) The free arsenic produced by the decomposition of arsine (AsH3)


auto-catalyses the reaction.
2AsH3 ~ 2As + 3H2
Catalyst
(d) Induced Catalysis: When one reaction influences the speed of
other, which is not possible under ordinary conditions, the phenomenon is
known as induced catalysis.
For example, sodium sulphite solution is readily oxidised in air but
sodium arsenite solution is not oxidised by passing a current of air through
it. However, if air is passed through a mixture of sodium sulphite and sodium
arsenite solution, the oxidation of both take place. Here the oxidation of
sodium sulphite acts as a catalyst for the oxidation of sodium arsenite solution.
[III] Characteristics of Catalytic Reactions
Although there are different types of catalytic reactions, the following
features or characteristics are common to most of them. These features are
often referred to as the criteria of catalysis.
(1) A catalyst remains unchanged in mass and chemical composition
at the end of the reaction.
Qualitative and quantitative analysis show that a catalyst undergoes no
change in mass or chemical composition. However, it may undergo a physical
change. Thus, granular manganese dioxide (Mn02) used as a catalyst in the
thermal decomposition of potassium chlorate is left as a fine powder at the
end of the reaction.
(2) A small quantity of catalyst is generally needed to produce
almost unlimited reaction.
Sometimes, a trace of a metal catalyst is required to affect very large
amounts ~f reactants. For example, one ten-millionth of its mass of finely
divided platinum is, however, needed to catalyse the decomposition of hy-
drogen peroxide.
On the other hand, there are catalysts which need to be present in
relatively large amount to be effective. Thus, in Friedel-Crafes reaction,
Anhy. Alel3
C6H6 + C 2HsCi ) C6H5CzH5 + HCI
Anhydrous aluminium chloride functions as a catalyst effectively when
present to the extent of 30 percent of the mass of benzene.
(3) A catalyst cannot, in general, initiate a reaction.
In most cases, a catalyst speeds up a reaction already in progress and
does not initiate (or start) the reaction. But there are certain reactions where
the reactants do not combine for very long period (perhaps years). For exam-
ple, a mixture of hydrogen and oxygen, which remains unchanged almost
CHEMICAL KINETICS & CATALYSIS 257

indefinitely at room temperature, can be brought to reaction by the catalyst


platinum black in a few seconds.
Room temp
) No reaction
Ptblack
2H2 + O2 ) 2H20
It is thus now considered that the catalyst can initiate a reaction.
According to this view, the reacting molecules (in the absence of catalyst) do
not posses minimum kinetic energy for successful collisions. The molecules
rebound from collisions without reacting at all.
(4) A catalyst is more effective when finely divided.
_ In heterogeneous catalysis, the solid catalyst is more effective when in
a state of fine sub-division than when used in bulk. So, a lump of platinum
will have much less catalytic activity than colloidal or platinised asbestos.
Finely divided nickel is a better catalyst than lumps of solid nickel, because
former occupies greater surface area than the latter.
(5) A catalyst is specific in its action.
While a particular catalyst works for one reaction, it will not necessarily
work for another reaction. Different catalysts, moreover, can bring about
completely different reactions for the same substance. For example. formic
acid gives carbon dioxide and hydrogen when passed over hot copper.
eu
HCOOH ~ CO 2 + H 2 ,
However, with hot aluminium oxide, formic acid gives carbon monox-
ide and water.
Al,O.
HCOOH ~ CO + H2 0
(6) Change of temperature changes the rate of a catalytic reaction
as it would do for the same reaction without a catalyst.
We have already studied the effect of temperature change on reversible
reactions under Le-Chatelier's principle. Some catalysts are, however, phys-
ically altered by a rise in temperature and hence their catalytic activity may
be decreased. This is particularly true with colloidal solutions like that of
platinum, since a rise in temperature may cause their coagulation. In such a
case, the rate of reaction increases up to a certain point and then gradually
decreases. The rate of reaction is maximum at a particular temperature
which is known as the optimum temperature.
(7) A catalyst does not alter the final position of equilibrium, al-
though it decreases the time required to establish it
It means that in a reversible reaction the catalyst accelerates the forward
and the reverse reactions equally. Thus, the ratio of the rates of two opposing
reactions, i.e., the equilibrium constant, remains unchanged.
The effect of a catalyst on the time required for equilibrium to be
established for the reaction
258 PHYSICAL CHEMISTRY-I

A+B ~ C+D
is shown in fig. (6). In the begin-
ning, the concentrations of A and
B are maximum and so the rate of
forward reaction is maximum. As
the time passes, the rate of the re-
action decreases till the equilib-
rium is established. For the reverse Equilibrium
reaction, the initial concentrations
of C and D are zero and the rate of
reaction is lowest. As the time
passes, the rate of reaction in-
creases till the equilibrium is estab- Time--.
lished. Similar curves of the rates
of reactions with the catalyst show
that the rates of the forward reac- :::::: time required for ih:e equilibrium ::::::
tion and the reverse reaction are )t. . . . '.' . . . . . . .~. ~. ~~~~.~~~~:......................... mm
changed equally but the equilib-
rium is established in a much shorter time.
For example, in the Haber's process for ammonia,
Fe
N2 + 3H2
2NH3 ~
the reaction is very slow. In the presence of the catalyst, the equilibrium is
reached much earlier but the percentage yield remains unchanged. The iron
catalyst decreases the time to attain equilibrium but cannot change the per-
centage yield.
Energy considerations also show that the final state of equilibrium
cannot be changed by the catalyst. Suppose the catalyst accelerates the for-
ward reaction more than the reverse reaction. This will shift the equilibrium
point, which cannot happen without the supply of energy to the system. But
a catalyst unchanged in mass and~composition at the end of the reaction,
cannot supply the required energy.
Problem 2. Write notes on the following:
(a) Catalytic promoters (b) Catalytic poisons
(A) Catalytic Promoters
The activity of a catalyst can often be increased by the addition of a
small quantity of a second material. This second substance is either not a
catalyst itself for the reaction or it may be a feeble catalyst.
A substance which, though itself not a catalyst, promotes the activity
of a catalyst is called a promoter or an activator.
[I] Examples of Promoters
(i) In some reactions, mixtures of catalysts are used to obtain the
maximum catalytic efficiency. For example, in the synthesis of methanol
CHEMICAL KINETICS & CATALYSIS 259

(CH 30H) from carbon monoxide and hydrogen, a mixture of zinc oxide and
chromium oxide is used as a catalyst.

(ii) Molybdenum (Mo) or aluminium oxide (Ali-03) promotes the ac-


tivity of iron catalyst in the Haber's synthesis for the manufacture of ammonia.
Fe
N2 + 3H2 ~ 2NH3
+Mo

[II] Explanation of Promoter's Action


The theory of promotion of a catalyst is not clearly understood. It may
be due to:

Peak

&
-~i-~i-~i-~i-~i- \~
1'% I I
-Ni-Ni-Ni
I I I I I I I I
-Ni-Ni-Ni-Ni-Ni- -Ni-Ni-Ni .;,:
I I I I I I I I ~
-Ni-Ni-Ni-Ni-Ni- -Ni-Ni-Ni U
I I I I I I I I I
-Ni-Ni-Ni-Ni-Ni- -Ni-Ni-Ni-Ni-Ni-
I I I I I I I I I I

(1) Increase of peaks and cracks. The presence of the promoter


increases the peaks and cracks on the catalyst surface. This increases the
concentration of the reactant molecules and hence the rate of reaction.
(2) Change of lattice spacing. The lattice spacing of the catalyst is
changed thus enhancing the spaces between the catalyst particles. The ad-
sorbed molecules of the reactant (say H 2) are further weakened and cleaved.
This makes the reaction go faster. The phenomenon of promotion is a common
feature of heterogeneous catalysis.
(8) Catalytic Poisons
Very often a heterogeneous catalyst is rendered ineffective by the pres-
ence of small amounts of impurities in the reactants.
A substance which destroys the activity of the catalyst to accelerate
a reQction, is called a catalytic poison and the process is called catalytic
poisoning.
260 PHYSICAL CHEMISTRY-I

IF''''''''·'''''''''''·''''''·'·'''·''·'·'·'·'·'''·'·'·'·'·''·"'·'·'·'·'·"·'·'·"~;?;S,:"·""""""·'·'·"·"'·'·""""·''''11

~ Distance between
catalyst particles ~
/f I"

:.:.. .:.:.:.:.:.:.:.:. .:.:.:.:.:.:. :.-.:.:.:.:.:.:.:.: . catalyst makes the reaction go faster? ':::'::-:::::": ;.:.: :::::
::::::::::::::-::::::::::::::.........................................................................................................................................................................::}}»}:.:;.:
(a) Examples of Catalytic Poisoning
(1) The platinum catalyst used in the oxidation of hydrogen is poisoned
by carbon monoxide.
Pt
2H2 + O2 ----+) 2H20
Poisoned
by CO
(2) The platinum catalyst used in the oxidation of sulphur dioxide
(contact process) is poisoned by arsenic oxide (AsP3)'
Pt
2S0 2 + O2 ----+) 2S03
Poisoned
by AS20 3

(3) The iron catalyst used in the synthesis of ammonia (Haber's process)
is poisoned by H 2S.
Fe
N2 + 3H2 ----+) 2NH3

(b) Explanation of Catalytic Poisoning


(1) The poison is adsorbed on the catalyst surface in preference to the
reactants: Even a monomolecular layer makes the surface unavailable for
further adsorption of the reactants. The poisoning of iron catalyst by H2S
comes in this class.
(2) The catalyst may combine chemically with the impurity: The
poisoning of iron catalyst by H2S comes in this class.
CHEMICAL KINETICS & CATALYSIS 261

o o o
II
c,
II
c
II
,
, , , , ,c,
, , ,, ,, , , ,,

~~;i;l Fig.9: Poisoning of platinum catalyst ~~~


by carbon monoxide.

Fe + H2S ~ FeS+H2
Problem 3. Discuss the theories of catalysis and also mention the indus-
trial applications of catalysts.
[I] Theories of Catalysis
Many theories have been put forward to explain the catalytic activity
of catalyst. A few important theories are given below :
1. Intermediate Compound Formation Theory
According to this theory, a catalyst first combines with one of the
reactants to form an intermediate compound of activity greater than that of
the reactants. This intermediate compound then reacts with another reactant
fufurm the product and so gives back the catalyst. If A and B are two reactants
and C is a catalyst, then according to this theory. .
A+C~ AC
AC+B ~ AB+C
A+B+[C] ~ AB+[C]
This theory can be fully explained by the following examples :
(1) In the oxidation of S02 by air, NO which acts as a catalyst, first
combines with oxygen to form N0 2 (intermediate compound) which oxidises
S02 and gives back nitric oxide.
2NO + O2 ~ 2N0 2
[S02 + N0 2 ~ S03 + NO] x 2
2S0 2 + O2 + [2NO] ~ 2S03 + [2NO]
(2) In the formation of ether from alcohol, H2S04 which is used as
catalyst first forms an intermediate compound C2H sHS0 4 .
C2HsOH + H2S04 ~ C2HsHS0 4 + HP
C 2HsHS04 + C2HsOH ~ C2HsOC2HS + H2S04
262 PHYSICAL CHEMISTRY-I

(3) Fonnation of methyl benzene, C6H5CH3 by reaction between ben-


zene (C6H6) and methyl chloride (CH3CI) using anhydrous aluminium chlor-
ide. AICI 3, as catalyst (Friedel-Craft's reaction).
AICI)
C6H6 + CH3CI ~ C6H5CH3 + HCI
Mechanism:
CH3CI + AICI 3 ~ [CH3 [AICI4 f t
Intermediate
compound
C6~ + [CH3t [AICI 4f ~ C6HSCH3 + AICI 3 + HCI
(4) Thermal decomposition of potassium chlorate (KCI0 3) in the pres-
ence of manganese dioxide (Mn02)'
MnO,
2KCI03 ~ 2KCI + 302
Mechanism:
2KCI0 3 + 6MnOz ~ 6Mn03 + 2KCI
Intermediate
compound
6Mn03 ~ 6MnOz + 30 2
It may be noted that the actual isolation of intermediate compounds
which would prove their existence is very difficult. As already stated, by their
very nature they are upstable. In general, the intermediate compounds sug-
gested as being formed are usually possible rather than proved.
2. Adsorption Theory or Modem Theory of Heterogeneous Catalysis
Adsorption theory explains the mechanism of a reaction between two
gases catalysed by a solid (heterogeneous or contact catalysis). According to
this theory, the catalyst acts by adsorption of the reacting molecules on its
surface. Generally speaking, four steps can be put forward for heterogeneous
catalysis. For example, for the following reaction,
Catalyst
A(g) + B(g) ~ C(g) + D(g)
the steps are as follows :
Step 1. Adsorption of reactant molecules. The reactant molecules
A and B strike the catalyst surface. They are held up at the surface by weak
vander Waals forces (physical adsorption) or by partial chemical bonds (clle-
misorptioll).
Step 2. Formation of activated complex. The particles of the re-
actants adjacent to one another JOIn to form an intermediate complex
(A - B). The activated complex is unstable. It has only a fleeting existence.
Step 3. Decomposition of activated complex. 'The activated com-
plex breaks to form the products C and D. The separated particles of the
products are held to the catalyst surface by partial chemical bonds.
CHEMICAL KINETICS & CATALYSIS 263

Step 4. Desorption of products. The particles of the products are


desorbed or released from the surface. They are stable and can lead to an

r··'·······'·N... . . ,. . . . ". . . . . . . . . . . . . '. .


independent existence.
;;;~; . . . . . . . . . ..
1 ''''~;;;''''''''''''''''''".S2,,''
Catalyst
2 """"""""~"''''''''''''''
Catalyst
."

:~; 3 """""''' """ """"''''' 4 """'''''''''''''''''''''''''''''''''''''''''',.

!!;,t;i;;;i·;;~;:;?~~;.·~~~~~~?~~;:::~t··g::.····.·
The mechanism of contact catalysis may vary in details, depending on
the nature of the reactants. Consider the example of hydrogenation of ethene
in presence of nickel. In this casse, ethene adds hydrogen in the presence of
nickel as a catalyst to yield ethane.
H H
I I
_N_i.....:(c_a_ta....;ly'-st-'-)~) H-C--C-H
I I
H H
Ethene gas Ethane (gas)

The catalyst functions according to the following steps.


Step 1. Adsorption of hydrogen molecules. Hydrogen molecules
are adsorbed on the nickel surface due to the residual valence bonds of the
nickel atoms.
Step 2. H-H bonds are broken. The H-H bond is smaller (0.74
A) than Ni-Ni bond. Therefore, the H-H bond of the adsorbed hydrogen
molecule is stretched and weakened. The weakened bond breaks, separating
the hydrogen atoms. The separated hydrogen atoms are held to the nickel
surface by chemical bonds.
Step 3. Formation of the activated complex. The chemisorbed
hydrogen atoms then attach to ethene molecule by partial chemical bonds.
Therefore, unstable activated complex is formed.
Step 4. Decomposition of the activated complex and desorption of
ethane molecule. The unstable activated complex decomposes to form
ethane molecule. The freed catalyst surface is again available for further
action.
264 PHYSICAL CHEMISTRY-I

il'::"""'''''''''~'''''''''''''''''''''''''''''''''''''''''M""""""""'·":'~

:1 H,,-- /H H,,-- 2;'--H I


I
Step 1 Step

j H/r:I"--H HH/C--C"--HH
)~ @--@ I
:'.! Step 3 Step 4 \

:::.:\::\U\ '~i~: .~~ ;.~~~~~ ~~.~~~ ~;~~~~~~~~~~. ~~. j/:::;:i{::::U/!::::·:.


\u:=::nt} .......... ...~~~.e~e..o.~. ~i.~~.~~.~~r:r~c~'....................I)/\/(I(U::
The adsorption theory explains the catalytic activity as follows:
(1) Metals in a state of fine sub-division or colloidal form are rich in
free valence bonds and hence they are more efficient catalysts than
the metal in lumps.
(2) A promoter increases the valence bonds on the catalyst surface by
changing the crystal lattice and thereby increasing the active cen-
tres.
(3) Catalytic poisoning occurs because the so-called poison blocks the
free valence bonds on its surface by preferential adsorption or by
chemical combination.
[II] Industrial Applications of Catalysts
Tht" presence of a catalyst is very useful in many industrially important
reactions, which are either very slow or take place at a very high temperature.
:.', .......:.<::.. ,,-:.:.:::- :-:-:: ',':::.: '';'' ',' ... ,',::-::-:,:::::::::,:':':-:::':::':,:',.;::.:::,:,:;:;::::';::'';;';''':-:-':;:::';''::::''\:{
:;:>'
."" .
CHEMICAL KINETICS & CATALYSIS 265

Hence. to decrease the cost of production it is essential to make use of a


suitable catalyst. A few important examples of heterogeneous catalytic reac-
tion of industrial applications are given as follows:

Reaction Catalyst and other favourable


conditions
1. Haber's process for the manufacture of Finely divided Fe + molybdenum (as
ammonia promoter) 200 atm. pressure and
N2 + 3H2 ~ 2NH3 temperature.400°-450°C.
2. The manufacture of chlorine by Deacon's Cupric chloride + excess of air at a
process temperature of 500°C.
4HCI + 02 .~ 2H20 + 2Cl2
3. Ostwald's process for the manufacture of Platinised asbestos + excess of air (as
HN0 3 promoter) and temp. 300°C.
4NH3 + 502 ~ 4NO + 6H20

2NO + 02 ~ 2N02
4N02 + 2H20 + 02 ~ 4HN03
4. Manufacture of hydrogen by Bosch's Ferric oxide + Cr203 (as a promoter) at a
process temp. of 400°-600°C.
(CO + Hz) + H20 ~ C02 + H20
Water gas
5. Manufacture of methyl alcohol from ZnO + Cr203 (as a promoter). 200 atms.
water gas pressure and temp of 450°C.
CO+H2 +H2 ~ CH30H
Water gas

6. Chamber process for the manufacture of Nitric oxide


H2S04
ZS02 + 02 + [NO] ~ 2S03 + [NO]
S03 + H20 ~ H2S04
7. Acetic acid from acetaldehyde
2CH3CHO + 02 ~ 2CH3COOH

Problem 4. What is biochemical or enzYme catalysis? Discuss the char-


acteristics and some examples of enzYme catalysis. Discuss the kinetics of
enZJme catalysis.
Or Explain enZJme catalysed reactions. (Meerut 2006, 2004)
Biochemical or Enzyme Catalysis
Enzymes are complex nitrogeneous organic compounds. They are pro-
duced in the living cells of plants and animals. On dissolving in water they
form the colloidal solution. hence they behave as very active catalysts in
certain biochemical reactions and are known as biochemical catalysts and
the phenomenon itself is known as biochemical catalysis.
[I] Characteristics of Enzyme Catalysts
(1) Enzymes form a colloidal solution in water and hence they are very
active catalysts.
1
266 PHYSICAL CHEMISTRY-I

(2) Like inorganic catalysts they cannot disturb the final state of equi-
librium of a reversible reaction.
(3) They are highly specific in nature, i.e., one catalyst cannot catalyse
more than one reaction.
(4) They are highly specific to temperature. The optimum temperature
of their activity is 35°C to 40°C. They are deactivated at 70°C.
(5) Their activity is increased in the presence of certain substances,
known as co-enzymes.
(6) A small quantity of enzyme catalyst is sufficient for a large change.
(7) They are destroyed by U.v. rays.
(8) Their efficiency is decreased in presence of electrolytes.
[II] Examples of Enzyme Catalysis
The following are some examples of biochemical or enzyme catalysis.
(1) Manufacture of ethyl alcohol from cane sugar

Glucose Fructose
Zymase
C6H120 6 ) 2C 2H50H + 2C0 2
(2) Manufacture of acetic acid from ethyl alcohol

C2H50H + O2 Mycoderma aceti C CO H 0


) H3 0 + H2
(3) Conversion of starch into maltose
Diastase
2(C~1005)n + nH20 --~ nC 12H220 II
Starch Maltose

Maltase
C 12H220 U + H20 ---~) 2C6H l2 0 6
Maltose Glucose

(4) In the estimation of urea


Urease enzyme completely converts urea into ammonium carbonate.
/,NH2
/' Urease
O=C",- + 2H20 ~ (NH4hC0 3
"'- NH2
(5) In digestive tract
(a) In stomach, pepsin enzyme converts proteins into peptides, whereas
in intestines, pancreas trypsin converts proteins into amino acids by hydrol-
ysis. These amino acids are absorbed by blood and are used in the building
of tissues.
(b) The enzyme ptyalin present in human saliva converts starch into
glucose.
Ptyalin
) nC 6H l2 0 6
Gluco;e

J
CHEMICAL KINETICS & CATALYSIS 267

[III] Kinetics of Enzyme Catalysis or Michaelis-Menten


Equation
A reactant in an enzyme catalysed reaction is known as substrate.
According to the mechanism of enzyme catalysis, the enzyme combines with
the substrate to form a complex, as suggested by Henri (1903). He also
suggested that this complex remains in equilibrium with the enzyme and the
substrate. Later on in 1925, Briggs and Haldane showed that a steady state
treatment could be easily applied to the kinetics of enzymes. Some photo-
chemical reactions and some enzymic reactions are reactions of the zero order.
With S representing substrate, E the enzyme, ES an enzyme-substrate
complex and P the products, the mechanism of the enzyme catalysed reaction
is presumed to be adequately represented by
kj k
E+S~ ES~E+P
k2

where kl' k2' k3 are the rate constants for the respective reactions.
The rate of formation of the complex ES is, evidently given by the
following equation,

d[!S] = 0 = kl [E] [S] - k2 [ES] - k3 [ES]

... (1)

where [E], [S] and [ES] represent molar concentrations of the free enzyme,
substrate and the complex, i.e., bound or reacted enzyme, respectively.
Now [E] cannot be experimentally measured. The equilibruim between
the free and bound enzyme is given by the enzyme conservation equation,
i.e.,
[E]o = [E] + [ES]

where [E]o refers to the total enzyme concentration. So,


[E] = [E]o - [ES]

On substituting the value of [E] in equation (1), we get

d ~S] = kl {[E]o - [ES]} [S] - (k'l + k3) [ES] =0 ... (2)

As the reaction proceeds, the intermediate complex formed in accor-


dance with the suggested mechanism, decomposes instantaneously according
to the same mechanism. On applying the steady state principle, we have

d [ES] = 0
dt
268 PHYSICAL CHEMISTRY-I

At the stationary state, equation (2) may be written as,


k\ {[E]o - [ES]} [S] = (k2 + k 3) [ES]
or k\ [E]o [S] = {(k2 + k3) + k\ [S]} [ES]

[ES] = k\ [E]o [S]


(k2 + k3) + kI [S]
[E]o [S]
=
k2 + k3 + [S]
kI
The rate of formation of the product, P, i.e., r is represented by the
equation,

r = d [P] =k3 [ES] ... (3)


dt
Substituting the value of [ES] in equation (3), we get
d [P] k3 [E]o [S]
r=--=
dt k2 + k3
k;-+[S]
. (k2 + k3)
The quantIty k is known as Michaelis constant and may be
denoted by Km. Therefok,
d [P] _ k3 [E]o [S]
... (4)
dt - Km + [S]
Equation (4) is known as Michaelis-Menten equation.
Further simplification of equation (4) can be made. If it is assumed that
all the enzyme has reacted with the substrate at high concentrations the
reaction will be going on at maximum rate. No free enzyme will remain so
that [E]o = [ES]. So, from equation (3), we get
rrnax = Vrnax = k3 [E]o
where "'max reters to maximum rate, using the notation of enzymology. So,
Michaelis-Menten equation can also be written as,
Vmax [S]
r=
Km + [S]
If r = Vma J2, i.e., if the rate of formation of product is equal to half of
the maximum rate at which the reaction proceeds at high concentration of
substrate, then
Km = [S]
Thus, Michaelis COli stant is equal to that concentration of substrate,
S at which the rate of formation of the product is half the maximum rate
obtained at a high cOllcentration of substrate.
CHEMICAL KINETICS & CATALYSIS 269

From equation (4), we can draw the following conclusions:


(i) If [S] is very small as compared to Km, the factor Km/[S] will be
very large as compared to unity and so the rate of formation of
P, i.e., d [P]Idt will be directly proportional to [8]. In other words,
the reaction will be of the first order with respect to S (Fig. 13).

. 1
Maxlmum rate, 2 Vmax

Vmax [SJIKm (first order)

Substrate concentration (S) _

(ii) If [S] is very large as compared to Km, the factor Km/[S] will be
negligibly small as compared to unity and so the rate of formation
of P, i.e., d [P]/dt will be independent of the concentration [S].
In other words, the reaction will be of zero order with respect to
S (Fig. 13).
(iii) If [S] is very small or very large, the reaction remains of the first
order with respect to the total concentration, [E]o of the enzyme.
Problems 5. mite a short note on acid-base catalysis. (Meerut. 2007)
As a result of the work of Bronsted, Lowry and others, it has become
evident in recent years that a variety of atomic, molecular and ionic species
are capable of catalysing chemical reactions. For some processes, hydrogen
ions appear to be effective, while other reactions are catalysed by hydroxyl
ions, cations of weak bases, anions of weak acids, undissociated molecules
of acids and bases etc. This general acid catalysis involves cases where all
acids act as catalysts, while general base catalysis refers to processes
catalysed by bases of all kinds. In some cases, both acids and bases are
effective, while in others a particular species is effective. General acid-base
catalysis is illustrated by mutarotation of glucose, which is catalysed by
hydrogen, hydroxyl and complex ions, as well as by the acids and bases,
though the most effective catalyst is the hydrogen ion.
270 PHYSICAL CHEMISTRY-I

Mechanism of acid-base catalysis: It is accepted that acid-base


catalysis involves a reversible acid-base reaction between the substrate and
catalyst. This is in agreement with the protonic concept of acids and bases,
since acid catalysis depends on the tendency of the acid to lose a proton, while
base catalysis depends upon the tendency of the base to gain a proton. The
mechanism of reaction involving Wand OIr ion catalysis may be expressed
as follows. by taking the example of hydrolysis of esters.
(a) With W ions as catalyst
ORO R 0 R
II I n+
I I+ HO
'
I+ I
CH,-C-O + n ---> CH,-C-O -H -'---> CH,-C/\H

H H

(b) With OS- ions as catalyst


ORO R 0 R
II I I I I I
CH3-C-O + OIr ~ CH3-C-O+ ~ CH3-C-O+-H
I I
H-O H-O
~ CH3COOH + ROH
The rate of reaction is given by
dx
-d = kH+ C H+ Cester + kOH- C OH Cester + kH,o CH,o Cester
t - -
where kH+ and koH- are the catalytic coefficients of hydrogen and hydroxyl
ions.

MULTIPLE CHOICE QUESTIONS

1. The dimension of first order rate constant is :


(i) time-1 (ii) time
(iii) time x conc (iv)time-l x conc- l
2. The dimension of second order rate constant is :
(i) times x conc (ii) time- l x conc- l
(iii) time x conc-1 (iv) time-1 x conc-2
CHEMICAL KINETICS & CATALYSIS 271

3. The hydrolysis of methyl acetate by acid is of :


(i) Zero (ii) First order
(iii)
Second order (iv) Third order
4. The hydrolysis of ethyl acetate by NaOH is of :
(i) Zero (ii) First order
(iii)
Second order (iv) Third order
5. The reaction rate of a reaction 2A ~ 3B is given by :
(a) _ d [A] (b) _! d [A]
~ 2 ~
(c) + ~ d ~~] (d) + d ~]
6. For a reaction A + B ~ Products, the reaction rate is given by, :
dx = k[A] [Br
dt
The order of reaction with respect to A and B are :
(i) 1,0 (ii) 1, 1 (iii)O. 1 (iv) 2, 0
7. The half life period of a first order reaction is 20 min. The time required for the
concentration of the reactant to change from 0.4 M to O.I,M is :
(i) 20 min (ii) 40 min (iii) 60 min (iv) 80 min
8. The ratio of the time required for 75% of a first on':,,! rr,ction to complete to
that required for 50% of the reaction is :
(i) 4: 3 (ii) I : 2 (iii)2: I ' (iv)3: 2
9. The time required to decompose half of the reaction for a n'h order reaction is
also doubled. The order of reaction is :
(i) 0 (ii) 1 (iii) 2 (iv) 3
10. The minimum energy required for reactant molecules to enter into chemical
reaction is known as :
(i) Kinetic energy (ii) Potential energy
(iii) Activation energy (iv) Threshold energy
11. A first order reaction is 75% completed in 32 minutes. For 50% completion, it
will take: :
(i) 4 min (ii) 8 min
(iii) 16 min (iv) 32 min
12. The elementary step of the reaction 2Na + Cl 2 ~ 2NaCI is found to follow
third order reaction kinetics. The molecularity of the reaction is :
(i) 0 (ii) 1
(iii) 2 (iv) 3
13. In a reaction A ~ B, the reaction rate is doubled on increasing the concen-
trations of reactants four times. The order of reaction is :
(i) 0, (ii) 1 (iii) 2 (iv) 3
14. On plotting loglok against Iff, the slope of the straight line is given by::
. Ea " Ea
(1) R (n) -R
E
(iii) 2.30; R
272 PHYSICAL CHEMISTRY-I

15. If the plot of 10glO [A] against time is a straight line with a negative slope, the
order of reaction is :
(i) 0 (ii) 1 (iii) 2 (iv) 3
16. In the hydrogenation of oils the catalyst used is :
(i) Iron (ii) Platinum
(iii) Nickel (iv) Molybdenum
17. The effect of a catalyst in a chemical reaction is to change the:
(i) Activation energy (ii) Equilibrium concentration
(iii) Heat of reaction (iv) Final product
18. The catalyst used in the contact process of sulphuric acid manufacture is :
(i) Oxides of nitrogen (ii) Nickel
(iii) Vanadium pentoxide (iv) Manganese dioxide
19. Which of the following is used as a contact catalyst?
(i) Boron (ii) Germanium
(iii) Nickel (iv) U~anium
20. Which of the following statements is universally correct?
(i) A catalyst remains unchanged at the end of the reaction
(ii) A catalyst physically changes at the end of the reaction
(iii) A catalyst takes part in the chemical reaction
(iv) A catalyst can induce chemical reaction
21. The catalyst used for the oxidation of ammonia to nitric acid is :
(i) Cupric chloride (ii) Iron oxide
(iii) Platinum (iv) Manganese dioxide
22. A substance that regards the rate of chemical reaction in the presence of a catalyst
is called :
(i) An inhibitor (ii) A positive catalyst
(iii) An auto-catalyst (iv)A promoter
23. A catalyst poison is essentially:
(i) A homogeneous catalyst (ii) A heterogeneous catalyst
(iii) An inhibitor (iv) An auto-catalyst
24. Catalyst poisons (temporary poisoning) act by :
(i) Chemically combining with the catalyst
(ii) Getting adsorbed on the active centres on the catalyst surface
(iii) Chemical combination with anyone of the reactants
(iv) Coagulating the catalyst
25. Which of the following types of the metal make the most effiCIent catalyst?
(i) Transition metals (ii) Alkali metals
(iii) Alkaline earth metals (iv) Coloured metals
26. Which one of the following statements is incorrect?
(i) Presence of a catalyst does not alter the equilibrium concentration in a
reversible reaction
(ii) Change of temperature alter the rate of catalysed reaction in the same
proportion as of the reaction without catalyst
(iii) Homogeneous catalysis depends upon the nature and extent of the surface
(iv) Change of a catalyst may change the nature of the reaction
27. Enzymes are:
(i) Micro-organisms (ii) Proteins
(iii) Inorganic compounds (iv) Moulds
CHEMICAL KINETICS & CATALYSIS 273

28. A pho~chemical reaction is :


(i) Catalysed by light
(ii) Initiated by light
' .. (iii) Accompanied with emission of light
1;
1
(iv) Accompained with absorption of light
29. Platinised asbestors is used as catalyst in the manufacture of H2S04, It is an
~ example of :
(i) Homogeneous catalyst (ii) Auto-catalyst
(iii) Heterogeneous catalyst (iv) Induced catalyst
30. The enzyme ptyalin used for digestion of food is present in :
(i) Saliva (ii) Blood
(iii) Intestine (iv) Adrenal gland
31. A catalyst is a substance with :
(i) Increase the equilibrium concentration of the product
(ii) Change the equilibrium constant of the reaction
. (iii) Shortens th time to each equilibrium
(iv) Supplies energy of the reaction
32. An example of auto-catalytic reaction is :
(i) The decomposition of nitroglycerine
(ii) Thermal decomposition of KCl0 3• Mn02 mixture
(iii) Break down of 14C6
(iv) Hydrogenation of vegetable oil using nickel catalyst
33. The efficiency of an enzyme in catalysing a reaction is due to its capacity:
(i) To form a strong enzyme-substrate complex
(ii) To decrease the bond energies in the substrate molecule
(iii) To change the shape of the substrate molecule
(iv) To lower the activation energy of the reaction
34. A catalyst:
(i) Increases the energy change in a reaction
(ll) Increases the free energy change in a reaction
(iii) Neither decreases nor increases the free energy change in a reaction
(iv) Can increase or decrease the free energy change in a reaction but it depends
on the catalyst
3? Starch is converted into disaccharide in the presence of :
(i) Diastase (ii) Maltase
(iii) Lactase (iv)Zymase
36. Glucose or fructose is converted into C2H50H in the presence of :
(i) Invertase (ii) Maltase
(iii) Zymase (iv) Diastase
37. A ~talyst increases the rate of a chemical reaction by :
(i) Increasing the activation energy
(ii) Decreasing the activation energy
(iii) Reacting with reactants
(iv) Reacting with products
38. Which of the following statement is correct?
(i) Enzymes are in colloidal state (ii) Enzymes are catalysts
(iii) Enzymes catalyse any reaction (iv) Urease in an enzyme
274 PHYSICAL CHEMISTRY-I

39. A catalyst is used:


(i) To vaporise the compound (ii) To kill the enzyme
(iii) To alter the velocity of reaction (iv) To balance the reaction
40. Enzymes Il.re :
(i) Substances made by chemists to activate washing powder
(ii) Very active vegetable catalysts
(iii) Catalysts found in organisms
(iv) Synthetic catalysts
41. Which of the following catalysts is used for preparing toluene by reacting
benzene with CH 3Cl?
(i) Ni (ii) Anhydrous AlCl 3
(iii) Pd (iv) Pt
42. The rusting of iron is catalysed by which of the following?
(i) Fe (ii) O2
(iii) Zn (iv) H+
43. In which of the commerical process, a catalyst is not used?
(i) Haber's process (ii) Deacon's process
(iii) Solvay process (iv) Lead chamber process
44. Which of the following statements is correct for a catalyst?
(i) It supplies energy to the system (ii) It alters the rate of the reaction
(iii) It alters the equilibrium constant (iv) It is used up in the reaction
45. Organic catalysts differ from inorganic catalysts in :
(i) By acting at very high temperature (ii) By acting at low temperature
(iii) Being used up (iv) Being proteneous in nature
46. Which one of the following statements regarding catalyst is not true?
(i) A catalyst can initiate a reaction
(ii) A catalyst remains unchanged at the end of the reaction
(iii) A catalyst does not alter the equilibrium in a reversible reaction
(iv) Catalysts are sometimes very specific in terms of reactions
47. Which statement is incorrect for heterogeneous catalysis?
(i) Catalyst is absorbed on the surface
(ii) Active centres are found on the surface of catalyst
(iii) Catalyst increases the energy of activation
(iv) None of these
48. Which of the following is used as a contact catalyst?
(i) Boron Oi) Nickel
(iii) Germanium (iv) Uranium
49. Which one of the following statements is incorrect in the case of heterogeneous
catalyst?
(i) The catalyst lowers the energy of activation
(ii) The catalyst actually forms a compound with the reactant
(iii) The surface of the catalyst plays a very important role
(iv) There is no change in the energy of activation

Fill in the Blanks


1. A first order reaction is 15% complete in 20 min. It will take ............ min to be
60% complete.
2. The rate ofreaction is nearly doubled on increasing the temperature by .......... .
CHEMICAL KINETICS & CATALYSIS 275

3. Order of reaction for the hydrolysis of ethyl acetate by HCI is ............ .


4. For a reaction A ~Products, the rate law is ~ = k [A]/2. The order of
reaction is ...... ... . t
5. The order of reaction of decomposition of H20 2 is ............... .
6. If the rate constant, k = .! . -x()' the order of reaction is ............. .
t a a-x
7. If the rate constant has tl-te UnIt sec-\ the order of reaction is ............. .
8. The equation k = Ae- EIRT is known as ................... equation.
9. In equation k = Ae- EIRT, A is known as ............... .
10. The hydrolysis of ethyl acette in acidic medium is ............. order reaction.
11. A substance which changes the rate of reaction is known as ............... .
12. The substance which retards the reaction rate is known as ................. cataryst.
13. The ................. of a catalyst increases when it is finely divided.
14. The substance which increases the efficiency of a catalyst is known as ............ .

15. In the conversion of urea into ammonium carbonate ................. acts as a catalyst.
16. 2502 + O2 ~ 2S0 3 is an example of .............. catalysis.
He!
17. CH 3COOCH 3 + H20~ CH 3COOH + C 2H50H is an example of .......... .
catalysis.
18. ~ 2NH , Mo acts as ............. .
In N2 + 3H2 ~ 3
19. A catalyst poison is essentially a ............... .
20. The presence of a catalyst ............ the activation energy of the reaction.

True or False
State whether the following statements are true (T) or
false (F)?
1. The value of temperature coefficient is nearly 10.
2. All radioactive emanations are of first order.
3. The inversion of cane sugar by HCI is of second order.
. ,, 4. The reaction rate is proportonal to the surface area of reactant.

5. The half life period of a first order reaction is O.~93.


6. Order of reaction and molecularity are similar.
7. Order of reaction can even be 4 or more.
8. Most of the reactions are of first and second order.
hv
9. The reaction H2 + Cl 2 ~ 2HCI is of zero order.
10. The rate of zero order reaction depends on the concentration of the reactant.
11. A catalyst is a substance which can only increase the reaction rate.
12. When one of the products formed in the reaction itself acts as a catalyst, the
phenomenon is called auto-catalysis.
13. A catalyst is specific in action.
14. A catalyst can change the position of equilibrium.
276 PHYSICAL CHEMISTRY-I

15. A catalyst remains unchanged in mass and chemical composition at the end of
a reaction.
16. A large quantity of catalyst is required to bring about a reaction.
17. The substance which increases the activity of a catalyst is called an activator.
18. A promoter decreases the peaks and cracks on the catalyst surface.
19. Enzyme ptyalin present in h)lman saliva changes starch into glucose.
20. The presence of a catalyst increases the activation energy of the reaction.
21. In homogeneous catalysis, the intermediate compound is formed at lower acti-
vation energy.
22. A catalyst cannot be recovered unchanged chemically at the end of the reaction.
ANSWERS
1. (a) ,2. (b) 3. (b) 4. (c) 5. (b) 6. (a) 7. (b) 8. (c) 9. (a) 10. (d) 11. (c) 12. (d)
llWK~~~~~nWm~~~·Wn~nWn~u~
25. (a) 26. (c) 27. (b) 28. (b) 29. (c) 30. (a) 31. (c) 32. (a) 33. (d) 34. (d) 35. (a) 36. (c)
37. (b) 38. (c) 39. (c) 40. (c) 41. (b) 42. (d) 43. (c) 44. (b) 45. (d) 46. (a) 47. (c) 48 (d)
49. (d)

Fill in the Blanks


1. 112.8 min 2. W'C 3. (F) one 4. 112
5. one 6. two 7. one 8. Arrhenius
9. Frequency factor 10. First 11. Catalyst 12. negative
13. efficiency 14. promoter 15. Urease 16. homogeneous
17. autolhomogeneous 18. promoter 19. inhibitor 20. decreases

True or False
1. (F), 2. (T), 3. (F), 4. en,
5. (T), 6. (F), 7. (F), 8. (T),
9. (T), 10. (F); 11. (F), 12. (T),
13. (T), 14. (F), 15. (T), 16. (F),
17. (T), 18. (F), 19. en, 20. (F),
21. (T), 22. (F)

DOD

You might also like